Dermoscopy: An Illustrated Self-Assessment Guide [1 ed.] 0071613552, 9780071613552

A case-based visual guide to learning dermoscopy complete with practical self-assessment 300 Full-color Illustrations —

390 107 24MB

English Pages 441 Year 2010

Report DMCA / Copyright

DOWNLOAD FILE

Polecaj historie

Dermoscopy: An Illustrated Self-Assessment Guide [1 ed.]
 0071613552, 9780071613552

Citation preview

Dermoscopy An Illustrated Self-Assessment Guide

NOTICE Medicine is an ever changing science. As new research and experience broaden our knowledge, changes in treatment and drug therapy are required. The authors and the publisher of this work have checked with sources believed to be reliable in their efforts to provide information that is complete and generally in accord with the standards accepted at the time of publication. However, in view of the possibility of human error, or changes in medical sciences, neither the authors nor the publisher nor any other party who has been involved in the preparation or publication of this work warrents that the information contained herein is in every respect accurate or complete, and they disclaim all responsibility for any errors or omissions or for the results obtained from use of the information contained in this work. Readers are encouraged to confirm the information contained herein with other sources. For example and in particular, readers are advised to check the product information sheet included in the package of each drug they plan to administer to be certain that the information contained in this work is accurate and that changes have not been made in the recommended dose or in the contraindications for administration. This recommendation is of particular importance in connection with new or infrequently used drugs.

Dermoscopy An Illustrated Self-Assessment Guide Robert H. Johr, MD Clinical Professor of Dermatology and Associate Clinical Professor of Pediatrics Pigmented Lesion Clinic University of Miami School of Medicine Miami, Florida

Prof. Wilhelm Stolz, MD Director Clinic of Dermatology, Allergology and Environmental Medicine Hospital München Schwabing, and Professor of Dermatology, Faculty of Medicine Ludwig-Maximilians-Universität Munich Germany

New York Chicago San Francisco Lisbon Milan New Delhi San Juan Seoul

London Madrid Mexico City Singapore Sydney Toronto

Copyright © 2010 by The McGraw-Hill Companies, Inc. All rights reserved. Except as permitted under the United States Copyright Act of 1976, no part of this publication may be reproduced or distributed in any form or by any means, or stored in a database or retrieval system, without the prior written permission of the publisher. ISBN: 978-0-07-161356-9 MHID: 0-07-161356-0 The material in this eBook also appears in the print version of this title: ISBN: 978-0-07-161355-2, MHID: 0-07-161355-2. All trademarks are trademarks of their respective owners. Rather than put a trademark symbol after every occurrence of a trademarked name, we use names in an editorial fashion only, and to the benefit of the trademark owner, with no intention of infringement of the trademark. Where such designations appear in this book, they have been printed with initial caps. McGraw-Hill eBooks are available at special quantity discounts to use as premiums and sales promotions, or for use in corporate training programs. To contact a representative please e-mail us at [email protected]. TERMS OF USE This is a copyrighted work and The McGraw-Hill Companies, Inc. (“McGrawHill”) and its licensors reserve all rights in and to the work. Use of this work is subject to these terms. Except as permitted under the Copyright Act of 1976 and the right to store and retrieve one copy of the work, you may not decompile, disassemble, reverse engineer, reproduce, modify, create derivative works based upon, transmit, distribute, disseminate, sell, publish or sublicense the work or any part of it without McGraw-Hill’s prior consent. You may use the work for your own noncommercial and personal use; any other use of the work is strictly prohibited. Your right to use the work may be terminated if you fail to comply with these terms. THE WORK IS PROVIDED “AS IS.” McGRAW-HILL AND ITS LICENSORS MAKE NO GUARANTEES OR WARRANTIES AS TO THE ACCURACY, ADEQUACY OR COMPLETENESS OF OR RESULTS TO BE OBTAINED FROM USING THE WORK, INCLUDING ANY INFORMATION THAT CAN BE ACCESSED THROUGH THE WORK VIA HYPERLINK OR OTHERWISE, AND EXPRESSLY DISCLAIM ANY WARRANTY, EXPRESS OR IMPLIED, INCLUDING BUT NOT LIMITED TO IMPLIED WARRANTIES OF MERCHANTABILITY OR FITNESS FOR A PARTICULAR PURPOSE. McGraw-Hill and its licensors do not warrant or guarantee that the functions contained in the work will meet your requirements or that its operation will be uninterrupted or error free. Neither McGraw-Hill nor its licensors shall be liable to you or anyone else for any inaccuracy, error or omission, regardless of cause, in the work or for any damages resulting therefrom. McGraw-Hill has no responsibility for the content of any information accessed through the work. Under no circumstances shall McGraw-Hill and/or its licensors be liable for any indirect, incidental, special, punitive, consequential or similar damages that result from the use of or inability to use the work, even if any of them has been advised of the possibility of such damages. This limitation of liability shall apply to any claim or cause whatsoever whether such claim or cause arises in contract, tort or otherwise.

Contents Foreword Preface Acknowledgments

vii ix xi

Chapter 1

Dermoscopy from A to Z

01

Chapter 2

Scalp, Face, Nose, and Ears

27

Chapter 3

Trunk and Extremities

95

Chapter 4

Palms, Soles, Nails

325

Chapter 5

Genitalia

389

Index

419

This page intentionally left blank

Foreword The new textbook, “Dermoscopy: An Illustrated SelfAssessment Guide” by Dr. Robert H. Johr and Prof. Wilhelm Stolz, is a welcome addition to the evolving world of dermoscopic knowledge. I have reviewed all of the images and the text and feel that the contributions are indeed unique. First of all, the exquisite nature of the photographic images deserves a comment. Also, the clarity of labeling for each image separates this text from all others I have seen. Specifically, all of the key features are isolated by means of circles, rectangles, asterisks, etc., and this clear demarcation has succeeded where many similar textbooks fail. Secondly, the questions, statements and the answer format draws the reader into the dialogue. You really cannot escape committing yourself to a diagnosis, or differential diagnosis, as you answer the questions and statements posed. I think these two unique aspects of the textbook serve the learning process in a very positive way. I should also add that I am an unabashed enthusiast of dermoscopy. Although I presently confine my clinical practice to pediatric dermatology, until 7 years ago, I saw adults on a daily basis and evaluated all of their pigmented lesions with dermoscopy. I have been committed to learning the technique for the last 14 years. Most of that learning curve has involved discussions and teaching by Dr. Robert Johr. Early on, we saw patients with complex pigmentary disorders such as xeroderma pigmentosum and evaluated numerous

lesions dermoscopically, helping to separate those lesions which deserved follow-up from those which deserved biopsy. To this day, I continue to use dermoscopy not only to evaluate pigmented lesions in children, but I also use it as an advanced “pocket magnifier”. Specifically, hair shaft disorders, scabies, questionable molluscum lesions, etc., are readily identified when separated out by means of dermoscopy. When I forget my dermascope or when I find it uncharged, I actually feel less qualified to evaluate my patients. I firmly believe that dermoscopy should be taught as an essential skill in every residency program and that all practitioners should strive to learn this technique. I certainly learned it when I was in my mid 50s, hence, a mature dog can learn new tricks. I am still surprised by the fact that virtually all children referred to me for evaluation of pigmented lesions have not been viewed with dermoscopy by their referring dermatologists. This needs to change. This technique adds a dimension which clearly makes us better diagnosticians and better dermatologists. The book by Drs, Johr and Stolz will help us in this effort. Ronald C. Hansen, MD Chief, Pediatric Dermatology Phoenix Children’s Hospital, Phoenix, Arizona Professor, Dermatology and Pediatrics University of Arizona College of Medicine, Tucson, Arizona

This page intentionally left blank

Preface Dermoscopy is itself a language full of terms that have specific meanings and even connotations, depending on any given lesion being examined. As in any language, its vocabulary is a work in progress based in consensus among its “expert” speakers who are striving to create a system to communicate dermoscopic findings based on observational data. In order for there to be mutual comprehension among users of this powerful technique, the language of dermoscopy must be spoken properly. This is not an easy task since there is a significant learning curve to master the technique as well as its terminology. It takes study, practice, and dedication. Dermoscopy is the standard of care around the world and is becoming very popular in the United States. Dermatologists and other groups of physicians are realizing what a valuable tool it is. The goal of this book is to teach what we believe are the important general principals and specific points of dermoscopy and to allow for users to “self-assess” their knowledge and skills using the techniques taught here. In an era of information overload, we designed the book to be short, sweet, and to the point. We want it to be an easy, enjoyable, and practical read. Important principles are often repeated which is a good way for them to be remembered. We “keep it real” with 191 cases that any busy clinician may have the opportunity to see in general dermatology clinic on a daily basis. Great clinical and dermoscopic images with short histories are followed by five “true or false” statements. As in real life, then comes the decision making in check box form: what is the potential risk and what is the diagnosis? Finally, the disposition of the case; whether to effect no intervention, follow-up, or to make a histopathologic diagnosis? The concept of dermoscopic differential diagnosis is found throughout the book. In most cases, we do not get into the controversial issue of the best technique to make a histopathologic diagnosis. We leave that up to you. Turn the page, and the answers to the statements are given in a format that separates our book from the others. The dermoscopic images are presented again with an extensive description of the criteria in the lesion. It is essential to evaluate as much as possible before making a diagnosis. There are many circles, boxes, arrows, and stars to point out

the important features of each case. Our goal is to fully demonstrate the global features and local criteria of each lesion. This is another very important unique teaching point of our book. Each case has a discussion of all of its salient features. Not in long drawn out paragraphs, but in outline form. We realize that your time is valuable and want to make the learning and recall process as easy as possible. Series of cases are organized into groups. For example, there are lesions in which the major feature might be pigment network, dots and globules, regression, pink, blue or black color, or vascular structures. There are similar-looking clinical and/or dermoscopic images grouped together in specific body locations, such as brownish spots on an ear lobe or in the genital area. This simulates real-life encounters. One case often flows into the next and knowledge gained from the previous case is needed to solve the next case. Melanocytic, nonmelanocytic, benign, malignant, or inflammatory pathology from head to toe with 95 melanomas and their most important simulators. Each case ends with a series of dermoscopic and/or clinical pearls based on years of experience treating patients with skin cancer. The patients’ well-being trumps political correctness. The book is sprinkled with general principles and specific points that are controversial but strongly embedded in our core beliefs. This book was a coproduction between United States and Germany. The text was developed in Florida and most of the cases were seen in Munich. Being a competent diagnostician must include the tissue-sparing and potentially life-saving technique called dermoscopy. Each of us has a profound responsibility for the wellbeing of every patient that walks through the door. Always regard each patient as someone’s precious loved one as if they were your own! Robert H. Johr Boca Raton, Florida

Prof. Wilhelm Stolz Munich, Germany

This page intentionally left blank

Acknowledgments I would like to thank James A. Ida, MD academic chief resident in the Department of Dermatology at Tufts Medical Center in Boston as well as a friend, for his excellent efforts as resident editor. James’s passion for language and grammatical style as well as keen interest in dermoscopy and in helping produce a superb text, made the tedious job of writing a book much more enjoyable. Thanks! I want to thank my wife of 38 years, Irma, for letting me be free in mind and spirit to spend the many hours it took to write this book. Her encouragement and willingness to listen to me when I thought or wrote something exciting was very helpful.

Dr. Robert H. Johr

First and foremost, I would like to express my deepest gratitude to my wife, Karola, who has lovingly shared my many dermoscopic and academic pursuits over the last two decades. For her tireless enthusiasm and skill with the beautiful color photography, a cornerstone of our text, I thank Mrs. Ulrike Brückl. Special thanks also go to our nurses, Mss. Carolin Mertens, Antje Angilotti, and Christa Meinhold, whose consistent passion and professionalism contributed greatly to our text. To the many physicians who supported me in my dermoscopy clinic and in the case preparations, most especially Drs. Brigitte Coras, Stefanie Guther, Anette Michael, Katrin Ramrath, Alexandra Tillmann, and Ulrike Weigert, I wish to extend my sincerest thanks. Finally, special thanks go to Mrs. Agnes Kaldewei, my long-time clinical and scientific assistant. Without her continuous and ever pleasant support, this text would not have been possible.

Dr. Wilhelm Stolz

This page intentionally left blank

Dermoscopy An Illustrated Self-Assessment Guide

This page intentionally left blank

Chapter 1

Dermoscopy from A to Z

This page intentionally left blank

Chapter 1

SYNONYMS ■ ■ ■ ■ ■ ■ ■

Dermatoscopy Skin surface microscopy Epiluminescence microscopy (ELM) Digital dermoscopy/digital ELM Auflichtmikroskopie (German) Dermoscopia/Dermatoscopia (Spanish/Italian) Grammatically, dermatoscopy is correct English; however, dermoscopy is the term used by the International Dermoscopy Society (IDS), experienced dermoscopists, and is most commonly used in the literature. This might change in the future and dermatoscopy could become the term of choice again.

■ ■ ■

■ ■







Dermoscopy is an in vivo, noninvasive technique in which oil or fluid (eg, mineral oil, gels, alcohol, and water) is placed on the lesion ■ Fluid eliminates reflection of light from the surface of the skin allowing visualization of color and structure in the epidermis, dermo-epidermal junction, and papillary dermis ■ The color and structure visualized cannot be seen with the naked eye or with typical magnification that clinicians use ■ Polarizing light and digital instrumentation do not require fluid When using polarized light dermoscopy ■ Light from a polarized light source penetrates the stratum corneum with less scatter ■ A second polarizer screens out scattered surface light resulting in the physician seeing primarily light from the deeper structures ■ This removes the need for contact with the skin and the need for immersion fluids, resulting in faster examination times











■ ■

Helps to differentiate melanocytic from nonmelanocytic skin lesions Helps to differentiate benign from malignant skin lesions ■ With dermoscopy the diagnostician’s sensitivity to diagnose melanoma is 90% compared to 74% when the technique is not used ■ Dermoscopy improves the diagnosis of melanoma by 16% ■ The number of benign lesions excised for each melanoma found (malignant/benign ratio) is significantly decreased with dermoscopy Increases the diagnosis of early melanoma Increases the diagnosis of melanoma incognito (false negative melanoma)

Helps to avoid unnecessary surgery Helps to plan surgery Helps one to work better with their pathologist (asymmetrical high risk criteria, collision tumors, dermoscopic–pathologic correlation) Patient reassurance Allows for follow up of patients with multiple nevi digitally to find changes over time

There are pigmented skin lesions that are not high risk enough to warrant immediate histopathologic diagnosis, yet not so banal that there is no concern at all There are melanomas that do not appear to be high risk clinically or with dermoscopy They are only diagnosed after monitoring for dermoscopic changes over time when comparing baseline with subsequent digital images Short-term monitoring is performed every three or four months ■ Any change over time could be a melanoma and is an indication to make a histopathologic diagnosis Long-term monitoring is done at six month to yearly intervals ■ Important changes include asymmetrical enlargement, asymmetrical changes within the lesion without enlargement, the appearance of high risk criteria (eg, irregular pigment network, dots, globules, streaks, blotches, regression, polymorphous vessels), disappearance of well-developed local criteria, new colors Single or multiple suspicious pigmented skin lesions can be chosen for short- or long-term digital monitoring

THE TWO-STEP ALGORITHM ■

BENEFITS OF DERMOSCOPY ■

3

Dermoscopic Digital Monitoring

DEFINITION ■

Dermoscopy from A to Z



The analysis of a suspicious skin lesion is a two-step process ■ Step one: determines if it is melanocytic or nonmelanocytic ■ Step two: if it has the criteria for a melanocytic lesion, the second step is to determine if it is low, intermediate, or high risk using the melanocytic algorithm of your choice Pattern analysis was the first melanocytic algorithm developed for this purpose and is most often used by experienced dermoscopists. Variations of pattern analysis (the simplified algorithms) have also been developed, including: ■ The ABCD rule of dermatoscopy (the second algorithm developed) (Table 1-1) ■ The seven-point checklist (Table 1-2) ■ Menzies method (eleven-point checklist) (Table 1-3) ■ The latest three-point checklist (Table 1-4)

4

DERMOSCOPY: AN ILLUSTRATED SELF-ASSESSMENT GUIDE

Table 1-1

ABCD RULE OF DERMATOSCOPY. IDENTIFY CRITERIA AND GIVE THEM POINTS TO DETERMINE THE TOTAL DERMATOSCOPY SCORE (TDS)

Table 1-3

DERMOSCOPIC CRITERION Negative Features Symmetry of pattern Presence of single color

DERMOSCOPIC CRITERION DEFINITION SCORE WEIGHT FACTOR Asymmetry in 0, 1, or 2 perpendicular axes; assess contour, colors and structures 0-2 Border abrupt ending of pigment pattern at periphery in 0-8 segments 0-8 Color presence of up to 6 colors (white, red, light-brown, dark-brown blue-gray, black) 1-6 Dermoscopic structures presence of network, structureless (homogeneous) areas, branched streaks, dots, and globules 1-5

Positive Features Blue-white veil Multiple brown dots Pseudopods (streaks) Radial streaming (streaks) Scar-like depigmentation Peripheral black dots/globules Multiple (5 or 6) colors Multiple blue/gray dots Broadened network

Formula for calculating total dermatoscopy score (TDS): (A score x 1.3) + (B score x 0.1) + (C score x 0.5) + (D score x 0.5) = TDS. Interpretation of total score: 5.45, lesion highly suspect for melanoma.

Melanocytic Lesion

Look for the criteria associated with a melanocytic lesion (Table 1-5). If one does not find them, the search is on for the criteria associated with seborrheic keratosis, basal cell carcinoma, dermatofibromas, vascular lesions, and others. ■ Not all of the possible criteria are needed to make a diagnosis ■ When there is an absence of criteria for a melanocytic lesion, seborrheic keratosis, basal cell carcinoma, dermatofibroma, or vascular lesion, you will have to deal with a melanocytic lesion by default ■ The “default category” is the last criterion used to diagnose a melanocytic lesion (Figure 1-1)

SEVEN-POINT CHECKLIST

DERMOSCOPIC CRITERION 1. 2. 3. 4. 5. 6. 7.

Atypical pigment network (major criteria) Blue-whitish veil Atypical vascular pattern Irregular streaks (minor criteria) Irregular dots/globules Irregular blotches Regression structure

For melanoma to be diagnosed both negative features must be absent and one or more of the 9 positive features must be present.

Criteria Defined

Step One: Identification of Criteria

Table 1-2

MENZIES SCORING METHOD. THE ELEVEN-POINT CHECKLIST

PIGMENT NETWORK/NETWORK ■ ■ ■



On the trunk and extremities Shades of black or brown Honeycomb-like, reticular, web-like line segments (elongated and hyperpigmented rete ridges) with hypopigmented holes (dermal papilla) There are nonmelanoctyic lesions that can have a pigment network (ie, solar lentigo, seborrheic keratosis, dermatofibroma)

PSEUDONETWORK/PSEUDOPIGMENT NETWORK ■

Because the skin of the head and neck is thin and does not have well-developed rete ridges, one sees ■ Appendageal openings/adnexal structures (sebaceous glands, hair follicles) ■ Uniform, round white or yellowish structures

Table 1-4 SCORES 2 2 2 1 1 1 1

By simple addition of the individual scores a minimum total score of 3 is required for the diagnosis of melanoma, whereas a total score of less than 3 is indicative of nonmelanoma.

THREE-POINT CHECKLIST TO DIAGNOSE HIGH RISK LESIONS (MELANOMA, BASAL CELL CARCINOMA)

Asymmetry of color and/or structure Irregular pigment network Blue and / or white color 2 out 3, 3 out 3 → excise The three-point check list is based on simplified pattern analysis and is intended to be used by nonexpert dermoscopists as a screening technique. Its aim is to diagnose melanocytic and nonmelanoctyic potentially malignant pathology.

Chapter 1

Table 1-5

CRITERIA FOR VARIOUS LESIONS

CRITERIA FOR A MELANOCYTIC LESION Pigment network (trunk and extremities) Aggregated globules Homogeneous blue color of a blue nevus Parallel patterns on acral sites By default







■ ■

CRITERIA FOR A SEBORRHEIC KERATOSIS Milia-like cysts Pseudofollicular openings Fissures and ridges Fat fingers Fingerprint pattern Hairpin-shaped vessels Moth-eaten borders Sharp demarcation CRITERIA FOR A BASAL CELL CARCINOMA Absence of pigment network Arborizing blood vessels Pigmentation Ulceration Spoke-wheel structures





Dermoscopy from A to Z

5

When they penetrate areas of diffuse pigmentation, reticular-like structures are formed that are referred to as the pseudonetwork Monomorphous appendageal openings can often be seen on the skin of the face without any pigmentation They should not be confused with the milia-like cysts seen in seborrheic keratosis It is not always possible to make the differentiation Consequences could be misdiagnosing lentigo maligna for a seborrheic keratosis This criterion can also be seen with nonmelanocytic lesions (ie, solar lentigo, seborrheic keratosis) It is not 100% diagnostic of a melanocytic lesion

DOTS AND GLOBULES ■ ■



CRITERIA FOR A DERMATOFIBROMA Central white patch Peripheral pigment network CRITERIA FOR A VASCULAR LESION Vascular spaces (lacunae )



Roundish structures distinguished only by their relative sizes Dots (0.1mm) are smaller than globules (greater than 0.1mm) Black, brown, gray, or red ■ When black, they can represent melanin or atypical melanocytes in the epidermis ■ Regular brown dots and globules represent nests of melanocytes at the dermo-epidermal junction ■ Irregular brown dots and globules represent nests of atypical melanocytes at the dermo-epidermal junction ■ Fine grayish dots (“peppering”) represent free melanin and/or melanophages in the papillary dermis, which can be seen in regression areas along with other criteria or alone in benign pathology such as late stage lichen planus-like keratosis or posttraumatic ■ Reddish globules can be seen in melanoma (neovascularization) It is written and taught that globules identify a melanocytic lesion with no mention of the smaller dots. The reality is that both dots and globules define a melanocytic lesion (Figure 1-2)

Figure 1-1 Amelanotic Melanoma. This is a melanocytic lesion by default because there is an absence of criteria for a melanocytic lesion, seborrheic keratosis, basal cell carcinoma, dermatofibroma, or hemangioma. The blue-white color (arrow) is a clue that this might be a melanocytic lesion. There are pinpoint/dotted (yellow boxes) and irregular linear (black boxes) vessels plus a general milky-red background color. Note: This interdigital melanoma was mistakenly treated as a tinea for two years. (Reproduced, with permission, from Journal of Drugs in Dermatology. New Methods and Technologies. May 2008-Vol 7-Issue 5. Fig 1b.)

Figure 1-2 Acquired Nevus. This is a melanoctic lesion because it has pigment network (black boxes) and globules (circles). There is a small hemangioma adjacent to the nevus (arrow).

6

DERMOSCOPY: AN ILLUSTRATED SELF-ASSESSMENT GUIDE

Figure 1-3 Blue Nevus. The classic homogenous blue color of a blue nevus.

Figure 1-4 Acral Nevus. This is a melanocytic lesion on acral skin with the benign parallel furrow pattern. Pigmentation is in the thin furrows (arrows) with globules (boxes) in the thicker ridges (stars).

HOMOGENEOUS BLUE PIGMENTATION ■





Structureless blue color usually in the absence of local criteria such as pigment network, dots, or globules (Figure 1-3) Many variations of homogeneous blue color with or without the white color of scarring usually represent a blue nevus The history is important because there is a differential diagnosis which could include ■ A lesion as banal as a radiation tattoo ■ One more ominous such as nodular or cutaneous metastatic melanoma

PARALLEL PATTERNS/ACRAL PATTERNS/VOLAR SURFACES ON GLABROUS NON HAIR BEARING SKIN ■



Furrows (also referred to as fissures) and ridges on the skin of the palms and soles ■ Dermatoglyphics are the skin markings that can form whorls, loops, and arches Can create parallel patterns

Thick fibrillar variant with thick oblique lines Thin filamentous variant with fine thin lines Pressure can change the lattice-like pattern into a fibrillar pattern Irregular thick and pigmented line segments could be seen in an acral melanoma ■ ■





GLOBULAR PATTERN (BENIGN) ■

Brown globules without a parallel component

RETICULAR PATTERN (BENIGN) ■

A lesion with only pigment network

HOMOGENEOUS PATTERN (BENIGN) ■ ■

Brown homogeneous color Absence of local criteria (ie, pigment network, globules)

PARALLEL FURROW PATTERN (BENIGN PATTERN)

Single thin or thick brown parallel lines in the furrows of the skin (crista superficialis limitans) ■ Variations include two brown lines on both sides of the hypopigmented furrows with or without dots and globules ■ Single line of dots and globules along the furrows (single-dotted variant) Double line of dots and globules parallel to the hypopigmented furrows (double-dotted variant) (Figure 1-4) ■

LATTICE-LIKE PATTERN (BENIGN PATTERN) ■ ■

Brown parallel lines in the furrows Brown lines running perpendicular to the furrows forming a ladder-like picture (Figure 1-5)

FIBRILLAR PATTERN (BENIGN PATTERN) ■

Uniform brown lines that run in an oblique (///////) direction

Figure 1-5 Acral Nevus. Brown lines in the furrows (black arrows) and perpendicular to the furrows (yellow arrows) characterize the lattice-like pattern. Pressure on the foot can change this into the fibrillar pattern with fine oblique (/////) lines.

Chapter 1

Figure 1-6 Acral Melanoma. The parallel ridge pattern diagnoses this acral melanoma with pigmentation in the thicker ridges (black arrows). The thin white lines are the furrows (yellow arrows).

Dermoscopy from A to Z

7

Figure 1-8 Acquired Nevus. There is an increased incidence of acral melanoma in darker skinned races. This nevus on the palm of an AfricanAmerican was without change and demonstrates the benign parallel ridge pattern. Pigmentation is seen in the ridges of the nevus (yellow arrows) and in the ridges of the entire palm (white arrows).

PARALLEL RIDGE PATTERN (IN SITU AND EARLY INVASIVE ACRAL MELANOMA) ■





Pigmentation is in the thicker ridges of the skin (crista profunda intermedia) (Figure 1-6) Sometimes there are monomorphous round white structures in the ridges that represent the acrosyringia of the sweat ducts that are said to look like a “string of pearls” ■ They are an important clue to help identify the ridges ■ They are never found in the furrows, always in the ridges Parallel ridge pattern is not 100% diagnostic for melanoma and can also be created by blood (black heel, talon noir) (Figure 1-7)



Parallel ridge pattern can be found on normal skin in darker skinned races (Figure 1-8)

DIFFUSE VARIEGATE PATTERN (MELANOMA) ■ ■

Pigmented blotches Black, brown, or gray

MULTICOMPONENT PATTERN (MELANOMA) ■ ■

Filled with regular and irregular criteria Multiple colors plus areas with several acral patterns (parallel furrow, lattice-like, fibrillar)

NONSPECIFIC PATTERN (MELANOMA) ■

If one cannot determine any of the above benign or malignant patterns; this represents a red flag for concern

PEARLS ■ ■







■ ■

There can be exceptions to every dermoscopic rule The history and clinical appearance of the lesion are important and should not be ignored If a pigmented lesion on the palms or soles is rapidly changing yet has a typical benign pattern, it still could be melanoma A supposedly benign acral pattern with irregularity to the components could be high risk The presence of blood at acral sites (palms, soles, nails) may or may not be associated with melanoma Look carefully for other important criteria If in doubt, cut it out!

Seborrheic Keratosis MILIA-LIKE CYSTS Figure 1-7 Acral Hemorrhage. The parallel ridge pattern created by



blood (white arrows).



Variously sized white or yellow structures Small or large, single or multiple

8





DERMOSCOPY: AN ILLUSTRATED SELF-ASSESSMENT GUIDE

They can appear opaque or bright like “stars in the sky” (epidermal horn cysts) Melanocytic lesions can have a few milia-like cysts

PSEUDOFOLLICULAR OPENINGS/COMEDO-LIKE OPENINGS ■ ■ ■









Sharply demarcated roundish structures Pigmented or nonpigmented Shape and size can vary, not only within a single lesion, but from lesion to lesion in an individual patient (Figure 1-9) Large keratin-filled irregularly-shaped openings are called crypts When pigmented, they can be brownish-yellow or even dark brown and black (keratin-filled invaginations of the epidermis) Pigmented pseudofollicular openings can be hard to differentiate from the pigmented dots and globules of a melanocytic lesion A few pseudofollicular openings can be found in melanocytic lesions

Figure 1-10 Seborrheic Keratosis. A striking brain-like pattern created by pigmented fissures (yellow arrows) and light ridges (black arrows). Many of the ridges look like “fat fingers.”

FISSURES AND RIDGES ■

Fissures (sulci) and ridges (gyri) seen in papillomatous seborrheic keratosis can create several patterns ■ Cerebriform or brain-like in which they resemble a sagittal section through the cerebral cortex ■ Mountain-like with variously sized or uniformly roundish structures representing mountains (ridges) and fine pigmented lines representing valleys (fissures) ■ Possible to confuse the mountain and valley pattern with the cobblestone pattern of a melanocytic lesion ■ Pigmented fissures should not be confused with the pigment network of a melanocytic lesion ■ Hypo- and hyperpigmented ridges can be digit-like (straight, kinked, circular, or branched) and are referred to as “fat fingers”



All of these patterns are commonly seen in this ubiquitous benign skin lesion (Figure 1-10)

FINGERPRINT PATTERN ■







Brown fine/thin parallel line segments that resemble fingerprints Differ from the pigment network where the line segments are honeycomb-like or reticular Fingerprint pattern can be seen in flat seborrheic keratosis or in solar lentigines Some authors believe that solar lentigines are flat seborrheic keratosis

HAIRPIN VESSELS ■

Elongated vessels (capillary loops) resembling hairpins (Figure 1-11)

Figure 1-9 Seborrheic Keratosis. Sharp borders (red arrows), milia-like cysts (black arrows), and pseudofollicular openings (boxes) characterize this seborrheic keratosis.

Figure 1-11 Seborrheic Keratosis. An especially well-formed hairpin vessel (box) in a seborrheic keratosis.

Chapter 1

■ ■



Dermoscopy from A to Z

9

May or may not be surrounded by hypopigmented halos Light halo indicates a keratinizing tumor and may be found in keratoacanthomas Irregular and thick hairpin vessels can be seen in melanoma

MOTH-EATEN BORDERS ■ ■

Flat or slightly raised brown seborrheic keratoses Well-demarcated, concave borders that are felt to resemble a “moth-eaten” garment

SHARP DEMARCATION ■



The majority of seborrheic keratoses have sharp, well-demarcated borders Not always indicative of melanoma in a pigmented lesion (Figure 1-9)

Basal Cell Carcinoma ABSENCE OF THE CRITERIA SEEN IN A MELANOCYTIC LESION ■ ■

Specifically, absence of a pigment network Pigmented dots and/or globules found in some basal cell carcinomas ■ Raises the issue of dermoscopic differential diagnosis of individual criterion

Figure 1-12 Basal Cell Carcinoma. This pigmented basal cell carcinoma has classic arborizing vessels (black arrows), linear vessels (yellow box), gray blotches (black boxes), blue globules (yellow arrows), and fine gray dots (circles). The three different presentations of pigmentation point out to how variable this criterion can be. (Reproduced, with permission, from Journal of Drugs in Dermatology. New Methods and Technologies. Sep 2007-Vol 6-Issue 9. Fig 2b.)

ARBORIZING VESSELS ■



One of the most sensitive and specific vascular structures seen with dermoscopy ■ Red tree-like branching telangiectatic blood vessels ■ In focus vessels because they are on the surface of the lesion ■ Out-of-focus arborizing vessels should raise a red flag for concern that the lesion might be a melanoma ■ Can be thick or thin ■ Most often there are different caliber vessels in a single lesion Can also be found in ■ Benign nevi ■ Recurrent nevi ■ Sebaceous gland hyperplasia ■ Scars ■ On sun-damaged skin ■ Melanoma

■ ■ ■

Blue Red White

ULCERATION ■



Single or multiple areas, where there is loss of epidermis with oozing blood or congealed blood and crusts (Figure 1-13) There should be no recent history of trauma

SPOKE-WHEEL STRUCTURES ■

To date, spoke-wheel structures are the only criterion with dermoscopy, which are 100% diagnostic ■ Can be found in up to 10% of basal cell carcinomas ■ May or may not be associated with the other criteria used to make the diagnosis

PIGMENTATION ■



Basal cell carcinoma may or may not contain pigment (pigmented nests or island of basal cell carcinoma in the dermis) that can range from ■ Fine dots to large leaf-like structures (bulbous extensions forming a leaf-like pattern) ■ Not necessary to try to determine if “leaf-like” structures (maple leaf-like areas) are present since in reality this is a difficult task ■ Blue-gray ovoid nets ■ Multiple blue-gray globules Colors that can be seen (Figure 1-12) ■ Black ■ Brown ■ Gray

Figure 1-13 Basal Cell Carcinoma. Arborizing vessels (black arrows) and ulceration (yellow arrows) characterize this nonpigmented basal cell carcinoma.

10







DERMOSCOPY: AN ILLUSTRATED SELF-ASSESSMENT GUIDE

Well-defined pigmented radial projections meeting at a darker central globule/central axle/hub Complete or incomplete variations of this structure can be seen and one often has to use their imagination to make the identification Finding spoke-wheel structures might be the only clue to the correct diagnosis

PEARLS ■

A nonhealing area in an adult that bleeds spontaneously is a basal cell carcinoma until proven otherwise

Dermatofibroma CENTRAL WHITE PATCH ■





■ ■

Most typical presentation of this criterion is ■ Centrally located ■ Scar-like ■ Bony or milky-white ■ Homogeneous area (scarring in this fibrohistiocytic tumor) Several variations such as white network-like structures (white network, negative pigment network, reticular depigmentation), which can also be seen in Spitz nevi and melanoma Telangiectatic vessels with different shapes can also be found anywhere in the lesion Not all dermatofibromas have a central white patch The clinically firm feel and “dimple” sign should be used to help make the diagnosis

Figure 1-15 Atypical Dermatofibroma. Regressive melanoma is in the dermoscopic differential diagnoses of this atypical dermatofibroma. There is asymmetry of color and structure, the multicomponent global pattern, irregular pigment network (box), irregular globules (red arrows), and irregular blotches (yellow arrows). This warrants a histopathologic diagnosis.



PIGMENT NETWORK ■

Dermatofibromas are one of the nonmelanocytic lesions that can have a pigment network; solar lentigines are another ■ In most cases, a fine peripheral pigment network with thin brown lines is seen ■ Ring-like structures that are a variation of a hyperpigmented network (Figure 1-14) ■ Not all dermatofibromas have a pigment network

Figure 1-14 Dermatofibroma. A classic central white patch (black arrow) and pigment network (black boxes) characterize this dermatofibroma. In this instance, ring-like structures (white arrows) make up the pigment network.

Atypical dermatofibromas with the following features are melanoma mimics that warrant a histopathologic diagnosis ■ Irregular pigment network ■ Irregular dots/globules/blotches ■ Pink color ■ Irregular regression-like white color ■ Polymorphous (different shapes) vascular structures (Figure 1-15)

Vascular Lesions LACUNAE/LAGOONS/SACCULES ■

Sharply demarcated bright red to bluish round or oval structures (dialated vascular spaces in the dermis) (Figure 1-16)

Figure 1-16 Hemangioma. Well-demarcated dark red lacunae (arrows) and blue-white color (stars) representing fibrous septa characterize this classic hemangioma.

Chapter 1





Figure 1-17 Cutaneous Metastatic Melanoma. This is one of many generalized cutaneous metastatic lesions in a 27-year-old white male with a history of a 7mm melanoma on his back. There are well-demarcated lacunae-like areas (arrows) and atypical vessels (boxes). A collision lesion, hemangioma, and amelanotic melanoma are in the dermoscopic differential diagnosis.



Step Two: Analysis of a Melanocytic Lesion Pattern Analysis Defined ■ Identify as many criteria in the lesion as possible and see if they fit into the known patterns associated with the variants of ■ Melanocytic Nevi ■ Congenital ■ Acquired ■ Recurrent ■ Halo ■ Combined ■ Blue ■ Dysplastic ■ Spitz

Melanoma ■ In situ ■ Superficial spreading ■ Nodular ■ Amelanotic ■ Lentigo maligna/Lentigo maligna melanoma ■ Nail apparatus ■ Acral ■ Mucosal Even though pattern analysis is considered a melanocytic algorithm, the same principles are used to diagnose all of the lesions that can be identified with the technique ■ Melanocytic ■ Nonmelanocytic ■ Benign ■ Malignant ■ Inflammatory

Step 1





11

Pattern Analysis Method ■

Different shades of red and blue colors can be seen in a single hemangioma ■ Lacunae should not be mistaken for the milky-red color seen in pigmented and amelanotic melanoma, which can have “out-of-focus” reddish globular-like structures ■ Black homogeneous structureless areas represent thrombosis ■ Significant scale or dryness (hyperkeratosis) can be seen in angiokeratomas Patchy white color or bluish-white color (fibrous septa) are commonly seen in hemangiomas ■ It should not be mistaken for the scar-like white color of regression or the blue color that can be found in melanomas Cutaneous metastatic melanoma and hemagioma-like melanoma can be indistinguishable from a hemangioma ■ A history of a previous melanoma will help make the diagnosis (Figure 1-17)

Dermoscopy from A to Z

■ ■







Determine symmetry or asymmetry of color and/or structure using the mirror image technique ■ Contour of the lesion is not important with this algorithm ■ The lesion is bisected by two lines that are placed 90° to each other ■ One does not physically place lines over the lesion. The lines are visually imagined ■ The lines should be placed to create as much symmetry as possible ■ Are the color and/or the structure on the left half of the lesion a mirror image of the right half Repeat the analysis for the upper and lower half of the lesion Perfect symmetry of color and/or structure is not often found in nature, and interobserver agreement may not be good with this assessment even among experienced dermoscopists Symmetry or asymmetry can also be determined along any axis through the center of the lesion (Menzies method) Significant asymmetry of color and/or structure is a very important clue that you might be dealing with high risk pathology Raise a red flag for concern and proceed with focused attention to what else you might find

Step 2 ■ ■

Determine the global/overall pattern of the lesion The predominant criteria seen throughout the lesion could be ■ Reticular ■ Globular ■ Cobblestone ■ Homogeneous ■ Parallel

12

DERMOSCOPY: AN ILLUSTRATED SELF-ASSESSMENT GUIDE

Starburst Multicomponent ■ Nonspecific There can be combinations of criteria in a single lesion such as reticular and homogeneous or reticular and globular ■ The “reticular homogeneous pattern” or “reticular globular pattern” ■









Step 3 ■



Identify the local criteria in the lesion ■ Pigment network ■ Dots and globules ■ Streaks (also called pseudopods and radial streaming) ■ Blotches ■ Blue-white veil ■ Regression ■ Colors ■ Vascular structures

Global Patterns Reticular ■





Determine if the criteria seen are ■ Regular or irregular ■ Good or bad ■ Low or high risk Melanoma-specific criteria are defined as criteria that can be seen in benign and malignant lesions but are more specific for high risk pathology such as ■ Dysplastic nevi ■ Spitzoid lesions ■ Melanoma All of the high risk criteria can be seen in benign pathology, and one should never tell patients that they have melanoma 100%

Table 1-6

Pigment network filling most of the lesion

Globular ■

Dots and globules filling most of the lesion

Cobblestone ■

Step 4 ■

Due to the different characteristics of the skin in these locations the criteria are different on ■ Trunk and extremities ■ Face, nose, and ears ■ Palms, soles, nail apparatus, and mucosa Thinner skin on the face, nose, ears versus the trunk and extremities, and thicker skin on the palms and soles with fissures and ridges The criteria found on the face, nose, ears, palms, soles, nails and mucosa are referred to as site-specific criteria (Table 1-6)

Larger angulated globules resembling street cobblestones filling most of the lesion (Figure 1-18)

Homogeneous ■

Diffuse pigmentation in the absence of local criteria such as pigment network, dots, and globules

Starburst (Spitzoid) ■

Streaks and/or dots and globules at the periphery of the lesion

Multicomponent ■ ■

Three or more different areas within a lesion Each zone can be composed of a single criterion or multiple criteria

Nonspecific ■

None of the above global patterns can be identified

MELANOMA-SPECIFIC CRITERIA IN DIFFERENT BODY REGIONS

TRUNK AND EXTREMITITES

FACE, NOSE, AND EARS

PALM AND SOLES

NAIL APPARATUS

GLOBAL CRITERIA Asymmetry of color and/ or structure Multicomponent pattern Nonspecific pattern LOCAL CRITERIA Irregular pigment network Irregular dots and globules Irregular streaks (pseudopods/ radial streaming) Irregular blotches Blue-white color Regression 5 or 6 colors Polymorphous vascular pattern

Asymmetrical pigmentation around follicular openings Annular- granular structures Rhomboid structures Circle within a circle Dark homogeneous areas Absence of the fingerprint pattern

Parallel ridge pattern Diffuse variegate pigmentation Multicomponent pattern Nonspecific pattern

Diffuse irregular pigmentation Irregular pigmented bands/melanonychia striata Loss of parallelism of pigmented bands Hutchinson and micro-Hutchinson sign

Chapter 1

Dermoscopy from A to Z

13

Regular Streaks ■ ■

■ ■





Figure 1-18 Acquired Nevus. Small dots and globules (boxes) and larger angulated globules (arrows) characterize this benign nevus. The mountain and valley pattern seen in seborrheic keratosis is in the dermoscopic differential diagnosis. A positive wobble sign in which the soft nevus moves from side to side with movement of instrumentation versus a stiff immoveable seborrheic keratosis helps to make the differentiation.

Black or brown linear projections of pigment Can be free standing or associated with a pigment network or dark blotches At all points along the periphery of the lesion Pseudopods and radial streaming are similar structures dermoscopically and histopathologically (aggregates of tumor cells running parallel to the epidermis that can be seen in Spitz nevi or represent the radial growth phase of melanoma), which are difficult to differentiate from each another To simplify the identification, the term “streaks” is now used by many but not all experienced dermoscopists to encompass all variations of this criterion The shape of the linear projections does not determine if they are regular or irregular, rather their distribution at the periphery of the lesion

Irregular Streaks ■ ■ ■

Black or brown linear projections Irregularly distributed at the periphery of a lesion Some but not all points at the periphery, “foci of streaks”

Regular Blotches ■

Local Criteria Regular Pigment Network ■ ■



Various shades of brown Honeycomb-like (web-like, reticular) line segments (elongated hyperpigmented rete ridges) Uniform color, thickness, and holes (dermal papilla)

Irregular Pigment Network ■ ■



Shades of black or brown Line segments that are thickened, branched, and broken up (enlarged, fused rete ridges) There may be a diffuse distribution or foci of irregular pigment network

Regular Dots and Globules ■









Brown roundish structures ■ Usually clustered ■ Dots (0.1mm) are smaller than globules (greater than 0.1mm) ■ Size, shape, and color are similar with an even distribution in the lesion (nest of melanocytes at the dermoepidermo junction) Dots and/or globules only found at the periphery can be seen in dysplastic, Spitz, or actively changing nevi Actively changing means if followed digitally the nevus will invariably enlarge over time Peripheral dots and globules are usually seen in younger patients with benign pathology Beware of this pattern in an adult

Irregular Dots and Globules ■

Black, brown, gray, or red roundish structures ■ Different sizes, shapes, and shades of color ■ Usually but not always asymmetrically located in the lesion



■ ■

Dark shades of black, brown or gray Structureless (ie, absence of network, dots, or globules) areas of color Bigger than dots and globules Uniform shape and color symmetrically located in the lesion (aggregates of melanin in the epidermis and/or dermis)

Irregular Blotches ■



Dark shades of black, brown or gray structureless areas of color Irregular in size and shape asymmetrically located in the lesion

Blue-White Veil ■ ■

Irregular, structureless area of confluent blue color Overlying whitish ground glass appearance ■ Orthokeratosis ■ Acanthosis ■ Hypergranulosis ■ Can represent heavily pigmented tumor cells in the dermis

Regression ■ ■







Bony or milky-white scar-like depigmentation (fibrosis) With or without gray pepper-like granules (free melanin and/or melanophages in the dermis) The white color should be lighter than the surrounding skin ■ The bony-white color should be differentiated from tan hypopigmentation commonly found in melanocytic lesions ■ Blue color can also be seen in areas of regression Regression by itself is an independently potentially high risk criterion The more the regression seen, the greater the chance the lesion is a melanoma

14

DERMOSCOPY: AN ILLUSTRATED SELF-ASSESSMENT GUIDE

Blue-White Color ■

■ ■

It is not always possible to identify classic regression or blue-white veil Blue and/or white colors of any intensity, shape or distribution A red flag for concern should be raised that the lesion could be a melanoma

1

Hypopigmentation ■



2

3

Commonly seen featureless areas of light brown color in all types of melanocytic lesions both benign and malignant Inexperienced dermoscopists can have trouble differentiating hypopigmentation from the bony-white color seen with true regression

Colors Seen with Dermoscopy ■













■ ■

Eumelanin’s location in the skin will determine the colors one sees with dermoscopy In a flat or slightly raised lesion, black indicates that melanin is superficially located in the epidermis Black is not always an ominous color but can be seen in benign pathology as well as in invasive melanoma Light and dark brown indicates pigment is at the dermoepidermal junction Gray in the papillary dermis represents free melanin and/or melanophages (“peppering”) As the pigment gets into the deeper dermis it looks blue (the Tyndall effect) Red and/or pink colors can be created by inflammation or neovascularization Sebaceous material and hyperkeratosis can look yellow The more colors seen, the greater the chance that one is dealing with high risk pathology (Figures 1-19, 1-20, 1-21)

Figure 1-20 Melanoma. This is a melanocytic lesion because there are globules (circle). There is asymmetry of color and structure (+) plus the multicomponent global pattern (1,2,3). Local criteria include: irregular dots and globules (circle), blue-white color (stars), and peppering (boxes). The classic blue-white veil is not seen. More than five colors including red are another melanoma-specific criterion. (Reproduced, with permission, from Journal of Drugs in Dermatology. New Methods and Technologies. Sep 2007- Vol 6-Issue 9. Fig 3b.)

Atypical Vascular Pattern/Polymorphous Vascular Pattern ■

Vessels that can be seen in melanoma are nonspecific; they can also be commonly found in other lesions, including ■ Benign ■ Malignant ■ Inflammatory

3

2 1

4

Figure 1-19 Melanoma. This is a melanocytic lesion because there is a pigment network (red arrows) and globules (circles). There is asymmetry of color and structure (+) plus the multicomponent global pattern (1,2,3,4). Local criteria include: irregular pigment network (red arrows), irregular dots and globules (circles), irregular dark blotches (black arrows), and blue-white color (stars). The classic blue-white veil is not seen. Peppering (yellow box) and gray blotches (yellow arrows) are part of the regression. More than five colors are seen including red. (Reproduced, with permission, from Journal of Drugs in Dermatology. New Methods and Technologies. May 2008-Vol 7-Issue 5. Fig 4b.)

Figure 1-21 Melanoma. This is a melanocytic lesion because there are globules (circles). There is an atypical starburst (Spitzoid) global pattern with foci of streaks at the periphery (boxes). Local criteria include: irregular dots and globules (circles), irregular streaks (boxes), and regression. The white and gray colors (yellow arrows) make up the regression. The black arrows point out where there are no streaks. Five colors including red round off the melanoma-specific criteria.

Chapter 1





When identified, they should raise a red flag for concern including ■ Dotted/pinpoint ■ Irregular linear ■ Irregular torturous/corkscrew (irregular, thick, coiled) ■ Irregular hairpin (irregular and thick hairpin-shaped) ■ Glomerular One must focus their attention to make out the shapes of these small vessels





■ ■



Minimal pressure with polarizing instrumentation and the use of ultrasound gel, hand cleaning gel or other fluid is the best technique to visualize small vessels

Milky-Red Areas ■





Localized or diffuse (amelanotic melanoma) pinkish-white color With or without reddish and/or bluish out-of-focus/ fuzzy globular structures (neovasculariztion) Not to be confused with the in focus lacunae seen in hemangiomas Diffuse or clustered fine coiled vessels that can be seen in ■ Bowen disease (Figure 1-22) ■ Melanoma ■ Pink lichen planus-like keratosis ■ Stasis and nonspecific dermatitis ■ Psoriasis

Asymmetrical Pigmentation Around Follicular Openings ■ ■

Seen only on the face, nose, and ears (“site-specific”) Irregular brown color outlining parts of the round follicular openings

The color does not completely encircle the openings (early proliferation of atypical melanocytes) A melanoma-specific criterion, used to diagnose lentigo maligna or lentigo maligna melanoma Seen only on the face, nose, and ears (“site-specific”) Brown or gray fine dots that surround follicular openings (melanophages and/or atypical melanocytes) This criterion and asymmetrical follicular pigmentation can be seen in ■ Lentigo maligna, lentigo maligna melanoma ■ Pigmented actinic keratosis ■ Posttraumatic ■ Late-stage lichen planus-like keratosis (Figures 1-23, 1-24) ■ Melasma

Rhomboid Structures ■ ■ ■

■ ■

Glomerular Vessels ■

15

Annular-Granular Pattern/Structures

PEARL ■

Dermoscopy from A to Z



Seen only on the face, nose, and ears (“site-specific”) Rhomboid is a parallelogram with unequal angles and sides Black, brown, or gray thickening around the follicular openings In reality true rhomboids are not regularly formed Any pigmented thickening around follicular openings is worrisome A melanoma-specific criterion used to diagnose lentigo maligna or lentigo maligna melanoma

Dark Homogeneous Areas ■ ■ ■ ■

On the face, nose, and ears Irregular in size and shape Black or brown homogeneous blotches of color Complete occlusion and disappearance of follicular openings due to invasive melanoma (lentigo maligna melanoma)

Figure 1-23 Lichen Planus-like Keratosis. Remnants of a fingerprint Figure 1-22 Bowen Disease. Typical glomerular vessels (black box) and large dotted vessels (yellow box) help diagnose this nonspecific pink scaly patch. (Reproduced, with permission, from Journal of Drugs in Dermatology. New Methods and Technologies. May 2008-Vol 7-Issue 5. Fig 2b.)

pattern (yellow boxes) of a flat seborrheic keratosis or solar lentigo and the gray annular-granular pattern (black boxes) around follicular openings (arrows) are the clues that this is not lentigo maligna. The gray dots represent melanophages and free melanin in the papillary dermis, not atypical melanocytes. A sub-set of lichen planus-like keratosis is thought to represent an immunologic event against flat seborrheic keratosis of solar lentigines.

16

DERMOSCOPY: AN ILLUSTRATED SELF-ASSESSMENT GUIDE

Figure 1-24 Lentigo Maligna (ear lobe). This case demonstrates varia-

Figure 1-25 Acrolentiginous/Acral Melanoma/Nail-Apparatus

tions of the classic criteria. The lesion is suspicious clinically but has a differential diagnosis that includes a seborrheic keratosis. The dermoscopic criteria for a seborrheic keratosis are not present. There is asymmetry of color and structure, asymmetrical pigmentation (black arrows) around follicular openings (red arrows), annular-granular structures (circles), and irregular dark blotches (boxes). One should have a mental checklist of the melanoma-specific criteria for this diagnosis because they are not always easy to find and identify.

Melanoma. The pigmented bands are not uniform in color and thickness (black arrows). Broken up line segments (box) represent loss of parallelism. There is also Hutchinson sign (yellow arrows).

Micro-Hutchinson Sign: Hutchinson Sign ■

PEARLS ■





Actinic keratosis and actinic/solar lentigines can be associated with lentigo maligna/lentigo maligna melanoma Use the areas where the high risk/melanoma-specific criteria are to perform an incisional biopsy, or the malignant diagnosis could be missed If you think the lesion is lentigo maligna yet the pathology report does not make the diagnosis, seek another histopathologic opinion or biopsy another area of the lesion



Nonmelanocytic Nail-Apparatus Bands ■

Benign Pigmented Nail Bands (Melanonychia Striata) ■

■ ■

Single or multiple nail involvement with brown longitudinal parallel lines Uniform color, spacing, and thickness A single band in a lighter skinned person with these findings is still worrisome

Malignant Pigmented Nail Bands (Atypical Melanonychia Striata) ■







Loss of parallelism (broken-up line segments) with brown, black, or gray parallel lines that demonstrate different shades of color, irregular spacing, and thickness (Figure 1-25) High risk dermoscopic criteria at this location in adults are usually not associated with high risk pathology when seen in children ■ Disfiguring nail-matrix biopsies can usually be avoided in children Any rapidly changing scenario warrants a histopathologic diagnosis no matter how old or young the patient Digital monitoring is helpful to monitor pigmentation in the nail apparatus to document stability or change over time

Pigmentation of the cuticle that can only be seen clearly with dermoscopy (micro-Hutchinson sign) Pigmentation of the cuticle easily seen without dermoscopy (Hutchinson sign) ■ Both are often associated with nail-apparatus melanoma yet not diagnostic (pseudo-Hutchinson sign) Uniform grayish bands can be seen with ■ Lentigo ■ Post-inflammatory ■ Post-traumatic ■ Drug-induced pigmentation multiple nails involved ■ Darker skinned races multiple nails involved

Nail-Apparatus Blood/Subungual Hematoma ■





The color of blood seen in the nail apparatus depends on how long the blood has been there (breakdown of heme) ■ Fresh blood looks red or purple/violaceous ■ Older blood can look yellowish brown or black A well-demarcated homogeneous area with parallel lines at the distal edge and/or globule-like blood spots/pebbles (Figure 1-26) Digital dermoscopy is helpful to follow nail-apparatus blood that should slowly move distally over several months

PEARLS ■ ■



Presence of blood in a nail does not rule out melanoma Search carefully for high risk criteria that might also be present Finding the Hutchinson sign and the parallel ridge pattern on the surrounding skin adjacent to the nail can help make the diagnosis of nail apparatus melanoma

Chapter 1

Dermoscopy from A to Z

17

Figure 1-26 Sub-ungual Hemantoma. Different colors plus blood pebbles (boxes) characterize this post-traumatic lesion. The white color (star) is secondary to trauma not regression. The brown (red arrows) and purple blotches (white arrows) result from the breakdown of blood/heme. No melanoma-specific criteria are seen.

Figure 1-27 Congenital Melanocytic Nevus. Terminal hairs with perifollicular hypopigmentation (boxes), irregular pigment network (circles), and regular dots (arrows) characterize this small banal congenital melanocytic nevus.

Longitudinal Erythronychia



■ ■

■ ■



An inconsistent finding in single or multiple finger nails Single or multiple longitudinal pinkish blanching bands of variable thickness (normal vasculature) Multiple nail involvement is a benign sign Single nail involvement can be seen in amelanotic nailapparatus melanoma, Bowen disease and onychomatricoma Associated with inflammatory skin diseases (ie, lichen planus, Darier’s disease)

Common Dermoscopic Patterns Congenital Nevi ■ ■











Diffuse homogeneous brown color Patchy or diffuse pigment network (target network may or may not be seen as network holes each with a small centrally located brown dot or pinpoint vessel) Globular and/or cobblestone pattern (target globules may or may not be seen as globules with a smaller centrally located brown dot or vessel) Islands of normal skin and islands of criteria such as pigment network, dots and globules Multicomponent pattern with three or more distinct areas of criteria Dark coarse terminal hairs (hypertrichosis) with or without surrounding hypopigmentation (perifollicular hypopigmentation) (Figure 1-27) Milia-like cysts and pseudofollicular openings, most often found in seborrhic keratosis, can be seen

Acquired Nevi ■ ■ ■ ■

Light/dark brown or pink color Regular pigment network Lacks sharp demarcation at the borders Globular or cobblestone pattern (the most common pattern seen in children)

■ ■ ■ ■

Symmetry of color and structure Comma-shaped blood vessels Hypopigmentation Milia-like cysts and pseudofollicular openings can be seen Pink nevi can be featureless (an absence of local criteria) or feature-poor (a few poorly developed local criteria) ■ A solitary flat pink lesion, with different shades of pink color, is more worrisome than multiple soft and compressible pink lesions

PEARL ■

Dermoscopy might not be helpful to diagnose pink macules and papules, which can be melanocytic, nonmelanocytic, benign, malignant, or inflammatory (Figure 1-28)

Blue Nevi ■



■ ■



Blue, blue-gray, or blue-black homogeneous color (Figure 1-3) Variable number of subtle blue globular-like structures may or may not be present Regression with white or gray areas commonly seen Radiation tattoo, nodular and cutaneous metastatic melanoma are in the dermoscopic differential diagnosis The history is essential to make the diagnosis

Combined Nevi ■

■ ■ ■

Light or dark brown homogeneous color +/− other local criteria (regular nevus) and a blue blotch (blue nevus) with a “fried egg” clinical appearance Diffuse brown homogeneous color with a blue border Diffuse blue homogeneous color with brown border Variable combinations of blue and brown colors

Recurrent Nevi/Pseudomelanoma ■ ■

Sharp border Irregular pigment network; irregular streaks

18

DERMOSCOPY: AN ILLUSTRATED SELF-ASSESSMENT GUIDE

Figure 1-28 Pink Lichen Planus-like Keratosis. This small papule was

Figure 1-30 Dysplastic Nevus. There are foci of irregular dots and glob-

only found after a complete skin examination. There are different shades of pink color, pinpoint (boxes), and comma-shaped vessels (yellow arrows) plus a milky-red area (black arrow). (Reproduced, with permission, from Journal of Drugs in Dermatology. New Methods and Technologies. Sep 2007- Vol 6-Issue 9. Fig 4b.)

ules (boxes), irregular dark blotches (black arrows), and multifocal hypopigmentation (red arrows).

■ ■ ■ ■ ■

Irregular dots and globules White scar-like areas with linear and/or arborizing vessels Any combination of criteria can be seen Pigmentation centrally located within the scar ■ If the pigmentation goes beyond the borders of the scar, raise a red flag for concern that the lesion might be a melanoma ■ A history of previous surgery and histopathology is important to help rule out melanoma ■ If possible, review the pathology from the lesion that was excised (Figure 1-29)

■ ■ ■ ■ ■ ■ ■

■ ■

Dysplastic Nevi ■

ABCDE clinical lesions can be indistinguishable from melanoma and look banal or high risk with dermoscopy



E for evolving/changing might be the only clue that a lesion is high risk (ie, dysplastic nevus, melanoma) Asymmetry of color and structure A multicomponent global pattern Irregular pigment network Irregular dots and globules Irregular blotches Multifocal hypopigmentation (Figure 1-30) Regression, blue-white color/blue-white veil, atypical vessels, and streaks are not usually seen Atypical pattern similar to superficial spreading melanoma Patients with multiple dysplastic nevi clinically, usually do not have many that look very atypical with dermoscopy Pink dysplastic nevi can be feature-poor or featureless with low or high-grade histopathology

PEARLS ■ ■

“Anything pink, stop and think!” Look for the clinical and/or dermoscopic “ugly duckling” to consider for biopsy or digital follow-up

Spitz Nevi

1



3

2





Figure 1-29 Recurrent Nevus. Asymmetry of color and structure (+), the multicomponent global pattern (1,2,3) irregular globules (boxes), irregular dark blotch (yellow arrow), brown color (white arrows), scar tissue (stars) with linear vessels (black arrows) characterize this recurrent nevus.

There are six patterns seen in Spitz nevi ■ Starburst ■ Globular ■ Homogeneous ■ Pink ■ Black pigment network ■ Atypical Spitzoid is the term used when any of the different six patterns is seen Starburst is the most common pattern (Figure 1-31) ■ Streaks and/or dots and globules at the periphery ■ Light/dark brown, black, or blue color centrally ■ Regular or irregular pattern depends on the location of the streaks

Chapter 1



Dermoscopy from A to Z

19

A Spitzoid melanoma histopathologically does not always have a Spitzoid dermoscopic features

In Situ Melanoma (Trunk and Extremities) ■ ■

■ ■ ■ ■ ■ ■ ■ ■

Figure 1-31 Spitz Nevus. A central regular dark blotch (stars) plus globules (black arrows) and a few streaks (red arrows) at all points along the periphery characterize this classic symmetrical starburst/Spitzoid pattern.

Lentigo Maligna/Lentigo Maligna Melanoma (Face, Nose, Ears) ■ ■ ■

Regular starburst pattern has symmetrical streaks and/or dots and globules around the lesion ■ Irregular starburst pattern has foci of streaks and/or dots and globules at the periphery ■ Beware! Symmetrical and asymmetrical starburst patterns can be seen in melanoma Globular is the second most common Spitzoid pattern ■ Filled with regular or irregular dots and/or globules ■ Blue color seen centrally is a clue that the lesion might be a Spitz nevus Homogeneous pattern ■ Featureless brown color Pink pattern ■ Featureless pink papule ■ Can have small vessels Black network pattern ■ The lesion is composed totally of a prominent black pigment network ■ Ink-spot lentigo and melanoma are in the differential diagnosis Atypical pattern ■ This can have any combination of melanoma-specific criteria ■ The histopathologic diagnosis is usually a surprise White pigment network/white network/negative pigment network/reticular depigmentation can be seen within the lesion ■ This is an important clue that the lesion is Spitzoid ■ Can also be seen in dermatofibromas and in melanoma ■













PEARLS ■

■ ■

Most but not all Spitzoid patterns require a histopathologic diagnosis especially in adults The pink variant could actually be amelanotic melanoma A melanoma with a Spitzoid pattern dermoscopically is not always Spitzoid histopathologically

Can be associated with a dysplastic nevus or develop de novo May or may not demonstrate the clinical ABCD clinical criteria Flat or slightly raised lesion Asymmetry of color and structure Multicomponent global pattern Black and/or dark brown irregular pigment network Irregular dots and globules Irregular dark blotches Regression and/or hypopigmentation Lacks the criteria associated with deeper melanomas (pink, red, or blue color, atypical vessels) (Figure 1-32)



■ ■



Asymmetrical follicular pigmentation Annular-granular structures Rhomboid structures Circle within a circle ■ Gray color and/or gray granules (atypical melanocytes) surrounding the periphery of follicular units with the presence of a centrally located hair shaft Absence of the fingerprint pattern With invasion (lentigo maligna melanoma) one sees irregular black blotches with or without the disappearance/ destruction of follicular openings Melanoma-specific criteria as seen in other locations with more advanced disease

Superficial Spreading Melanoma ■ ■

Starts in an existing nevus or de novo Demonstrates the clinical ABCD criteria

3 1

2 3

Figure 1-32 In Situ Melanoma. This is a melanocytic lesion because there is a pigment network (black box) and globules (circles). There is asymmetry of color and structures (+), the multicomponent global pattern (1,2,3), irregular pigment network (black box), irregular dots and globules (circles), and reticular depigmentation (white box). The hypopigmentation (black stars) should not be confused with regression. There is diffuse erythema (red stars) and only three other colors. (Reproduced, with permission, from Journal of Drugs in Dermatology. New Methods and Technologies. May 2008-Vol 7-Issue 5. Fig 3b.)

20



DERMOSCOPY: AN ILLUSTRATED SELF-ASSESSMENT GUIDE

Contains a variable number of the melanoma-specific criteria found on the trunk and extremities (Figures 1-19, 1-20, 1-21)

Nodular Melanoma ■ ■ ■ ■

■ ■



■ ■



Starts in an existing nevus or de novo May or may not be fast growing Pigmented, hypomelanotic, or amelanotic Can be mistaken for benign nevus or squamous cell carcinoma Usually lacks the clinical ABCD criteria Due to the absence of the radial growth phase there is a scarcity of local criteria (pigment network, globules, streaks) Remnants of local criteria may or may not be present at the periphery of the lesion Large intense irregular dark blotches Multiple deeper skin colors seen such as blue, white, pink, milky-red Atypical vessels

PEARLS ■





The clinical appearance of a lesion (flat, palpable or nodular, presence or absence of the ABCD criteria) plus the colors and structures seen with dermoscopy can help estimate if you are dealing with a thin, intermediate or thick melanoma Flat melanomas are usually in situ or early invasive,with black and brown colors plus well-developed local criteria Thick melanomas tend to be elevated or nodular and can have a paucity or absence of local criteria such as pigment network, plus blue-white veil/color, multiple colors, and the atypical vascular pattern

Amelanotic Melanoma ■ ■ ■



■ ■



Flat, palpable, or nodular Hypopigmented, pink or red May or may not have the melanoma-specific criteria typically seen in pigmented melanomas Different shades of pink color and polymorphous vascular pattern Milky-red areas are important clues to the correct diagnosis Pediatric patients have a high proportion of amelanotic melanomas Amelanotic melanoma should always be in the differential diagnosis of a pyogenic granuloma (Figure 1-33)

Figure 1-33 Amelanotic Melanoma (face). This is a melanocytic lesion because it has globules (boxes). Histopathologically it was an amelanotic melanoma even though brown color is seen with dermoscopy. There is an absence of melanoma-specific criteria found on the face with different shades of pink and brown color plus ulceration (yellow arrows). Follicular openings (black arrows) should not be confused with the milia-like cysts of a seborrheic keratosis.

Featureless Melanoma ■

Melanoma Incognito/False Negative Melanoma ■







■ ■ ■

■ ■

Dermoscopy might not be as helpful to make the diagnosis as the history of a melanoma being previously excised Single or multiple Pigmented or nonpigmented All different sizes, shapes, and colors can be seen in each patient with or without atypical vessels Any combination of criteria can be seen Benign patterns such as a hemangioma-like cutaneous metastatic melanoma (Figure 1-17)

Feature-poor Melanoma ■

Melanoma with subtle nondiagnostic criteria (Figure 1-33)

Clinically and/or dermoscopically the lesion does not look like melanoma With dermoscopy there can be obvious or subtle clues to make the diagnosis Clues to help make the diagnosis ■ History of dermoscopic change over time ■ A Spitzoid pattern in a lesion that does not look Spitzoid clinically ■ Areas of regression as the major high risk criterion ■ Polymorphous vessels in a pink lesion The “little red riding hood sign” is when the lesion looks clinically benign from a distance but not when seen close up with dermoscopy

PEARL ■

Cutaneous Metastatic Melanoma ■

Melanoma without dermoscopic criteria at all

Dermoscopy should not only be used on clinically suspicious lesions if one wants to diagnose melanoma incognito

Nail-Apparatus Melanoma ■ ■ ■

■ ■ ■ ■ ■

Amelanotic diffuse reddish color/amelanotic tumor nodule Longitudinal erythronychia in a single nail Diffuse melanonychia with different shades of black, brown, or gray color Irregular pigmented bands (Figure 1-25) A single uniform band does not rule out melanoma Irregular dots and globules Blood identified in 25% of lesions Variable amounts of nail-plate destruction with advanced disease

Chapter 1

■ ■

+/− Hutchinson sign The parallel ridge pattern can be seen on the adjacent skin

Ink-spot Lentigo ■ ■ ■ ■

Black macule or macules on sun-exposed areas Prominent thick black pigment network Usually a very easy clinical and dermoscopic diagnosis Melanoma could be in the clinical and dermoscopic differential diagnosis ■ Look for melanoma-specific criteria (ie, blue-white color, irregular blotches) that should not be present in an ink-spot lentigo

Solar Lentigo/Actinic Lentigo ■ ■ ■ ■

Macules and/or patches Different shades of homogeneous brown color Sharply demarcated moth-eaten concave borders Fingerprint pattern with straight and/or wavy linear line segments



Nonpigmented actinic keratosis ■ Scaly surface ■ Pinkish-red pseudopigment network Pigmented actinic keratosis ■ Mimicks lentigo maligna ■ Asymmetrical follicular pigmentation ■ Annular-granular structures ■ Rhomboid structures

PEARL ■

Multiple scaly lesions favor the diagnosis of actinic keratosis over lentigo maligna. Both can have pigmented and nonpigmented variants.

21

PEARL ■

The yellowish clinical appearance and globules are the main features used to differentiate sebaceous gland hyperplasia from basal cell carcinoma.

Collision Tumor ■

■ ■



Actinic Keratosis ■

Dermoscopy from A to Z



Lesion with the dermoscopic criteria for two different pathologies ■ One can find a triple collision lesion with three different pathologies Collision tumors are commonly seen Diagnostic criteria can be side by side or one can be seen within the other Examples include ■ Seborrheic keratosis, basal cell carcinoma ■ Seborrheic keratosis, in situ or invasive squamous cell carcinoma ■ Seborrheic keratosis, amelanotic or pigmented melanoma ■ Seborrheic keratosis, eccrine porocarcinoma ■ Basal cell carcinoma, seborrheic keratosis, clear cell acanthoma Any combination is possible (Figure 1-34)

Other Diagnoses Made with Dermoscopy Scabies ■ Burrows appear as discrete yellowish scaly linear areas ■ Mites can be seen as a small triangle/gray delta structure that corresponds to the front section of the body with its mouth/biting apparatus and legs ■ Higher magnification and oil/fluid increases the visibility of the mite, stool, and eggs

Bowen Disease (In Situ Squamous Cell Carcinoma) ■ ■ ■



Pink or reddish scaly macules, papules, patches, plaques Pinpoint and/or glomerular vessels Clusters or diffuse distribution of vessels throughout the lesion With or without foci of pigmentation

Keratoacanthoma ■

■ ■ ■

A rapidly growing reddish scaly nodule of short duration on sun exposed areas Centrally located yellowish keratinous material Peripheral whitish background Hairpin vessels with or/without a white halo at the periphery

Sebaceous Gland Hyperplasia ■ ■ ■

Delled yellow papules seen clinically Multiple grouped white or yellow globules Small caliber basal cell carcinoma-like vessels ■ The vessels have been termed crown or wreath-like vessels ■ Supposedly never to reach the center of the lesion ■ This is a misnomer because in reality the vessels rarely meet this criterion and can be found anywhere in the lesion

Figure 1-34 Collison Tumor—Squamous Cell Carcinoma and Seborrheic Keratosis. A rapidly growing nodule (arrows) representing a squamous cell carcinoma and the mountain and valley pattern of a seborrheic keratosis (box) characterizes this lesion. The cobblestone pattern of a nevus is in the dermoscopic differential diagnosis.

22

DERMOSCOPY: AN ILLUSTRATED SELF-ASSESSMENT GUIDE

Pediculosis Capitis ■ Direct visualization of the parasite and nits ■ It is possible to see if the nits are full (vital nits) or empty, which helps determine the success or failure of treatment Pediculosis Pubis ■ It is possible to easily see the parasite attached to adjacent pubic hairs or hairs at other sites Lichen Planus ■ Peppering ■ Brown blotches ■ White reticular areas (Wickham striae) ■ Negative pigment network/white pigment network/reticular depigmentation is in the dermoscopic differential diagnosis of Wickham striae Warts ■ Red and/or black dots (thrombosed capillaries) ■ With or without a white halo Psoriasis ■ Red scaly plaque or plaques ■ Diffuse distribution of pinpoint and/or glomerular vessels identical to Bowen disease ■ Distribution of lesions will help differentiate Psoriasis from Bowen disease ■ Both can have single or multiple lesions

Scleroderma Pattern ■ The triad of ■ Rarefied capillaries (less than 6 loops per mm) ■ Thin loops ■ Megacapillaries ■ Pearly shining sclerosis “cotton balls” Dermatomyositis ■ Mega, twisted, branched loops, microhemorrhage Lupus Erythematosus ■ Considerable variation of loops, branching, twisted, megacapillaries, microhemorrhage Trichoscopy ■ The use of dermoscopy to evaluate scalp skin and hair follicles ■ Structures that can be visualized include ■ Hair shafts ■ Hair-follicle openings ■ Perifollicular epidermis ■ Cutaneous microvasculature ■ Higher magnifications (20-70 fold) with digital systems and fluid (70% ethanol) are preferred ■ Hand-held instrumentation with lower magnification and other fluids such as emersion oil or gels can also be used Genetic Hair-Shaft Abnormalities Monilethrix

Nail Folds ■ Normal capillary loops are hairpin-shaped and run parallel to the axis of the nail ■ The main value of nail fold dermoscopy is the early diagnosis of scleroderma before there are positive clinical and serologic findings (Figure 1-35)







Multiple constrictions of the hair shaft alternating with elliptical nodosities that look like a “pearl necklace” A high tendency to fracture, which gives hair a stubblelike appearance Hair shafts bend regularly in multiple places and curving in different directions “regularly bended ribbon sign”

Figure 1-35 Nail Fold Capillaries in Cutaneous Lupus. Uniform hairpin-shaped capillary loops are replaced with large (mega) irregularly-shaped capillary loops that still have a hairpin shape. The surrounding skin is atrophic with diffuse telangietatic vessels.

Chapter 1

Netherton Syndrome ■



Trichorrhexis invaginata/bamboo hair/golf tee type characterized by invagination of the distal portion of the hair shaft into its proximal portion forming a ball-in-cup appearance Diagnosis can easily be made without the need for hair sampling and microscopic examination

Pili Annulati ■



■ ■ ■ ■ ■



■ ■ ■

ANDROGENIC ALOPECIA







23

Terminal to vellus hair ratio can be calculated without skin biopsies Increased percentage of thin hairs Decreased average hair diameter Predominance of hair-follicle units with single hairs Hyperkeratotic plugs Perifollicular pigmentation

ALOPECIA AREATA

Alternating light and dark bands are seen clinically ■ Light bands represent cavities within the cortex ■ Cavities appear as whitish areas within a darker hair shaft ■ The opposite is true with light microscopy

Acquired Hair Diseases ■

Dermoscopy from A to Z

Variable hair-shaft diameter Digital systems allow the precise measurement and monitoring of hair-shaft thickness Identify and count vellus hairs (thin hairs less than 0.03mm in width)

Regularly distributed hyperkeratotic plugs in hair follicles (yellow dots) Cadaverized hairs (black dots) Dystrophic hairs Micro-exclamation point hairs Fibrosis with white dots in long-standing cases

CICATRICIAL ALOPECIA ■



Scarring alopecia of different etiologies looks the same with fibrosis of follicular ostia visible as white dots With more advanced disease, the dots coalesce to form bony white areas without visible ostia

QUESTIONS 1. Which criteria can be used to diagnose a melanocytic lesion? a. Milia-like cysts and pigmented pseudofollicular openings. b. Arborizing vessels, ulceration, and pigmentation. c. A central white patch plus fine peripheral pigment network. d. Lacunae and black homogenous blotches. e. Pigment network, brown globules, homogeneous blue color, or parallel patterns.

3. Which criteria can be used to diagnose a seborrheic keratosis? a. Milky-red areas, irregular streaks, and pigmented pseudofollicular openings. b. Streaks, irregular blotches, and regression. c. Fissures, ridges, sharp border demarcation, milialike cysts, pseudofollicular openings, fat fingers, and hairpin vessels. d. Rhomboid structures and/or circle within a circle. e. Diffuse brown color, glomerular vessels, and milia-like cysts.

2. Diagnosing a melanocytic lesion by default means? a. There are high risk criteria at the periphery of the lesion that are hard to identify. b. There are criteria for a seborrheic keratosis or basal cell carcinoma associated with pigment network and brown globules. c. There is an absence of criteria to diagnose a melanocytic lesion, seborrheic keratosis, dermatofibroma, pyogenic granuloma, or ink-spot lentigo, therefore the lesion should be considered melanocytic. d. There is an absence of criteria to diagnose a melanocytic lesion, seborrheic keratosis, basal cell carcinoma, dermatofibroma, or hemangioma, therefore the lesion should be considered melanocytic. e. None of the above.

4. Which criteria can be used to diagnose a basal cell carcinoma? a. Pigment network and arborizing vessels. b. Arborizing and pinpoint vessels plus multifocal hypopigmentation. c. The absence of a pigment network, arborizing vessels, pigmentation, ulceration, and spoke-wheel structures. d. Glomerular vessels, ulceration, and blue ovoid nests of pigmentation. e. Islands of black blotches, arborizing vessels, and moth-eaten borders.

24

DERMOSCOPY: AN ILLUSTRATED SELF-ASSESSMENT GUIDE

5. Vascular lesions can contain the following criteria: a. Out of focus lacunae-like globules. b. A variable number of red, sharply demarcated vascular spaces called lacunae and fibrous septae. c. Ten to twenty major and minor lacunae and thromboses. d. A minimal of two well-developed glomerular vessels. e. Fibrous septae, peppering, and blue dark lacunae.

6. Dermatofibromas can be associated with the following criteria: a. Pigment network, arborizing vessels, and central white patch. b. A central white patch that is never located at the periphery. c. A central white patch and peripheral pigment network. d. A complete absence of blood vessels and a few milia-like cysts. e. Multifocal hypopigmentation, arborizing vessels, and a central bluish-white veil.

7. Melanoma-specific criteria on the trunk and extremities can contain the following combination of criteria: a. Asymmetry of color and structure, a cobblestone global pattern, and regular globules or blotches. b. A multicomponent global pattern, symmetry of color and structure, regular network, regular globules, and regression. c. Polymorphous vessels, arborizing vessels, two colors, and regular streaks. d. Irregular pigment network, irregular globules, irregular blotches, and regression. e. Rhomboid structures and the parallel ridge pattern.

8. Dysplastic nevi typically have the following combination of criteria: a. Symmetry of color and structure and no melanomaspecific criteria. b. Asymmetry of color and structure, irregular pigment network, regular blotches, and regular streaks. c. Multifocal regression, peppering, regular pigment network, regular dots, and globules. d. Pinpoint, arborizing, and glomerular vessels plus several melanoma-specific criteria. e. Asymmetry of color and structure plus several melanoma-specific criteria.

9. Which of the following statements is true about Spitz nevi? a. They can have ten different patterns. b. A Spitzoid lesion only refers to the starburst or pink patterns. c. Melanoma is not in the differential diagnosis of regular starburst pattern. d. In an adult, most Spitzoid lesions do not need to be excised. e. Symmetrical and asymmetrical starburst patterns can be seen in melanoma.

10. Which of the following statements best describes the criteria seen in superficial spreading melanomas? a. Criteria associated with a benign nevus are never seen. b. They contain several well-developed melanomaspecific criteria such as symmetry of color and structure and one prominent color. c. Usually they have several well-developed melanomaspecific criteria such as asymmetry of color and structure, multicomponent global pattern, regular network, regular globules, and regular streaks. d. They contain a variable number of melanomaspecific criteria such as asymmetry of color and structure, multicomponent global pattern, irregular local criteria, five or six colors, and polymorphous vessels. e. Are usually feature-poor or featureless.

ANSWERS 1. E. Criteria to diagnose a melanocytic lesion include any variation of pigment network (regular and/or irregular), multiple brown dots and/or globules, homogeneous blue color of a blue nevus, and parallel patterns seen on acral skin. The default category is the last way to diagnose

a melanocytic lesion. Milia-like cysts and pseudofollicular openings can be seen in melanocytic lesions but are not primary criteria to make the diagnosis. Answers a, b, c and d diagnose a seborrheic keratosis, basal cell carcinoma, dermatofibroma, and hemangioma.

Chapter 1

2. D. Diagnosing a melanocytic lesion by default means that one does not see criteria for a melanocytic lesion, seborrheic keratosis, basal cell carcinoma, dermatofibroma, or hemangeoma. Default is an absence of criteria. One has to memorize all of the criteria from each specific potential diagnosis to be able to diagnose a melanocytic lesion by default. Dermoscopy cannot be mastered by osmosis. It is essential to study and practice the technique routinely in ones daily practice. Ink-spot lentigo and pyogenic granuloma are not in this algorithm. 3. C. All of the criteria used to diagnose seborrheic kerotosis are commonly seen in daily practice. Melanomaspecific criteria can also be seen in atypical seborrheic keratosis. Beware of seborrheic keratosis—like melanomas. Milky-red areas, irregular streaks, regression, rhomboid structures, and circle within a circle are all melanomaspecific criteria that are more sensitive and specific for melanoma but could be found in seborrheic keratosis. Glomerular vessels are a primary criterion to diagnose Bowen disease and are not seen in seborrheic keratosis. 4. C. Basal cell carcinomas are usually a clinical diagnosis and dermoscopy is used to confirm ones clinical impression. Dermoscopy gives one an on the spot second opinion. By definition, if one sees pigment network the lesion could not be a basal cell carcinoma. A sub-set of melanomas can be difficult to differentiate from basal cell carcinoma with pigmentation and arborizing vessels. Pinpoint and glomerular vessels could be seen but they would be out shadowed by arborizing vessels. If not, one could be dealing with a basal cell-like melanoma. Motheaten borders are seen in lentigines and flat seborrheic keratosis, never in basal cell carcinomas. 5. B. The hallmark of vascular lesions are lacunae, vascular spaces with well-demarcated sharp borders. There is no set number of lacunae needed to make the diagnoses. At times one has to use their imagination to decide if the margins fit the criteria for vascular spaces. Different shades of red, blue, and even black are typically seen. Black homogeneous color usually represents thombosis. Major and minor lacunae do not exist. Fibrous whitish septa and/or bluishwhite color are routinely seen in typical hemangiomas. At times it is not possible to differentiate lacunae and red color of a hemangioma from the milky-red areas that can contain out-of-focus reddish globules seen in melanoma. 6. C. Dermatofibromas are ubiquitous benign tumors and in most cases dermoscopy is not needed to make the diagnosis. A central white patch and pigment network, the primary criteria to make the diagnosis, may or may not be present. It might not be possible to differentiate an atypical dermatofibroma from a melanoma if melanoma-specific

Dermoscopy from A to Z

25

criteria are identified. There are innumerable ways that the central white patch can appear, and in many cases it is not centrally located. Telectangietatic vessels with polymorphous shapes are commonly seen but basal cell carcinoma-like arborizing vessels would make the diagnosis of a dermatofibroma unlikely.

7. D. Irregularity is the name of the game if criteria are to be considered melanoma-specific. Melanoma-specific criteria can be seen in both benign and malignant pathology but are more sensitive and specific for melanoma. There is not a single melanoma-specific criterion that is pathognomonic for melanoma. One should learn their definitions, and study as many classic textbook examples as possible. Rhomboid structures help diagnose melanoma on the face and the parallel ridge pattern can be seen in acral melanomas. 8. E. Dysplastic nevi are ubiquitous in the light-skinned population and can be indistinguishable clinically and dermoscopically from melanoma. They usually look more benign than malignant with dermoscopy; however, there are melanomas that do not have well-developed melanoma-specific criteria. Vessels of any kind are not typically seen except in pink feature-poor dysplastic nevi. They can have a variable number of melanoma-specific criteria (eg, irregular pigment network, irregular dots, and/or globules, irregular blotches) that are not as well developed as those seen in melanoma. Streaks, regression, and many colors are not usually seen and should raise a red flag for concern that the lesion might be a melanoma. 9. E. Spitzoid lesions are always a red flag for concern. Even symmetrical patterns can be seen in melanoma. There are only six patterns (starburst, globular, homogeneous, pink, black network, and atypical). One often has to use their imagination to diagnose a Spitzoid lesion. Since symmetrical and asymmetrical Spitzoid patterns can be found in melanoma, they should all be excised in children as well as in adults. A dermatopathologist that specializes in melanocytic lesions is good, while one that has expertise in Spitzoid lesions is ideal. Even experienced dermatopathologists have trouble differentiating atypical Spitzoid lesions from melanoma, and atypical Spitzoid lesions have the potential to metastasize because in reality they are melanomas. 10. D. Superficial spreading melanoma can have it all as far as the spectrum of melanoma-specific criteria goes. The criteria can be well developed or difficult to identify. Criteria associated with benign melanocytic lesions can also be seen. The more the high risk criteria identified in the lesion, the greater the chance that one is dealing with a melanoma. Nodular and amelanotic melanoma are more likely to be feature-poor or featureless.

26

DERMOSCOPY: AN ILLUSTRATED SELF-ASSESSMENT GUIDE

SUGGESTED READINGS Altamura D, Avramidis M, Menzies SW. Assessment of the optimal interval for and sensitivity of short-term sequential digital dermoscopy monitoring for the diagnosis of melanoma. Arch Dermatol. 2008; 144(4):502-506. Argenziano G, Zalaudek I, Johr RH et al. Dermoscopy features of melanoma incognito: indications for biopsy. J Am Acad Dermatol. 2007; 56(3):508-513. Argenziano G, Soyer HP, Chimenti S et al. Dermoscopy of pigmented skin lesions: results of a consensus meeting via the internet. J Am Acad Dermatol. 2003; 48:679-693. Argenziano G, Zalundek I et al. Vascular structures in skin tumors: a dermoscopy study. Arch Dermatol. 2004; 140:1485-1489. Bowling J, Agenziano G et al. Dermoscopy key points: recommendations from the International Dermoscopy Society. Dermatol. 2007; 214:3-5. Cari P, DeGiorgi V, Crocetti E et al. Improvement of the malignant/benign ratio in excised melanocytic lesions in the dermoscopy era: a retrospective study 1997-2001. Br J Dermatol. 2004; 150(4):687-692. Johr RH, Soyer HP, Argenziano G. Dermoscopy: The Essentials. Edinburgh: Mosby; 2004.

Johr RH. Pink lesions. Clin Dermatol. 2002; 20:289-296. Kittler H, Guitera P, Riedl E et al. Identification of clinically featureless incipient melanoma using sequential dermoscopy imaging. Arch Dermatol. 2006; 142(9):1113-1119. Malvehy J, Puig S, Braun R, Marghoob A, Kopf AW. Handbook of Dermoscopy. London (UK): Taylor and Francis; 2006. Malvehy J, Puis S et al. Dermoscopy report: proposal for standardization. Results of a consensus meeting of the International Dermoscopy Society. J Am Acad Dermatol. 2007; 57:84-95. Marghoob AA, Braun PR, Kopf AW eds. Atlas of Dermoscopy. London (UK): Taylor and Francis; 2005 Menzies S, Zalaudek I. Why perform dermoscopy? The evidence for its role in the routine management of pigmented skin lesions. Arch Dermatol. 2006; 142(9):1211-1212. Stolz W, Braun–Falco O et al. Color Atlas of Dermatoscopy. 2nd ed. Berlin: Blackwell Publishing; 2002. Vestergaard ME, Macaskill P et al. Dermoscopy compared with naked eye examination for the diagnosis of primary melanoma: a meta-analysis of studies performed in a clinical setting. Br J Dermatol. 2008; 159:669-676.

Chapter 2

Scalp, Face, Nose, and Ears

.

General Instructions: You will find a list of True/False statements following each case history. Select any statements, which you believe to be true. There may be one, more than one or no true statements for any given case. Choose the correct risk, diagnosis and disposition for each case. Then, turn the page to find a detailed discussion and pearls for each case.

Chapter 2

Scalp, Face, Nose, and Ears

RISK ❑ Low ❑ Intermediate ❑ High

2-1a

DIAGNOSIS ❑ ❑ ❑ ❑ ❑ ❑ ❑ ❑

Nevus Seborrheic keratosis Basal cell carcinoma Vascular Dermatofibroma Squamous cell carcinoma Melanoma Other

DISPOSITION

2-1b

❑ No intervention ❑ Follow-up ❑ Histopathologic diagnosis

CASE 1 HISTORY An 85-year-old man noticed the significant darkening of a long time facial lesion. 1. Clinically and dermoscopically this could be lentigo maligna. 2. Multiple milia-like cysts and pseudofollicular openings diagnose a seborrheic keratosis. 3. It is not always possible to differentiate follicular openings from pseudofollicular openings and milia-like cysts. 4. The dark blotch with the bluish tinge is always high risk. 5. The dark blotch has histopathologic correlates that include atypical melanocytes.

29

30

DERMOSCOPY: AN ILLUSTRATED SELF-ASSESSMENT GUIDE

RISK ✔ Low ❑ ❑ Intermediate ❑ High

DIAGNOSIS Nevus Seborrheic keratosis Basal cell carcinoma Vascular Dermatofibroma Squamous cell carcinoma Melanoma Other

DISPOSITION ❑ ✔ No intervention ❑ Follow-up ❑ Histopathologic diagnosis

2-1c

❑ ✔ ❑ ❑ ❑ ❑ ❑ ❑ ❑

ANSWERS Answers: 2,3,5

Discussion: ■

■ ■

DERMOSCOPIC CRITERIA ■ ■

■ ■



Follicular openings (boxes) Pigmented pseudofollicular openings (yellow arrows) Milia-like cysts (black arrows) Hyperpigmentation (white arrows) Moth-eaten borders (red arrows)



■ ■ ■ ■

Multiple milia-like cysts and pseudofollicular openings characterize this seborrheic keratosis. Both criteria can also be found in benign nevi and melanoma. Dark colors (hyperpigmentation) are commonly found in seborrheic keratosis. Concave “moth-eaten” borders are one criterion seen in flat seborrheic keratosis and solar lentigines. Blue and/or white color in any intensity should raise a red flag for concern. Bluish color is not always high risk. In a flat lesion subtle blue and/or white color usually is low risk In a raised lesion intense blue and/or white color is potentially high risk.

PEARLS ■





It is essential to differentiate the pseudofollicular openings and milia-like cysts of a seborrheic keratosis from the follicular openings seen in lentigo maligna or lentigo maligna melanoma. One should not stop and diagnose a seborrheic keratosis if milia-like cysts and pseudofollicular openings are identified before examining the entire lesion. High risk criteria pointing to the correct diagnosis might be found elsewhere and then they would be missed.

Chapter 2

Scalp, Face, Nose, and Ears

RISK ❑ Low ❑ Intermediate ❑ High

2-2a-2

2-2a-1

DIAGNOSIS ❑ ❑ ❑ ❑ ❑ ❑ ❑ ❑

Nevus Seborrheic keratosis Basal cell carcinoma Vascular Dermatofibroma Squamous cell carcinoma Melanoma Other

DISPOSITION

2-2b

❑ No intervention ❑ Follow-up ❑ Histopathologic diagnosis

CASE 2 HISTORY The mother of this 7-year-old female is concerned about a blue spot on her daughter’s face. She wants it treated immediately because the patient’s uncle was recently diagnosed with a melanoma. The lesion has been present for two years and is not changing. 1. Historically, clinically, and dermoscopically this clearly falls within the range of a blue nevus. 2. This could be a combined nevus. 3. This polychromatic lesion with white, blue, and shades of brown color indicates that it is high risk. 4. Different shades of blue, brown, black, and white can be seen in blue nevi. 5. The multiple follicular openings seen here are as expected on the face.

31

32

DERMOSCOPY: AN ILLUSTRATED SELF-ASSESSMENT GUIDE

RISK ✔ Low ❑ ❑ Intermediate ❑ High

DIAGNOSIS Nevus Seborrheic keratosis Basal cell carcinoma Vascular Dermatofibroma Squamous cell carcinoma Melanoma Other

2-2c

✔ ❑ ❑ ❑ ❑ ❑ ❑ ❑ ❑

ANSWERS DISPOSITION ✔ No intervention ❑ ❑ Follow-up ❑ Histopathologic diagnosis

Answers: 1,2,4,5

Discussion: ■

■ ■



DERMOSCOPIC CRITERIA ■ ■ ■

Follicular openings (circles) Brown color (arrows) Bluish-white color

■ ■









This blue nevus has a dermoscopic differential diagnosis that includes a combined nevus. In most cases blue nevi are easy to diagnose. The classic blue nevus is characterized by homogenous featureless blue color with well-demarcated borders. Hypopigmentation often scar-like is commonly seen. Globular-like structures can be seen in blue nevi. Combined nevi contain two different cell populations. ■ Blue nevi and compound nevi make up most combined nevi. Combined nevi can contain various shades and distributions of blue and brown colors. “Black” blue nevi are created by the black lamella (pigmented parakeratosis). ■ The black lamella looks like a jet black homogeneous blotch with a glazed appearance. Tape stripping can remove the black lamella, which is a positive clinical sign pointing toward low risk pathology. ■ The same holds true in whatever lesions that have a black lamella (ie, junctional nevus). Polychromatic blue nevi should raise a red flag for concern. ■ The concept of “raising a red flag for concern” means that there is a clue or clues that a lesion might be high risk. Proceed with focused attention with the complete analysis of the lesion.

PEARLS ■ ■

■ ■

The history is essential to help make the correct diagnosis of a blue nevus. Melanoma and cutaneous metastatic melanoma can be identical clinically and dermoscopically with a blue nevus. A “blue” nevus developing in an adult should always raise a red flag for concern. To allay the anxiety of the parents that this could be a melanoma, a punch excision of the lesion was performed which confirmed that this was in fact a benign blue nevus.

Chapter 2

Scalp, Face, Nose, and Ears

RISK ❑ Low ❑ Intermediate ❑ High

DIAGNOSIS

2-3a

❑ ❑ ❑ ❑ ❑ ❑ ❑ ❑

Nevus Seborrheic keratosis Basal cell carcinoma Vascular Dermatofibroma Squamous cell carcinoma Melanoma Other

DISPOSITION

2-3b

❑ No intervention ❑ Follow-up ❑ Histopathologic diagnosis

CASE 3 HISTORY This young patient was referred to you for a second opinion because their dermatologist recommended excision of this “fried egg” appearing nevus. It was never examined with dermoscopy and the referring pediatric dermatologist thought that dermoscopic examination would be beneficial to help avoid surgery on the child’s face. 1. “Fried egg” appearing nevi are always at high risk, histopathologically, and, therefore, dermoscopic evaluation is not indicated. 2. In this age group “fried egg” appearing nevi are usually not high risk and dermoscopy can help to prove this. 3. Regular dots, globules, and a regular blotch characterize this combined nevus. 4. The dark blotch in an adult should raise a red flag for concern. 5. Sequential digital clinical and dermoscopic monitoring is an alternative to skin biopsy or excision.

33

34

DERMOSCOPY: AN ILLUSTRATED SELF-ASSESSMENT GUIDE

RISK ✔ Low ❑ ❑ Intermediate ❑ High

DIAGNOSIS Nevus Seborrheic keratosis Basal cell carcinoma Vascular Dermatofibroma Squamous cell carcinoma Melanoma Other

2-3c

✔ ❑ ❑ ❑ ❑ ❑ ❑ ❑ ❑

DISPOSITION ❑ No intervention ✔ Follow-up ❑ ❑ Histopathologic diagnosis

ANSWERS Answers: 2,3,4,5

Discussion: ■ “Fried egg” appearing nevi should be evaluated with dermoscopy before

DERMOSCOPIC CRITERIA: ■

■ ■

Regular dots and globules (circles) Islands of normal skin (arrows) Regular bluish-brown blotch (star)

■ ■

■ ■





more invasive procedures are performed. In this age group the chances of high risk pathology are extremely low. This congenital combined nevus has two cell populations: a blue nevus (the bluish-brown blotch) and compound nevus. Combined nevi do not always have a “fried egg” appearance. If found in a teenager or an adult, the bluish-brown blotch is potentially at more high risk. The bluish-brown blotch has a differential diagnosis that includes melanocytic atypia or melanoma. Islands of normal skin and islands of criteria are one of the patterns commonly seen in congenital melanocytic nevi.

PEARLS ■



Sequential digital monitoring and confident reassurance to the parents that an excision is not needed would be the optimal cutting-edge approach to handle this case. Whatever global patterns and local criteria are identified, one has to decide if they are regular or irregular, good or bad, low or high risk.

Chapter 2

Scalp, Face, Nose, and Ears

RISK ❑ Low ❑ Intermediate ❑ High

2-4a

DIAGNOSIS ❑ ❑ ❑ ❑ ❑ ❑ ❑ ❑

Nevus Seborrheic keratosis Basal cell carcinoma Vascular Dermatofibroma Squamous cell carcinoma Melanoma Other

DISPOSITION

2-4b

❑ No intervention ❑ Follow-up ❑ Histopathologic diagnosis

CASE 4 HISTORY A 76-year-old man came for a consultation of a slowly enlarging growth on his nose that started one year ago. 1. Milia-like cysts and pigmented pseudofollicular openings diagnose this pigmented seborrheic keratosis. 2. Irregular globules and blotches, polymorphous vessels, and regression characterize this melanoma. 3. The absence of pigment network, arborizing vessels, plus pigmentation diagnoses this basal cell carcinoma. 4. This pattern of pigmentation and vessels can be found in a basal cell carcinoma or melanoma. 5. Leaf-like structures are present throughout this lesion.

35

36

DERMOSCOPY: AN ILLUSTRATED SELF-ASSESSMENT GUIDE

RISK ❑ Low ❑ Intermediate ✔ High ❑

DIAGNOSIS Nevus Seborrheic keratosis Basal cell carcinoma Vascular Dermatofibroma Squamous cell carcinoma Melanoma Other

2-4c

❑ ❑ ✔ ❑ ❑ ❑ ❑ ❑ ❑

DISPOSITION ❑ No intervention ❑ Follow-up ✔ Histopathologic diagnosis ❑

ANSWERS Answers: 3,4

Discussion: ■ ■

DERMOSCOPIC CRITERIA ■ ■









Arborizing vessels (red arrows) Irregular grayish-white blotches (black arrows) Follicular openings inside the lesion (yellow arrows) Follicular openings outside the lesions (yellow boxes) Pseudofollicular openings (white arrows) Bony-white color (stars)













An experienced dermoscopist can easily make the diagnosis. A less-experienced dermoscopist should evaluate all of the criteria found in the lesion with full knowledge of their definitions. Melanoma is in the differential diagnosis because of the irregular blackishgray blotches and diffuse bony- white color (regression). White color is commonly found in basal cell carcinomas and does not represent true regression. Pinpoint; irregular linear and other irregularly-shaped telangectatic vessels (polymorphous vascular pattern) are absent. The dermoscopic differential diagnosis of the large pseudofollicular opening includes ulceration. Ulceration is much more commonly found in basal cell carcinoma than in melanoma. Arborizing vessels are not found in seborrheic keratosis.

PEARLS ■

■ ■





There are innumerable shapes and sizes of pigmentation that can be found in basal cell carcinomas. One has to only determine if pigment is present or absent. Adjectives (ie, ovoid, leaf-like) describing the shape of pigmentation are not needed. The irregular grayish-black blotch at the superior pole fits the definition of a “leaf-like” structure, yet it has no resemblance to a leaf. This is an inaccurate concept that is written and taught, which should be abandoned.

Chapter 2

Scalp, Face, Nose, and Ears

RISK ❑ Low ❑ Intermediate ❑ High

2-5a

DIAGNOSIS ❑ ❑ ❑ ❑ ❑ ❑ ❑ ❑

Nevus Seborrheic keratosis Basal cell carcinoma Vascular Dermatofibroma Squamous cell carcinoma Melanoma Other

DISPOSITION

2-5b

❑ No intervention ❑ Follow-up ❑ Histopathologic diagnosis

(Reprinted, with permission, from Stolz W, Braun-Falco O, Bilek P et al. (2002) Color Atlas of Dermatoscopy. Second Edition. Blackwell Publishing, Oxford.)

CASE 5 HISTORY This previously long-standing asymptomatic lesion suddenly became raised and tender, but only for a few days. The patient’s primary care physician was concerned about the sudden change and thought it was a skin cancer. 1. 2. 3. 4.

This is the classic clinical and dermoscopic picture of an inflamed molluscum contagiosum. This collision tumor consists of a squamous and basal cell carcinoma. Arborizing vessels and regression characterize this melanoma. Arborizing “basal cell-like” vessels and whitish-yellow globules characterize this typical sebaceous gland hyperplasia. 5. Basal cell carcinoma is in the clinical and dermoscopic differential diagnosis of sebaceous gland hyperplasia.

37

38

DERMOSCOPY: AN ILLUSTRATED SELF-ASSESSMENT GUIDE

RISK ✔ Low ❑ ❑ Intermediate ❑ High

DIAGNOSIS Nevus Seborrheic keratosis Basal cell carcinoma Vascular Dermatofibroma Squamous cell carcinoma Melanoma Other

2-5c

❑ ❑ ❑ ❑ ❑ ❑ ❑ ✔ ❑

DISPOSITION ✔ No intervention ❑ ❑ Follow-up ❑ Histopathologic diagnosis

ANSWERS Answers: 4,5

Discussion: ■ ■

DERMOSCOPIC CRITERIA ■





Basal cell-like vessels (black arrows) Sebaceous globules (yellow arrows) Follicular openings (red arrows)





■ ■



This is a classic clinical and dermoscopic sebaceous gland hyperplasia. The yellowish-white globules would not be seen in a basal cell carcinoma. By definition “crown” or “wreath-like” vessels said to characterize sebaceous gland hyperplasia surround and penetrate the lesion, but never reach the center. It is not necessary to adhere to this strict definition because it is rarely found in typical lesions. “Crown” or “wreath-like” vessels are poor terms that should be abandoned. Basal cell-like vessels in any distribution plus sebaceous globules make the diagnosis. Molluscum contagiosum typically presents with a clear papule that has a central dell. ■ Irregular whitish globules filling a lesion and peripheral linear vessels along the border have been reported.

PEARLS ■



The dramatic and worrisome short-lived change was created by an isolated pimple that coincidentally developed under the long-standing sebaceous gland hyperplasia. In general, it is not always necessary to rush into making a histopathologic diagnosis in a changing lesion when the change is of a short duration and the etiology of the change not clear.

Chapter 2

Scalp, Face, Nose, and Ears

RISK ❑ Low ❑ Intermediate ❑ High

2-6a

DIAGNOSIS ❑ ❑ ❑ ❑ ❑ ❑ ❑ ❑

Nevus Seborrheic keratosis Basal cell carcinoma Vascular Dermatofibroma Squamous cell carcinoma Melanoma Other

DISPOSITION

2-6b

❑ No intervention ❑ Follow-up ❑ Histopathologic diagnosis

CASE 6 HISTORY A 64-year-old ski enthusiast with a history of many sunburns presented with a nonhealing lesion on his nose. 1. 2. 3. 4. 5.

This featureless lesion is of no concern. This feature-poor lesion does not have enough criteria to diagnose a basal cell carcinoma. Foci of asymmetrical follicular pigmentation are an important clue that this is a melanoma. Milia-like cysts, pink and gray color diagnose this irritated seborrheic keratosis. This feature poor melanoma has asymmetry of color and structure, asymmetrical follicular pigmentation, pink and gray color, polymorphous vessels, and ulceration.

39

40

DERMOSCOPY: AN ILLUSTRATED SELF-ASSESSMENT GUIDE

RISK ❑ Low ❑ Intermediate ❑ ✔ High

DIAGNOSIS Nevus Seborrheic keratosis Basal cell carcinoma Vascular Dermatofibroma Squamous cell carcinoma Melanoma Other

2-6c

❑ ❑ ❑ ❑ ❑ ❑ ✔ ❑ ❑

DISPOSITION ❑ No intervention ❑ Follow-up ✔ Histopathologic diagnosis ❑

ANSWERS Answers: 2,3,5

Discussion: ■ ■

DERMOSCOPIC CRITERIA ■

■ ■

■ ■ ■ ■

Asymmetry of color and structure Follicular openings (arrows) Asymmetrical follicular pigmentation (boxes) Diffuse pinkish color Grayish-white color (black stars) Polymorphous vessels (circle) Ulceration (yellow star)







This lesion is feature-poor not featureless. Feature-poor means that the important criteria are present but not welldeveloped and easy to identify. Clinically one suspects a basal cell carcinoma but the ulceration is not enough to make that diagnosis. Asymmetrical follicular pigmentation is the most specific criteria to diagnose a melanoma. The polymorphous vessels (ie, pinpoint, linear) could easily be missed if too much pressure is applied with instrumentation.

PEARLS ■ ■ ■



Pink color should always be a red flag for concern in any lesion. “If there’s pink, stop and think.” If one has to spend too much time thinking about what is going on, it is time to make a histopathologic diagnosis. This diagnosis was a big surprise!

Chapter 2

Scalp, Face, Nose, and Ears

RISK ❑ Low ❑ Intermediate ❑ High

2-7a

DIAGNOSIS ❑ ❑ ❑ ❑ ❑ ❑ ❑ ❑

Nevus Seborrheic keratosis Basal cell carcinoma Vascular Dermatofibroma Squamous cell carcinoma Melanoma Other

DISPOSITION

2-7b

❑ No intervention ❑ Follow-up ❑ Histopathologic diagnosis

CASE 7 HISTORY A 39-year-old woman was concerned about a strange-looking spot on her face that did not wash off with soap. 1. 2. 3. 4. 5.

This is a typical make-up tattoo that should be treated with a laser. This could only be a melanocytic lesion because it has a pigment network. Lentigines and dermafibromous are nonmelanocytic lesions that can have a pigment network. Branched streaks and irregular globules diagnose this in situ melanoma. The black stellate macule seen clinically and thickened, branched black pigment network diagnose this classic ink-spot lentigo.

41

42

DERMOSCOPY: AN ILLUSTRATED SELF-ASSESSMENT GUIDE

RISK ✔ Low ❑ ❑ Intermediate ❑ High

DIAGNOSIS Nevus Seborrheic keratosis Basal cell carcinoma Vascular Dermatofibroma Squamous cell carcinoma Melanoma Other

DISPOSITION ✔ No intervention ❑ ❑ Follow-up ❑ Histopathologic diagnosis

2-7c

❑ ❑ ❑ ❑ ❑ ❑ ❑ ✔ ❑

ANSWERS Answers: 3,5

Discussion: ■ ■



DERMOSCOPIC CRITERIA ■



Thick, branched pigment network (arrows) Regular dots (circle)



■ ■

The diagnosis of an ink-spot lentigo can usually be made clinically. Look for high risk criteria so that you do not miss an ink-spot lentigo-like melanoma. The criteria seen here are not the ones used to diagnose in situ melanoma on the face (ie, asymmetrical follicular pigmentation, annular-granular structures, rhomboid structures, circle within a circle). Follicular openings that one expects to find on the face are not found in every patient. The dot-like structures represent remnants of the pigment network. Not all skin lesions with a pigment network are melanocytic (ie, lentigo, dermatofibroma).

PEARLS ■ ■



Ink-spot lentigines can be associated with a history of excessive sun exposure. A complete skin examination should be performed to look for more high risk pathology (ie, skin cancer). Complete skin examinations should be performed on all patients at the time one deems appropriate.

Chapter 2

Scalp, Face, Nose, and Ears

RISK ❑ Low ❑ Intermediate ❑ High

2-8a

DIAGNOSIS ❑ ❑ ❑ ❑ ❑ ❑ ❑ ❑

Nevus Seborrheic keratosis Basal cell carcinoma Vascular Dermatofibroma Squamous cell carcinoma Melanoma Other

DISPOSITION

2-8b

❑ No intervention ❑ Follow-up ❑ Histopathologic diagnosis

CASE 8 HISTORY A 45-year-old man noticed the appearance of a new spot on his face that was getting darker. 1. Clinically and dermoscopically this could be an ink-spot lentigo. 2. The absence of a thick black branched pigment network should raise a red flag for concern that his might not be an ink-spot lentigo. 3. The irregular black blotch, irregular globules, and foci of blue-white color characterize this in situ melanoma. 4. This is too small to be a melanoma; therefore the high risk dermoscopic picture should be ignored. 5. The isolated pseudofollicular opening, milia-like cysts, and sharp demarcation diagnose this small seborrheic keratosis.

43

44

DERMOSCOPY: AN ILLUSTRATED SELF-ASSESSMENT GUIDE

RISK ❑ Low ✔ Intermediate ❑ ❑ High

DIAGNOSIS Nevus Seborrheic keratosis Basal cell carcinoma Vascular Dermatofibroma Squamous cell carcinoma Melanoma Other

2-8c

❑ ❑ ❑ ❑ ❑ ❑ ✔ ❑ ❑

DISPOSITION ❑ No intervention ❑ Follow-up ✔ Histopathologic diagnosis ❑

ANSWERS Answers: 1,2,3

Discussion: ■ ■

DERMOSCOPIC CRITERIA ■





■ ■

Irregular black blotch (white star) Follicular openings (red arrows) Pseudofollicular opening (white arrow) Irregular dots (white circles) Blue-white color (black arrows)



■ ■



The history and dermoscopic picture are worrisome. An ink-spot lentigo might look like this dermoscopically. However, without the prominent pigment network it falls out of the range of a classic lesion. One has to create a dermoscopic differential diagnosis for the round white structures (ie, milia-like cysts vs follicular openings). Milia-like cysts and pseudofollicular openings can be seen in melanoma. This entire picture warrants a histopathologic diagnosis even if it is in a cosmetically sensitive area. Melanomas can start as a speck and remain less than 6mm for long periods of time.

PEARLS ■







Destructive therapy without a positive histopathologic diagnosis could be disastrous. If after destructive therapy without a previous histopathologic diagnosis a lesion like this recurs, a histopathologic diagnosis should now be made posthaste. “Melanoma incognito” aka false negative melanoma can mimic benign or other malignant lesions. “Melanoma incognito” can clinically and/or dermoscopically mimic seborrheic keratosis, vascular lesions, basal cell carcinoma combined, recurrent, Spitz, blue and dysplastic nevi, actinic keratosis, or solar lentigines.

Chapter 2

Scalp, Face, Nose, and Ears

RISK ❑ Low ❑ Intermediate ❑ High

DIAGNOSIS

2-9a

❑ ❑ ❑ ❑ ❑ ❑ ❑ ❑

Nevus Seborrheic keratosis Basal cell carcinoma Vascular Dermatofibroma Squamous cell carcinoma Melanoma Other

DISPOSITION

2-9b

❑ No intervention ❑ Follow-up ❑ Histopathologic diagnosis

CASE 9 HISTORY There is only a two-month-history of this asymptomatic pigmented skin lesion on the ear of a 30-year-old man. 1. 2. 3. 4.

This is a melanocytic lesion because there are multiple brown globules. This is the typical dermoscopic appearance of a secondarily infected wound. This is an atypical Spitzoid global pattern because there are foci of irregular streaks. Asymmetry of color and structure, irregular globules, irregular streaks, an irregular black blotch plus regression characterizes this melanoma. 5. An atypical Spitzoid pattern seen with dermoscopy does not always correlate with a “Spitzoid” melanoma histopathologically.

45

46

DERMOSCOPY: AN ILLUSTRATED SELF-ASSESSMENT GUIDE

RISK ❑ Low ❑ Intermediate ✔ High ❑

DIAGNOSIS Nevus Seborrheic keratosis Basal cell carcinoma Vascular Dermatofibroma Squamous cell carcinoma Melanoma Other

2-9c

❑ ❑ ❑ ❑ ❑ ❑ ✔ ❑ ❑

DISPOSITION ❑ No intervention ❑ Follow-up ✔ Histopathologic diagnosis ❑

ANSWERS Answers: 1,3,4,5

Discussion: ■

DERMOSCOPIC CRITERIA ■



■ ■



Asymmetry of color and structure Irregular dots and globules (circles) Irregular streaks (black arrows) Irregular black blotch (white star, white arrows) Regression (black star)







■ ■

A regular/good/low risk Spitzoid pattern, aka “Starburst” pattern, can have streaks and/or dots and globules at most points along the periphery of the lesion. An atypical Spitzoid pattern can have foci of streaks and/or dots and globules at some points along the periphery of the lesion. There are only streaks connected to the irregular black blotch which fits the definition of irregular streaks. Inter-observer agreement among experienced dermoscopists is not always good and another clinician might not agree that this is an atypical Spitzoid global pattern. The grayish-white regression area fills half of the lesion. “Spitzoid” dermoscopic patterns are not always Spitzoid histopathologically.

PEARLS ■ ■





■ ■

Even regular Spitzoid patterns can be seen in melanoma. All lesions with this pattern of criteria should be removed no matter how old or young the patient. There would not be a good dermoscopic–pathologic correlation if the histopathologic report was that of a benign lesion. Speak with your pathologist about the asymmetrical dermoscopic appearance of the criteria. A reevaluation of the specimen might be indicated. Do not hesitate to ask for another histopathologic opinion with a pigmented-lesion expert if there is not a good dermoscopic–pathologic correlation.

Chapter 2

Scalp, Face, Nose, and Ears

RISK ❑ Low ❑ Intermediate ❑ High

2-10a

DIAGNOSIS ❑ ❑ ❑ ❑ ❑ ❑ ❑ ❑

Nevus Seborrheic keratosis Basal cell carcinoma Vascular Dermatofibroma Squamous cell carcinoma Melanoma Other

DISPOSITION

2-10b

❑ No intervention ❑ Follow-up ❑ Histopathologic diagnosis

CASE 10 HISTORY During a routine total body skin examination, this pigmented skin lesion was found on the scalp of an 83-year-old woman. 1. 2. 3. 4.

The clinical and dermoscopic differential diagnosis includes a seborrheic keratosis. The clinical and dermoscopic differential diagnosis includes an ulcerated basal cell carcinoma. The clinical and dermoscopic differential diagnosis includes invasive melanoma. The dermoscopic differential diagnosis of the irregular brown color includes irregular blotch vs ulceration. 5. The absence of arborizing vessels rules out a basal cell carcinoma.

47

48

DERMOSCOPY: AN ILLUSTRATED SELF-ASSESSMENT GUIDE

RISK ❑ Low ❑ Intermediate ✔ High ❑ 1 2

DIAGNOSIS Nevus Seborrheic keratosis Basal cell carcinoma Vascular Dermatofibroma Squamous cell carcinoma Melanoma Other

3

1

2-10c

❑ ❑ ❑ ✔ ❑ ❑ ❑ ❑ ❑

ANSWERS Answers: 2,3,4

Discussion:

DISPOSITION



❑ No intervention ❑ Follow-up ✔ Histopathologic diagnosis ❑

DERMOSCOPIC CRITERIA ■







■ ■

Asymmetry of color and structure Multicomponent global pattern (1,2,3) Irregular dots and globules (white boxes) Irregular grayish-black blotches (yellow arrows) Ulceration (white arrows) Regression (yellow stars)

■ ■ ■



■ ■





Strict analysis of the criteria favors the diagnosis of a melanoma. ■ Globules identify a melanocytic lesion: ■ Basal cell carcinomas can have pigmented globular-like structures that cannot be differentiated from the dots and globules of a melanocytic lesion. ■ Asymmetry of color and structure ■ Multicomponent global pattern ■ Irregular dots and globules ■ Irregular blotches ■ Regression Basal cell carcinomas do not always have arborizing vessels. Basal cell carcinomas do not always have pigment. Some of the grayish-black blotches fit the definition of “leaf like” structure, yet none look like any type of leaf! Spoke-wheel structures, which are not seen here, might be the only clue to diagnose a basal cell carcinoma. It is not always possible to be 100% sure of global patterns and local criteria. One has to be able to create a dermoscopic differential diagnosis for global patterns and local criteria. The multicomponent global pattern (three or more different areas within a lesion) is not diagnostic of melanoma. The multicomponent global pattern can be seen in melanocytic, nonmelanocytic, benign, and malignant lesions.

PEARLS ■







In this case, dermoscopy has been clinically helpful. Clearly this is not a seborrheic keratosis that needs no intervention. Even though one cannot be sure if this is a basal cell carcinoma or melanoma, we know that a histopathologic diagnosis is indicated. It is essential to be familiar with all of the classic examples of global patterns and local criteria. One will need that knowledge to be able to create a dermoscopic differential diagnosis when the occasion arises. Regularly one has to think in terms of dermoscopic differential diagnosis.

Chapter 2

Scalp, Face, Nose, and Ears

RISK ❑ Low ❑ Intermediate ❑ High

2-11a

DIAGNOSIS ❑ ❑ ❑ ❑ ❑ ❑ ❑ ❑

Nevus Seborrheic keratosis Basal cell carcinoma Vascular Dermatofibroma Squamous cell carcinoma Melanoma Other

DISPOSITION

2-11b

❑ No intervention ❑ Follow-up ❑ Histopathologic diagnosis

CASE 11 HISTORY While washing this seven-year-old girl’s hair, her mother found this asymptomatic spot on her scalp. 1. 2. 3. 4. 5.

More than 50% of this lesion demonstrates regressive changes pathognomonic of a melanoma. There are no clues to differentiate hypopigmentation from regression in this lesion. Gray color and foci of “peppering” are the criteria to diagnose the regression in this lesion. The more regression in a lesion, the greater the chance it is a melanoma. Irregular dots and globules, irregular blotches, plus regression characterize this dysplastic nevus.

49

50

DERMOSCOPY: AN ILLUSTRATED SELF-ASSESSMENT GUIDE

RISK ❑ Low ✔ Intermediate ❑ ❑ High

DIAGNOSIS Nevus Seborrheic keratosis Basal cell carcinoma Vascular Dermatofibroma Squamous cell carcinoma Melanoma Other

2-11c

❑ ✔ ❑ ❑ ❑ ❑ ❑ ❑ ❑

DISPOSITION ❑ No intervention ❑ Follow-up ❑ ✔ Histopathologic diagnosis

ANSWERS Answers: 3,4,5

Discussion: ■ ■

DERMOSCOPIC CRITERIA ■

■ ■ ■ ■

Irregular dots and globules (boxes) Irregular blotches (black arrows) Regression (star) Gray color (white arrows) Peppering (circles)





■ ■ ■ ■



Regression is not diagnostic of melanoma. Regression-like areas can be seen in melanocytic, nonmelanocytic, benign, and malignant lesions. The more regression, the greater the chance of high risk pathology (ie, dysplastic nevi, melanoma). It is not possible to differentiate this severely dysplastic nevus from a melanoma using dermoscopy. One might not be able to differentiate hypopigmentation from true regression. The white color of regression should be lighter than the surrounding skin. Gray color indicates that melanin is in the papillary dermis. The fine granules of “peppering” represent free melanin and/or melanophages in the papillary dermis. Gray color and “peppering” are found in regression not in hypopigmentation.

PEARLS ■ ■ ■





■ ■

The identification of regression is a red flag for concern. Proceed with focused attention and look for other high risk criteria. In this age group, and in this location, one would expect to find a globular pattern with regular dots and globules. There is not a good clinico-dermoscopic correlation—another red flag for concern. Not all scalp nevi in children need to be excised, especially if they have a banal clinical and dermoscopic appearance. The scalp is a common location to find dyspalstic nevi in children. It is essential to include the scalp in a complete skin examination.

Chapter 2

Scalp, Face, Nose, and Ears

RISK ❑ Low ❑ Intermediate ❑ High

2-12a

DIAGNOSIS ❑ ❑ ❑ ❑ ❑ ❑ ❑ ❑

Nevus Seborrheic keratosis Basal cell carcinoma Vascular Dermatofibroma Squamous cell carcinoma Melanoma Other

DISPOSITION

2-12b

❑ No intervention ❑ Follow-up ❑ Histopathologic diagnosis

CASE 12 HISTORY A first-year-dermatology resident found this small pigmented lesion after a complete skin examination in a pediatric dermatology clinic. The parents had not been aware of it. 1. 2. 3. 4.

Regular dots and globules characterize this classic cobblestone pattern. Irregular dots and globules are one important feature of this dysplastic nevus. The multifocal hypopigmentation seen here is a site-specific criterion just found in the scalp. Asymmetry of color and structure, irregular dots and globules, and hypopigmentation characterize this dysplastic nevus. 5. Asymmetry of color and structure, irregular dots and globules, regression, and a milky-red area characterize this dysplastic nevus.

51

52

DERMOSCOPY: AN ILLUSTRATED SELF-ASSESSMENT GUIDE

RISK ❑ Low ✔ Intermediate ❑ ❑ High

DIAGNOSIS Nevus Seborrheic keratosis Basal cell carcinoma Vascular Dermatofibroma Squamous cell carcinoma Melanoma Other

DISPOSITION ❑ No intervention ❑ Follow-up ✔ Histopathologic diagnosis ❑

2-12c

❑ ✔ ❑ ❑ ❑ ❑ ❑ ❑ ❑

ANSWERS Answers: 2,5

Discussion: ■ ■ ■

DERMOSCOPIC CRITERIA ■







Asymmetry of color and structure Irregular dots and globules (circles) Regression/whitish gray color (white arrows) Milky-red area/pink color (box)



■ ■



■ ■

As in Case 11, there is not a good clinico-dermoscopic correlation here A red flag for concern. One would expect to see the globular or cobblestone global pattern with regular dots and globules or larger, angulated globlules. Dots and globules with different sizes and shapes, scattered throughout the lesion are considered irregular by definition. The grayish-white area of regression is widespread. Multifocal hypopigmentations, not found here, would be one criterion commonly found in dysplastic nevi. There are no site-specific criteria found only on the scalp that are not found elsewhere on the body. The milky-red area does not contain out-of-focus, globular-like structures. In this case, the milky-red area is nonspecific and is not indicative of melanoma.

PEARLS ■ ■ ■





Do not be surprised to find nevi in the scalp of children. Do not be surprised to find atypical dermoscopic criteria in this location. In this case if the pathology report was not at least a dysplastic nevus, there would not be a good dermoscopic–pathologic correlation. When one does not have a good dermoscopic–pathologic correlation, speak with your pathologist. When one does not have a good dermoscopic–pathologic correlation, consider getting a pigmented-lesion expert pathologist’s opinion.

Chapter 2

Scalp, Face, Nose, and Ears

RISK

2-13a

❑ Low ❑ Intermediate ❑ High

DIAGNOSIS ❑ ❑ ❑ ❑ ❑ ❑ ❑ ❑

Nevus Seborrheic keratosis Basal cell carcinoma Vascular Dermatofibroma Squamous cell carcinoma Melanoma Other

DISPOSITION

2-13b

❑ No intervention ❑ Follow-up ❑ Histopathologic diagnosis

CASE 13 HISTORY A 78-year-old female had a basal cell carcinoma excised 8 years ago from her scalp. There is now a new lesion adjacent to the surgery scar. 1. 2. 3. 4. 5.

This recurrent basal cell carcinoma is filled with typical arborizing vessels. This melanoma is filled with pinpoint and cork screw vessels. This lesion is filled with hairpin vessels. Hairpin vessels and gray color characterize this irritated seborrheic keratosis. At times, it may be challenging to accurately categorize small vessels but an attempt always helps.

53

54

DERMOSCOPY: AN ILLUSTRATED SELF-ASSESSMENT GUIDE

RISK ✔ Low ❑ ❑ Intermediate ❑ High

DIAGNOSIS Nevus Seborrheic keratosis Basal cell carcinoma Vascular Dermatofibroma Squamous cell carcinoma Melanoma Other

DISPOSITION ✔ No intervention ❑ ❑ Follow-up ❑ Histopathologic diagnosis

2-13c

❑ ✔ ❑ ❑ ❑ ❑ ❑ ❑ ❑

ANSWERS Answers: 3,4,5

Discussion: ■ ■ ■

DERMOSCOPIC CRITERIA ■



■ ■ ■ ■

Classic hairpin vessels (black boxes) Nonperfect-shaped hairpin vessels (yellow boxes) Pinpoint vessels (circles) Linear vessels (red boxes) Gray color (black arrows) Brown color (yellow arrows)

■ ■

■ ■

Clinically, but not dermoscopically, this appears to be a basal cell carcinoma. Arborizing vessels are not seen anywhere. Basal cell carcinomas do not always have classic arborizing vessels. This lesion is filled with classic and not-so-classic hairpin vessels. There are no thick, irregular hairpin-shaped vessels that can be found in melanoma. The grayish color is the clue that there is inflammation. The brown color and pinpoint vessels have no diagnostic significance.

PEARLS ■ ■







A recurrent basal cell carcinoma after so many years would be unusual. A new primary basal cell carcinoma may happen to arise coincidentally in the same area. None of the specific shape designations given to vessels are pathonomonic of any specific pathology. Inter-observer agreement on the exact shape of blood vessels within a given lesion might vary with the personal experience of the individual. Polarizing instrumentation, ultrasound gel/hand cleaning gel, and minimal pressure should be applied to better visualize small vessels.

Chapter 2

Scalp, Face, Nose, and Ears

RISK ❑ Low ❑ Intermediate ❑ High

2-14a

DIAGNOSIS ❑ ❑ ❑ ❑ ❑ ❑ ❑ ❑

Nevus Seborrheic keratosis Basal cell carcinoma Vascular Dermatofibroma Squamous cell carcinoma Melanoma Other

DISPOSITION

2-14b

❑ No intervention ❑ Follow-up ❑ Histopathologic diagnosis

CASE 14 HISTORY A mother was worried about an enlarging growth by the right ear of her six-year-old child. 1. Clinically and dermoscopically this could be an amelanotic melanoma. 2. Clinically and dermoscopically this could be a Spitz nevus. 3. Different shades of pink color and polymorphous vessels are diagnostic of this amelanotic melanoma. 4. One can see pinpoint, linear, and hairpin vessels. 5. A vascular pattern in a pink lesion is one of the sub-types of Spitz nevi.

55

56

DERMOSCOPY: AN ILLUSTRATED SELF-ASSESSMENT GUIDE

RISK ❑ Low ❑ Intermediate ✔ High ❑

DIAGNOSIS Nevus Seborrheic keratosis Basal cell carcinoma Vascular Dermatofibroma Squamous cell carcinoma Melanoma Other

2-14c

✔ ❑ ❑ ❑ ❑ ❑ ❑ ❑ ❑

DISPOSITION ❑ No intervention ❑ Follow-up ✔ Histopathologic diagnosis ❑

ANSWERS Answers: 1,2,4,5

Discussion: ■ ■

DERMOSCOPIC CRITERIA ■ ■ ■ ■ ■ ■

Different shades of pink color White color Linear vessels (black arrows) Pinpoint vessels (boxes) Hairpin vessel (yellow arrow) Milia-like cyst (circle)



■ ■ ■ ■ ■

Amelanotic melanoma is a common subtype in this age group. Different shades of pink color and centrally-located vessels are commonly found in amelanotic melanoma, yet are not diagnostic. There are six dermoscopic patterns seen in Spitz nevi: starburst, globular, homogeneous, reticular/black pigment network, vascular, and atypical. All of the patterns can be seen in children. This lesion demonstrates the vascular pattern. Absence of pigment and vessels creates the pink color. The milia-like cyst has no diagnostic significance. Structures identical to milia-like cysts can represent mucoid degeneration and can be seen in melanomas.

PEARLS ■ ■ ■

Dermoscopy is more sensitive and specific to diagnose pigmented lesions. Classic patterns and their variations are commonly found in Spitz nevi. Pink lesions in any age group should raise a red flag for concern.

Chapter 2

Scalp, Face, Nose, and Ears

RISK ❑ Low ❑ Intermediate ❑ High

DIAGNOSIS

2-15a

❑ ❑ ❑ ❑ ❑ ❑ ❑ ❑

Nevus Seborrheic keratosis Basal cell carcinoma Vascular Dermatofibroma Squamous cell carcinoma Melanoma Other

DISPOSITION

2-15b

❑ No intervention ❑ Follow-up ❑ Histopathologic diagnosis

CASE 15 HISTORY An 80-year-old female with a history of recurrent lentigo maligna on the right side of her face developed a new blue papule by her left ear. 1. This could be a de novo melanoma characterized by irregular blotches, blue-white color, and regression. 2. This could be a heavily pigmented basal cell carcinoma without arborizing vessels and ulceration. 3. Commonly, basal cell carcinomas have such extensive pigmentation. 4. Uncommonly, basal cell carcinomas have such extensive pigmentation. 5. Without arborizing vessels and spoke-wheel structures, one cannot diagnose a basal cell carcinoma.

57

58

DERMOSCOPY: AN ILLUSTRATED SELF-ASSESSMENT GUIDE

RISK ❑ Low ❑ Intermediate ✔ High ❑

DIAGNOSIS Nevus Seborrheic keratosis Basal cell carcinoma Vascular Dermatofibroma Squamous cell carcinoma Melanoma Other

2-15c

❑ ❑ ✔ ❑ ❑ ❑ ❑ ❑ ❑

DISPOSITION ❑ No intervention ❑ Follow-up ✔ Histopathologic diagnosis ❑

ANSWERS Answers: 1,2,4

Discussion: ■

DERMOSCOPIC CRITERIA ■ ■ ■

Ulceration (yellow arrows) Bluish-white color (stars) Purplish color (red arrows)

■ ■ ■ ■ ■ ■ ■ ■ ■ ■ ■

With strict adherence to the definitions of the dermoscopic criteria, it is not possible to differentiate a basal cell carcinoma from melanoma. The bluish-white color has a dermoscopic differential diagnosis. Found in basal cell carcinomas and melanomas. Islands of pigmented basal cell carcinoma cells. Deep dermal heavily pigmented atypical melanocytes. The irregular dark color has a dermoscopic differential diagnosis. Ulceration vs irregular blotches. Blotches representing heavily pigmented atypical melanocytes. The reddish color represents vascularization. Most basal cell carcinomas are not so heavily pigmented. The minority of basal cell carcinomas (10 %) have spoke-wheel structures. Basal cell carcinomas do not always have arborizing vessels.

PEARLS ■ ■ ■

In general it is not always possible to be 100% sure of a given diagnosis. Be prepared to create a dermoscopic differential diagnosis. Err on the side of the most aggressive pathology when planning your surgical approach.

Chapter 2

Scalp, Face, Nose, and Ears

RISK ❑ Low ❑ Intermediate ❑ High

2-16a

DIAGNOSIS ❑ ❑ ❑ ❑ ❑ ❑ ❑ ❑

Nevus Seborrheic keratosis Basal cell carcinoma Vascular Dermatofibroma Squamous cell carcinoma Melanoma Other

DISPOSITION

2-16b

❑ No intervention ❑ Follow-up ❑ Histopathologic diagnosis

y

CASE 16 HISTORY A barber found this lesion on the scalp of a 61-year-old man. 1. Compared to Case 15, this basal cell carcinoma is easier to diagnose because there are classic arborizing vessels. 2. A bluish-white veil and irregular blotches plus corkscrew vessels characterize this melanoma. 3. Arborizing vessels are always thick and branch at 90° angles. 4. Typically the arborizing vessels seen in basal cell carcinomas are in-focus because they are located on the surface of the lesion. 5. In or out-of-focus arborizing vessels can also be found in melanomas.

59

60

DERMOSCOPY: AN ILLUSTRATED SELF-ASSESSMENT GUIDE

RISK ❑ Low ❑ Intermediate ✔ High ❑

DIAGNOSIS Nevus Seborrheic keratosis Basal cell carcinoma Vascular Dermatofibroma Squamous cell carcinoma Melanoma Other

2-16c

❑ ❑ ✔ ❑ ❑ ❑ ❑ ❑ ❑

DISPOSITION

ANSWERS

❑ No intervention ❑ Follow-up ❑ ✔ Histopathologic diagnosis

Answers: 1,4,5

DERMOSCOPIC CRITERIA ■ ■ ■ ■

Arborizing vessels (black arrows) Ulceration (yellow arrows) Bluish-white color (stars) Brown color (white arrows)

Discussion: ■



■ ■ ■

■ ■ ■

■ ■ ■

There are no criteria to diagnose a melanocytic lesion (ie, pigment network, globules), unless one considers the default category. ■ The foci of ovoid brown color are not a criterion that can be used to diagnose a melanocytic lesion. Arborizing vessels plus ulceration are the two major criteria to help make the diagnosis. Thick and thin bored arborizing vessels are classic. The vessels are very thin in this lesion. Arborizing vessels can also be found in melanomas but much less frequently than in basal cell carcinomas. Out-of-focus vessels should raise a red flag for concern. Out-of-focus arborizing vessels favor a diagnosis of melanoma. It is not necessary to use a protractor to determine the angles of branching vessels. Angles are not of diagnostic significance. The “classic” bluish-white veil is not present. Blue and white colors are commonly found in basal cell carcinomas.

PEARLS ■



The stool quaic test can be used to differentiate an ulceration with blood from an irregular melanocytic blotch, which would not contain heme. At any location on the body, do not hesitate to cut off some hair to get a better view.

Chapter 2

Scalp, Face, Nose, and Ears

RISK ❑ Low ❑ Intermediate ❑ High

2-17a

DIAGNOSIS ❑ ❑ ❑ ❑ ❑ ❑ ❑ ❑

Nevus Seborrheic keratosis Basal cell carcinoma Vascular Dermatofibroma Squamous cell carcinoma Melanoma Other

DISPOSITION

2-17b

❑ No intervention ❑ Follow-up ❑ Histopathologic diagnosis

CASE 17 HISTORY This 88-year-old woman has a biopsy proven basal cell carcinoma under a dark plaque. Clinically, both lesions seem to be connected. 1. 2. 3. 4.

Leaf-like structures fill the pigmented component of this basal cell carcinoma. Asymmetrically pigmented follicular openings are plentiful. A few rhomboid structures can be identified. Irregular blotches with and without follicular openings indicate dermal invasion in this melanoma. 5. Extensive regression characterizes this heavily pigmented basal cell carcinoma.

61

62

DERMOSCOPY: AN ILLUSTRATED SELF-ASSESSMENT GUIDE

RISK ❑ Low ❑ Intermediate ✔ High ❑

DIAGNOSIS Nevus Seborrheic keratosis Basal cell carcinoma Vascular Dermatofibroma Squamous cell carcinoma Melanoma Other

DISPOSITION ❑ No intervention ❑ Follow-up ✔ Histopathologic diagnosis ❑

2-17c

❑ ❑ ❑ ❑ ❑ ❑ ✔ ❑ ❑

ANSWERS Answers: 2,3,4

Discussion: ■ ■

DERMOSCOPIC CRITERIA ■















Follicular openings (yellow arrows) Asymmetrical follicular pigmentation (black arrows) Rhomboid structures (yellow box) Irregular blotch with follicular openings (black box) Irregular blotch without follicular openings (stars) Irregular dots and globules (yellow circles) Areas of regression (white arrows) Polymorphous vessels (red circles)













Clinically and dermoscopically, this is an obvious invasive melanoma. The first step in the progression of melanoma on the face is represented by asymmetrical follicular pigmentation. ■ The follicular openings should not be confused with milia-like cysts seen in seborrheic keratosis. The next step in the progression of melanoma on the face is the formation of rhomboid structures. True “rhomboid” forms (parallelogram with unequal angles and sides) are not necessary. Any pigmented thickening that surrounds follicular openings should be considered “rhomboid” structures. The irregular black blotch with follicular openings is the first sign of dermal invasion. The irregular black blotch without follicular openings represents complete obliteration of follicular openings indicating further dermal invasion. By definition, this is a collision tumor, representing a basal cell carcinoma and melanoma.

PEARLS ■ ■ ■

One does not need dermoscopy to realize that this is high risk pathology. The diagnosis of melanoma on the face is not always this easy. Dermoscopy is essential to help make the diagnosis in most cases of early lentigo maligna, which offers the patients their best chance for survivial.

Chapter 2

Scalp, Face, Nose, and Ears

RISK ❑ Low ❑ Intermediate ❑ High

2-18a

DIAGNOSIS ❑ ❑ ❑ ❑ ❑ ❑ ❑ ❑

Nevus Seborrheic keratosis Basal cell carcinoma Vascular Dermatofibroma Squamous cell carcinoma Melanoma Other

DISPOSITION

2-18b

❑ No intervention ❑ Follow-up ❑ Histopathologic diagnosis

CASE 18 HISTORY Over the past few months this asymptomatic pigmented skin lesion on a 74-year-old man has been increasing in size and getting darker. 1. Milia-like cysts and pseudofollicular openings characterize this seborrheic keratosis. 2. The irregular blotch and areas of regression are important clues that this could be a melanoma. 3. The multicomponent global pattern, asymmetry of color and structure, irregular dots and globules, and an irregular blotch plus regression characterize this melanoma. 4. Multiple follicular openings are not identifiable. 5. Based on the history, clinical, and dermoscopic appearance, sequential digital dermoscopic monitoring is indicated rather than a biopsy.

63

64

DERMOSCOPY: AN ILLUSTRATED SELF-ASSESSMENT GUIDE

RISK 1

❑ Low ❑ Intermediate ✔ High ❑

3 2

DIAGNOSIS Nevus Seborrheic keratosis Basal cell carcinoma Vascular Dermatofibroma Squamous cell carcinoma Melanoma Other

2-18c

❑ ❑ ❑ ❑ ❑ ❑ ✔ ❑ ❑

DISPOSITION ❑ No intervention ❑ Follow-up ✔ Histopathologic diagnosis ❑

ANSWERS Answers: 2,3,4

Discussion: ■ ■ ■

DERMOSCOPIC CRITERIA ■







■ ■ ■

Asymmetry of color and structure Multicomponent global pattern (1,2,3) Irregular dots and globules (circles) Irregular black blotch (yellow arrows) Regression (white arrows) Peppering (box) Follicular openings (black arrows)



■ ■ ■



The history, clinical, and dermoscopic picture are worrisome. There are no criteria to diagnose a seborrheic keratosis. The irregular black blotch could be seen in a heavily pigmented seborrheic keratosis. The typical site-specific and melanoma-specific criteria from the face are not seen. ■ Asymmetrical follicular pigmentation ■ Annular-granular structures ■ Rhomboid structures ■ Circle within a circle The regression area is not seen clinically. The regression area consists of blue and white colors plus peppering. One can expect to see multiple follicular openings on the face which are not seen here. Follicular openings can be found outside of the lesion on normal skin.

PEARLS ■







A large dark pigmented lesion on the face should always raise a red flag for concern. A large facial lesion could be a congenital melanocytic nevus, solar lentigo, or melanoma. The absence of criteria that favor low risk pathology (ie, fingerprint pattern) should raise a red flag for concern. If the expected site-specific criteria are absent, use the pattern analysis algorithm for the trunk and extremities or an algorithm of your choice to evaluate the lesion.

Chapter 2

Scalp, Face, Nose, and Ears

RISK ❑ Low ❑ Intermediate ❑ High

DIAGNOSIS

2-19a

❑ ❑ ❑ ❑ ❑ ❑ ❑ ❑

Nevus Seborrheic keratosis Basal cell carcinoma Vascular Dermatofibroma Squamous cell carcinoma Melanoma Other

DISPOSITION

2-19b

❑ No intervention ❑ Follow-up ❑ Histopathologic diagnosis

CASE 19 HISTORY A total body skin examination was performed on this 70-year-old man at his first visit with a new dermatologist. The patient was not aware of the lesion on his right ear lobe that the dermatologist found. 1. This is a classic pigmented seborrheic keratosis that does not need dermoscopic evaluation. 2. In order not to miss melanoma incognito, aka false negative melanomas, one should routinely examine lesions that do not look atypical clinically. 3. Asymmetry of color and structure, irregular dots, globules and blotches, asymmetrical follicular pigmentation and regression characterize this melanoma. 4. Symmetry of color and structure, regular dots and globules, and blotches characterize this small congenital melanocytic nevus. 5. The dramatic asymmetric follicular pigmentation should raise a red flag for concern.

65

66

DERMOSCOPY: AN ILLUSTRATED SELF-ASSESSMENT GUIDE

RISK ❑ Low ❑ Intermediate ✔ High ❑

DIAGNOSIS Nevus Seborrheic keratosis Basal cell carcinoma Vascular Dermatofibroma Squamous cell carcinoma Melanoma Other

2-19c

❑ ❑ ❑ ❑ ❑ ❑ ✔ ❑ ❑

DISPOSITION

ANSWERS

❑ No intervention ❑ Follow-up ✔ Histopathologic diagnosis ❑

Answers: 2,3,5

Discussion: ■

■ ■

DERMOSCOPIC CRITERIA ■













Asymmetry of color and structure Follicular openings (white arrows) Asymmetrical follicular pigmentation (black arrows) Asymmetrical follicular pigmentation units (white boxes) Irregular dots and globules (circles) Irregular dark blotches (black boxes) Regression (stars)

■ ■





■ ■

The clinical differential diagnosis includes a seborrheic keratosis or melanoma. This would be an unusual location for both pathologies. The fingerprint pattern, milia-like cysts, and pseudofollicular openings used to diagnose a seborrheic keratosis are absent. The asymmetrical follicular pigmentation units are dramatic. One should not confuse the follicular openings with milia-like cysts of a seborrheic keratosis. One infers that there is regression based on the clinical appearance of the lesion. The regression areas are not well developed because the color is not whiter than the surrounding skin. Gray color and “peppering” are absent. Criteria can be present yet do not always fit into the exact definitions given to them.

PEARLS ■

■ ■



On the face, nose, and ears, it is not always easy to identify the high risk criteria. It is essential to learn the definitions of the site-specific criteria. It is essential to have a color atlas of dermoscopy images, in your mind, of the important criteria. Focus your attention and make a careful search for the melanoma-specific criteria.

Chapter 2

Scalp, Face, Nose, and Ears

RISK ❑ Low ❑ Intermediate ❑ High

2-20a

DIAGNOSIS ❑ ❑ ❑ ❑ ❑ ❑ ❑ ❑

Nevus Seborrheic keratosis Basal cell carcinoma Vascular Dermatofibroma Squamous cell carcinoma Melanoma Other

DISPOSITION

2-20b

❑ No intervention ❑ Follow-up ❑ Histopathologic diagnosis

CASE 20 HISTORY A 74-year-old man has this spot on his face, which has not changed for many years. 1. “Moth-eaten” convex borders and pseudopigment network, aka pseudonetwork, characterize this solar lentigo. 2. “Moth-eaten” concave borders, pseudonetwork, remnants of a fingerprint pattern, and homogenous color characterize this solar lentigo. 3. The pigment network and pseudonetwork are both created by elongated hyperpigmented rete ridges. 4. The pseudonetwork is found only on the face, nose, and ears, and diagnoses melanocytic lesions. 5. The pseudonetwork can be seen in melanocytic and nonmelanocytic lesions.

67

68

DERMOSCOPY: AN ILLUSTRATED SELF-ASSESSMENT GUIDE

RISK ✔ Low ❑ ❑ Intermediate ❑ High

DIAGNOSIS Nevus Seborrheic keratosis Basal cell carcinoma Vascular Dermatofibroma Squamous cell carcinoma Melanoma Other

2-20c

❑ ❑ ❑ ❑ ❑ ❑ ❑ ✔ ❑

DISPOSITION ✔ No intervention ❑ ❑ Follow-up ❑ Histopathologic diagnosis

ANSWERS Answers: 2,5

Discussion: ■ ■

DERMOSCOPIC CRITERIA ■ ■

■ ■ ■

Sharp border demarcation Concave “moth-eaten” borders (black arrows) Pseudonetwork (boxes) Homogeneous color (stars) Remnants of the fingerprint pattern (white arrows)













The criteria to diagnose melanoma are not seen. Typically, solar lentigines have sharp borders. Typically, solar lentigines have concave borders, said to resemble a moth-eaten garment, known as “moth-eaten borders.” The pigment network found on the trunk and extremities is created by elongated hyperpigmented rete ridges and is not usually seen on the face, nose, or ears. Follicular and other appendageal openings that penetrate areas of pigmentation create the pseudonetwork. The terms pseudonetwork and pseudopigment network are used interchangeably. The fingerprint pattern made up of fine, light brown, parallel line segments resembling finger prints differs from the honeycomb-like line segments of the pigment network. Well developed or fragments of the fingerprint pattern are commonly seen. ■ In this case the fragments of the fingerprint pattern are very difficult to see and could be easily overlooked.

PEARLS ■





Despite common teaching, there is no evidence to substantiate that the pseudonetwork is a primary criterion to diagnose melanocytic lesions. The pseudonetwork is site-specific and can be seen in melanocytic and nonmelanocytic lesions. One has to identify other criteria to make a diagnosis once the pseudonetwork is identified.

Chapter 2

Scalp, Face, Nose, and Ears

RISK ❑ Low ❑ Intermediate ❑ High

2-21a

DIAGNOSIS ❑ ❑ ❑ ❑ ❑ ❑ ❑ ❑

Nevus Seborrheic keratosis Basal cell carcinoma Vascular Dermatofibroma Squamous cell carcinoma Melanoma Other

DISPOSITION

2-21b

❑ No intervention ❑ Follow-up ❑ Histopathologic diagnosis

CASE 21 HISTORY The pigmented skin lesion on the face of this 8-year-old boy was scheduled for excision by a plastic surgeon, under general anesthesia, after the patient was referred by a pediatric dermatologist. The dermatologist thought that it looked atypical clinically and there was a history of change. The lesion was never examined with dermoscopy. 1. Asymmetry of color and structure, irregular dots and globules characterizes this dysplastic nevus. 2. Nodular melanoma is in the clinical and dermoscopic differential diagnosis. 3. Minimal asymmetry of color and structure, foci of pseudopigment network, and heavy pigmentation characterize this small congenital melanocytic nevus. 4. Asymmetry of color and structure are commonly seen in congenital melanocytic nevi. 5. A history of change in a nevus is not always high risk, especially in a child.

69

70

DERMOSCOPY: AN ILLUSTRATED SELF-ASSESSMENT GUIDE

RISK ✔ Low ❑ ❑ Intermediate ❑ High

DIAGNOSIS Nevus Seborrheic keratosis Basal cell carcinoma Vascular Dermatofibroma Squamous cell carcinoma Melanoma Other

2-21c

✔ ❑ ❑ ❑ ❑ ❑ ❑ ❑ ❑

DISPOSITION ✔ No intervention ❑ ❑ Follow-up ❑ Histopathologic diagnosis

ANSWERS Answers: 3,4,5

Discussion: ■ ■

DERMOSCOPIC CRITERIA ■







Asymmetry of color and structure Appendageal openings (arrows) Pseudopigment network complex (homogeneous color and appendageal openings) Regular dots and globules (boxes)



■ ■ ■

Clinically and dermoscopically this is not high risk. Asymmetry of color and structure are commonly found in congenital melanocytic nevi of all sizes. Other experienced dermoscopists might not agree that there is asymmetry of color and structure. The pseudopigment network is easy to identify and expected in this location. The terms appendageal and follicular openings can be used interchangeably. At first blush, this lesion is similar to Case 18 however, there are no welldeveloped melanoma-specific criteria.

PEARLS ■ ■ ■

The majority of pigmented skin lesions in children are not high risk. Unnecessary surgery can be avoided in most children if dermoscopy is used. No child should ever have a pigmented skin lesion biopsied or removed before it is examined with dermoscopy.

Chapter 2

Scalp, Face, Nose, and Ears

RISK ❑ Low ❑ Intermediate ❑ High

2-22a

DIAGNOSIS ❑ ❑ ❑ ❑ ❑ ❑ ❑ ❑

Nevus Seborrheic keratosis Basal cell carcinoma Vascular Dermatofibroma Squamous cell carcinoma Melanoma Other

DISPOSITION

2-22b

❑ No intervention ❑ Follow-up ❑ Histopathologic diagnosis

CASE 22 HISTORY A 65-year-old man was referred by another dermatologist to evaluate this lesion with dermoscopy to determine if it should be biopsied. This history of the lesion is not known by the patient. 1. Clinically and dermoscopically a solar lentigo and in situ melanoma are in the differential diagnosis. 2. There are no high risk dermoscopic features in this solar lentigo. 3. The linear tracks and globules diagnose a melanoma. 4. Linear tracks and linear globules can be seen in benign and malignant pathology. 5. The “peppering” seen in this lesion indicates that there is a component of trauma.

71

72

DERMOSCOPY: AN ILLUSTRATED SELF-ASSESSMENT GUIDE

RISK ❑ Low ✔ Intermediate ❑ ❑ High 1

4

DIAGNOSIS

2

Nevus Seborrheic keratosis Basal cell carcinoma Vascular Dermatofibroma Squamous cell carcinoma Melanoma Other

DISPOSITION ❑ No intervention ❑ Follow-up ✔ Histopathologic diagnosis ❑

3

2-22c

❑ ❑ ❑ ❑ ❑ ❑ ❑ ✔ ❑

ANSWERS Answers: 1,4,5

Discussion: ■



DERMOSCOPIC CRITERIA ■











Asymmetry of color and structure Multicomponent global pattern (1,2,3,4) Irregular linear globules (yellow box) Irregular linear lines aka train tracks (arrows) Irregular thick network (black box) Peppering (white box)

■ ■

■ ■



Most solar lentigines on the lips have a banal dermoscopic appearance (ie, homogeneous brown color, fine parallel lines). It is not always possible to differentiate a solar lentigo from in situ melanoma clinically and with dermoscopy. In such situations a biopsy is indicated. Several distinct patterns of criteria can be seen on the lips and in genital mucosa (nonglabrous skin): ■ Regular or irregular linear globules ■ Parallel lines of globules sometimes creating a “fingerprint” pattern ■ Single thin or thick lines ■ Two parallel lines the “train track” pattern ■ Multiple parallel line segments another “fingerprint” pattern ■ Thin or thick network-like structures “Peppering” indicates trauma unless other high risk criteria are seen. Asymmetry of color and structure, a multicomponent global pattern, and irregular criteria should raise a red flag for concern. Blue-white color, dark irregular blotches, ulceration, and polymorphous vessels not identified here can be seen in invasive melanoma.

PEARLS ■







In this location surgery can be avoided in most cases after a banal picture is seen with dermoscopy. Avoiding surgery and the formation of potentially raised scars that can be very annoying to the patient, is an important benefit of dermoscopy. In gray-zone lesions sequential digital clinical and dermoscopic monitoring can be used. The most atypical dermoscopic area should be used for an incisional biopsy.

Chapter 2

Scalp, Face, Nose, and Ears

RISK ❑ Low ❑ Intermediate ❑ High

2-23a

DIAGNOSIS ❑ ❑ ❑ ❑ ❑ ❑ ❑ ❑

Nevus Seborrheic keratosis Basal cell carcinoma Vascular Dermatofibroma Squamous cell carcinoma Melanoma Other

DISPOSITION

2-23b

❑ No intervention ❑ Follow-up ❑ Histopathologic diagnosis

CASE 23 HISTORY This 57-year-old man’s main complaint was a slowly enlarging spot on his nose. 1. Arborizing telangiectases must always be present to diagnose a basal cell carcinoma on the nose. 2. The vessels in this lesion fall within the range of vessels that can be seen in basal cell carcinomas. 3. Blue, black, gray, brown, red, and white colors can be seen in basal cell carcinomas. 4. Asymmetry of color and structure, multiple colors, dots and globules, plus telangiectatic vessels characterize this pigmented basal cell carcinoma. 5. Pigment network, ovoid nets of blue pigment, ulceration, spoke-wheel structures, plus arborizing vessels are the criteria used to diagnose basal cell carcinomas.

73

74

DERMOSCOPY: AN ILLUSTRATED SELF-ASSESSMENT GUIDE

RISK ❑ Low ❑ Intermediate ✔ High ❑

DIAGNOSIS ❑ ❑ ✔ ❑ ❑ ❑ ❑ ❑ ❑

Nevus Seborrheic keratosis Basal cell carcinoma Vascular Dermatofibroma Squamous cell carcinoma Melanoma Other

DISPOSITION 2-23c

❑ No intervention ❑ Follow-up ✔ Histopathologic diagnosis ❑

ANSWERS Answers: 2,3,4

DERMOSCOPIC CRITERIA ■



■ ■ ■



Asymmetry of color and structure Nonarborizing telangiectatic “basal cell–like” vessels (black arrows) Blue globules (yellow arrows) Brown dots and globules (circle) Multiple colors—brown, blue, white Appendageal openings (red arrows)

Discussion: ■ ■







■ ■

Classic thick and thin arborizing vessels are not always found. On occasion, pinpoint, hairpin, linear, kinked, and other shaped telangiectatic vessels can be seen with or without arborizing vessels. The classic description of pigmentation that can be found in basal cell carcinomas includes: large gray-blue ovoid nests, gray-blue dots and globules, and leaf-like structures. Be aware that there are innumerable variations of size and shape of the pigmentation found in basal cell carcinomas. The absence of a pigment network helps to differentiate this from a melanoma. Pigment network is not seen in basal cell carcinomas. In order to diagnose a basal cell carcinoma there must not be a pigment network.

PEARLS ■







It is important to learn the definitions of the types of pigment that can be found in basal cell carcinomas. It is not essential to perfectly fit pigmentation into the specifically defined shapes and colors. If one follows the definition of how leaf-like structures should look, one will realize that they never look like any type of leaf! The concept of leaf-like structures is a misnomer that should be abandoned.

Chapter 2

Scalp, Face, Nose, and Ears

RISK ❑ Low ❑ Intermediate ❑ High

2-24a

DIAGNOSIS ❑ ❑ ❑ ❑ ❑ ❑ ❑ ❑

Nevus Seborrheic keratosis Basal cell carcinoma Vascular Dermatofibroma Squamous cell carcinoma Melanoma Other

DISPOSITION

2-24b

❑ No intervention ❑ Follow-up ❑ Histopathologic diagnosis

CASE 24 HISTORY A 71-year-old man’s barber found this asymptomatic lesion on his scalp. 1. Multiple brown globules categorize this as a melanocytic lesion. 2. The multicomponent global pattern and asymmetry of color and structure are significant red flags for concern. 3. Irregular dots and globules and a large irregular black blotch are more reason for concern. 4. The blue-white and gray colors of regression points in the direction of a melanoma. 5. Even though streaks and polymorphous vessels are absent, this still could be a melanoma.

75

76

DERMOSCOPY: AN ILLUSTRATED SELF-ASSESSMENT GUIDE

RISK ❑ Low ❑ Intermediate ✔ High ❑

1

DIAGNOSIS Nevus Seborrheic keratosis Basal cell carcinoma Vascular Dermatofibroma Squamous cell carcinoma Melanoma Other

DISPOSITION ❑ No intervention ❑ Follow-up ❑ ✔ Histopathologic diagnosis

2 3 4

2-24c

❑ ❑ ❑ ❑ ❑ ❑ ✔ ❑ ❑

ANSWERS Answers: 1,2,3,4,5

Discussion: ■ ■



DERMOSCOPIC CRITERIA ■





■ ■

Asymmetry of color and structure (+) Multicomponent global pattern (1,2,3,4) Irregular dots and globules (circles) Irregular black blotch (stars) Regression (arrows)

■ ■



It is essential to include the scalp in a total body skin examination. Criteria used to diagnose benign pathology are not seen (ie, cobblestone pattern, milia-like cysts, brain-like pattern). Superficial spreading and nodular melanoma are typically found on the scalp. One should have a systematic check list of criteria that need to be identified. Go over the checklist with each lesion: ■ Step 1: Is the lesion melanocytic or nonmelanocytic? If the lesion is melanocytic: ■ Step 2: To determine if it is low, intermediate, or high risk using the algorithm of your choice. If using pattern analysis: ■ Step 3: What is the global pattern? ■ Step 4: Is there symmetry or asymmetry of color and structure? ■ Step 5: Identify the local criteria and determine if they are regular or irregular, good or bad, low or high risk. ■ Step 6: Put everything together and make a dermoscopic diagnosis. Do not make a dermoscopic diagnosis before identifying all of the criteria in a lesion.

PEARLS ■ ■



Bright light is essential for a good scalp examination. The entire scalp should be carefully checked when doing a complete skin examination. The sooner the checklists are memorized, the sooner one is on their way to becoming a well-organized dermoscopist.

Chapter 2

Scalp, Face, Nose, and Ears

RISK ❑ Low ❑ Intermediate ❑ High

DIAGNOSIS

2-25a

❑ ❑ ❑ ❑ ❑ ❑ ❑ ❑

Nevus Seborrheic keratosis Basal cell carcinoma Vascular Dermatofibroma Squamous cell carcinoma Melanoma Other

DISPOSITION

2-25b

❑ No intervention ❑ Follow-up ❑ Histopathologic diagnosis

CASE 25 HISTORY A 77-year-old man was not aware of the pigmented lesion on his ear. 1. There are multiple asymmetrically pigmented follicular openings. 2. There are foci of rhomboid structures. 3. The irregular black blotches with and without follicular openings indicates dermal invasion of this lentigo maligna melanoma. 4. The irregular black blotches without follicular openings represents the “black lamella” indicating this pigmented lesion is benign. 5. If one has good eyesight and can perform a biopsy, dermoscopy is not helpful to diagnose lentigo maligna and its variants.

77

78

DERMOSCOPY: AN ILLUSTRATED SELF-ASSESSMENT GUIDE

RISK ❑ Low ❑ Intermediate ✔ High ❑

DIAGNOSIS Nevus Seborrheic keratosis Basal cell carcinoma Vascular Dermatofibroma Squamous cell carcinoma Melanoma Other

DISPOSITION ❑ No intervention ❑ Follow-up ✔ Histopathologic diagnosis ❑

2-25c

❑ ❑ ❑ ❑ ❑ ❑ ✔ ❑ ❑

ANSWERS Answers: 1,2,3

Discussion: ■



DERMOSCOPIC CRITERIA ■











Asymmetry of color and structure Follicular openings (red arrows) Asymmetrical follicular pigmentation (black arrows) Irregular black blotch with follicular openings (white arrows) Irregular black blotch without follicular openings (yellow arrows) Rhomboid structures (white boxes)





■ ■ ■ ■

Clinically, but not dermoscopically, this could be a seborrheic keratosis or melanoma. There are no dermoscopic criteria to diagnose a seborrheic keratosis (ie, milia-like cysts, pseudofollicular openings, fingerprint pattern). The “black lamella” could look like this—a black homogenous blotch with a glazed look. The “black lamella” created by pigmented parakeratosis is not seen in melanoma. Tape stripping often removes the “black lamella.” Tape stripping will not remove the irregular black blotch in this melanoma. By now the diagnosis should be relatively easy. There are many faces of melanoma and the diagnosis will not always be so easy.

PEARLS ■





One should memorize the criteria that are needed to diagnose lentigo maligna and lentigo maligna melanoma. One should have images of the classic examples of the high risk criteria cemented in their minds. Focus ones attention to make the diagnosis.

Chapter 2

Scalp, Face, Nose, and Ears

RISK ❑ Low ❑ Intermediate ❑ High

2-26a

DIAGNOSIS ❑ ❑ ❑ ❑ ❑ ❑ ❑ ❑

Nevus Seborrheic keratosis Basal cell carcinoma Vascular Dermatofibroma Squamous cell carcinoma Melanoma Other

DISPOSITION

2-26b

❑ No intervention ❑ Follow-up ❑ Histopathologic diagnosis

CASE 26 HISTORY A 76-year-old man has several gray scaly spots on his face that are sensitive to touch. 1. The clinical impression of a melanoma with regression is supported by the dermoscopic picture. 2. The gray pseudonetwork seen here is created by confluent annular-granular structures. 3. Annular-granular structures can be seen in pigmented actinic keratosis, lichen-planus keratosis, and lentigo maligna. 4. Annular-granular structures are the only criterion that is 100% diagnostic of lentigo maligna. 5. Multiple scaly lesions favor a diagnosis of pigmented actinic keratosis.

79

80

DERMOSCOPY: AN ILLUSTRATED SELF-ASSESSMENT GUIDE

RISK ❑ Low ✔ Intermediate ❑ ❑ High

DIAGNOSIS Nevus Seborrheic keratosis Basal cell carcinoma Vascular Dermatofibroma Squamous cell carcinoma Melanoma Other

2-26c

❑ ❑ ❑ ❑ ❑ ❑ ❑ ✔ ❑

DISPOSITION ❑ No intervention ❑ Follow-up ✔ Histopathologic diagnosis ❑

ANSWERS Answers: 2,3,5

Discussion: ■

DERMOSCOPIC CRITERIA ■ ■

■ ■

Follicular openings (arrows) Annular-granular structures (circles) Gray pseudonetwork (box) Brown color (star)



■ ■ ■





Annular-granular structures are created by fine gray or brown dots that surround follicular openings. When gray they can represent melanophages and/or free melanin or atypical melanocytes. Annular-granular structures can be localized or diffuse. Confluent annular-granular structure creates the gray pseudonetwork. The differential diagnosis in this case includes: pigmented actinic keratosis, lichen planus keratosis, and lentigo maligna. Clinical sensitivity often expressed by patients favors the diagnosis of pigmented actinic keratosis. Diffuse “peppering” and remnants of an actinic/solar lentigo or flat seborrheic keratosis (fingerprint pattern) are characteristic of a chronic long standing lichen planus-like keratosis.

PEARLS ■



■ ■

Pigmented actinic keratosis and solar lentigines can be associated with lentigo maligna. Look for other criteria associated with lentigo maligna before making a final diagnosis of a pigmented actinic keratosis. Use the most atypical area for an incisional biopsy. A rough scaly pigmented area is a clue that a lesion with this dermoscopic picture is a pigmented actinic keratosis.

Chapter 2

Scalp, Face, Nose, and Ears

RISK ❑ Low ❑ Intermediate ❑ High

2-27a

DIAGNOSIS ❑ ❑ ❑ ❑ ❑ ❑ ❑ ❑

Nevus Seborrheic keratosis Basal cell carcinoma Vascular Dermatofibroma Squamous cell carcinoma Melanoma Other

DISPOSITION

2-27b

❑ No intervention ❑ Follow-up ❑ Histopathologic diagnosis

CASE 27 HISTORY A 68-year-old man has an asymptomatic spot on his nose that was discovered by his wife. 1. 2. 3. 4. 5.

This barely visible tan macule does not warrant dermoscopic examination. Multiple milia-like cysts and pseudofollicular openings diagnose a seborrheic keratosis. Asymmetry of color and structure are a red flag for concern. Asymmetrical follicular pigmentation is another red flag for concern. For cosmetic reasons this lesion could be treated with liquid nitrogen without a biopsy.

81

82

DERMOSCOPY: AN ILLUSTRATED SELF-ASSESSMENT GUIDE

RISK ❑ Low ❑ Intermediate ✔ High ❑

DIAGNOSIS Nevus Seborrheic keratosis Basal cell carcinoma Vascular Dermatofibroma Squamous cell carcinoma Melanoma Other

2-27c

❑ ❑ ❑ ❑ ❑ ❑ ✔ ❑ ❑

DISPOSITION ❑ No intervention ❑ Follow-up ✔ Histopathologic diagnosis ❑

ANSWERS Answers: 3,4

Discussion: ■



DERMOSCOPIC CRITERIA ■







■ ■

Asymmetry of color and structure Different shades of brown and gray color Follicular openings (black circles) Asymmetrical follicular pigmentation (boxes) Irregular globules (yellow circle) Poorly defined border (black arrows)

■ ■





■ ■ ■

Compared with all of the cases with the same diagnosis, the dermoscopic picture is not dramatic. One should not confuse the follicular openings with milia-like cysts of a seborrheic keratosis. The overall picture is that of an irregular pseudonetwork. A uniform pseudonetwork would be made up of light brown homogeneous color studded with monomorphous round follicular openings. There would not be asymmetrical follicular pigmentation. One must focus attention to find the asymmetrically pigmented follicular openings. Use the area with asymmetrically pigmented follicular openings if an incisional biopsy is performed. The criteria for a solar lentigo (fingerprint pattern) are absent. Poorly defined borders seen here do not favor a solar lentigo. Sharp concave “moth-eaten” border favors a solar lentigo.

PEARLS ■ ■







Banal appearing pigmentation can represent lentigo maligna. Dermoscopy should not only be used to examine atypically pigmented skin lesions in order not to miss melanoma incognito. In this clinical scenario, an important clue is the absence of the fingerprint pattern. Immediately one should think this could be lentigo maligna and carefully search for the diagnostic criteria. This case points out the benefits of dermoscopy to diagnose early lentigo maligna. Use a Woods light to determine the extent of the pigmentation.

Chapter 2

Scalp, Face, Nose, and Ears

RISK ❑ Low ❑ Intermediate ❑ High

2-28a

DIAGNOSIS ❑ ❑ ❑ ❑ ❑ ❑ ❑ ❑

Nevus Seborrheic keratosis Basal cell carcinoma Vascular Dermatofibroma Squamous cell carcinoma Melanoma Other

DISPOSITION

2-28b

❑ No intervention ❑ Follow-up ❑ Histopathologic diagnosis

CASE 28 HISTORY A 36-year-old woman has this brown spot on her face for a few years. It is asymptomatic and there is no history of change. 1. Different shades of brown and gray colors plus asymmetry of color and structure are a red flag for concern. 2. The uniform pseudonetwork rules out lentigo maligna. 3. There are only two asymmetrically pigmented follicular openings. 4. The light areas have a differential diagnosis that includes hypopigmentation, areas of inflammation, or regression. 5. Well developed “moth-eaten” borders and fingerprint structures diagnose a solar lentigo.

83

84

DERMOSCOPY: AN ILLUSTRATED SELF-ASSESSMENT GUIDE

RISK ❑ Low ❑ Intermediate ✔ High ❑

DIAGNOSIS Nevus Seborrheic keratosis Basal cell carcinoma Vascular Dermatofibroma Squamous cell carcinoma Melanoma Other

2-28c

❑ ❑ ❑ ❑ ❑ ❑ ✔ ❑ ❑

DISPOSITION ❑ No intervention ❑ Follow-up ✔ Histopathologic diagnosis ❑

ANSWERS Answers: 1,3,4

Discussion: ■ ■ ■

DERMOSCOPIC CRITERIA ■













Asymmetry of color and structure Different shades of brown and gray color Follicular openings (yellow boxes) Asymmetrical follicular pigmentation (black boxes) Irregular dots and globules (circles) Multifocal hypopigmentation (stars) Moth-eaten borders (arrows)





■ ■ ■

The dermoscopic picture is not perfectly banal appearing. A uniform pseudonetwork is not seen. One cannot determine if there are classic “moth-eaten” borders and sharp demarcation. There are only two asymmetrically pigmented follicular openings (a clue that this could be lentigo maligna) that even experienced dermoscopist might not see. The hypopigmented areas have a grayish tinge, which suggests that there is inflammation. True regression is not seen. Well-formed site-specific melanoma- specific criteria are absent. The irregular dots and globules are a minor criterion that has no diagnostic significance.

PEARLS ■







Sequential short-term (3 months) digital clinical and dermoscopic monitoring would be one alternative to a biopsy in this cosmetically important area. One must be as knowledgeable as possible with difficult cases like this, in order to form a meaningful differential diagnosis. Look carefully for high risk criteria before making a benign dermoscopic diagnosis. Only subtle dermoscopic clues helped us diagnose this melanoma incognito in the form of lentigo maligna!

Chapter 2

Scalp, Face, Nose, and Ears

RISK ❑ Low ❑ Intermediate ❑ High

DIAGNOSIS ❑ ❑ ❑ ❑ ❑ ❑ ❑ ❑

Nevus Seborrheic keratosis Basal cell carcinoma Vascular Dermatofibroma Squamous cell carcinoma Melanoma Other

2-29a

DISPOSITION ❑ No intervention ❑ Follow-up ❑ Histopathologic diagnosis

CASE 29 HISTORY A less-experienced colleague e-mails you this image and wants to know if you think a biopsy is indicated. A history and clinical image were not supplied. 1. 2. 3. 4. 5.

There are annular-granular structures. There is asymmetry of color and structure. There is asymmetrical follicular pigmentation. The borders are poorly defined. The differential diagnosis includes a pigmented actinic keratosis vs lentigo maligna.

85

86

DERMOSCOPY: AN ILLUSTRATED SELF-ASSESSMENT GUIDE

RISK ❑ Low ❑ Intermediate ✔ High ❑

DIAGNOSIS Nevus Seborrheic keratosis Basal cell carcinoma Vascular Dermatofibroma Squamous cell carcinoma Melanoma Other

DISPOSITION ❑ No intervention ❑ Follow-up ✔ Histopathologic diagnosis ❑

2-29b

❑ ❑ ❑ ❑ ❑ ❑ ✔ ❑ ❑

ANSWERS Answers: 1,2,3,4,5

Discussion: ■



DERMOSCOPIC CRITERIA ■











Asymmetry of color and structure Different shades of tan and gray colors Follicular openings (black arrows) Asymmetrical follicular pigmentation (boxes) Annular-granular structures (circles) “Moth-eaten” borders (red arrows)









The striking dermoscopic findings are the annular-granular structures and asymmetrical follicular pigmentation. Once again there is an irregular pseudonetwork indicating the lesion is on the face. The annular-granular structures and asymmetrical follicular pigmentation are together in this lesion and are enough criteria to diagnose lentigo maligna. Interobserver agreement might be in conflict regarding the presence or absence of “moth-eaten” borders. Your advice to your colleague would include the differential diagnosis: pigmented actinic keratosis vs lentigo maligna. Your colleague should biopsy the area with annular-granular structures and asymmetrical follicular pigmentation.

PEARLS ■

■ ■



With knowledge and experience one can come up with a diagnosis, differential diagnosis, and plan without a history or clinical image just based on the dermoscopic picture. Make sure that your pathologist is a pigmented lesion expert. The histopathology of an actinic keratosis could be confused with lentigo maligna. If one thinks the lesion is a lentigo maligna but the biopsy reports an actinic keratosis biopsy another area.

Chapter 2

Scalp, Face, Nose, and Ears

RISK ❑ Low ❑ Intermediate ❑ High

2-30a

DIAGNOSIS ❑ ❑ ❑ ❑ ❑ ❑ ❑ ❑

Nevus Seborrheic keratosis Basal cell carcinoma Vascular Dermatofibroma Squamous cell carcinoma Melanoma Other

DISPOSITION

2-30b

❑ No intervention ❑ Follow-up ❑ Histopathologic diagnosis

CASE 30 HISTORY Two dermatologists saw this lesion and said it was nothing to worry about. They did not examine it with dermoscopy. 1. 2. 3. 4. 5.

The roundish white structures could be milia-like cysts. The roundish white structures could be follicular openings. “Moth-eaten” borders diagnose a solar lentigo. Irregular dots and globules are a red flag for concern. Foci of asymmetrical follicular pigmentation are present.

87

88

DERMOSCOPY: AN ILLUSTRATED SELF-ASSESSMENT GUIDE

RISK ❑ Low ❑ Intermediate ✔ High ❑

❑ ❑ ❑ ❑ ❑ ❑ ✔ ❑ ❑

Nevus Seborrheic keratosis Basal cell carcinoma Vascular Dermatofibroma Squamous cell carcinoma Melanoma Other

DISPOSITION ❑ No intervention ❑ Follow-up ✔ Histopathologic diagnosis ❑

Lentigo Maligna Solar Lentigo

ANSWERS Answers: 1,2,4,5

Discussion: ■

DERMOSCOPIC CRITERIA ■











Asymmetry of color and structure Different shades of brown color Follicular openings (black arrows) Annular-granular structures (circle) Asymmetrical follicular pigmentation (boxes) “Moth-eaten” borders (red arrows)

2-30c

DIAGNOSIS









■ ■ ■

A large solitary lesion on the face with different shades of brown color are the first red flags for concern. ■ Now is the time to focus one’s attention and look for high risk criteria associated with lentigo maligna. The differential diagnosis of the round structures includes follicular openings versus milia-like cysts. The entire picture is that of an irregular pseudonetwork filled with follicular openings. With foci of criteria associated with lentigo maligna, the “moth-eaten” borders are not enough to diagnose a large solar lentigo. Knowing that there is an association between solar lentigo and lentigo maligna was important in this case. Two biopsies were performed for academic reasons. The right side without any high risk criteria diagnosed a solar lentigo. The left side diagnosed lentigo maligna. ■ The biopsy was performed in the darker area where asymmetrical follicular pigmentation and annular-granular structures were seen with dermoscopy.

PEARLS ■



If one cannot differentiate a seborrheic keratosis from a melanocytic lesion based on the dermoscopy, look and palpate the lesion to help make the differentiation. Putting as much of the clinical picture together as possible is the best way to make the correct diagnosis.

Chapter 2

Scalp, Face, Nose, and Ears

RISK ❑ Low ❑ Intermediate ❑ High

DIAGNOSIS

2-31a

❑ ❑ ❑ ❑ ❑ ❑ ❑ ❑

Nevus Seborrheic keratosis Basal cell carcinoma Vascular Dermatofibroma Squamous cell carcinoma Melanoma Other

DISPOSITION

2-31b

❑ No intervention ❑ Follow-up ❑ Histopathologic diagnosis

CASE 31 HISTORY A 55-year-old man thought that a lesion on his forehead was getting darker. 1. Pseudofollicular openings, peppering, and remnants of the fingerprint pattern characterize this irritated seborrheic keratosis. 2. The gray pseudonetwork diagnoses a pigmented actinic keratosis. 3. Multiple circles within a circle are a red flag for concern and diagnose lentigo maligna. 4. Asymmetrical follicular pigmentation could be confused with circle within a circle structures. 5. Multiple milia-like cysts are enough to diagnose a seborrheic keratosis.

89

90

DERMOSCOPY: AN ILLUSTRATED SELF-ASSESSMENT GUIDE

RISK ❑ Low ❑ Intermediate ✔ High ❑

DIAGNOSIS Nevus Seborrheic keratosis Basal cell carcinoma Vascular Dermatofibroma Squamous cell carcinoma Melanoma Other

DISPOSITION ❑ No intervention ❑ Follow-up ✔ Histopathologic diagnosis ❑

2-31c

❑ ❑ ❑ ❑ ❑ ❑ ✔ ❑ ❑

ANSWERS Answers: 3,4

Discussion: ■



DERMOSCOPIC CRITERIA ■



■ ■





■ ■ ■

Asymmetry of color and structure Different shades of brown and gray color Hair shafts (black arrows) Follicular pigmentation (yellow arrows) Circle within a circle units (black boxes) Asymmetrical follicular pigmentation (white box) Fingerprint pattern (red box) Peppering (yellow box) Gray color (stars)









At first blush, the striking asymmetry of color and structure and different shades of brown and gray color should be a red flag for concern. Annular-granular structures (atypical melanocytes) vs peppering (melanophages and/or free melanin) seen throughout the lesion is the differential diagnosis for the gray color—another red flag for concern Circle within a circle is a poorly studied unique structure composed of a central hair shaft and gray pigmentation (atypical melanocytes and/or peppering) that surround the hair shaft. ■ More work needs to be done to study this criterion to better delineate the histopathologic correlation. The differential diagnosis for the circle within a circle would be asymmetrical follicular pigmentation, pseudofollicular openings, and milia-like cysts. The central hair shaft differentiates the circle within a circle from follicular openings with pigmentation. A focus of well-developed fingerprint pattern (parallel line segments) makes the dermoscopic association of lentigo maligna and solar lentigo.

PEARLS ■

■ ■

■ ■ ■

The German philosopher Johann Wolfgang Goethe once wrote “One cannot see what one does not know.” (In German of course!) The concept of circle within a circle is not well known. Circle within a circle might be the only criterion seen to diagnose lentigo maligna. Mastering dermoscopy by “osmosis” cannot be done. Put in the time to learn as much dermoscopy as possible. The learning curve is steep!

Chapter 2

Scalp, Face, Nose, and Ears

RISK ❑ Low ❑ Intermediate ❑ High

2-32a

DIAGNOSIS ❑ ❑ ❑ ❑ ❑ ❑ ❑ ❑

Nevus Seborrheic keratosis Basal cell carcinoma Vascular Dermatofibroma Squamous cell carcinoma Melanoma Other

DISPOSITION

2-32b1

❑ No intervention ❑ Follow-up ❑ Histopathologic diagnosis

CASE 32 HISTORY These two lesions were found on the forehead of a 70-year-old man with a history of actinic keratosis, basal, and squamous cell carcinoma.

LESION 1 1. Globules diagnose a melanocytic lesion. 2. Irregular pigmented pseudofollicular openings and milia-like cysts diagnose a seborrheic keratosis. 3. On close inspection there is significant asymmetry of color and structure. 4. The blue-white color and irregular dark blotches are a red flag for concern. 5. This could not be a melanoma because the typical site-specific and melanoma-specific criteria are not seen.

91

92

DERMOSCOPY: AN ILLUSTRATED SELF-ASSESSMENT GUIDE

RISK ❑ Low ❑ Intermediate ✔ High ❑

1

2

DIAGNOSIS Nevus Seborrheic keratosis Basal cell carcinoma Vascular Dermatofibroma Squamous cell carcinoma Melanoma Other

3

2-32b2

❑ ❑ ❑ ❑ ❑ ❑ ✔ ❑ ❑

DISPOSITION ❑ No intervention ❑ Follow-up ✔ Histopathologic diagnosis ❑

ANSWERS Answers: 1,3,4

Discussion: ■ ■

DERMOSCOPIC CRITERIA ■









■ ■

Asymmetry of color and structure Multicomponent global pattern (1,2,3) vs Irregular cobblestone global pattern (irregular, large angulated globules) Irregular dots and globules (circles) Irregular blotches (star and yellow arrows) Blue-white color (white arrows) Asymmetrical follicular pigmentation (red arrows)









At first blush this looks more like a seborrheic keratosis than melanoma. The irregular dots and globules could be confused with pigmented pseudofollicular openings characteristic of a seborrheic keratosis. A melanoma on the face with a cobblestone-like pattern is very unusual. ■ There are no well-developed site-specific and melanoma-specific criteria to diagnose this in situ melanoma. The bizarre-shaped blotches (yellow arrows) favor the diagnosis of irregular pseudofollicular openings of a seborrheic keratosis. The bluish-white color and large dark blotch could be found in a seborrheic keratosis. This difficult case could lead to a misdiagnosis of a seborrheic keratosis with the initiation of inappropriate destructive therapy before making a histopathologic diagnosis.

PEARLS ■





This lesion points out the need for a differential diagnosis for global patterns, local criteria, and the potential diagnosis. If destructive therapy in a lesion like this is not successful, it is better to make a histopathologic diagnosis as soon as possible rather than carrying out additional blind destructive therapies. Dermoscopy is not a perfect technique. Don’t expect it to be!

Chapter 2

Scalp, Face, Nose, and Ears

RISK ❑ Low ❑ Intermediate ❑ High

DIAGNOSIS Nevus Seborrheic keratosis Basal cell carcinoma Vascular Dermatofibroma Squamous cell carcinoma Melanoma Other

2-32c1

❑ ❑ ❑ ❑ ❑ ❑ ❑ ❑

DISPOSITION ❑ No intervention ❑ Follow-up ❑ Histopathologic diagnosis

LESION 2 1. 2. 3. 4.

The classic pseudonetwork diagnoses a solar lentigo. Ulceration and pigmentation characterize this basal cell carcinoma. Different shades of pink and brown are a red flag for concern. Several asymmetrically pigmented follicular openings are an important clue that this could be another melanoma. 5. This could be a feature-poor melanoma.

93

94

DERMOSCOPY: AN ILLUSTRATED SELF-ASSESSMENT GUIDE

RISK ❑ Low ❑ Intermediate ✔ High ❑

DIAGNOSIS Nevus Seborrheic keratosis Basal cell carcinoma Vascular Dermatofibroma Squamous cell carcinoma Melanoma Other

2-32c2

❑ ❑ ❑ ❑ ❑ ❑ ✔ ❑ ❑

DISPOSITION ❑ No intervention ❑ Follow-up ✔ Histopathologic diagnosis ❑

ANSWERS Answers: 3,4,5

Discussion: ■

DERMOSCOPIC CRITERIA ■

■ ■



■ ■

Different shades of pink and brown color Regression (stars) Asymmetrical follicular pigmentation (yellow boxes) Irregular dots and globules (blaick box) Scale (red arrows) Ulceration (black arrows)











■ ■

The pink color is the first clue of potentially high risk pathology. ■ “If there’s pink, stop and think !” Two completely different, melanomas in the same patient appearing side by side, is extremely unusual. Complete clinical and histopathologic separation rules out a single melanoma. The lesion is feature-poor because there are not well-defined high risk criteria. ■ This feature-poor pink lesion turned out to be an early invasive melanoma! The asymmetrically pigmented follicular openings could be missed even by the most experienced dermoscopist. Different shades of brown and especially pink color are clues to the seriousness of this lesion. The irregular dots and globules are a clue that the lesion is melanocytic. The ulceration was created by trauma.

PEARLS ■

■ ■

Even the most experienced dermoscopists routinely have difficult cases in which dermoscopy is inconclusive. It is a fact of life that everyone misses or misdiagnoses melanoma. Dermoscopists have a much better chance to avoid this unpleasant situation.

Chapter 3

Trunk and Extremities

General Instructions: You will find a list of True/False statements following each case history. Select any statements, which you believe to be true. There may be one, more than one or no true statements for any given case. Choose the correct risk, diagnosis and disposition for each case. Then, turn the page to find a detailed discussion and pearls for each case.

Chapter 3

Trunk and Extremities

RISK ❑ Low ❑ Intermediate ❑ High

DIAGNOSIS 3-1a

❑ ❑ ❑ ❑ ❑ ❑ ❑ ❑

Nevus Seborrheic keratosis Basal cell carcinoma Vascular Dermatofibroma Squamous cell carcinoma Melanoma Other

DISPOSITION

3-1b

❑ No intervention ❑ Follow-up ❑ Histopathologic diagnosis

CASE 1 HISTORY A 32-year-old man has many small banal appearing nevi on his trunk and extremities. A nevus on his left arm looked different than the others. 1. 2. 3. 4. 5.

This is a melanocytic lesion because there is a pigment network. It has a reticular global pattern. There is symmetry of color and structure. A uniform, regular pigment network fills most of the lesion. Multifocal hypopigmentation and irregular pigment network diagnose a moderately dysplastic nevus.

97

98

DERMOSCOPY: AN ILLUSTRATED SELF-ASSESSMENT GUIDE

RISK ✔ Low ❑ ❑ Intermediate ❑ High

DIAGNOSIS Nevus Seborrheic keratosis Basal cell carcinoma Vascular Dermatofibroma Squamous cell carcinoma Melanoma Other

3-1c

✔ ❑ ❑ ❑ ❑ ❑ ❑ ❑ ❑

DISPOSITION

ANSWERS

✔ No intervention ❑ ❑ Follow-up ❑ Histopathologic diagnosis

Answers: 1,2,3,4

Discussion: ■



DERMOSCOPIC CRITERIA ■





Regular pigment network (black boxes) Irregular pigment network (white boxes) Multifocal hypopigmentation (black arrows)











■ ■





Melanoma is in the clinical differential diagnosis because there are slightly irregular borders, several dark colors, and it is an “ugly duckling.” One will not see the entire picture until the lesion is examined with dermoscopy. It could have looked much worse! The “ugly duckling” is a pigmented skin lesion that clinically and/or dermoscopically is different from the other lesions on a patient. An “ugly duckling” lesion is a red flag for concern; however, it is not always high risk. The dermoscopic picture seen here should give one reassurance that this is not a melanoma. By definition this is the reticular global pattern because the lesion is made up of pigment network. There are foci of irregular pigment network with broadened, branched, and broken up line segments. Irregular pigment network is not always associated with high risk pathology. Multifocal hypopigmentation and irregular pigment network can be found in dysplastic nevi. The hypopigmentaion could represent normal skin that commonly is found in melanocytic nevi. There are no well-developed melanoma-specific criteria, aka high risk criteria.

PEARLS ■ ■





If in doubt cut it out. With experience one will become less and less concerned with this entire clinical scenario. Digital clinical and dermoscopic monitoring for changes over time is a cutting-edge way to handle this case. An experienced dermoscopist would find this lesion perfectly banal appearing.

Chapter 3

Trunk and Extremities

RISK ❑ Low ❑ Intermediate ❑ High

DIAGNOSIS

3-2a

❑ ❑ ❑ ❑ ❑ ❑ ❑ ❑

Nevus Seborrheic keratosis Basal cell carcinoma Vascular Dermatofibroma Squamous cell carcinoma Melanoma Other

DISPOSITION

3-2b

❑ No intervention ❑ Follow-up ❑ Histopathologic diagnosis

CASE 2 HISTORY This asymptomatic pigmented skin lesion was found by a dermatologist while performing total body skin examination. 1. Compared to Case 1, the global dermoscopic picture is more worrisome. 2. There is definite asymmetry of color and structure. 3. Asymmetry of color and structure, irregular blotches, and foci of regression characterize this in situ melanoma. 4. This could be a dysplastic nevus characterized by asymmetry of color and structure, irregular pigment network, and multifocal hypopigmentation. 5. Skin markings traversing the lesion are creating the linear hypopigmentation.

99

100

DERMOSCOPY: AN ILLUSTRATED SELF-ASSESSMENT GUIDE

RISK ✔ Low ❑ ❑ Intermediate ❑ High

DIAGNOSIS Nevus Seborrheic keratosis Basal cell carcinoma Vascular Dermatofibroma Squamous cell carcinoma Melanoma Other

3-2c

✔ ❑ ❑ ❑ ❑ ❑ ❑ ❑ ❑

DISPOSITION ✔ No intervention ❑ ❑ Follow-up ❑ Histopathologic diagnosis

ANSWERS Answers: 1,2,4,5

Discussion: ■

DERMOSCOPIC CRITERIA ■





Asymmetry of color and structure (+) Irregular pigment network (black boxes) Multifocal hypopigmentation (black arrows)

■ ■

■ ■

■ ■

Compared to Case 1 the global/overall dermoscopic picture is more worrisome. The irregular pigment network is more widespread than in Case 1. Linear hypopigmentation is commonly seen and is created by the normal skin markings that traverse the lesion. The bony-white color of regression is not seen. A banal acquired nevus and mildly dysplastic nevus are in this differential diagnosis. Melanoma is not in the clinical and dermoscopic differential diagnosis. Multiple similar appearing nevi are less worrisome than a single lesion that looks like this.

PEARLS ■



For the novice dermoscopist, digital clinical and dermoscopic monitoring for changes over time would be a cutting-edge way to handle this case. An experienced dermosopist might find this lesion perfectly banal appearing and find no need for follow-up.

Chapter 3

Trunk and Extremities

RISK ❑ Low ❑ Intermediate ❑ High

DIAGNOSIS

3-3a

❑ ❑ ❑ ❑ ❑ ❑ ❑ ❑

Nevus Seborrheic keratosis Basal cell carcinoma Vascular Dermatofibroma Squamous cell carcinoma Melanoma Other

DISPOSITION

3-3b

❑ No intervention ❑ Follow-up ❑ Histopathologic diagnosis

CASE 3 HISTORY The daughter of this 52-year-old woman noticed an “irregular mole” on her mother’s back. 1. Pigment network and globules identify a melanocytic lesion. 2. There is significant asymmetry of color and structure. 3. A regular pigment network, regular dots and globules plus regular dark blotches diagnose a congenital melanocytic nevus. 4. Asymmetry of color and structure, irregular dots and globules, irregular dark blotches, plus the suggestion of regression diagnose a melanoma. 5. One can tell by the colors and structures that this is not a deep melanoma.

101

102

DERMOSCOPY: AN ILLUSTRATED SELF-ASSESSMENT GUIDE

RISK ❑ Low ❑ Intermediate ✔ High ❑

1

2

DIAGNOSIS Nevus Seborrheic keratosis Basal cell carcinoma Vascular Dermatofibroma Squamous cell carcinoma Melanoma Other

2 2 3

3

3-3c

❑ ❑ ❑ ❑ ❑ ❑ ✔ ❑ ❑

1

DISPOSITION ❑ No intervention ❑ Follow-up ✔ Histopathologic diagnosis ❑

ANSWERS Answers: 1,2,4,5

Discussion: ■ ■ ■

DERMOSCOPIC CRITERIA ■











Asymmetry of color and structure (+) Multicomponent global pattern (1, 2, 3) Regular pigment network (black boxes) Irregular dots and globules (circles) Irregular dark blotches (black arrows) Regression (stars)



■ ■ ■

■ ■



Compared to Cases 1 and 2 the dermoscopic picture is worse. There are no irregular dark blotches in Cases 1 and 2. The presence of irregular dark blotches is a major red flag for concern. The asymmetry of color and structure, and irregular dark blotches but not the irregular dots and globules are the main high risk features in this lesion. It is debatable if true regression is present. The grayish color favors regression vs. hypopigmentation. The global pattern could also be described as reticular-homogeneous with foci of pigment network and homogeneous featureless dark color. In situ melanoma and a dysplastic nevus are in the differential diagnosis. In a flat or slightly raised lesion, the presence of local criteria (ie, pigment network, dots and globules) and light plus dark brown plus gray colors are all clues that this will not be a deep melanoma. These are all superficially located criteria. ■ Black color represents pigment in the epidermis. ■ In a nodular lesion, black color can be seen in deeply invasive melanoma ■ Light and dark brown color represents pigment at the dermal-epidermal junction. ■ Gray represents pigment in the papillary dermis. As a melanoma gets more invasive, local criteria tend to be less prominent and one see colors associated with a deeper location in the dermis (ie, blue, steel blue, pink, red).

PEARLS ■ ■

Clinically and dermoscopically, one can diagnose an in situ melanoma. With this information in hand, one can plan an excision with a 5mm border that would be adequate for an in situ melanoma, thereby getting the job done with one rather than typical two surgical procedures needed for most melanomas.

Chapter 3

Trunk and Extremities

RISK ❑ Low ❑ Intermediate ❑ High

DIAGNOSIS

3-4a

❑ ❑ ❑ ❑ ❑ ❑ ❑ ❑

Nevus Seborrheic keratosis Basal cell carcinoma Vascular Dermatofibroma Squamous cell carcinoma Melanoma Other

DISPOSITION

3-4b

❑ No intervention ❑ Follow-up ❑ Histopathologic diagnosis

CASE 4 HISTORY Two lesions catch your eye as potential melanomas when examining this patient. 1. 2. 3. 4. 5.

Pigment network and globules identify a melanocytic lesion. There is asymmetry of color and structure and the multicomponent global pattern. The pigment network, globules, and dark blotches are irregular. Regression fills the lower half of the lesion. Blue and/or white color represents another high risk criterion.

103

104

DERMOSCOPY: AN ILLUSTRATED SELF-ASSESSMENT GUIDE

RISK ❑ Low ❑ Intermediate ✔ High ❑

2 1

DIAGNOSIS 4

Nevus Seborrheic keratosis Basal cell carcinoma Vascular Dermatofibroma Squamous cell carcinoma Melanoma Other

DISPOSITION ❑ No intervention ❑ Follow-up ✔ Histopathologic diagnosis ❑

3

3-4c

❑ ❑ ❑ ❑ ❑ ❑ ✔ ❑ ❑

ANSWERS Answers: 1,2,3,4,5

Discussion: ■ ■

DERMOSCOPIC CRITERIA ■









■ ■ ■

Asymmetry of color and structure (+) Multicomponent global pattern (1, 2, 3, 4) Irregular pigment network (boxes) Irregular dots and globules (circles) Irregular dark blotches (white arrows) Blue-white color (stars) Regression (black arrows) Multiple colors









Step 1 is to determine if the lesion is melanocytic or nonmelanocytic. If melanocytic, Step 2 is to determine if it is high risk (ie, dysplastic nevus or melanoma). Pattern analysis: ■ What is the global pattern? ■ Is there symmetry or asymmetry of color and structure? ■ Identify all of the local criteria. ■ Are the local criteria regular or irregular? ■ Put everything together and make a dermoscopic diagnosis. This invasive melanoma has foci with overlapping of criteria (ie, irregular pigment network and irregular dark blotches, regression and remnants of fine pigment network, bluish-white color over brown color). Bluish-white color but not the classic “bluish-white veil” is present → a high risk criterion. The irregular dark blotches could be considered hyperpigmentation.

PEARL ■

If one wants to be thorough with the analysis of each lesion: ■ Memorize a checklist of things to look for. ■ Go over the checklist with every lesion. ■ Analyze all of the criteria in a lesion before making a diagnosis.

Chapter 3

Trunk and Extremities

RISK ❑ Low ❑ Intermediate ❑ High

3-5a1

3-5a2

DIAGNOSIS ❑ ❑ ❑ ❑ ❑ ❑ ❑ ❑

Nevus Seborrheic keratosis Basal cell carcinoma Vascular Dermatofibroma Squamous cell carcinoma Melanoma Other

DISPOSITION

3-5b

❑ No intervention ❑ Follow-up ❑ Histopathologic diagnosis

CASE 5 HISTORY This was the other suspicious lesion found on back of the patient from the last case. 1. 2. 3. 4. 5.

Clinically but not dermoscopically this could be diagnosed as a seborrheic keratosis. Gray dots and globules diagnose a melanocytic lesion. Brown dots and globules diagnose a melanocytic lesion. Pinkish-white color and peppering characterize the area of regression. There are foci of polymorphous vessels; however, they are hard to differentiate from the peppering.

105

106

DERMOSCOPY: AN ILLUSTRATED SELF-ASSESSMENT GUIDE

RISK 1

❑ Low ❑ Intermediate ✔ High ❑

2

DIAGNOSIS Nevus Seborrheic keratosis Basal cell carcinoma Vascular Dermatofibroma Squamous cell carcinoma Melanoma Other

DISPOSITION ❑ No intervention ❑ Follow-up ✔ Histopathologic diagnosis ❑

3

ANSWERS Answers: 1,3,4,5

Discussion: ■

■ ■ ■

DERMOSCOPIC CRITERIA ■





■ ■ ■ ■

Asymmetry of color and structure (+) Multicomponent global pattern (1, 2, 3) Irregular brown dots and globules (black boxes) Regression (stars) Peppering (red boxes) Polymorphous vessels (circles) Hyperkeratosis (arrows)

3

3-5c

❑ ❑ ❑ ❑ ❑ ❑ ✔ ❑ ❑





■ ■

Brown, not gray, dots and globules are one criterion used to diagnose a melanocytic lesion. There are no criteria to suggest that this is a seborrheic keratosis. Regression fills most of the lesion. The gray dots and globules “peppering” plus the pinkish-white color make up the area of regression. Asymmetry of color and structure plus the multicomponent global pattern are easy to identify if one knows how they are defined. Foci of pinpoint and linear vessels (aka the polymorphous vascular pattern) are hard to differentiate from the peppering. ■ Gray color favors peppering vs. red color favoring pinpoint vessels. ■ Inter-observer agreement even among expert dermoscopists might differ on the best name for all of the small dots. ■ This brings up the concept of dermoscopic differential diagnosis. ■ Peppering with foci of vessels vs pure peppering without vessels. The hyperkeratosis has no significance. Putting everything together one could not diagnose anything other than a regressive melanoma.

PEARLS ■



■ ■ ■

One is obligated to examine most of the pigmented skin lesions in a patient with so many lesions. A good plan with such a difficult case is to have the patient return another day for a re-examination. Final conclusions should not be made hastily. This patient would benefit from total body photography. This patient would benefit from mapping out her nevi dermoscopically for sequential monitoring to look for significant changes over time.

Chapter 3

Trunk and Extremities

RISK ❑ Low ❑ Intermediate ❑ High

DIAGNOSIS

3-6a

❑ ❑ ❑ ❑ ❑ ❑ ❑ ❑

Nevus Seborrheic keratosis Basal cell carcinoma Vascular Dermatofibroma Squamous cell carcinoma Melanoma Other

DISPOSITION

3-6b

❑ No intervention ❑ Follow-up ❑ Histopathologic diagnosis

CASE 6 HISTORY This clinically banal looking nevus was found on the back of a 7-year-old girl. 1. 2. 3. 4. 5.

There isn’t a good clinico-dermoscopic correlation. Brown peripheral globules identify a melanocytic lesion. The asymmetry of color and structure is a red flag for concern. There are foci of markedly irregular pigment network. Asymmetry of color and structure, irregular pigment network, irregular dots and globules, multifocal hypopigmentation, and diffuse erythema characterize this dysplastic nevus.

107

108

DERMOSCOPY: AN ILLUSTRATED SELF-ASSESSMENT GUIDE

RISK ❑ Low ✔ Intermediate ❑ ❑ High

1

2

DIAGNOSIS Nevus Seborrheic keratosis Basal cell carcinoma Vascular Dermatofibroma Squamous cell carcinoma Melanoma Other

DISPOSITION ❑ No intervention ❑ Follow-up ✔ Histopathologic diagnosis ❑

3

3-6c

✔ ❑ ❑ ❑ ❑ ❑ ❑ ❑ ❑

ANSWERS Answers: 1,2,3,4,5

Discussion: ■



DERMOSCOPIC CRITERIA ■











Asymmetry of color and structure (+) Multicomponent global pattern (1, 2, 3) Irregular pigment network (boxes) Irregular dots and globules (circles) Multifocal hypopigmentation (stars) Diffuse erythema











■ ■





In this age group, a nevus that looks like this clinically should have a globular or cobblestone pattern with dermoscopy. Since it does not look so, there is not a good clinico-dermoscopic correlation → a red flag for concern. When there is not a good clinico-dermoscopic correlation, one should proceed with caution and look for high risk criteria. Peripheral dots and globules are routinely found in children and indicate an actively changing nevus. The peripheral dots and globules that can be seen in Spitz nevi would be more numerous. Sequential monitoring of a nevus with peripheral dots and globules will show that it enlarges over time. The pigment network is markedly irregular. ■ Foci are asymmetrically located. ■ They are irregularly pigmented, broken-up, branched and thickened. Diffuse erythema is commonly found in dysplastic nevi. Pressure with instrumentation blanched away the erythema revealing multifocal hypopigmentation. Multifocal hypopigmenation, not to be confused with regression, is commonly found in dysplastic nevi. This lesion is more benign than malignant looking with a typical pattern seen in dysplastic nevi.

PEARL ■

In order not to miss incognito lesions (ie, lesions that look banal clinically but not dermoscopically) such as dysplastic nevi, Spitz nevi, and melanoma: ■ Routinely examine with dermoscopy both banal and atypical appearing skin lesions.

Chapter 3

Trunk and Extremities

RISK ❑ Low ❑ Intermediate ❑ High

DIAGNOSIS

3-7a

❑ ❑ ❑ ❑ ❑ ❑ ❑ ❑

Nevus Seborrheic keratosis Basal cell carcinoma Vascular Dermatofibroma Squamous cell carcinoma Melanoma Other

DISPOSITION

3-7b

❑ No intervention ❑ Follow-up ❑ Histopathologic diagnosis

CASE 7 HISTORY This lesion was found on the hairy chest of a 56-year-old man. 1. 2. 3. 4.

Clinically and dermoscopically this is a typical benign appearing nevus spilus. Clinically and dermoscopically this could be a benign agminated nevus. Pigment network and globules identifies a melanocytic lesion. Irregular pigment network, irregular dots and globules, hyperpigmentation and areas of regression are the main features of this melanoma. 5. Islands of normal skin plus foci of regular pigment network and regular blotches characterize this benign congenital melanocytic nevus.

109

110

DERMOSCOPY: AN ILLUSTRATED SELF-ASSESSMENT GUIDE

RISK ❑ Low ❑ Intermediate ✔ High ❑

DIAGNOSIS Nevus Seborrheic keratosis Basal cell carcinoma Vascular Dermatofibroma Squamous cell carcinoma Melanoma Other

DISPOSITION ❑ No intervention ❑ Follow-up ✔ Histopathologic diagnosis ❑

3-7c

❑ ❑ ❑ ❑ ❑ ❑ ✔ ❑ ❑

ANSWERS Answers: 3, 4

Discussion: ■ ■

DERMOSCOPIC CRITERIA ■











Asymmetry of color and structure Irregular pigment network (boxes) Irregular dots and globules (circles) Irregular brown pigmentation (hyperpigmentation) Multifocal hypopigmentation (white arrows) Regression (stars)







The tan lentiginous background characteristic of a nevus spilus is not seen. Agminated nevi consisting of multiple grouped similar pigmented lesions remain in the clinical and dermoscopic differential diagnosis. ■ Agminated nevi have been reported in association with congenital, acquired, blue, and Spitz nevi. The prominent irregular pigment network and areas of regression are the main diagnostic features of this early invasive melanoma. The areas of regression are not whiter than the surrounding skin and do not contain “peppering.” ■ Criteria do not always conform to exact definitions. ■ Clinically regression is clearly apparent. The irregular dots and globules and hyperpigmentation are more high risk criteria that help make the diagnosis.

PEARLS ■ ■







Dysplastic nevi and melanoma can rarely develop in a nevus spilus. As a general rule, everything that one evaluates with dermoscopy should be categorized as being regular or irregular, good or bad, low or high risk. ■ There are no exceptions to this rule. It is essential to study images with the classic examples of all the important dermoscopic principles. Routinely one encounters variations of the classic morphology of global patterns and local criteria. Do not hesitate to cut away some hair if you need to get a clearer look in a hairy area.

Chapter 3

Trunk and Extremities

RISK ❑ Low ❑ Intermediate ❑ High

DIAGNOSIS

3-8a

❑ ❑ ❑ ❑ ❑ ❑ ❑ ❑

Nevus Seborrheic keratosis Basal cell carcinoma Vascular Dermatofibroma Squamous cell carcinoma Melanoma Other

DISPOSITION

3-8b

❑ No intervention ❑ Follow-up ❑ Histopathologic diagnosis

CASE 8 HISTORY A 70-year-old woman was concerned about this changing lesion on her left shin. 1. Historically and clinically, more so than dermoscopically, this is suspicious for a melanoma. 2. There are no dramatic high risk dermoscopic criteria. 3. The asymmetry of color and structure and a focus of an irregular pigment network are not high risk criteria. 4. Asymmetry of color and structure, irregular pigment network, and multifocal hypopigmentation are diagnostic of this dysplastic nevus. 5. Different shades of brown color are a red flag for concern.

111

112

DERMOSCOPY: AN ILLUSTRATED SELF-ASSESSMENT GUIDE

RISK ❑ Low ❑ Intermediate ✔ High ❑

DIAGNOSIS Nevus Seborrheic keratosis Basal cell carcinoma Vascular Dermatofibroma Squamous cell carcinoma Melanoma Other

DISPOSITION ❑ No intervention ❑ Follow-up ✔ Histopathologic diagnosis ❑

3-8c

❑ ❑ ❑ ❑ ❑ ❑ ✔ ❑ ❑

ANSWERS Answers: 1,2,5

Discussion: ■ ■ ■

DERMOSCOPIC CRITERIA ■



■ ■



Asymmetry of color and structures (+) Irregular pigment network (boxes) Irregular globules (circle) Multifocal hypopigmentation (stars) Different shades of brown color

■ ■ ■



This is a gray zone lesion that looks more benign than malignant. The history of change is very important. The clinical appearance is more worrisome than the overall dermoscopic picture. There are no dramatic melanoma- specific criteria. Clinically and dermoscopically this looks more like a dysplastic nevus. Different shades of brown are a red flag for concern. ■ Different shades of any color are a red flag for concern. Do not confuse areas of hypopigmentation with the bony-white color of regression.

PEARLS ■ ■

■ ■

The most experienced clinician could miss this in situ melanoma. Sequential clinical and dermoscopic monitoring can be used with gray zone lesions. Short-term monitoring → check again for changes in 3 to 4 months. Long-term monitoring → check again for changes in 6 months to a year.

Chapter 3

Trunk and Extremities

RISK ❑ Low ❑ Intermediate ❑ High

3-9a

DIAGNOSIS ❑ ❑ ❑ ❑ ❑ ❑ ❑ ❑

Nevus Seborrheic keratosis Basal cell carcinoma Vascular Dermatofibroma Squamous cell carcinoma Melanoma Other

DISPOSITION

3-9b

❑ No intervention ❑ Follow-up ❑ Histopathologic diagnosis

CASE 9 HISTORY A 64-year-old woman had a pigmented lesion on her left lower leg for several years that recently began to get bigger. 1. Pigment network and brown globules diagnose a melanocytic lesion. 2. There is significant asymmetry of color and structure as well as the multicomponent global pattern. 3. There is an irregular pigment network and irregular brown not red dots and globules. 4. The nodular component of this melanoma is amelanotic. 5. In a nodular lesion suspicious for melanoma, sequential digital monitoring is contraindicated.

113

114

DERMOSCOPY: AN ILLUSTRATED SELF-ASSESSMENT GUIDE

1

RISK ❑ Low ❑ Intermediate ❑ ✔ High

2 1

DIAGNOSIS ❑ ❑ ❑ ❑ ❑ ❑ ✔ ❑ ❑

Nevus Seborrheic keratosis Basal cell carcinoma Vascular Dermatofibroma Squamous cell carcinoma Melanoma Other

2

DISPOSITION

3

3-9c

❑ No intervention ❑ Follow-up ✔ Histopathologic diagnosis ❑

DERMOSCOPIC CRITERIA ■









■ ■

Asymmetry of color and structure (+) Multicomponent global pattern (1,2,3) Regular pigment network (yellow box) Irregular pigment network (black boxes) Irregular brown dots and globules (circles) Milky-red area (arrows) Amelanotic nodule (stars)

ANSWERS Answers: 1,2,3,4,5

Discussion: ■

■ ■





Compared with Case 8, melanoma-specific criteria are much easier to identify. This invasive melanoma is not mistakable for a benign lesion. Ask your pathologist to examine additional sections and reevaluate this case, if a benign lesion is reported. ■ Consider the opinion of a dermato-pathologist with special expertise in pigmented skin lesions if this is not reported to be a melanoma. ■ There should always be a good dermoscopic-pathologic correlation! This pinkish-white area has a dermoscopic differential diagnosis that includes regression or the amelanotic component of the melanoma. The milky-red area represents neovascularization.

PEARLS ■





A nodular lesion suggestive of a melanoma should be fully excised posthaste, allowing for pathologic examination of the entire lesion as well as tumor staging. Sequential monitoring is contraindicated in a nodular lesion that could be a melanoma. Even a short delay in making the diagnosis of a nodular melanoma could worsen the patient’s prognosis.

Chapter 3

Trunk and Extremities

RISK ❑ Low ❑ Intermediate ❑ High

DIAGNOSIS

3-10a

❑ ❑ ❑ ❑ ❑ ❑ ❑ ❑

Nevus Seborrheic keratosis Basal cell carcinoma Vascular Dermatofibroma Squamous cell carcinoma Melanoma Other

DISPOSITION

3-10b

❑ No intervention ❑ Follow-up ❑ Histopathologic diagnosis

CASE 10 HISTORY This asymptomatic lesion on a 24-year-old man was discovered by the patient who wanted to know what it was. 1. 2. 3. 4. 5.

This is a nodular amelanotic melanoma arising in a nevus. Milky-red areas and markedly atypical pigment network characterize this melanoma. This is a typical benign collision tumor consisting of a melanocytic nevus and hemangioma. Dermoscopy in general is not helpful to diagnose collision tumors. Benign and malignant pathology in any combination is possible in collision tumors.

115

116

DERMOSCOPY: AN ILLUSTRATED SELF-ASSESSMENT GUIDE

RISK ✔ Low ❑ ❑ Intermediate ❑ High

DIAGNOSIS Nevus Seborrheic keratosis Basal cell carcinoma Vascular Dermatofibroma Squamous cell carcinoma Melanoma Other

DISPOSITION ✔ No intervention ❑ ❑ Follow-up ❑ Histopathologic diagnosis

3-10c

✔ ❑ ❑ ❑ ✔ ❑ ❑ ❑ ❑ ❑

ANSWERS Answers: 3,5

Discussion: ■ ■ ■

DERMOSCOPIC CRITERIA ■

■ ■

Regular pigment network (boxes) Homogeneous red color (stars) Lacunae (circles)



■ ■

■ ■

Collision tumors consist of two distinct pathologies within the same lesion. Collision tumors are commonly encountered. The most common tumors found in collision are: seborrheic keratoses, melanocytic nevi, hemangiomas, basal cell carcinomas, in situ and invasive squamous cell carcinomas, in situ, invasive amelanotic or pigmented melanoma. Triple collision tumors exist yet are not common (eg, basal cell carcinoma, seborrheic keratosis and clear cell acanthoma). Any combination is possible. Even though the pigment network is thickened and branched, it is uniform throughout → it is not considered an irregular pigment network. The lacuanae are visible but not well developed. Milky-red areas are in the differential diagnosis and consist of pinkish-white homogeneous color that can contain red and/or blue globular-like structures. ■ Hemanagiomas typically have sharp in focus vascular spaces. ■ Homogeneous red color along with lacunae can be seen. ■ Milky-red areas typically have out-of-focus/blurry globular-like structures. ■ At times this distinction cannot be made.

PEARLS ■



As a general rule, identify the criteria in a collision tumor and determine if they are regular or irregular just as you would do with a noncollision tumor. If there is any suggestion of high risk pathology → make a histopathologic diagnosis ASAP.

Chapter 3

Trunk and Extremities

RISK ❑ Low ❑ Intermediate ❑ High

DIAGNOSIS

3-11a

❑ ❑ ❑ ❑ ❑ ❑ ❑ ❑

Nevus Seborrheic keratosis Basal cell carcinoma Vascular Dermatofibroma Squamous cell carcinoma Melanoma Other

DISPOSITION

3-11b

❑ No intervention ❑ Follow-up ❑ Histopathologic diagnosis

CASE 11 HISTORY This lesion was found on the arm of a 52-year-old man. 1. This is a benign halo nevus characterized by a cobblestone pattern and surrounding white halo. 2. This is a benign halo hemangioma characterized by well-dermarcated lacunae and hypopigmentation. 3. The brown color of the lacunae-like structures is a clue that this might be a melanocytic lesion. 4. Clinically and dermoscopically, this could be a melanoma with halo nevus-like regression. 5. Brown, blue, gray, and white colors plus irregular vessels are clues that this is a melanoma.

117

118

DERMOSCOPY: AN ILLUSTRATED SELF-ASSESSMENT GUIDE

RISK

1

❑ Low ❑ Intermediate ✔ High ❑

1

DIAGNOSIS

2

Nevus Seborrheic keratosis Basal cell carcinoma Vascular Dermatofibroma Squamous cell carcinoma Melanoma Other

DISPOSITION ❑ No intervention ❑ Follow-up ✔ Histopathologic diagnosis ❑

3

1

ANSWERS Answers: 3,4,5

Discussion: ■ ■

DERMOSCOPIC CRITERIA ■





■ ■ ■



Asymmetry of color and structure Multicomponent global pattern (1,2,3) Irregular brown globules (white boxes) Regression (stars) Gray color (black arrows) Irregular blue blotch (yellow box) Irregular vessels (red arrows)

1

3-11c

❑ ❑ ❑ ❑ ❑ ❑ ✔ ❑ ❑



■ ■



■ ■

A halo nevus is in the clinical and dermoscopic differential diagnosis. With an eccentrically located lesion and several high risk dermoscopic criteria, an invasive melanoma with halo nevus-like regression is a more likely diagnosis. The differential diagnosis of the tumor includes an irregular cobblestone pattern of a melanocytic nevus or lacunae-like vascular structures of a hemangioma. The brown color rules out a vascular lesion. Halo phenomenon can occur around melanocytic and nonmelanocytic lesions: ■ Congenital, acquired, dysplastic, blue, Spitz nevi, and melanoma. ■ Seborrheic keratosis, neurofibroma, basal cell carcinoma. ■ But not around hemangiomas. ■ a Vscular lesions typically have white color but within the lesion (fibrous septae). Cutaneous metastatic melanoma could have this dermoscopic picture with lacunae-like structures. Bony– white, blue, and gray color make up the area of regression. The fine gray granules (“peppering”) that can be seen in regression are not identified.

PEARLS ■ ■





A halo nevus-like lesion in an adult is melanoma until proven otherwise. Halo nevi can be found in children and in most cases are not high risk lesions. Halo phenomenon in a patient with a history of melanoma can be a clue that the patient has metastatic melanoma. One should search for irregular criteria before diagnosing a benign halo nevus.

Chapter 3

Trunk and Extremities

RISK ❑ Low ❑ Intermediate ❑ High

DIAGNOSIS

3-12a

❑ ❑ ❑ ❑ ❑ ❑ ❑ ❑

Nevus Seborrheic keratosis Basal cell carcinoma Vascular Dermatofibroma Squamous cell carcinoma Melanoma Other

DISPOSITION

3-12b

❑ No intervention ❑ Follow-up ❑ Histopathologic diagnosis

CASE 12 HISTORY This isolated lesion was thought to be high risk by the referring general practitioner, and was located on the back of a 34-year-old woman. 1. Milky-red areas and a blue-white veil characterize this nodular-melanoma. 2. Well-dermarcated vascular spaces, fibrous septa, and the absence of criteria seen in a melanocytic lesion characterize this classic hemangioma. 3. By definition, hemangiomas must have tightly spaced lacunae. 4. Bright red, violaceous, blue, and black colors can be found in hemangiomas. 5. Pyogenic granulomas can have similar lacunae-like structures and a white peripheral border.

119

120

DERMOSCOPY: AN ILLUSTRATED SELF-ASSESSMENT GUIDE

RISK ✔ Low ❑ ❑ Intermediate ❑ High

DIAGNOSIS Nevus Seborrheic keratosis Basal cell carcinoma Vascular Dermatofibroma Squamous cell carcinoma Melanoma Other

DISPOSITION ✔ No intervention ❑ ❑ Follow-up ❑ Histopathologic diagnosis

3-12c

❑ ❑ ❑ ✔ ❑ ❑ ❑ ❑ ❑

ANSWERS Answers: 2,4

Discussion: ■ ■



DERMOSCOPIC CRITERIA ■ ■ ■

Lacunae (boxes) Fibrous septa (white arrows) Fibrous border (black arrows)



■ ■

■ ■





■ ■ ■

Clinically and dermoscopically this is a classic hemangioma. There are no criteria for a melanocytic → there should be no criteria for a melanocytic lesion. The main criteria are lacunae (aka saccules, lagoons). Lacunae → sharply demarcated, vary in size and distribution round to oval structures. Lacunae can be tightly packed or loosely distributed throughout the lesion. The color is determined by the location of vascular spaces in the dermis. ■ Bright red (cherry/senile hemangiomas) the vascular spaces are high up in the papillary dermis. ■ The deeper the vascular spaces, the darker the color (ie, darker shades of red, violaceous, blue). ■ Black color indicates thrombosis. Individual hemangiomas can have single or multiple colors. Bluish-white or white fibrous septa are commonly seen and should not be confused with the bluish-white or bony- white color found in melanomas. Milky-red areas in melanomas can have lacunae-like structures but the borders are not sharply demarcated. Pyogenic granulomas can have septa-like white lines in the lesion and at the periphery but lack lacunae. Angiokeratomas are hemangioma-like with varying amounts of scale or crust. Lymphangiomas have clear vascular-like spaces. Lymph-hemangiomas have a combination of clear and red vascular spaces.

PEARLS ■



It is essential to determine if the vascular spaces have sharp well-demarcated borders. It is essential to look for the criteria associated with melanocytic lesions (eg, melanoma).

Chapter 3

Trunk and Extremities

RISK ❑ Low ❑ Intermediate ❑ High

DIAGNOSIS

3-13a

❑ ❑ ❑ ❑ ❑ ❑ ❑ ❑

Nevus Seborrheic keratosis Basal cell carcinoma Vascular Dermatofibroma Squamous cell carcinoma Melanoma Other

DISPOSITION

3-13b

❑ No intervention ❑ Follow-up ❑ Histopathologic diagnosis

CASE 13 HISTORY This is from the abdomen of a 19-year-old patient, and has been getting “scabby” over a 3-month period. 1. This nodular melanoma is characterized by an irregular black blotch, ulceration, and amelanotic milky-red areas. 2. A hemorrhagic crust, remnants of lacunae, and peripheral erythema characterize this thrombosed hemangioma. 3. This could be a partially thrombosed angiokeratoma. 4. A hemorrhagic crust is never found in nodular melanoma. 5. Black color in a vascular lesion represents thrombosis.

121

122

DERMOSCOPY: AN ILLUSTRATED SELF-ASSESSMENT GUIDE

RISK ❑ Low ✔ Intermediate ❑ ❑ High

DIAGNOSIS Nevus Seborrheic keratosis Basal cell carcinoma Vascular Dermatofibroma Squamous cell carcinoma Melanoma Other

DISPOSITION ❑ No intervention ❑ Follow-up ✔ Histopathologic diagnosis ❑

3-13c

❑ ❑ ❑ ✔ ❑ ❑ ❑ ❑ ❑

ANSWERS Answers: 2,3,5

Discussion: ■ ■



DERMOSCOPIC CRITERIA ■

■ ■

Hemorrhagic crust (aka thrombosis) (stars) Lacunae (white arrows) Peripheral erythema (black arrows)





■ ■



This is a worrisome gray zone lesion that could be a nodular melanoma. The lacunae and homogenous reddish color represent what is left of the hemangioma. Milky-red areas are in the differential diagnosis of the hemangioma component. The peripheral erythema could represent the amelanotic component of a nodular melanoma. The criteria seen here, point out the need for one to create a dermoscopic differential diagnosis. A pyogenic granuloma would not have such a large hemorrhagic crust. A thrombosed angiokeratoma could look like this especially since there are areas of yellow color that represents hyperkeratosis. Melanoma can have ulceration and crust formation.

PEARLS ■





If one has to spend abundant time thinking about what is going on, it is time to make a histopathologic diagnosis especially if nodular melanoma is in the differential diagnosis. A positive stool guaiac test might help differentiate the hemorrhagic crust (guaiac )+from a nonulcerated black blotch in a melanoma (guaiac −) The often dramatic jet-black color in thrombosed vascular lesions is as black as ever can be seen when using-dermoscopy, with the exception of the black lamella.

Chapter 3

Trunk and Extremities

RISK ❑ Low ❑ Intermediate ❑ High

DIAGNOSIS

3-14a

❑ ❑ ❑ ❑ ❑ ❑ ❑ ❑

Nevus Seborrheic keratosis Basal cell carcinoma Vascular Dermatofibroma Squamous cell carcinoma Melanoma Other

DISPOSITION

3-14b

❑ No intervention ❑ Follow-up ❑ Histopathologic diagnosis

CASE 14 HISTORY You were consulted because it was felt that this 16-year-old girl has a melanoma. 1. Pigment network and globules diagnose a melanocytic lesion. 2. The dark round structures could be irregular globules or thrombosed lacunae. 3. The reddish oval structures could represent lacunae of a hemangioma or the poorly defined globules seen in milky-red areas. 4. This melanoma is characterized by asymmetry of color and structure, a multicomponent global pattern, irregular pigment network, irregular globules, and a blue-white veil. 5. This thrombosed hemangioma is characterized by lacunae, thrombosed lacunae, and the bluish-white color created by hyperkeratosis.

123

124

DERMOSCOPY: AN ILLUSTRATED SELF-ASSESSMENT GUIDE

RISK ❑ Low ✔ Intermediate ❑ ❑ High

3 1 2

DIAGNOSIS Nevus Seborrheic keratosis Basal cell carcinoma Vascular Dermatofibroma Squamous cell carcinoma Melanoma Other

2 3

3-14c

❑ ❑ ❑ ✔ ❑ ❑ ❑ ❑ ❑

DISPOSITION

ANSWERS

❑ No intervention ❑ Follow-up ✔ Histopathologic diagnosis ❑

Answers: 2,3,5

Discussion: ■ ■



DERMOSCOPIC CRITERIA ■



■ ■ ■





Asymmetry of color and structure (+) Multicomponent global pattern (1,2,3) Lacunae (black boxes) Thrombosed lacunae (arrows) Milky-red globular-like structures (white boxes) Network-like structures (yellow box) Bluish-white color (stars)





■ ■

■ ■

As in Case 13, the dermoscopic picture is not clear. The global pattern and local criteria have a differential diagnosis → vascular vs melanocytic. Several well-demarcated lacunae are an important clue that this is a vascular lesion. The presence of the lacunae suggests that the black round structures could be thrombosed lacunae rather than irregular globules of a melanocytic lesion. The milky-red globular-like structures are still lacunae→ lacunae do not always have sharp borders. The histopathologic correlate of the network-like structure is not known. Clinically, the lesion is dry and scaly → this creates the blue color seen by dermoscopy. Blue and/or white color is not always high risk and has a differential diagnosis. A multicomponent global pattern with three or more different areas within a lesion is not diagnostic of a melanoma and can be seen in benign and malignant pathologies (ie, congenital and dysplastic nevi or basal cell carcinomas).

PEARLS ■ ■



One cannot see what one does not know. Study hard to better serve your patients whose lives can depend on your correct dermoscopic diagnosis. There is a significant learning curve to become a competent dermoscopist.

Chapter 3

Trunk and Extremities

RISK ❑ Low ❑ Intermediate ❑ High

DIAGNOSIS

3-15a

❑ ❑ ❑ ❑ ❑ ❑ ❑ ❑

Nevus Seborrheic keratosis Basal cell carcinoma Vascular Dermatofibroma Squamous cell carcinoma Melanoma Other

DISPOSITION

3-15b

❑ No intervention ❑ Follow-up ❑ Histopathologic diagnosis

CASE 15 HISTORY A 63-year-old man has a growing lesion on his chest. 1. Pigment network and globules identify this melanocytic lesion. 2. There is asymmetry of color and structure plus a multicomponent global pattern. 3. There are foci of irregular pigment network, irregular dots and globules, irregular streaks, irregular pigmentation, and blue-white color. 4. Regression, milky-red areas, and polymorphous vessels are important high risk criteria that are not seen. 5. The differential diagnosis includes a dysplastic nevus or melanoma.

125

126

DERMOSCOPY: AN ILLUSTRATED SELF-ASSESSMENT GUIDE

RISK ❑ Low ❑ Intermediate ✔ High ❑ 1

3

DIAGNOSIS Nevus Seborrheic keratosis Basal cell carcinoma Vascular Dermatofibroma Squamous cell carcinoma Melanoma Other

2

3-15c

❑ ❑ ❑ ❑ ❑ ❑ ✔ ❑ ❑

DISPOSITION

ANSWERS

❑ No intervention ❑ Follow-up ✔ Histopathologic diagnosis ❑

Answers: 1,2,3,4,5

Discussion: ■

DERMOSCOPIC CRITERIA ■







■ ■ ■

Asymmetry of color and structure (+) Multicomponent global pattern (1,2,3) Irregular pigment network (black boxes) Irregular dots and globules (circles) Irregular streaks (yellow box) Irregular brown pigmentation Bluish-white color (stars)







The order of the questions is the order in which one should evaluate each lesion. ■ Step 1 → Is the lesion melanocytic or not? ■ Step 2 → Is there symmetry or asymmetry of color and/or structure? ■ Step 3 → What is the global pattern? ■ Step 4 → Find all of the local criteria (pigment network, dots and globules, streaks, blotches, regression, colors, vessels). ■ Step 5 → Determine if the local criteria are regular or irregular. ■ Step 6 → Double check to make sure you have not missed anything. ■ Step 7 → Could there be a dermoscopic differential diagnosis for the global pattern or any of the local criteria? ■ Step 8 → Put the entire clinical scenario together and make a diagnosis. ■ Step 9 → What is the best disposition for the lesion? The foci of irregular streaks are not that easy to identify in this invasive melanoma. One could consider the foci of irregular pigment network foci or irregular streaks. Different shades of brown and/or any other colors are a high risk criterion.

PEARLS ■ ■

Memorize all of the features one should look for in each case. Do not make a final diagnosis before you find and analyze all of the features in a lesion.

Chapter 3

Trunk and Extremities

RISK ❑ Low ❑ Intermediate ❑ High

DIAGNOSIS

3-16a

❑ ❑ ❑ ❑ ❑ ❑ ❑ ❑

Nevus Seborrheic keratosis Basal cell carcinoma Vascular Dermatofibroma Squamous cell carcinoma Melanoma Other

DISPOSITION

3-16b

❑ No intervention ❑ Follow-up ❑ Histopathologic diagnosis

CASE 16 HISTORY A 48-year-old man has a nonchanging brownish lesion in his left axilla for many years. 1. 2. 3. 4. 5.

Clinically but not dermoscopically this looks similar to the melanoma in Case 15. There are no criteria to diagnose a melanocytic lesion. There are no criteria to diagnose a basal cell carcinoma. Milia-like cysts and pigmented pseudofollicular openings characterize this seborrheic keratosis. Asymmetry of color and structures, a multicomponent global pattern, irregular globules, bluewhite veil, and regression characterize this melanoma.

127

128

DERMOSCOPY: AN ILLUSTRATED SELF-ASSESSMENT GUIDE

RISK ❑ Low ✔ Intermediate ❑ ❑ High

❑ ✔ ❑ ❑ ❑ ❑ ❑ ❑ ❑

Nevus Seborrheic keratosis Basal cell carcinoma Vascular Dermatofibroma Squamous cell carcinoma Melanoma Other

3-16c

DIAGNOSIS

ANSWERS Answers: 1,2,3,4

DISPOSITION ❑ No intervention ❑ Follow-up ✔ Histopathologic diagnosis ❑

Discussion: ■ ■ ■





DERMOSCOPIC CRITERIA ■ ■

■ ■

Milia-like cysts (black boxes) Pigmented pseudofollicular openings (arrows) Bluish-white colors (stars) Peppering (yellow box)



■ ■







■ ■

The history, if it is accurate, and clinical appearance are not high risk. The global dermoscopic appearance on the other hand is worrisome. Multiple milia-like cysts and pigmented pseudofollicular openings favor the diagnosis of a seborrheic keratosis. The differential diagnosis of the pigmented pseudofollicular openings includes irregular dots and globules of a melanocytic lesion. If one considers that this could be melanocytic then the asymmetry of color and structure, multicomponent global pattern, irregular dots and globules, blue-white and other colors are very worrisome. Milia-like cysts and pigmented pseudofollicular openings can be found in melanoma. Typically seborrheic keratosis can be heavily pigmented. The blue color in this benign lesion can be created by pigment deeper in the dermis similar to a blue nevus → the Tyndall effect. Peppering → gray color with fine gray dots indicates that there may be an inflammatory component (irritated seborrheic keratosis). The asymmetry of color and structure and multicomponent global pattern has no significance in nonmelanocytic lesions. A collision tumor, (eg, melanoma or nevus and a seborrheic keratosis), is in the differential diagnosis. This lesion should be very worrisome for the novice dermoscopist. One should always listen to the patient’s history. However, patient histories are not always reliable → in this case there was no history of change for many years.

PEARLS ■



■ ■ ■ ■

Commonly, pathology reports of melanomas have seborrheic keratosis as the primary diagnosis or seborrheic keratosis is in the differential diagnosis. Even the most experienced clinician could miss a seborrheic keratosis-like melanoma aka melanoma incognito → false negative melanoma. Atypical seborrheic keratosis can be impossible to differentiate from melanoma. Ask a more experienced dermosopist for their opinion with difficult cases. A consensus of minds is often the way to go with difficult cases. If in doubt, cut it out.

Chapter 3

Trunk and Extremities

RISK ❑ Low ❑ Intermediate ❑ High

DIAGNOSIS

3-17a

❑ ❑ ❑ ❑ ❑ ❑ ❑ ❑

Nevus Seborrheic keratosis Basal cell carcinoma Vascular Dermatofibroma Squamous cell carcinoma Melanoma Other

DISPOSITION

3-17b

❑ No intervention ❑ Follow-up ❑ Histopathologic diagnosis

CASE 17 HISTORY A 63-year-old man was in for a yearly skin examination, and this solitary pigmented lesion was found on his upper back. There is no other history. 1. 2. 3. 4. 5.

Pigment network identifies a melanocytic lesion. There is asymmetry of color and structure. There is a homogeneous reticular global pattern. The pigment network is irregular. The irregular pigmentation is a red flag for concern.

129

130

DERMOSCOPY: AN ILLUSTRATED SELF-ASSESSMENT GUIDE

RISK ❑ Low ✔ Intermediate ❑ ❑ High

DIAGNOSIS Nevus Seborrheic keratosis Basal cell carcinoma Vascular Dermatofibroma Squamous cell carcinoma Melanoma Other

DISPOSITION ❑ No intervention ✔ Follow-up ❑ ❑ Histopathologic diagnosis

3-17c

✔ ❑ ❑ ❑ ❑ ❑ ❑ ❑ ❑

ANSWERS Answers: 1,2,3,4,5

Discussion: ■



DERMOSCOPIC CRITERIA ■









Asymmetry of color and structure (+) Homogeneous—reticular global pattern Irregular pigment network (boxes) Irregular hyperpigmentation (stars) Background normal skin (arrows)



■ ■

■ ■ ■



One would expect to see this lesion filled with a regular pigment network → there is not a good clinico– dermoscopic correlation—a red flag for concern. Whatever the global pattern (eg, homogeneous—reticular; reticular—globular) one has to determine if the criteria are regular or irregular. By definition, the pigment network (reticular component of the global pattern) is irregular → thick, branched, broken up line segments. Irregular pigment network can be found in low risk nevi (eg, junctional nevi). The homogeneous darker color (homogeneous component of the global pattern) has a differential diagnosis that includes epidermal and/or dermal pigmentation with or without atypical melanocytes. The differential diagnosis of this lesion includes a nevus vs dysplastic nevus. Melanoma is not in the differential diagnosis. There would not be a good dermoscopic–pathologic correlation if this was thought to be a melanoma by the pathologist. Normal background skin is commonly found within a lesion and has no diagnostic significance → it must be differentiated from hypopigmentation or regression.

PEARLS ■



■ ■

Good communication with one’s pathologist is essential when there is not a good dermoscopic–pathologic correlation. A single lesion with this dermoscopic appearance is more worrisome than several lesions that look like this. For the novice dermoscopist → consider making histopathologic diagnosis. For the experienced dermoscopist → sequential digital monitoring.

Chapter 3

Trunk and Extremities

RISK ❑ Low ❑ Intermediate ❑ High

DIAGNOSIS

3-18a

❑ ❑ ❑ ❑ ❑ ❑ ❑ ❑

Nevus Seborrheic keratosis Basal cell carcinoma Vascular Dermatofibroma Squamous cell carcinoma Melanoma Other

DISPOSITION ❑ No intervention ❑ Follow-up ❑ Histopathologic diagnosis

CASE 18 HISTORY A colleague e-mailed this image for your opinion of whether this could be a melanoma. It was located on the back of a 72-year-old man. 1. Pigment network identifies a melanocytic lesion. 2. Asymmetry of color and structure, a multicomponent global pattern, irregular pigment network, and irregular globules characterize this melanoma. 3. This is a classic fingerprint pattern often found in flat seborrheic keratosis. 4. A fingerprint pattern and nonpigmented pseudofollicular openings characterize this seborrheic keratosis. 5. Parallel line segments rather than honeycomb-like line segments differentiate the fingerprint pattern of a seborrheic keratosis from the pigment network of a melanocytic lesion.

131

132

DERMOSCOPY: AN ILLUSTRATED SELF-ASSESSMENT GUIDE

RISK ✔ Low ❑ ❑ Intermediate ❑ High

DIAGNOSIS Nevus Seborrheic keratosis Basal cell carcinoma Vascular Dermatofibroma Squamous cell carcinoma Melanoma Other

DISPOSITION ✔ No intervention ❑ ❑ Follow-up ❑ Histopathologic diagnosis

3-18b

❑ ✔ ❑ ❑ ❑ ❑ ❑ ❑ ❑

ANSWERS Answers: 3,4,5

Discussion: ■

DERMOSCOPIC CRITERIA ■ ■



Fingerprint pattern (boxes) Nonpigmented pseudofollicular openings (yellow arrows) Concave moth-eaten borders (black arrows)

■ ■



■ ■ ■ ■

This is a classic fingerprint pattern with brown parallel line segments that resemble the dermatoglyphics of the hand → fingerprints. ■ The differential diagnosis includes an early brain-like pattern also seen in seborrheic keratosis or an atypical pigment network. This pattern can be found in flat seborrheic keratosis or solar lentigines. There are multiple nonpigmented pseudofollicular openings that would not be seen in a solar lentigo. The nonpigmented pseudofollicular openings should not be confused with globules of a melanocytic lesion. The line segments of the fingerprint pattern can be seen throughout the lesion. There may only be remnants of a fingerprint pattern found in a lesion. There are no melanoma-specific criteria in this lesion. Once the definition of the fingerprint pattern is learned and one sees images of the pattern, it usually is easy to recognize.

PEARLS ■ ■

Caution! The fingerprint pattern can be associated with lentigo maligna. Look for high risk criteria throughout a lesion before diagnosing seborrheic keratosis or solar lentigo.

Chapter 3

Trunk and Extremities

RISK ❑ Low ❑ Intermediate ❑ High

DIAGNOSIS

3-19a

❑ ❑ ❑ ❑ ❑ ❑ ❑ ❑

Nevus Seborrheic keratosis Basal cell carcinoma Vascular Dermatofibroma Squamous cell carcinoma Melanoma Other

DISPOSITION

3-19b

❑ No intervention ❑ Follow-up ❑ Histopathologic diagnosis

CASE 19 HISTORY A 45-year-old man with signs of extensive sun damage has this pigmented skin lesion on his chest. 1. This could be a false negative dermoscopic melanoma because the clinical but not the dermoscopic appearance suggests a melanoma. 2. This is a solar lentigo with the fingerprint pattern and moth-eaten borders. 3. This is a melanocytic lesion because there is a pigment network. 4. Hypopigmentation filling the lesion rules out a melanoma. 5. Asymmetry of color and structure, a multicomponent global pattern, irregular pigment network, and regression characterize this invasive melanoma.

133

134

DERMOSCOPY: AN ILLUSTRATED SELF-ASSESSMENT GUIDE

RISK 1

❑ Low ❑ Intermediate ✔ High ❑

2

2

2

DIAGNOSIS 1

Nevus Seborrheic keratosis Basal cell carcinoma Vascular Dermatofibroma Squamous cell carcinoma Melanoma Other

3 3

3-19c

❑ ❑ ❑ ❑ ❑ ❑ ✔ ❑ ❑

DISPOSITION ❑ No intervention ❑ Follow-up ✔ Histopathologic diagnosis ❑

ANSWERS Answers: 1,3,5

Discussion: ■

DERMOSCOPIC CRITERIA ■









Asymmetry of color and structure (+) The multicomponent global pattern (1,2,3) Regular pigment network (yellow boxes) Irregular pigment network (black boxes) Regression (stars)

■ ■

■ ■

■ ■ ■

The lesion looks worse clinically than dermoscopically. ■ It is not completely banal appearing dermoscopically. ■ It looks more benign than malignant dermoscopically. The clinical appearance overrides the dermoscopic picture. Regular pigment network with uniform line segments and holes (dermal papilla) can be contrasted with an irregular pigment network. The parallel lines of a fingerprint pattern are not seen. The areas of regression look more like hypopigmentation → they are inferred to be present because one sees regression clinically. ■ Hypopigmentation has no specific diagnostic significance. There are no dramatic melanoma- specific criteria. The differential diagnosis includes a dysplastic nevus or melanoma. This is a false negative dermoscopic melanoma.

PEARLS ■









When a lesion looks low risk clinically, a high risk dermoscopic appearance should not be ignored. When a lesion looks high risk clinically but not dermoscopically, the clinical appearance should not be ignored. In most cases, high risk looking lesions clinically will have a low risk dermoscopic appearance. With experience one will feel more comfortable handling all of these clinical scenarios. Gut feelings about the entire clinical scenario should never be ignored → if one has an uncomfortable feeling about a lesion it should probably be excised.

Chapter 3

Trunk and Extremities

RISK ❑ Low ❑ Intermediate ❑ High

DIAGNOSIS

3-20a

❑ ❑ ❑ ❑ ❑ ❑ ❑ ❑

Nevus Seborrheic keratosis Basal cell carcinoma Vascular Dermatofibroma Squamous cell carcinoma Melanoma Other

DISPOSITION

3-20b

❑ No intervention ❑ Follow-up ❑ Histopathologic diagnosis

CASE 20 HISTORY A 32-year-old man with a slowly growing pigmented lesion on his chest for many years is referred for your evaluation. 1. This medium sized congenital melanocytic nevus is characterized by regular pigment network, regular dots and globules, plus islands of normal skin. 2. The parallel fingerprint pattern helps to diagnose a traumatized flat seborrheic keratosis. 3. The clinical appearance, asymmetry of color and structure, regular pigment network, regular dots and globules, plus regression characterize this melanoma. 4. The clinical appearance, asymmetry of color and structure, a multicomponent global pattern, irregular pigment network, irregular dots and globules, plus regression characterize this invasive melanoma. 5. Clinically this could only be a melanoma and dermoscopic evaluation would not add useful information.

135

136

DERMOSCOPY: AN ILLUSTRATED SELF-ASSESSMENT GUIDE

RISK ❑ Low ❑ Intermediate ✔ High ❑

DIAGNOSIS Nevus Seborrheic keratosis Basal cell carcinoma Vascular Dermatofibroma Squamous cell carcinoma Melanoma Other

DISPOSITION ❑ No intervention ❑ Follow-up ✔ Histopathologic diagnosis ❑

3-20c

❑ ❑ ❑ ❑ ❑ ❑ ✔ ❑ ❑

ANSWERS Answers: 4

Discussion: ■



DERMOSCOPIC CRITERIA ■

■ ■







Asymmetry of color and structure Multicomponent global pattern Irregular pigment network (black boxes) Fingerprint-like pigment network (yellow boxes) Irregular dots and globules (circles) Regression (arrows)

■ ■ ■









A medium-sized congenital melanocytic nevus and seborrheic keratosis are in the clinical, but not the dermoscopic differential diagnosis. It would be unlikely that a 32-year-old would have a solitary large seborrheic keratosis. There are no criteria to suggest this is a seborrheic keratosis. Congenital melanocytic nevi can have atypical dermoscopic features. Irregular criteria in a congenital melanocytic nevus should always raise a red flag for concern. The dramatic foci of parallel pigment network resembling the fingerprint pattern are only one criterion and not enough to diagnose a solar lentigo or seborrheic keratosis. All of the other melanoma-specific criteria must be taken into consideration. ■ Asymmetry of color and structure ■ Multicomponent global pattern ■ Irregular pigment network ■ Irregular dots and globules ■ Regression Dermoscopy adds essential information to confirm that this is a very high risk lesion. Dermoscopy rules out low risk pathology.

PEARLS ■ ■

One should use the areas with most atypical features for an incisional biopsy. Based on the clinical appearance and high risk dermoscopic criteria, one would be justified to completely excise this lesion without a preliminary biopsy even though it would create a big scar.

Chapter 3

Trunk and Extremities

RISK ❑ Low ❑ Intermediate ❑ High

DIAGNOSIS

3-21a

❑ ❑ ❑ ❑ ❑ ❑ ❑ ❑

Nevus Seborrheic keratosis Basal cell carcinoma Vascular Dermatofibroma Squamous cell carcinoma Melanoma Other

DISPOSITION

3-21b

❑ No intervention ❑ Follow-up ❑ Histopathologic diagnosis

CASE 21 HISTORY A 20-year-old woman was worried about a mole that was getting darker. 1. 2. 3. 4. 5.

Globules identify a melanocytic lesion. This could be the “mountain and valley” pattern of a seborrheic keratosis. This seborrheic keratosis is filled with pigmented pseudofollicular openings. Irregular dots and globules suggest that this is a changing dysplastic nevus or melanoma. This is the classic cobblestone global pattern of a banal nevus.

137

138

DERMOSCOPY: AN ILLUSTRATED SELF-ASSESSMENT GUIDE

RISK ✔ Low ❑ ❑ Intermediate ❑ High

DIAGNOSIS Nevus Seborrheic keratosis Basal cell carcinoma Vascular Dermatofibroma Squamous cell carcinoma Melanoma Other

DISPOSITION ✔ No intervention ❑ ❑ Follow-up ❑ Histopathologic diagnosis

3-21c

✔ ❑ ❑ ❑ ❑ ❑ ❑ ❑ ❑

ANSWERS Answers: 1,2,5

Discussion: ■ ■ ■ ■

DERMOSCOPIC CRITERIA ■

■ ■





Symmetry of color and structure (+) Cobblestone global pattern Regular dots and globules (circles) Cobblestone globules (red arrows) Hypopigmented fissures (yellow arrows)





The history of a changing nevus is not always high risk. One expects nevi to change in children. Clinically and dermoscopically this is a banal lesion. This is the classic cobblestone global pattern: ■ Filled with large angulated brown globules resembling street cobblestones. ■ Smaller dots and globules can also be found. The clinical and dermoscopic differential diagnosis includes a seborrheic keratosis with a “mountain and valley” pattern. ■ Globules→ mountains ■ Fissures → valleys ■ There are no other criteria to diagnose a seborrheic keratosis → milia-like cysts, pseudofollicular openings, “fat fingers”, or hairpin vessels. An actively changing nevus typically has dots and globules only at the periphery of the lesion.

PEARLS ■



When the clinical and dermoscopic differential diagnosis is between a nevus and a seborrheic keratosis, look at and feel the lesion: ■ Soft and compressible→ nevus ■ Immovable dry and scaly → seborrheic keratosis A positive “wobble” sign (applying pressure with instrumentation [ie, Dermlite] and gentle side-to-side motion) helps diagnose a nevus → ■ A nevus will have a positive “wobble” sign with side-to-side movement of the lesion. ■ A seborrheic keratosis will have a negative “wobble” sign without any movement.

Chapter 3

Trunk and Extremities

RISK ❑ Low ❑ Intermediate ❑ High

DIAGNOSIS

3-22a

❑ ❑ ❑ ❑ ❑ ❑ ❑ ❑

Nevus Seborrheic keratosis Basal cell carcinoma Vascular Dermatofibroma Squamous cell carcinoma Melanoma Other

DISPOSITION

3-22b

❑ No intervention ❑ Follow-up ❑ Histopathologic diagnosis

CASE 22 HISTORY A 59-year-old woman with a history of melanoma returned for a routine skin examination. You notice that in general the color of her skin is unusual. 1. This is a classic example of cutaneous metastatic melanoma. 2. Globules identify a melanocytic lesion. 3. The jaundiced color of the lesion is a clue that the patient has metastatic melanoma to the liver. 4. The unusual color of the lesion is created by a self-tanner. 5. This is the cobblestone pattern of a benign nevus.

139

140

DERMOSCOPY: AN ILLUSTRATED SELF-ASSESSMENT GUIDE

RISK ✔ Low ❑ ❑ Intermediate ❑ High

DIAGNOSIS Nevus Seborrheic keratosis Basal cell carcinoma Vascular Dermatofibroma Squamous cell carcinoma Melanoma Other

DISPOSITION ✔ No intervention ❑ ❑ Follow-up ❑ Histopathologic diagnosis

3-22c

✔ ❑ ❑ ❑ ❑ ❑ ❑ ❑ ❑

ANSWERS Answers: 2,4,5

Discussion: ■



DERMOSCOPIC CRITERIA ■ ■



■ ■

Cobblestone global pattern Symmetry of color and structure Cobblestone globules (black arrows) Fissures (yellow arrows) “Fat fingers” (white arrows)





■ ■

Commonly, one encounters patients with a history of melanoma that still want to be tan. Patients that use self tanners can usually be easily recognized. Self tanners do affect what one sees with dermoscopy, although the changes are not significant. Once again as in Case 21 the differential diagnosis is between a nevus with a cobblestone pattern and a seborrheic keratosis with a “mountain and valley” pattern. Foci of small “fat fingers” favor a seborrheic keratosis. This case points out that “fat fingers” are not diagnostic of seborrheic keratosis and can be seen in melanocytic lesions.

PEARLS ■



Suggest to a patient that uses self-tanners that its use might adversely affect your dermoscopic examination. Try to get a feel if your patient has psychological issues related to the history of melanoma that might benefit from a psychological evaluation.

Chapter 3

Trunk and Extremities

RISK ❑ Low ❑ Intermediate ❑ High

DIAGNOSIS

3-23a

❑ ❑ ❑ ❑ ❑ ❑ ❑ ❑

Nevus Seborrheic keratosis Basal cell carcinoma Vascular Dermatofibroma Squamous cell carcinoma Melanoma Other

DISPOSITION

3-23b

❑ No intervention ❑ Follow-up ❑ Histopathologic diagnosis

CASE 23 HISTORY A 37-year-old pregnant woman developed multiple pigmented skin lesions during her 7th month of pregnancy. She was especially worried about this one. 1. 2. 3. 4. 5.

This could be the cobblestone pattern of a melanocytic nevus. This could be the fissure and ridge pattern of a seborrheic keratosis. Highly irregular hypopigmented globules and regression characterize this nodular melanoma. Without milia-like cysts one cannot diagnose a seborrheic keratosis. “Fat fingers,” nonpigmented pseudofollicular openings, fissures, crypts, and sharp border demarcation diagnose a seborrheic keratosis.

141

142

DERMOSCOPY: AN ILLUSTRATED SELF-ASSESSMENT GUIDE

RISK ✔ Low ❑ ❑ Intermediate ❑ High

DIAGNOSIS Nevus Seborrheic keratosis Basal cell carcinoma Vascular Dermatofibroma Squamous cell carcinoma Melanoma Other

3-23c

❑ ✔ ❑ ❑ ❑ ❑ ❑ ❑ ❑

ANSWERS Answers: 1,2,5

DISPOSITION ✔ No intervention ❑ ❑ Follow-up ❑ Histopathologic diagnosis

Discussion: ■ ■





DERMOSCOPIC CRITERIA ■



■ ■ ■



Sharp border demarcation (red arrows) Nonpigmented pseudofollicular openings (yellow arrows) Fissures (white arrows) Crypts (stars) Ridges aka “fat fingers” (black arrows) Incipient seborrheic keratosis (blue arrow)



■ ■ ■



This seborrheic keratosis is characterized by fissures and ridges. Several nonpigmented pseudofollicular openings should not be confused with globules of a melanocytic lesion. When the typical pseudofollicular openings expand into larger irregular keratin filled areas they are referred to as crypts. Crypts can be found in papillomatous nevi. Hypo or hyperpigmented ridges can be digit-like in shape and are referred to as “fat fingers.” ■ “Fat fingers” are commonly encountered. ■ “Fat fingers” can be straight, kinked, circular, or branched. ■ “Fat fingers” might be the only clue to diagnose a seborrheic keratosis. Seborrheic keratosis can be devoid of milia-like cysts. A melanocytic nevus could look exactly like this. Palpate the lesion to help differentiate: ■ A nevus will be soft and compressible while a seborrheic keratosis will be firm and not compressible. There is an independent small seborrheic keratosis seen clinically and with dermoscopy.

PEARLS ■



There are several misnomers in the dermoscopic language, which we think should not be used. ■ The “crown vessels” seen in sebaceous gland hyperplasia are defined as basal cell-like vessels at the border of the lesion that penetrate the lesion but never reach the center. Rarely will you see this distribution of vessels in typical sebaceous gland hyperplasia. ■ “Leaf-like” structures represent one type of pigmentation found in basal cell carcinomas that have finger-like projections said to look like a “maple leaf ” or be “leaf-like.” If one finds pigmentation in a basal cell carcinoma that fits this description it will never look like any type of a leaf. “Fat fingers” are not a misnomer because they are an excellent description of one morphologic variation of ridges.

Chapter 3

Trunk and Extremities

RISK ❑ Low ❑ Intermediate ❑ High

DIAGNOSIS

3-24a

❑ ❑ ❑ ❑ ❑ ❑ ❑ ❑

Nevus Seborrheic keratosis Basal cell carcinoma Vascular Dermatofibroma Squamous cell carcinoma Melanoma Other

DISPOSITION

3-24b

❑ No intervention ❑ Follow-up ❑ Histopathologic diagnosis

CASE 24 HISTORY On routine skin examination, this lesion was found on the back of a 28-year-old man. 1. Peripheral globules are seen clinically suggesting this could be a Spitz nevus. 2. This could be a dysplastic nevus characterized by irregular dots and globules and different shades of brown color. 3. Peripheral dots and globules indicate this could be an actively changing nevus. 4. Clinically and dermoscopically this could be a banal acquired nevus. 5. A symmetrical “Spitzoid” pattern may be found in melanoma.

143

144

DERMOSCOPY: AN ILLUSTRATED SELF-ASSESSMENT GUIDE

RISK ❑ Low ✔ Intermediate ❑ ❑ High

✔ ❑ ❑ ❑ ❑ ❑ ❑ ❑ ❑

Nevus Seborrheic keratosis Basal cell carcinoma Vascular Dermatofibroma Squamous cell carcinoma Melanoma Other

3-24c

DIAGNOSIS

ANSWERS Answers: 1,2,3,4,5

DISPOSITION ❑ No intervention ❑ Follow-up ✔ Histopathologic diagnosis ❑

Discussion: ■ ■ ■



DERMOSCOPIC CRITERIA ■

■ ■



Symmetry of color and structure (+) “Spitzoid” global pattern Irregular dots and globules (circles) Hypopigmentation (arrows)





■ ■ ■ ■





This is a melanocytic lesion because there are brown globules. These are irregular dots and globules because they are of different sizes and shapes. Globally this has a “Spitzoid” pattern because it resembles the “starburst” pattern. ■ There are peripheral globules without streaks. Any of the six dermoscopic pictures representing Spitz nevi are also referred to as being “Spitzoid.” ■ All “Spitzoid” lesions do not have a “starburst” pattern. ■ A symmetrical “Spitzoid” pattern is less worrisome than an irregular “Spitzoid” pattern. This is a symmetrical “Spitzoid” pattern because the dots and globules surround the entire lesion. The differential diagnosis includes: ■ Nevus ■ Spitz nevus (correct diagnosis) ■ Dysplastic nevus ■ Melanoma Even a symmetrical “Spitzoid” pattern could be found in melanoma. All “Spitzoid” lesions should be removed, especially in adults. When present, different shades of dark color are a red flag for concern. One should search carefully for other high risk criteria that might not be that easy to find. Peripheral dots and globules can be a clue of an actively changing nevus → it will enlarge over time. Inter-observer disagreement and decision-making in a lesion like this exist: ■ This is a perfectly banal appearing Spitz nevus. ■ This could be a symmetrical “Spitzoid” melanoma. ■ This is a banal lesion with peripheral globules that will change over time.

PEARLS ■



The history and clinical appearance of the lesion are very helpful to determine the disposition in this particular case. For the experienced dermoscopist, sequential digital follow-up is one alternative to excision.

Chapter 3

Trunk and Extremities

RISK ❑ Low ❑ Intermediate ❑ High

DIAGNOSIS

3-25a

❑ ❑ ❑ ❑ ❑ ❑ ❑ ❑

Nevus Seborrheic keratosis Basal cell carcinoma Vascular Dermatofibroma Squamous cell carcinoma Melanoma Other

DISPOSITION

3-25b

❑ No intervention ❑ Follow-up ❑ Histopathologic diagnosis

CASE 25 HISTORY A 45-year-old woman with a history of lentigo maligna on her face presented with this new pigmented skin lesion on her left anterior thigh. 1. Globules identify a melanocytic lesion. 2. There is significant asymmetry of color and structure plus a multicomponent global pattern. 3. Irregular pigment network, irregular globules, a focus of purplish-white color, and regression are all red flags for concern. 4. The lesion was excised and reported to be a dysplastic nevus which is a good dermoscopic– pathologic correlation. 5. Diagnosing this histopathologically as a dysplastic nevus is not a good dermoscopic– pathologic correlation.

145

146

DERMOSCOPY: AN ILLUSTRATED SELF-ASSESSMENT GUIDE

RISK ❑ Low ❑ Intermediate ✔ High ❑

1

2

DIAGNOSIS Nevus Seborrheic keratosis Basal cell carcinoma Vascular Dermatofibroma Squamous cell carcinoma Melanoma Other

3

3-25c

❑ ❑ ❑ ❑ ❑ ❑ ✔ ❑ ❑

ANSWERS Answers: 1,2,3,5

Discussion: ■



DISPOSITION ❑ No intervention ❑ Follow-up ✔ Histopathologic diagnosis ❑



■ ■

DERMOSCOPIC CRITERIA ■





■ ■





Asymmetry of color and structure (+) Multicomponent global pattern (1,2,3) Irregular pigment network (box) Irregular globules (circles) Different shades of homogeneous brown color Purplish-white color (black arrows) Regression (stars)













This new, small pigmented skin lesion looks high risk clinically and with dermoscopy. The asymmetry of color and structure is striking and is an important red flag for concern → proceed with focused attention and look for other high risk criteria. The globules are also unusually large and asymmetrically located at the periphery, another red flag for concern. The focus of pigment network is unusually thick and irregular. The pigment network and irregular globules could be considered to be irregular streaks. ■ Whatever one decides these structures are, they are still atypical and high risk. The light color seen throughout has a differential diagnosis → hypopigmentation vs regression. Different shades of homogeneous brown color is another high risk criterion to put into the entire pattern analysis algorithm. The purplish -white color does not fit the description of a blue-white veil and could represent neovacularization. There would be no point to do an incisional biopsy on such a small lesion that clinically and dermoscopically could be a melanoma. A good dermoscopic– pathologic correlation: ■ The histopathologic diagnosis should correspond with your dermoscopic diagnosis. When there is not a good dermoscopic– pathologic correlation: ■ Speak with your pathologist. ■ Consider getting another dermato-pathologists opinion.

PEARLS ■ ■



There should always be a good dermoscopic– pathologic correlation. Having digital dermoscopic images of a potentially high risk lesion is the best way to go back and make sure that there is a good dermoscopic– pathologic correlation. Addendum: Initially the lesion was excised with clear margins. Based on a poor dermoscopic–pathologic correlation (histopathologic diagnosis was a dysplastic nevus), another histopathologic opinion was obtained with a pigmented lesion expert. The second opinion was that of an early invasive melanoma, which led to an appropriate re-excision. The patient is doing well 5 years post diagnosis.

Chapter 3

Trunk and Extremities

RISK ❑ Low ❑ Intermediate ❑ High

DIAGNOSIS

3-26a

❑ ❑ ❑ ❑ ❑ ❑ ❑ ❑

Nevus Seborrheic keratosis Basal cell carcinoma Vascular Dermatofibroma Squamous cell carcinoma Melanoma Other

DISPOSITION

3-26b

❑ No intervention ❑ Follow-up ❑ Histopathologic diagnosis

CASE 26 HISTORY A 47-year-old woman with a history of melanoma and multiple seborrheic keratosis developed this new lesion in the epigastric area. 1. 2. 3. 4.

Clinically and dermoscopically with “fat fingers” this looks like a seborrheic keratosis. Globules identify a melanocytic lesion. Large peripheral globules create a “Spitzoid” global pattern. Clinically and dermoscopically the lesion appears to be a seborrheic keratosis which is a good clinical– dermoscopic correlation. 5. A “Spitzoid” lesion in an adult is not a red flag for concern.

147

148

DERMOSCOPY: AN ILLUSTRATED SELF-ASSESSMENT GUIDE

RISK ❑ Low ✔ Intermediate ❑ ❑ High

DIAGNOSIS Nevus Seborrheic keratosis Basal cell carcinoma Vascular Dermatofibroma Squamous cell carcinoma Melanoma Other

DISPOSITION ❑ No intervention ❑ Follow-up ✔ Histopathologic diagnosis ❑

3-26c

✔ ❑ ❑ ❑ ❑ ❑ ❑ ❑ ❑

ANSWERS Answers: 2,3

Discussion: ■

■ ■

DERMOSCOPIC CRITERIA ■

■ ■ ■

Symmetry of color and structure (+) “Spitzoid” global pattern Irregular globules (arrows) Multifocal hypopigmentation (stars)









■ ■

This new lesion was less than 6 mm and looked like a seborrheic keratosis clinically. It was a surprise to see this dermoscopic picture. If it were not for the history that this was a new lesion, it might not have been examined with dermoscopy. To avoid missing high risk incognito lesions, one should routinely examine clinically banal appearing lesions. This was a clinically false negative high risk moderately dysplastaic nevus with atypical “Spitzoid” features histopathologically. This is an excellent example of a bad clinico-dermoscopic correlation → a red flag for concern. ■ The clinical and dermoscopic pictures do not match up well. Even though the globules are large, they do not look like “fat fingers.” ■ Inter-observer agreement even among expert dermoscopists is not always good. ■ Another dermoscopist might think that the globules do resemble “fat fingers.” Symmetry of color and structure can still be found in high risk pathology. The hypopigmentation has no significance and should not be confused with regression, which would be whiter.

PEARLS ■





It can be difficult for even an experienced dermato-pathologist to differentiate a benign “Spitzoid” lesion from a “Spitzoid” melanoma. Clarify with the dermato-pathologist if a benign “Spitzoid” lesion with atypical features is really not a melanoma. Several experienced dermato-pathologists might not be able to correctly diagnose a “Spitzoid” melanoma. ■ Consider sentinel node biopsy with atypical “Spitzoid” lesions if they have sufficient depth of invasion.

Chapter 3

Trunk and Extremities

RISK ❑ Low ❑ Intermediate ❑ High

DIAGNOSIS

3-27a

❑ ❑ ❑ ❑ ❑ ❑ ❑ ❑

Nevus Seborrheic keratosis Basal cell carcinoma Vascular Dermatofibroma Squamous cell carcinoma Melanoma Other

DISPOSITION

3-27b

❑ No intervention ❑ Follow-up ❑ Histopathologic diagnosis

CASE 27 HISTORY The pediatrician of this 5-year-old referred the patient for evaluation of a congenital melanocytic nevus that was getting bigger, and recently developed a few new dark spots. The patient’s mother was very worried. 1. Dots and globules identify a melanocytic lesion. 2. The dots and globules are more or less similar in size and shape, and symmetrically located throughout the lesion. 3. Asymmetry of color and structure, irregular dots and globules, and irregular blotches characterize this dysplastic congenital nevus. 4. Islands of normal skin and the globular pattern characterize this banal congenital nevus. 5. The presence of high risk criteria (eg, a multicomponent global pattern) in a congenital melanocytic nevus is always high risk.

149

150

DERMOSCOPY: AN ILLUSTRATED SELF-ASSESSMENT GUIDE

RISK ✔ Low ❑ ❑ Intermediate ❑ High

DIAGNOSIS Nevus Seborrheic keratosis Basal cell carcinoma Vascular Dermatofibroma Squamous cell carcinoma Melanoma Other

3-27c

✔ ❑ ❑ ❑ ❑ ❑ ❑ ❑ ❑

ANSWERS DISPOSITION

Answers: 1,2,4

❑ No intervention ✔ Follow-up ❑ ❑ Histopathologic diagnosis

Discussion: ■ ■

■ ■

DERMOSCOPIC CRITERIA ■

■ ■









Symmetry of color and structure Globular global pattern Regular dots and globules (circles) Hyperpigmented globules (black boxes) Islands of normal skin (black arrows) Perifollicular hypopigmentation (yellow boxes) Lanugo hairs (yellow arrows)





■ ■



Clinically and dermoscopically this is a banal congenital melanocytic nevus. There is symmetry of color and structure → this pattern is seen throughout the entire lesion. There are uniform, regular dots and globules. Foci of darker color (hyperpigmentation) are routinely seen in congenital nevi and are not a sign of atypia. Islands of normal skin may or may not be found around hairs typically found in congenital nevi. ■ Perifolicular hypopigmentation Fine lanugo or thick dark terminal hairs are commonly but not always seen in congenital nevi. There are no high risk criteria in this lesion. High risk criteria (eg, irregular dark blotches or irregular bluish globules) would be a red flag for concern and a histopathologic diagnosis should be considered. It is expected that congenital melanocytic nevi will enlarge as this child grows.

PEARLS ■ ■



The management of congenital melanocytic nevi is controversial. Most authorities believe that they should either be excised or monitored to identify early changes of malignancy. Digital gross and digital dermoscopic imaging for sequential monitoring would be the cutting-edge way to follow any congenital melanocytic nevus → small, medium, or large. ■ Side-by-side comparisons of baseline and follow-up images could be made to look for important changes over time such as: ■ Asymmetrical enlargement ■ The presence of new high risk criteria ■ The presence of new colors ■ Regression ■ Disappearance of criteria

Chapter 3

Trunk and Extremities

RISK ❑ Low ❑ Intermediate ❑ High

DIAGNOSIS

3-28a

❑ ❑ ❑ ❑ ❑ ❑ ❑ ❑

Nevus Seborrheic keratosis Basal cell carcinoma Vascular Dermatofibroma Squamous cell carcinoma Melanoma Other

DISPOSITION

3-28b

❑ No intervention ❑ Follow-up ❑ Histopathologic diagnosis

CASE 28 HISTORY Over the past 3 months this acquired pigmented skin lesion on the back of a 31-year-old woman got bigger. 1. A central white patch and peripheral pigment network characterize this dermatofibroma. 2. Regular pigment network, regular globules, and hypopigmentation characterize this small congenital nevus. 3. The irregular black blotch and regression are clues that this is a high risk lesion. 4. Regular pigment network, regular globules, symmetry of color and structure, plus a “Spitzoid” global pattern characterize this Spitz nevus. 5. Asymmetry of color and structure, a multicomponent global pattern, irregular pigment network, irregular dots and globules, an irregular black blotch and regression characterize this melanoma.

151

152

DERMOSCOPY: AN ILLUSTRATED SELF-ASSESSMENT GUIDE

RISK

1

❑ Low ❑ Intermediate ✔ High ❑ 2

3

DIAGNOSIS Nevus Seborrheic keratosis Basal cell carcinoma Vascular Dermatofibroma Squamous cell carcinoma Melanoma Other

DISPOSITION ❑ No intervention ❑ Follow-up ✔ Histopathologic diagnosis ❑

3-28c

❑ ❑ ❑ ❑ ❑ ❑ ✔ ❑ ❑

ANSWERS Answers: 3,5

Discussion: ■

DERMOSCOPIC CRITERIA ■









■ ■

Asymmetry of color and structure Multicomponent global pattern (1,2,3) Irregular pigment network (black boxes) Irregular dots and globules (circles) Irregular black blotch (yellow box) Regression (stars) “Peppering” (white boxes)

■ ■

■ ■





Clinically, one sees a persistent mark on the skin which is created by pressure from the instrumentation. This sign could mean: ■ This is an interesting case that many people looked at. ■ This is a difficult case that a novice dermoscopist spent a lot of time looking at. This is a melanocytic lesion because there is pigment network and globules. There are foci where the network and globules seem to be a unit → this is an unusual union. The area of regression is widespread and well developed. The lesion is more malignant then benign looking, and has a differential diagnosis → dysplastic nevus. The central patch of a dermatofibroma is not always centrally located but it is always bony– white, not gray, in color. This could not be a congenital nevus because it recently developed and the overall/global pattern would not be seen in a congenital lesion.

PEARL ■

If one needs too much time to figure it out, then it’s time to cut it out!

Chapter 3

Trunk and Extremities

RISK ❑ Low ❑ Intermediate ❑ High

DIAGNOSIS

3-29a

❑ ❑ ❑ ❑ ❑ ❑ ❑ ❑

Nevus Seborrheic keratosis Basal cell carcinoma Vascular Dermatofibroma Squamous cell carcinoma Melanoma Other

DISPOSITION

3-29b

❑ No intervention ❑ Follow-up ❑ Histopathologic diagnosis

CASE 29 HISTORY A 50-year-old man was found to have this asymptomatic lesion on his left lower leg. 1. Pigment network and globules identify a melanocytic lesion. 2. Pigment network and central white patch diagnose a dermatofibroma. 3. The central white patch of this dermatofibroma is characterized by a negative pigment network (ie, white pigment network, reticular depigmentation) and homogenous white color. 4. Reticular depigmentation (white pigment network) can be found in dermatofibromas, Spitz nevi, and melanoma. 5. A white pigment network might be the only clue that a lesion is a melanoma.

153

154

DERMOSCOPY: AN ILLUSTRATED SELF-ASSESSMENT GUIDE

RISK ✔ Low ❑ ❑ Intermediate ❑ High

DIAGNOSIS Nevus Seborrheic keratosis Basal cell carcinoma Vascular Dermatofibroma Squamous cell carcinoma Melanoma Other

DISPOSITION ✔ No intervention ❑ ❑ Follow-up ❑ Histopathologic diagnosis

3-29c

❑ ❑ ❑ ❑ ✔ ❑ ❑ ❑ ❑

ANSWERS Answers: 2,3,4,5

Discussion: ■



DERMOSCOPIC CRITERIA ■









Regular pigment network (black boxes) Central white patch (black arrows) Reticular depigmentation (white boxes) Globular-like structures (yellow arrows) Pink color



■ ■



■ ■

The pink color seen clinically and with dermoscopy represents diffuse erythema. Diffuse erythema and/or small vessels with different shapes (eg, pinpoint, linear, branched) are commonly found in dermatofibromas and are not high risk. Presence of a pigment network is not always diagnostic of a melanocytic lesion because it can also be seen in lentignes and dermatofibromas. The central patch of a dermatofibroma should be milky/bony–white. The central white patch in this lesion has two components: ■ Homogenous white color ■ Reticular white color→ reticular depigmentation, white network, negative pigment network, white pigment network. The globular-like structures are not true globules of a melanocytic lesion (ie, nests of melanocytes) but are created by the reticular depigmentation over a background of brown color. A reticulated central white patch is commonly found in dermatofibromas. A reticulated white pigment network can also be found in banal acquired nevi, Spitz nevi, and melanoma.

PEARLS ■





Typically, dermatofibromas have a characteristically firm feel on palpitation → use your fingers and touch the lesion in question. Look carefully for melanoma-specific criteria when there is a negative pigment network, so as not to miss a dermatofibroma-like melanoma. A reticulated white pigment network might be the only clue that a lesion is a melanoma.

Chapter 3

Trunk and Extremities

RISK ❑ Low ❑ Intermediate ❑ High

DIAGNOSIS

3-30a

❑ ❑ ❑ ❑ ❑ ❑ ❑ ❑

Nevus Seborrheic keratosis Basal cell carcinoma Vascular Dermatofibroma Squamous cell carcinoma Melanoma Other

DISPOSITION

3-30b

❑ No intervention ❑ Follow-up ❑ Histopathologic diagnosis

CASE 30 HISTORY A 38-year-old woman noticed a color change over a few week period in this pigmented lesion located on her left arm. 1. A central white patch and reticular depigmentation characterize this dermatofibroma. 2. An eccentric focus of reticular depigmentation is a clue that this night not be a dermatofibroma but a melanoma. 3. Globules identify a melanocytic lesion. 4. Asymmetry of color and structure, a multicomponent global pattern, multifocal hypopigmentation, and regular dots and globules characterize this mildly dysplastic nevus. 5. White and gray homogeneous colors plus “peppering” characterize the regression in this melanoma.

155

156

DERMOSCOPY: AN ILLUSTRATED SELF-ASSESSMENT GUIDE

RISK ❑ Low ❑ Intermediate ✔ High ❑

1

DIAGNOSIS 2

Nevus Seborrheic keratosis Basal cell carcinoma Vascular Dermatofibroma Squamous cell carcinoma Melanoma Other

DISPOSITION ❑ No intervention ❑ Follow-up ✔ Histopathologic diagnosis ❑

3

3-30c

❑ ❑ ❑ ❑ ❑ ❑ ✔ ❑ ❑

ANSWERS Answers: 2,3,5

Discussion: ■ ■

DERMOSCOPIC CRITERIA ■









■ ■

Asymmetry of color and structure Multicomponent global pattern (1,2,3) Irregular dots and globules (black boxes) Bony-white color of regression (black arrows) Gray homogeneous color of regression (yellow arrows) “ Peppering “ (yellow box) Reticular depigmentation (white arrows)











This is a melanocytic lesion because there are brown globules. There is asymmetry of color and structure because: ■ The gray homogeneous color on the right side is not seen on the left side. ■ The gray homogeneous color seen in the lower half is not seen in the upper half. ■ Reticular depigmentation is mostly on the right side. ■ Dots and globules are just on the right side. The central white patch of a dermatofibroma is in the differential diagnosis of the bony-white color of regression. The white color should not be confused with hypopigmentation → It is too white. An absence of pigment network does not rule out a dermatofibroma → there are dermatofibromas without pigment network or central white patch. Homogenous gray color and the fine gray dots “peppering” are part of the regression. The large area of regression is much easier to see than the foci of reticular depigmentation.

PEARLS ■ ■

■ ■

Typically there is no history of change with dermatofibromas. The entire clinical scenario is important: ■ Personal and family history ■ History of the lesion ■ Clinical and dermoscopic features. Dermoscopy in a vacuum is not as helpful as knowing the whole story. Think differential diagnosis for local criteria and global patterns.

Chapter 3

Trunk and Extremities

RISK ❑ Low ❑ Intermediate ❑ High

DIAGNOSIS

3-31a

❑ ❑ ❑ ❑ ❑ ❑ ❑ ❑

Nevus Seborrheic keratosis Basal cell carcinoma Vascular Dermatofibroma Squamous cell carcinoma Melanoma Other

DISPOSITION

3-31b

❑ No intervention ❑ Follow-up ❑ Histopathologic diagnosis

CASE 31 HISTORY A 33-year-old woman recently noticed an irregularly shaped skin lesion on her left lower leg. There was no history of any changes. 1. Globules diagnose a melanocytic lesion. 2. Multifocal hypopigmentation, symmetry of color and structure, and regular dots and globules diagnose a benign nevus. 3. Clinically and dermoscopically, there is the suggestion of regression, a red flag for concern. 4. The absence of reticular depigmentation rules out a melanoma. 5. A multicomponent global pattern, asymmetry of color and structure, irregular dots and globules, regression and reticular depigmentation characterize this invasive melanoma.

157

158

DERMOSCOPY: AN ILLUSTRATED SELF-ASSESSMENT GUIDE

RISK ❑ Low ❑ Intermediate ✔ High ❑

1

DIAGNOSIS Nevus Seborrheic keratosis Basal cell carcinoma Vascular Dermatofibroma Squamous cell carcinoma Melanoma Other

DISPOSITION ❑ No intervention ❑ Follow-up ✔ Histopathologic diagnosis ❑

2

3

ANSWERS Answers: 1,3,5

Discussion: ■ ■ ■

DERMOSCOPIC CRITERIA ■





■ ■

Asymmetry of color and structure Multicomponent global pattern (1,2,3) Irregular dots and globules (circles) Regression (stars) Reticular depigmentation (boxes)

3-31c

❑ ❑ ❑ ❑ ❑ ❑ ✔ ❑ ❑

The reticular depigmentation is more widespread than in Case 30. Compared to Case 30 the regression is not bony-white. Multifocal hypopigmentation would be in the differential diagnosis of the areas of regression. ■ Gray color favors regression over hypopigmentation. ■ Hypopigmentation is light brown color and does not contain gray color. ■ Regression is observed clinically, which is a point in favor of regression over hypopigmentation.

PEARLS ■ ■



The important features could be missed if one is in a hurry. If there are clues that a lesion might be high risk, think to yourself slow down, focus your attention, and look carefully at the lesion. High risk criteria are not always easy to find (ie, reticular depigmentation) but might be the only clues to suggest the seriousness of a lesion.

Chapter 3

Trunk and Extremities

RISK ❑ Low ❑ Intermediate ❑ High

DIAGNOSIS

3-32a

❑ ❑ ❑ ❑ ❑ ❑ ❑ ❑

Nevus Seborrheic keratosis Basal cell carcinoma Vascular Dermatofibroma Squamous cell carcinoma Melanoma Other

DISPOSITION

3-32b

❑ No intervention ❑ Follow-up ❑ Histopathologic diagnosis

CASE 32 HISTORY A 63-year-old man had this lesion on his chest that has been getting bigger and darker over a 1 year period. 1. Clinically and dermoscopically, the eccentric pigmentation is diagnostic of a melanoma. 2. Peripheral pigment network and a black central white patch characterize a senile dermatofibroma. 3. The irregular pigment network and irregular black blotch are melanoma-specific criteria. 4. Symmetry of color and structure, regular pigment network, a regular black blotch, and regression characterize this melanoma. 5. Asymmetry of color and structure, a multicomponent global pattern, irregular pigment network, irregular black blotch, and hypopigmentation characterize this melanoma.

159

160

DERMOSCOPY: AN ILLUSTRATED SELF-ASSESSMENT GUIDE

RISK ❑ Low ❑ Intermediate ✔ High ❑ 2

DIAGNOSIS Nevus Seborrheic keratosis Basal cell carcinoma Vascular Dermatofibroma Squamous cell carcinoma Melanoma Other

DISPOSITION ❑ No intervention ❑ Follow-up ✔ Histopathologic diagnosis ❑

1 3

3-32c

❑ ❑ ❑ ❑ ❑ ❑ ✔ ❑ ❑

ANSWERS Answers: 3, 5

Discussion: ■ ■ ■

DERMOSCOPIC CRITERIA ■









Asymmetry of color and structure (+) Multicomponent global pattern (1,2,3) Irregular pigment network (boxes) Irregular black blotch (yellow arrows) Hypopigmentation (stars)











Pigment network identifies a melanocytic lesion. A black central white patch does not exist in dermatofibromas. To date senile dermatofibromas have not been reported. Eccentric criteria (eg, pigment network, dots and globules, blotches) are a red flag for concern. Eccentric criteria may or may not be high risk → melanoma-specific criteria. ■ One must determine if eccentric criteria are regular or irregular, low or high risk, good or bad. The black blotch is irregular because: ■ It is asymmetrically located. ■ There are different shades of dark color. ■ The borders are irregular. The pigment network is irregular because: ■ There are only a few foci of network. ■ The line segments are thick and branched. The hypopigmentation should not be confused with regression. ■ It is not bony-white ■ The hypopigmentation has no diagnostic significance.

PEARLS ■ ■

Eccentric criteria are a red flag for concern. Proceed with focused attention and analyze all of the criteria in the lesion.

Chapter 3

Trunk and Extremities

RISK ❑ Low ❑ Intermediate ❑ High

DIAGNOSIS

3-33a

❑ ❑ ❑ ❑ ❑ ❑ ❑ ❑

Nevus Seborrheic keratosis Basal cell carcinoma Vascular Dermatofibroma Squamous cell carcinoma Melanoma Other

DISPOSITION

3-33b

❑ No intervention ❑ Follow-up ❑ Histopathologic diagnosis

CASE 33 HISTORY A 24-year-old, fair-skinned, blue-eyed, red head with multiple banal appearing nevi was found to have this lesion on his abdomen. 1. 2. 3. 4. 5.

This is a melanocytic lesion by default. There is asymmetry of color and structure and a multicomponent global pattern. The differential diagnosis of the light area includes hypopigmentation or regression. The eccentrically located regular blue blotch is a red flag for concern. The dermoscopic differential diagnosis includes a banal nevus, combined nevus, or regressive melanoma.

161

162

DERMOSCOPY: AN ILLUSTRATED SELF-ASSESSMENT GUIDE

RISK ❑ Low ✔ Intermediate ❑ ❑ High

1

2

DIAGNOSIS Nevus Seborrheic keratosis Basal cell carcinoma Vascular Dermatofibroma Squamous cell carcinoma Melanoma Other

DISPOSITION ❑ No intervention ❑ Follow-up ✔ Histopathologic diagnosis ❑

3

4

3-33c

✔ ❑ ❑ ❑ ❑ ❑ ❑ ❑ ❑

ANSWERS Answers: 1,2,3,4,5

Discussion: ■ ■

DERMOSCOPIC CRITERIA ■



■ ■ ■

■ ■

Asymmetry of color and structure Multicomponent global pattern (1,2,3,4) Regular blue blotch (box) Hypopigmentation (stars) Irregular dot and globular-like structures (circles) Irregular purplish-brown color Gray homogenous color (yellow arrows)

■ ■







Compared to Case 32, the entire picture is not so clear. This is a melanocytic lesion by default because there are no clear cut criteria for a melanocytic lesion, seborrheic keratosis, basal cell carcinoma, dermatofibroma, or vascular lesion. White color is always a red flag for concern. The white and gray colors favor regression over hypopigmentation. ■ Histopathologically, there was no regression found in this lesion. One cannot tell if there are true dots and globules or they are created by the irregularity of the dark color. The blue color is considered a blotch because it is larger than dots or globules. The blue blotch has a histopathologic differential diagnosis that includes: ■ The blue nevus component of a combined nevus ■ Melanocytic atypia ■ Malignant melanocytes

PEARLS ■





This case points out the need for one to be as knowledgeable as possible and the need to think in terms of dermoscopic and histopathologic differential diagnosis. Dermoscopic criteria are not always what they appear to be (eg, there is no regression in this lesion). Dermoscopy is not a perfect technique. Don’t expect it to be!

Chapter 3

Trunk and Extremities

RISK ❑ Low ❑ Intermediate ❑ High

DIAGNOSIS

3-34a

❑ ❑ ❑ ❑ ❑ ❑ ❑ ❑

Nevus Seborrheic keratosis Basal cell carcinoma Vascular Dermatofibroma Squamous cell carcinoma Melanoma Other

DISPOSITION

3-34b

❑ No intervention ❑ Follow-up ❑ Histopathologic diagnosis

CASE 34 HISTORY A 17-year-old presented with multiple small congenital nevi. There was no history of change in any of the lesions. While examining all of the nevi with dermoscopy, this “ugly duckling” nevus with an irregular black blotch was found. Initially, the blotch was not seen clinically because it was very small. 1. This is a banal appearing congenital combined nevus. 2. Dermoscopically and clinically this is an “ugly duckling” lesion because it was the only nevus with a black blotch. 3. On close inspection, cobblestone-like globules are seen asymmetrically located in the lesion. 4. The irregular black blotch could represent transepidermal elimination of melanin, melanocytic atypia, or melanoma arising in a small congenital melanocytic nevus. 5. To avoid a large keloid with a complete excision, one could biopsy the black blotch to make the diagnosis.

163

164

DERMOSCOPY: AN ILLUSTRATED SELF-ASSESSMENT GUIDE

RISK

1

❑ Low ❑ Intermediate ✔ High ❑ 2

DIAGNOSIS Nevus Seborrheic keratosis Basal cell carcinoma Vascular Dermatofibroma Squamous cell carcinoma Melanoma Other

DISPOSITION ❑ No intervention ❑ Follow-up ✔ Histopathologic diagnosis ❑

3

3-34c

✔ ❑ ❑ ❑ ❑ ❑ ❑ ❑ ❑

ANSWERS Answers: 2,3,4,5

Discussion: ■



DERMOSCOPIC CRITERIA ■





■ ■







Asymmetry of color and structure Multicomponent global pattern (1,2,3) Irregular black blotch (yellow arrows) Bluish-white color (yellow star) Hypopigmentation (black stars) Irregular cobblestone globules (white box) Irregular dots and globules (circles) Scale (yellow boxes)









■ ■

It was a surprise to find the black blotch in what appeared to be a banal small congenital melanocytic nevus. All of the other nevi had a classic cobblestone global pattern. ■ Uniform large angulated globules filling the lesion. Retrospectively, a black speck was seen clinically but only after it was found with dermoscopy. One should always try to examine clinically banal as well as atypical lesions no matter how many nevi a patient has. ■ Have the patient come back another time if there is not enough time to fully examine most if not all of the lesions. An irregular cobblestone pattern is in the differential diagnosis of the multicomponent global pattern. The irregular black blotch is a red flag for concern and the histopathologic correlates include: ■ Transepidermal elimination of melanin ■ Melanocytic atypia ■ Malignant melanocytes The black color rules out a combined nevus in which the color should be bluish. There is significant scaliness, which creates areas that look like reticular depigmentation. ■ A quick swipe with an alcohol prep will eliminate the scaliness to confirm that there is no reticular depigmentation.

PEARLS ■ ■ ■



Always eliminate dryness over a lesion to get a better dermoscopic view. Melanoma arising in a small satellite congenital melanocytic nevus is a rarity. Beauty is in the eye of the beholder. The “ugly duckling” is not always ugly especially to its mother. A clinical and/or dermoscopically “ugly duckling” lesion is always a red flag for concern but it is not always high risk.

Chapter 3

Trunk and Extremities

RISK ❑ Low ❑ Intermediate ❑ High

DIAGNOSIS

3-35a

❑ ❑ ❑ ❑ ❑ ❑ ❑ ❑

Nevus Seborrheic keratosis Basal cell carcinoma Vascular Dermatofibroma Squamous cell carcinoma Melanoma Other

DISPOSITION

3-35b

❑ No intervention ❑ Follow-up ❑ Histopathologic diagnosis

CASE 35 HISTORY A 55-year-old man developed a dark papule in a pre-existing pigmented skin lesion on his abdomen. 1. 2. 3. 4. 5.

Globules identify a melanocytic lesion. Milia-like cysts and pigmented pseudofollicular openings diagnose a seborrheic keratosis. The large blue ovoid nest of pigment is enough to diagnose a pigmented basal cell carcinoma. Different shades of brown color are clues that this could be a melanoma. If it were not for the history of change, this could be a combined nevus.

165

166

DERMOSCOPY: AN ILLUSTRATED SELF-ASSESSMENT GUIDE

RISK

1

❑ Low ❑ Intermediate ✔ High ❑

1

2

DIAGNOSIS Nevus Seborrheic keratosis Basal cell carcinoma Vascular Dermatofibroma Squamous cell carcinoma Melanoma Other

3 1

ANSWERS Answers: 1,4,5

Discussion: ■



DISPOSITION ❑ No intervention ❑ Follow-up ✔ Histopathologic diagnosis ❑

■ ■



DERMOSCOPIC CRITERIA ■









■ ■ ■

Asymmetry of color and structure Multicomponent global pattern (1,2,3) Irregular brown dots and globules (black circles) Irregular blue globules (white circle) Irregular bluish-white blotch (white arrows) Hypopigmentation (stars) Different shades of brown color Milia-like cysts (boxes)

3-35c

❑ ❑ ❑ ❑ ❑ ❑ ✔ ❑ ❑











The development of a dark papule, in a pre-existing melanocytic lesion, is melanoma until proven otherwise. The skin markings, seen clinically, do not favor a benign lesion. ■ Skin markings and hairs can be found in melanomas. The irregular brown dots and globules could be easily overlooked. The blotch is irregular because: ■ There is bluish-white color. ■ There is a focus of irregular blue dots and globules. ■ It is asymmetrically located within the lesion. The blue dots and globules could be created by nests of melanocytes deeper in the dermis or by irregularities in the homogeneity of the bluish-white blotch. Different shades of homogenous brown color are a red flag for concern. ■ Different shades of any homogeneous color should always be a red flag for concern. There are a few milia-like cysts. ■ To diagnose a seborrheic keratosis there should be multiple milia-like cysts plus other criteria. ■ The irregular brown dots and globules do not look like the pigmented pseudofollicular openings seen in seborrheic keratosis. The only resemblance this melanoma has to a pigmented basal cell carcinoma is the dark bluish-white blotch. ■ A pigmented basal cell carcinoma would not have diffuse brown homogenous color. ■ Arborizing vessels are not needed to diagnose a basal cell carcinoma. A combined nevus is in the dermoscopic but not in the historical or clinical differential diagnosis. The areas of hypopigmentation are not white enough to be considered regression.

PEARLS ■





One does not need dermoscopy to realize that this could be a melanoma. ■ Dermoscopy increases the diagnosis of melanoma by at least 16% when compared to skin examination without it. Dermoscopy is an enjoyable tool that allows one to see colors and structures not visible with the naked eye or with the typical magnification that clinicians use. Dermoscopy helps confirm with an on the spot second opinion clinical impressions of high risk pathology (eg, dysplastic, Spitz nevi, squamous and basal cell carcinoma, melanoma).

Chapter 3

Trunk and Extremities

RISK ❑ Low ❑ Intermediate ❑ High

DIAGNOSIS

3-36a

❑ ❑ ❑ ❑ ❑ ❑ ❑ ❑

Nevus Seborrheic keratosis Basal cell carcinoma Vascular Dermatofibroma Squamous cell carcinoma Melanoma Other

DISPOSITION

3-36b

❑ No intervention ❑ Follow-up ❑ Histopathologic diagnosis

CASE 36 HISTORY This was the only pigmented lesion on the back of a 45-year-old man. The rest of his skin was devoid of melanocytic nevi, lentigines, hemangiomas, seborrheic, or actinic keratosis, typically found on people that live in Florida. 1. Globules identify a melanocytic lesion. 2. There is asymmetry of color and structure and a multicomponent global pattern. 3. There are irregular dots and globules, irregular streaks, irregular blackish-gray blotches, and blue-white color. 4. The dermoscopic differential diagnosis includes a dysplastic nevus and melanoma. 5. One could speculate that this will not be an in situ melanoma based on an evaluation of the entire dermoscopic picture.

167

168

DERMOSCOPY: AN ILLUSTRATED SELF-ASSESSMENT GUIDE

RISK ❑ Low ❑ Intermediate ✔ High ❑ 1 2

DIAGNOSIS Nevus Seborrheic keratosis Basal cell carcinoma Vascular Dermatofibroma Squamous cell carcinoma Melanoma Other

3 4

3-36c

❑ ❑ ❑ ❑ ❑ ❑ ✔ ❑ ❑

ANSWERS Answers: 1,2,3,5

DISPOSITION ❑ No intervention ❑ Follow-up ✔ Histopathologic diagnosis ❑

Discussion: ■



DERMOSCOPIC CRITERIA ■





■ ■





Asymmetry of color and structure Multicomponent global pattern (1,2,3,4) Irregular dots and globules (circles) Irregular streaks (black arrows) Irregular blackish-gray blotches (white arrows) Irregular blue blotch (yellow arrow) Bluish-white color (stars)







■ ■

Clinically, the lesion is not as dramatic as the dermoscopic picture. ■ All of the important melanoma-specific criteria are easy to identify. One should go over the entire checklist of points that need to be evaluated: ■ Melanocytic vs nonmelanocytic. ■ Symmetry vs asymmetry of color and structure. ■ What is the global pattern? ■ Identify and evaluate all of the local criteria. ■ Make a dermoscopic diagnosis. There is blue and white color but not the blue-white veil. ■ Blue and/or white color in any form is a high risk criterion. One can tell by the multiple colors and paucity of local criteria that this is not an in situ melanoma. Clinically and dermoscopically this looks more like a superficial spreading rather than a nodular melanoma. ■ Histopathologically it was a nodular melanoma. ■ This is not a perfect dermoscopic-pathologic correlation even though it is still a melanoma. ■ One should confirm with the dermatopathologist that this is really a nodular melanoma. A dysplastic nevus would not have so much diversity of color and structure. Foci of streaks at the periphery not their shape determines that they are irregular. ■ Regular streaks would be found symmetrically around the lesion at the periphery. ■ As in this case, streaks can be an extension of the pigment network or come off a dark blotch (tumor body).

PEARLS ■ ■

A dramatic dermoscopic picture like this one can come as a surprise. When that happens, one must refrain from outbursts of surprise with statements such as “Oh my God” or “This looks very bad.” Patients don’t like that!

Chapter 3

Trunk and Extremities

RISK ❑ Low ❑ Intermediate ❑ High

DIAGNOSIS

3-37a

❑ ❑ ❑ ❑ ❑ ❑ ❑ ❑

Nevus Seborrheic keratosis Basal cell carcinoma Vascular Dermatofibroma Squamous cell carcinoma Melanoma Other

DISPOSITION

3-37b

❑ No intervention ❑ Follow-up ❑ Histopathologic diagnosis

CASE 37 HISTORY The patient’s neighbor noticed this skin lesion on the upper thigh of a 42-year-old man. 1. A central white patch and blue blotch characterize this small dermatofibroma. 2. Multifocal hypopigmentation, regular dots and globules, and a regular blue blotch characterize this dysplastic nevus. 3. Several globules identify a melanocytic lesion. 4. The blue blotch in the context of a melanocytic lesion diagnoses this combined nevus. 5. Asymmetry of color and structure, a multicomponent global pattern, irregular globules, an irregular bluish-white blotch, regression, and pinpoint vessels characterize this nodular melanoma.

169

170

DERMOSCOPY: AN ILLUSTRATED SELF-ASSESSMENT GUIDE

RISK ❑ Low ❑ Intermediate ✔ High ❑

1

DIAGNOSIS Nevus Seborrheic keratosis Basal cell carcinoma Vascular Dermatofibroma Squamous cell carcinoma Melanoma Other

DISPOSITION ❑ No intervention ❑ Follow-up ✔ Histopathologic diagnosis ❑

2

3

3-37c

❑ ❑ ❑ ❑ ❑ ❑ ✔ ❑ ❑

ANSWERS Answers: 3,5

Discussion: ■

DERMOSCOPIC CRITERIA ■













Asymmetry of color and structure Multicomponent global pattern (1,2,3) Irregular brown globules (black boxes) Irregular bluish-white blotch (yellow arrows) White component of regression (stars) Gray component of regression (white arrows) Pinpoint vessels (yellow boxes)

■ ■

■ ■

As compared to Case 36, clinically and dermoscopically, this looks more like a nodular melanoma because there is: ■ A nodule not flat lesion ■ Paucity of local criteria ■ Pinpoint vessels The gray homogenous color favors regression over hypopigmentation. Purplish color differentiates the pinpoint vessels from dots and globules of a melanocytic lesion, which should be brown. ■ One cannot always tell the difference. A combined nevus would not have so much diversity of criteria. An atypical dermatofibroma could look like this. ■ Melanoma is in the differential diagnosis of atypical dermatofibromas. ■ Atypical dermatofibromas should be removed.

PEARLS ■





Any papule or nodule with even a suggestion that it could be a melanoma should be excised posthaste. Sequential digital monitoring is contraindicated in any lesion that could be a nodular melanoma. Any delay in making the diagnosis could affect the patient’s survival chances.

Chapter 3

Trunk and Extremities

RISK ❑ Low ❑ Intermediate ❑ High

3-38a

DIAGNOSIS ❑ ❑ ❑ ❑ ❑ ❑ ❑ ❑

Nevus Seborrheic keratosis Basal cell carcinoma Vascular Dermatofibroma Squamous cell carcinoma Melanoma Other

DISPOSITION

3-38b

❑ No intervention ❑ Follow-up ❑ Histopathologic diagnosis

CASE 38 HISTORY The daughter of this 81-year-old man found this lesion on the arm of her father and brought it to the attention of his dermatologist. 1. 2. 3. 4. 5.

Dermoscopy is not needed to diagnose this melanoma. Globules identify a melanocytic lesion. There is significant asymmetry of color and structure and a multicomponent global pattern. The dots and globules are irregular. An irregular dark blotch, bluish-white color, and regression are other melanoma-specific criteria.

171

172

DERMOSCOPY: AN ILLUSTRATED SELF-ASSESSMENT GUIDE

RISK ❑ Low ❑ Intermediate ✔ High ❑

4

2

DIAGNOSIS

1

Nevus Seborrheic keratosis Basal cell carcinoma Vascular Dermatofibroma Squamous cell carcinoma Melanoma Other

3 5 3-38c

❑ ❑ ❑ ❑ ❑ ❑ ✔ ❑ ❑

ANSWERS Answers: 1,2,3,4,5

DISPOSITION

Discussion:

❑ No intervention ❑ Follow-up ✔ Histopathologic diagnosis ❑







DERMOSCOPIC CRITERIA ■







■ ■ ■



Asymmetry of color and structure Multicomponent global pattern (1,2,3,4,5) Irregular dots and globules (circles) Irregular black blotch (yellow arrows) Regression (stars) Peppering (boxes) Bluish-white color (white arrows) Irregular blue blotch (black arrow)











It is not necessary to examine the entire lesion to diagnose this melanoma. ■ Examine the entire lesion before making a dermoscopic diagnosis. Asymmetry of color and structure. ■ The left and right sides are not mirror images of each other. ■ The lower and upper halves are not mirror images of each other. A multicomponent global pattern. ■ Three or more different areas within the lesion. ■ A single criterion or multiple criteria can make up each zone. Irregular dots and globules. ■ Different sizes and shapes ■ Asymmetrical location Irregular black blotch. ■ Irregular in size and shape ■ Asymmetrical location Regression. ■ Bony/milky-white color ■ Peppering → fine gray dots Bluish-white color. ■ This is not a blue-white veil → irregular homogenous blue color with an overlying ground glass appearance. ■ Blue and/or white color is any form is a high risk criterion. ■ The bluish-white color lies over the irregular black blotch. Irregular blue blotch. ■ Larger than a dot or globule ■ Irregular border ■ Asymmetrical location

PEARLS ■

■ ■ ■

Seeing examples of well-developed melanoma-specific criteria is essential if one has a chance to identify them when they are not so well developed. Look carefully for subtle criteria before determining that they are absent. All of the melanoma-specific (high risk) criteria can be found in benign lesions. No single criterion is pathognomonic of a melanoma.

Chapter 3

Trunk and Extremities

RISK ❑ Low ❑ Intermediate ❑ High

DIAGNOSIS

3-39a

❑ ❑ ❑ ❑ ❑ ❑ ❑ ❑

Nevus Seborrheic keratosis Basal cell carcinoma Vascular Dermatofibroma Squamous cell carcinoma Melanoma Other

DISPOSITION

3-39b

❑ No intervention ❑ Follow-up ❑ Histopathologic diagnosis

CASE 39 HISTORY A 27-year-old man moved from another city to Miami. He came in for a routine skin examination because he has a history of dysplastic nevi. This lesion on his very hairy chest was being followed in another pigmented lesion clinic with regular photography and not dermoscopy. There was no history of change. 1. 2. 3. 4.

One’s first impression is that this is a melanoma. The bluish-white veil clinches the melanoma diagnosis. The milky-red globules are diagnostic of a melanoma. The lesion was excised with a diagnosis of a dysplastic nevus. That is a good dermoscopic– pathologic correlation. 5. Asymmetry of color and structure, a homogenous global pattern, regular dots and globules, regular blotch, veil, and globular vessels characterize this melanoma.

173

174

DERMOSCOPY: AN ILLUSTRATED SELF-ASSESSMENT GUIDE

1

RISK ❑ Low ❑ Intermediate ✔ High ❑

2 3

4

DIAGNOSIS Nevus Seborrheic keratosis Basal cell carcinoma Vascular Dermatofibroma Squamous cell carcinoma Melanoma Other

1

5

6

3-39c

✔ ❑ ❑ ❑ ❑ ❑ ❑ ❑ ❑

2

ANSWERS Answers: 1

DISPOSITION ❑ No intervention ❑ Follow-up ✔ Histopathologic diagnosis ❑

Discussion: ■



DERMOSCOPIC CRITERIA ■





■ ■

Asymmetry of color and structure Multicomponent global pattern (1,2,3,4,5,6) Irregular dots and globules (black circles) Bluish -white color (stars) Milky-red globules (red circle)







The dermoscopic picture is dramatic compared to the not so high risk looking clinical lesion. The histopathologic diagnosis of a dysplastic nevus was not a good dermoscopic-pathologic correlation. ■ Dermoscopically it looked like a melanoma but it was not a histopathologic melanoma. ■ When there is not a good dermoscopic–pathologic correlation, seek other dermatopathologists’ opinions. ■ This is a false positive dermoscopic melanoma. ■ In this case, none of the three dermatopathologists diagnosed a melanoma. There is blue and white color but not the blue-white veil as it is defined. ■ The bluish-white color is over poorly defined irregular dark color. Milky-red globules are not diagnostic of a melanoma but are a red flag for concern. ■ It is hard to tell if some of the globules are reddish or brown. The melanoma-specific criteria include: ■ Asymmetry of color and structure ■ Multicomponent global pattern ■ Irregular dots and globules ■ Bluish-white color ■ Milky-red globules ■ Multiple colors

PEARLS ■



■ ■

Never tell a patient that they have melanoma 100%. The worst clinical scenario including the dermoscopic picture might still be benign. With difficult cases, one can send digital dermoscopic images to expert dermoscopists for their opinion. ■ Take dermoscopy to a higher level by having equipment to take digital dermoscopic images. Do not hesitate to seek other dermatopathologists opinions with difficult cases. At times it is necessary to cut hair away to get a better clinical and dermoscopic look at a suspicious skin lesion.

Chapter 3

Trunk and Extremities

RISK ❑ Low ❑ Intermediate ❑ High

DIAGNOSIS

3-40a

❑ ❑ ❑ ❑ ❑ ❑ ❑ ❑

Nevus Seborrheic keratosis Basal cell carcinoma Vascular Dermatofibroma Squamous cell carcinoma Melanoma Other

DISPOSITION

3-40b

❑ No intervention ❑ Follow-up ❑ Histopathologic diagnosis

CASE 40 HISTORY A 55-year-old man came in for a skin examination only to make sure that he was okay. This lesion was found on the upper back. He was not aware of its presence. 1. Globules identify a melanocytic lesion. 2. There is asymmetry of color and structure and a multicomponent global pattern. 3. One sees foci of irregular pigment network, irregular dots and globules, an irregular brown blotch, regression, and bluish-white color. 4. The lesion was excised and reported to be a dysplastic nevus, which is not a good dermoscopic-pathologic correlation. 5. The arborizing vessels, surrounding the lesion, indicate that there is significant sun-damaged skin.

175

176

DERMOSCOPY: AN ILLUSTRATED SELF-ASSESSMENT GUIDE

RISK ❑ Low ❑ Intermediate ✔ High ❑ 1

DIAGNOSIS Nevus Seborrheic keratosis Basal cell carcinoma Vascular Dermatofibroma Squamous cell carcinoma Melanoma Other

2

3

3-40c

❑ ❑ ❑ ❑ ❑ ❑ ✔ ❑ ❑

ANSWERS Answers: 1,2,3,4,5

DISPOSITION ❑ No intervention ❑ Follow-up ✔ Histopathologic diagnosis ❑

Discussion: ■



DERMOSCOPIC CRITERIA ■









■ ■ ■

Asymmetry of color and structure Multicomponent global pattern (1,2,3) Irregular pigment network (yellow arrows) Irregular dots and globules (circles) Irregular brown blotch (white arrow) Regression (stars) Bluish-white color (yellow box) Arborizing vessels (black arrows)







The dermoscopic diagnosis but not the histopathologic diagnosis was thought to be a melanoma. ■ That is not a good dermoscopic–pathologic correlation ■ It was sent for review to a national pigment lesion expert dermatopathologist who reported this was “definitely” not a melanoma. ■ With free margins and a benign pathology report, was the patient in the clear? ■ No, this is a false negative pathologic, but not dermoscopic melanoma. This case is the opposite of Case 39 in which the lesion was truly benign. Some of the melanoma-specific criteria are easy to see: ■ Asymmetry of color and structure ■ Multicomponent global pattern ■ Regression ■ Bluish-white color Some of the melanoma-specific criteria are not easy to see: ■ Irregular pigment network ■ Irregular dots and globules ■ Irregular brown blotch Arborizing vessels are not diagnostic of a basal cell carcinoma and can also be found in/on: ■ Nevi ■ Sebaceous gland hyperplasia ■ Scar tissue ■ Sun-damaged skin → individual mats or diffuse arborizing vessels ■ It is not always possible to differentiate a solitary mat of arborizing vessels from a basal cell carcinoma→ if in doubt, cut it out!

PEARLS ■



■ ■

Even the most experienced pigmented lesion expert dermatopathologists do not always get it right. A consensus of opinions might be better than a single opinion (eg, dermoscopists, dermatopathologists). Every one misses and/or misdiagnoses melanoma. It is a sad fact of life!

Chapter 3

Trunk and Extremities

RISK ❑ Low ❑ Intermediate ❑ High

DIAGNOSIS

3-41a

❑ ❑ ❑ ❑ ❑ ❑ ❑ ❑

Nevus Seborrheic keratosis Basal cell carcinoma Vascular Dermatofibroma Squamous cell carcinoma Melanoma Other

DISPOSITION

3-41b

❑ No intervention ❑ Follow-up ❑ Histopathologic diagnosis

CASE 41 HISTORY The patient of Case 40 returned for a 6-month follow-up skin examination and some spots were seen in the melanoma excision site. 1. This is a typical combined nevus that coincidentally is near the melanoma excision site. 2. The brown, blue, and white colors could be seen in melanoma. 3. The history, clinical, and dermoscopic appearances of the new spots are consistent with cutaneous metastatic melanoma. 4. The brown and bluish-white colors are irregular. 5. Without the history and visible surgical scar, the dermoscopic differential diagnosis includes: a combined, blue, Spitz nevus, nodular, and cutaneous metastatic melanoma.

177

178

DERMOSCOPY: AN ILLUSTRATED SELF-ASSESSMENT GUIDE

RISK ❑ Low ❑ Intermediate ✔ High ❑

❑ ❑ ❑ ❑ ❑ ❑ ✔ ❑ ❑

Nevus Seborrheic keratosis Basal cell carcinoma Vascular Dermatofibroma Squamous cell carcinoma Melanoma Other

3-41c

DIAGNOSIS

ANSWERS Answers: 2,3,4,5

Discussion: ■

DISPOSITION ❑ No intervention ❑ Follow-up ✔ Histopathologic diagnosis ❑







DERMOSCOPIC CRITERIA ■





Irregular bluish-white color (stars) Irregular brown color (white arrows) Surgical scar (black arrows)





The histopathology was said to be benign, however, the large blue and small brown spots made clear that the original diagnosis was not correct. → It was a melanoma! Cutaneous and lymph node metastasis might be the only way to diagnose melanoma in clinically and histopathologically equivocal cases. The histopathologic diagnosis of the bluish-brown area was consistent with nodular or cutaneous metastatic melanoma. ■ A sentinel lymph node biopsy was positive → metastatic melanoma. ■ The patient is disease free 6 years post diagnosis Without the history, the dermoscopic differential diagnosis includes: ■ Combined nevus ■ Spitz nevus ■ Blue nevus ■ Nodular melanoma ■ Cutaneous metastatic melanoma Dermoscopic features of cutaneous metastatic melanoma are not specific. ■ Pigmented ■ Nonpigmented → hypo or amelanotic (pink or reddish in color), ■ With or without polymorphous vessels → differently shaped small or larger telangiectatic vessels. The history of a past melanoma excision thick enough to metastasize is usually more helpful than the dermoscopic features to diagnose cutaneous metastatic melanoma.

PEARLS ■ ■

This case points out how treacherous melanoma can be. Luck might be as important as depth of invasion for patient survival. ■ The diagnosis of an in situ amelanotic melanoma in the scalp of a young man with a lot of hair → good luck. ■ A young man with an “irritated” mole who thinks it is caused by his belt, does nothing about it, and then dies at age 33 → bad luck.

Chapter 3

Trunk and Extremities

RISK ❑ Low ❑ Intermediate ❑ High

DIAGNOSIS

3-42a

❑ ❑ ❑ ❑ ❑ ❑ ❑ ❑

Nevus Seborrheic keratosis Basal cell carcinoma Vascular Dermatofibroma Squamous cell carcinoma Melanoma Other

DISPOSITION

3-42b

❑ No intervention ❑ Follow-up ❑ Histopathologic diagnosis

CASE 42 HISTORY This lesion was found in the posterior upper arm of a 42-year-old woman. 1. 2. 3. 4. 5.

Clinically and dermoscopically this could be a nodular melanoma. Clinically and dermoscopically this could be a blue nevus. Clinically and dermoscopically this could be cutaneous metastatic melanoma. This is the classic homogenous blue global pattern characteristic of a blue nevus. Irregular dots and globules plus different shades of blue and brown color are diagnostic of a nodular melanoma.

179

180

DERMOSCOPY: AN ILLUSTRATED SELF-ASSESSMENT GUIDE

RISK ✔ Low ❑ ❑ Intermediate ❑ High

DIAGNOSIS Nevus Seborrheic keratosis Basal cell carcinoma Vascular Dermatofibroma Squamous cell carcinoma Melanoma Other

3-42c

✔ ❑ ❑ ❑ ❑ ❑ ❑ ❑ ❑

DISPOSITION ✔ No intervention ❑ ❑ Follow-up ❑ Histopathologic diagnosis

ANSWERS Answers: 1,2,3,4

Discussion: ■

DERMOSCOPIC CRITERIA ■ ■

■ ■

Homogenous global pattern Irregular bluish dots and globules (boxes) Milia-like cyst (circle) Homogeneous brown color (arrows)











This is a classic blue nevus with a homogeneous global pattern. Based on the history, there could be other diagnostic possibilities: ■ Rapidly growing nodule → nodular melanoma. ■ A history of a melanoma in the area → cutaneous metastatic melanoma. Typically, one finds a solitary blue papule or nodule that the patient knows has been present for years without any changes. There are different shades of blue, bluish-white, and some suggestion of brown color. ■ This combination of colors could also be found in a combined nevus. The irregular dots and globules (nests of melanocytes) are blue because they are deeper in the dermis (ie, the “Tyndall” effect). ■ They could also be created by irregularities of the bluish-white color over the background of diffuse blue color. The milia-like cyst has no diagnostic significance. ■ Milia-like cysts are not exclusively found in seborrheic keratosis. ■ Milia-like cysts can be found in melanocytic nevi and in melanomas.

PEARLS ■ ■

The history is essential to help diagnose a blue nevus. In general, the family and personal history plus the history and clinical appearance of a lesion will help one make a more accurate dermoscopic diagnosis.

Chapter 3

Trunk and Extremities

RISK ❑ Low ❑ Intermediate ❑ High

DIAGNOSIS

3-43a

❑ ❑ ❑ ❑ ❑ ❑ ❑ ❑

Nevus Seborrheic keratosis Basal cell carcinoma Vascular Dermatofibroma Squamous cell carcinoma Melanoma Other

DISPOSITION ❑ No intervention ❑ Follow-up ❑ Histopathologic diagnosis

CASE 43 HISTORY This dermoscopic image was on the dermatology board examination without a history or clinical image. 1. Globules suggest that this could be a melanocytic lesion. 2. Ulceration, an ovoid blue nest of pigmentation, and a spoke-wheel structure suggest this could be a basal cell carcinoma. 3. Asymmetry of color and structure, a multicomponent global pattern, irregular brown globules, irregular streaks, an irregular bluish-black blotch, and regression suggest that this could be a melanoma. 4. Ulceration is present but does not help distinguish a basal cell carcinoma from a melanoma. 5. Spoke-wheel structures have not yet been reported in melanoma and are exclusively found in basal cell carcinomas.

181

182

DERMOSCOPY: AN ILLUSTRATED SELF-ASSESSMENT GUIDE

RISK ❑ Low ❑ Intermediate ✔ High ❑

1 2

DIAGNOSIS Nevus Seborrheic keratosis Basal cell carcinoma Vascular Dermatofibroma Squamous cell carcinoma Melanoma Other

3

4

3-43b

❑ ❑ ✔ ❑ ❑ ❑ ❑ ❑ ❑

ANSWERS Answers: 1,2,3,4,5

Discussion: ■

DISPOSITION ❑ No intervention ❑ Follow-up ✔ Histopathologic diagnosis ❑

■ ■



DERMOSCOPIC CRITERIA ■











■ ■

Asymmetry of color and structure Multicomponent global pattern (1,2,3,4) Irregular streak-like structures (black arrows) Irregular dot and globular-like structures (circles) Blue ovoid nest of pigmentation (white stars) Finger-like projections (yellow arrows) Ulceration (yellow box) Spoke-wheel structure (black box)







■ ■

This pigmented basal cell carcinoma can not be distinguished from melanoma. ■ There are no arborizing vessels. ■ Basal cell carcinomas can be without arborizing vessels. Multicomponent global pattern is not exclusively found in melanomas. Ulceration can be found in basal cell carcinomas and in melanomas. ■ Statistically, ulceration is found more often in basal cell carcinomas. ■ Ulceration in this case favors the diagnosis of a basal cell carcinoma. There are globular-like and streak-like structures, which means: ■ These structures resemble globules and streaks, respectively, but morphologically they are not true globules or streaks. ■ The histopathologic correlation of these criteria is not known in this case because it was not specifically looked for by the dermatopathologist. ■ They are variations of pigmentation seen in basal cell carcinomas. The blue ovoid nest of pigmentation has a differential diagnosis that includes an irregular dark blotch of a melanocytic lesion. ■ Finger-like projections coming off the blotch favor blue-ovoid basal cell nest. ■ The differential diagnosis includes irregular streaks of a melanocytic lesion. ■ This is also how “leaf-like” structures are said to look. ■ There is no resemblance to any type of leaf! ■ “Leaf-like” structures is a misnomer and the concept should be abandoned. The whitish color suggests regression. ■ White color can be found in basal cell carcinomas. The spoke-wheel structure has a central brown globule (“hub”) with a suggestion of linear structures radiating from the center (spokes). ■ Imagination may be needed to diagnose spoke-wheel structures. ■ Number of radial projections varies. ■ One or more spoke-wheels can be found in a basal cell carcinoma. ■ Spoke-wheel structures might be the only criterion to diagnose a basal cell carcinoma → no arborizing vessels, pigmentation, or ulceration. One cannot always differentiate melanoma from basal cell carcinoma. Dermoscopy shows → histopathologic diagnosis is indicated.

PEARLS ■



Use the potentially most high risk diagnosis to plan a surgical approach: ■ Biopsy a basal cell carcinoma ■ Excision of a potential melanoma This case points out the need to create a dermoscopic differential diagnosis.

Chapter 3

Trunk and Extremities

RISK ❑ Low ❑ Intermediate ❑ High

DIAGNOSIS

3-44a

❑ ❑ ❑ ❑ ❑ ❑ ❑ ❑

Nevus Seborrheic keratosis Basal cell carcinoma Vascular Dermatofibroma Squamous cell carcinoma Melanoma Other

DISPOSITION

3-44b

❑ No intervention ❑ Follow-up ❑ Histopathologic diagnosis

CASE 44 HISTORY This asymptomatic dark blue lesion was found on the thigh of an 85-year-old man. 1. 2. 3. 4.

Globules identify a melanocytic lesion. Multiple blue ovoid nests diagnose a basal cell carcinoma. Large, well-demarcated blue vascular spaces diagnose a hemangioma. Asymmetry of color and structure, a multicomponent global pattern, regular dots and globules, regular blotches and regression diagnose a nodular melanoma. 5. Areas with purplish color, suggest that this is a vascular lesion.

183

184

DERMOSCOPY: AN ILLUSTRATED SELF-ASSESSMENT GUIDE

RISK ✔ Low ❑ ❑ Intermediate ❑ High

1

DIAGNOSIS Nevus Seborrheic keratosis Basal cell carcinoma Vascular Dermatofibroma Squamous cell carcinoma Melanoma Other

3 3-44c

❑ ❑ ❑ ✔ ❑ ❑ ❑ ❑ ❑

2

DISPOSITION ❑ ✔ No intervention ❑ Follow-up ❑ Histopathologic diagnosis

ANSWERS Answers: 3,5

Discussion: ■

DERMOSCOPIC CRITERIA ■





■ ■ ■

Asymmetry of color and structure Multicomponent global pattern (1,2,3) Large blue vascular spaces (white stars) Purplish color (arrows) Whitish color (black stars) Globular-like structures (circles)

■ ■ ■









This vascular lesion points out that asymmetry of color and structure and a multicomponent global pattern are not diagnostic of a melanoma. Vascular lesions do not always have small lacunae. Vascular lesions can have large vascular spaces. The vascular spaces have a differential diagnosis that includes: ■ Ovoid nests of pigmentation → basal cell carcinoma ■ Irregular blotches → melanoma ■ Agminated (grouped in the same area) blue nevi The purplish color is a clue that this could be a vascular lesion. ■ Vascular lesions can have different shades of red, blue, and/or black colors (representing thrombosis). Vascular lesions typically have white color created by hyperkeratosis or by fibrous septa. The globular-like structures are artifacts created by oil placed over the lesion to eliminate the dryness. The less-experienced dermoscopist might consider this to be a melanoma. ■ If in doubt, cut it out!

PEARL ■

Always eliminate dryness when it exists to get a better dermoscopic view of a lesion. ■ A quick swipe with an alcohol prep gets the job done but does not last long and might need to be repeated. ■ Oil or gel (eg, ultrasound gel, hand washing gel) can be used if one wants to take digital dermoscopic images because it lasts much longer than alcohol to eliminate dryness and enhances digital image quality.

Chapter 3

Trunk and Extremities

RISK ❑ Low ❑ Intermediate ❑ High

DIAGNOSIS

3-45a

❑ ❑ ❑ ❑ ❑ ❑ ❑ ❑

Nevus Seborrheic keratosis Basal cell carcinoma Vascular Dermatofibroma Squamous cell carcinoma Melanoma Other

DISPOSITION

3-45b

❑ No intervention ❑ Follow-up ❑ Histopathologic diagnosis

CASE 45 HISTORY This was found on the left forearm arm of a 67-year-old woman. 1. 2. 3. 4. 5.

Globules identify a melanocytic lesion. There is asymmetry of color and structure and a multicomponent global pattern. There are irregular dots and globules, bluish-white color, and regression. A few milia-like cysts are not enough to diagnose a pigmented seborrheic keratosis. A focus of milky-red globules helps diagnose this melanoma.

185

186

DERMOSCOPY: AN ILLUSTRATED SELF-ASSESSMENT GUIDE

RISK ❑ Low ❑ Intermediate ✔ High ❑

4

1 3

DIAGNOSIS Nevus Seborrheic keratosis Basal cell carcinoma Vascular Dermatofibroma Squamous cell carcinoma Melanoma Other

DISPOSITION ❑ No intervention ❑ Follow-up ✔ ❑ Histopathologic diagnosis

2 2

3-45c

❑ ❑ ❑ ❑ ❑ ❑ ✔ ❑ ❑

ANSWERS Answers: 1,2,3,4,5

Discussion: ■ ■ ■

DERMOSCOPIC CRITERIA ■







■ ■ ■



Asymmetry of color and structure Multicomponent global pattern (1,2,3,4) Irregular dots and globules (circles) Bluish-white color (white arrows) Regression (stars) Milky-red area (yellow arrows) Normal skin markings (red arrows) Milia-like cysts (black arrows)

■ ■



■ ■

Melanoma arose in a nevus → area 2 of multicomponent global pattern. Shows the importance of a systematic analysis of a suspicious lesion. This is melanocytic because there are multiple globules. There is obvious asymmetry of color and structure. The global pattern is multicomponent because there are four distinct areas with different criteria. Local criteria include: ■ Irregular dots and globules ■ Bluish-white color → there is no “veil” ■ Areas of regression are questionably present ■ A small focus of milky-red area (neovascularization) Regression not easily seen with dermoscopy was present histopathologically. The skin markings have no diagnostic significance. ■ They crisscross the homogeneous brown color of the benign nevus component of the lesion.

PEARLS ■

■ ■

Memorizing a systematic plan of attack requires motivation to master the technique, dedication, and some hard work. Having a written cheat sheet of the important criteria may aid learning. Better communication between the dermoscopist and pathologist could lead to a more accurate histopathologic diagnosis. ■ Regression is seen with dermoscopy but not reported histopathologically. ■ Important criteria might be asymmetrically located in the lesion but not reported histopathologically → more cuts might be indicated.

Chapter 3

Trunk and Extremities

RISK ❑ Low ❑ Intermediate ❑ High

DIAGNOSIS

3-46a

❑ ❑ ❑ ❑ ❑ ❑ ❑ ❑

Nevus Seborrheic keratosis Basal cell carcinoma Vascular Dermatofibroma Squamous cell carcinoma Melanoma Other

DISPOSITION

3-46b

❑ No intervention ❑ Follow-up ❑ Histopathologic diagnosis

CASE 46 HISTORY A 36-year-old man was concerned about this darkening lesion on his right arm. 1. Clinically and dermoscopically, the blue and brown areas represent cutaneous metastatic melanoma. 2. This is the classic “starburst” global pattern of a Spitz nevus. 3. There are irregular streaks at the periphery. 4. There are regular streaks at the periphery. 5. The majority of the lesion is filled with a regular black blotch.

187

188

DERMOSCOPY: AN ILLUSTRATED SELF-ASSESSMENT GUIDE

RISK ✔ Low ❑ ❑ Intermediate ❑ High

DIAGNOSIS Nevus Seborrheic keratosis Basal cell carcinoma Vascular Dermatofibroma Squamous cell carcinoma Melanoma Other

DISPOSITION ❑ No intervention ❑ Follow-up ✔ ❑ Histopathologic diagnosis

3-46c

✔ ❑ ❑ ❑ ❑ ❑ ❑ ❑ ❑

ANSWERS Answers: 2,4,5

Discussion: ■

DERMOSCOPIC CRITERIA ■ ■

■ ■ ■ ■

“Starburst” global pattern Symmetry of color and structure Regular streaks (arrows) Regular black blotch (stars) Bluish-white color Nevus (box)









■ ■



This is a classic “starburst” global pattern: ■ Regular streaks ■ Homogeneous dark color ■ Bluish-white color The streaks are considered to be regular because they are located at most points along the periphery. Streaks are considered irregular when they are not located at most points along the periphery (ie, when there are only foci of streaks, they are considered irregular). ■ Location at the periphery, not the shape, determines if streaks are regular or irregular. Dots and globules at the periphery can be components of the “starburst” pattern → they are not present in this lesion. Any of the six dermoscopic patterns representing Spitz nevi are referred to as being “Spitzoid”: ■ Starburst ■ Globular ■ Homogeneous ■ Vascular (pink or reddish) ■ Black pigment network (reticular) ■ Nonspecific A regular “Spitzoid” pattern is less worrisome than an irregular “Spitzoid” pattern. Cutaneous metastatic melanomas could have a homogeneous dark blotch but not a symmetrical ring of streaks at the periphery. The brown color with a fine regular pigment network represents a nevus that has nothing to do with the Spitz nevus. ■ It is not uncommon to have different pathologies adjacent to each other (eg, seborrheic keratoses, nevi, and/or hemangiomas).

Chapter 3



A “black lamella” (ie, pigmented parakeratosis, histopathologically) is in the differential diagnosis of the black blotch. ■ It is a jet black thin appearing blotch with an overall shiny appearance → similar to the black color of a black widow spider. ■ Tape stripping can help to make the differentiation → specks from the “black lamella” can become stuck to the tape. ■ The “black lamella” does not always come off with tape stripping.

PEARLS ■ ■ ■

An acquired “Spitzoid” lesion in an adult is always a red flag for concern. Melanoma can have a symmetrical “Spitzoid” pattern. All “Spitzoid” lesions should be considered for excision, not only in adults, and regardless of the age of the patient.

Trunk and Extremities

189

This page intentionally left blank

Chapter 3

Trunk and Extremities

RISK ❑ Low ❑ Intermediate ❑ High

DIAGNOSIS

3-47a

❑ ❑ ❑ ❑ ❑ ❑ ❑ ❑

Nevus Seborrheic keratosis Basal cell carcinoma Vascular Dermatofibroma Squamous cell carcinoma Melanoma Other

DISPOSITION

3-47b

❑ No intervention ❑ Follow-up ❑ Histopathologic diagnosis

CASE 47 HISTORY Friends of this 44-year-old man were concerned about this darkening lesion on the upper arm. 1. Similar to Case 46, this is a symmetrical “Spitzoid” or “starburst” global pattern. 2. This is an atypical “Spitzoid” pattern composed of irregular dots and globules at the periphery, an irregular black blotch, and bluish-white color. 3. The bluish-white color and irregular black blotch are melanoma-specific criteria. 4. “Spitzoid” melanomas always have a “Spitzoid” dermoscopic pattern. 5. One can have “Spitzoid” dermoscopic pattern without a “Spitzoid” histopathologic pattern.

191

192

DERMOSCOPY: AN ILLUSTRATED SELF-ASSESSMENT GUIDE

RISK ❑ Low ❑ Intermediate ✔ High ❑

DIAGNOSIS Nevus Seborrheic keratosis Basal cell carcinoma Vascular Dermatofibroma Squamous cell carcinoma Melanoma Other

3-47c

❑ ❑ ❑ ❑ ❑ ❑ ✔ ❑ ❑

DISPOSITION ❑ No intervention ❑ Follow-up ✔ ❑ Histopathologic diagnosis

ANSWERS Answers: 2,3,5

Discussion: ■

DERMOSCOPIC CRITERIA ■







■ ■

Irregular “Spitzoid” global pattern Asymmetry of color and structure (+) Irregular dots and globules (circles) Irregular black blotch (white stars) Bluish-white color (arrows) Peripheral erythema (black stars)













This is an irregular “Spitzoid” global pattern. ■ There is asymmetry of color and structure. ■ There are foci of irregular dots and globules at the periphery → some of the globules look like streaks. ■ There is bluish-white color typically found in “Spitz” nevi. ■ There is an irregular black blotch→ black color can also be found in “Spitz” nevi. Any combination of dots, globules, and/or streaks at the periphery plus light/dark brown, black, or blue color located within the lesion may be found in “Spitz” nevi. A negative or white pigment network (reticular depigmentation) is a clue that a lesion might be a “Spitz” nevus. One often has to stretch their imagination to diagnose a “Spitzoid” pattern. Experienced dermoscopists might not think this is a “Spitzoid” pattern. ■ It could be considered a multicomponent global pattern. The melanoma-specific criteria are striking and enough to make the diagnosis without further categorization as “Spitzoid.” ■ This was not a “Spitzoid” melanoma histopathologically. The peripheral erythema has no diagnostic significance in this case. ■ One can see a corona of erythema clinically around “Spitz” nevi. ■ In general, a corona of peripheral erythema seen clinically is a clue that the lesion might be a “Spitz” nevus.

PEARLS ■ ■

“Spitzoid” lesions are very commonly encountered. Any atypical “Spitzoid” dermoscopic and/or histopathologic diagnosis could actually be a melanoma.

Chapter 3

Trunk and Extremities

RISK ❑ Low ❑ Intermediate ❑ High

DIAGNOSIS

3-48a

❑ ❑ ❑ ❑ ❑ ❑ ❑ ❑

Nevus Seborrheic keratosis Basal cell carcinoma Vascular Dermatofibroma Squamous cell carcinoma Melanoma Other

DISPOSITION

3-48b

❑ No intervention ❑ Follow-up ❑ Histopathologic diagnosis

(Reprinted, with permission, from Stolz W, Braun-Falco O, Bilek P et al. (2002) Color Atlas of Dermatoscopy. Second Edition. Blackwell Publishing, Oxford.)

CASE 48 HISTORY Over a 6-month period, this black spot on the back of a 5-year-old girl has been getting bigger. 1. 2. 3. 4. 5.

Irregular streaks and black color characterize this de novo nodular melanoma. In general, this is a “Spitzoid” global pattern with the sub-category of a “starburst” pattern. Regular streaks are the major clue that this is a Spitz nevus. The black color is diagnostic of a thrombosed hemangioma. The homogenous black color could be created by a “black lamella.”

193

194

DERMOSCOPY: AN ILLUSTRATED SELF-ASSESSMENT GUIDE

RISK ✔ Low ❑ ❑ Intermediate ❑ High

✔ ❑ ❑ ❑ ❑ ❑ ❑ ❑ ❑

Nevus Seborrheic keratosis Basal cell carcinoma Vascular Dermatofibroma Squamous cell carcinoma Melanoma Other

3-48c

DIAGNOSIS

ANSWERS Answers: 2,3,5

Discussion: ■

DISPOSITION



❑ No intervention ❑ Follow-up ✔ Histopathologic diagnosis ❑ ■



DERMOSCOPIC CRITERIA ■

■ ■ ■

Symmetry of color and structure “Starburst” global pattern Regular streaks (arrows) Regular black blotch (stars)









In this age group, acquired black papules are most likely a Spitz nevus, also called Reed nevi when dark. Nodular melanoma is in clinical but not dermoscopic differential. ■ This is a classic “starburst” global pattern. ■ Melanomas do not usually demonstrate so much symmetry. ■ The more the asymmetry of color and structure the greater chance the lesion is high risk (eg, dysplastic or atypical Spitz nevus, melanoma). Regular streaks (sometimes termed “pseudopods” and “radial streaming”) are finger-like projections along the periphery. Streaks represent variations of the same criterion with the same histopathologic correlates: ■ Intraepidermal or junctional confluent radial nest of melanocytes, which can be benign or malignant. ■ The radial growth phase of melanoma. Even though jet black color may be seen in thrombosed hemangiomas, hemangiomas do not show symmetrical streaks at the periphery. This lesion could have a “black lamella” created by pigmented parakeratoses. ■ Tape strip to see if fragments of pigmented “black lamella” adhere to tape. ■ Fragments of “black lamella” may or may not stick to the tape. Spitz nevi have been shown to be dynamic, changing patterns over time (eg, a globular pattern morphing into a “starburst” pattern). Even though this lesion demonstrates striking symmetry of color and structure, histopathologically it could still be an atypical “Spitzoid” lesion or melanoma. For this reason a histopathologic diagnosis should be made. ■ Atpyical “Spitzoid” lesions are not uncommonly found in children. ■ “Spitzoid” melanomas can be found in children.

PEARLS ■ ■



One of the major benefits of dermoscopy is patient reassurance. One should take the opportunity to alleviate the anxiety most parents would have in this situation by confidently making the diagnosis of a benign “Spitz” nevus. Parents should be informed of a very small chance this could be higher risk.

Chapter 3

Trunk and Extremities

RISK

3-49a

❑ Low ❑ Intermediate ❑ High

DIAGNOSIS ❑ ❑ ❑ ❑ ❑ ❑ ❑ ❑

Nevus Seborrheic keratosis Basal cell carcinoma Vascular Dermatofibroma Squamous cell carcinoma Melanoma Other

DISPOSITION

3-49b

❑ No intervention ❑ Follow-up ❑ Histopathologic diagnosis

CASE 49 HISTORY A 45-year-old man was worried about a dark spot on his chest, which started to itch. 1. This is a melanocytic lesion because there are brown globules. 2. There is asymmetry of color and structure and a multicomponent global pattern. 3. This could be considered an atypical “Spitzoid” global pattern with irregular globules and bluish-white color. 4. Symmetry of color and structure, bluish-white color and regular globules rules out a melanoma. 5. Significant asymmetry of color and structure and several melanoma-specific local criteria suggest that this could be a dysplastic nevus or melanoma.

195

196

DERMOSCOPY: AN ILLUSTRATED SELF-ASSESSMENT GUIDE

RISK ❑ Low ❑ Intermediate ✔ High ❑ 1 2

DIAGNOSIS Nevus Seborrheic keratosis Basal cell carcinoma Vascular Dermatofibroma Squamous cell carcinoma Melanoma Other

DISPOSITION ❑ No intervention ❑ Follow-up ✔ ❑ Histopathologic diagnosis

DERMOSCOPIC CRITERIA ■











Asymmetry of color and structure (+) Multicomponent global pattern (1,2,3) Irregular dots and globules (circles) Bluish-white color (white arrows) Different shades of brown color Hypopigmentation (black arrows)

3

3-49c

❑ ❑ ❑ ❑ ❑ ❑ ✔ ❑ ❑

ANSWERS Answers: 1,2,3,5

Discussion: ■

■ ■

■ ■







The multicomponent global pattern could also be considered as atypical “Spitzoid.” ■ Bluish-white color ■ Irregular globules → globular “Spitzoid” pattern Both global patterns are high risk and a significant red flag for concern. This atypical “Spitzoid” lesion actually turned out to be a “Spitzoid” melanoma. ■ All atypical “Spitzoid” dermoscopic patterns are not histopathologically “Spitzoid” melanomas. Blue or bluish-white color is a clue that the lesion might be “Spitzoid.” There is a good clinico–dermoscopic correlation. ■ Both clinical and dermoscopic pictures are potentially high risk. There is a good dermoscopic-pathologic correlation. ■ The dermoscopy looks bad enough to be a melanoma. The hypopigmentation is light brown and should not be confused with the bony-white color of regression. This lesion is more malignant looking than benign but it is not definitively malignant. ■ Case 39 looks similar to this and turned out to be a dysplastic nevus. ■ Always consider a dermoscopic differential diagnosis.

PEARLS ■





Never tell a patient that they have melanoma 100% because you might be wrong. Your reputation in the community will suffer if that happens. Always be as positive with the patient as possible even if there is a chance that they have a melanoma. Put yourself in their shoes and be empathetic.

Chapter 3

Trunk and Extremities

RISK ❑ Low ❑ Intermediate ❑ High

DIAGNOSIS

3-50a

❑ ❑ ❑ ❑ ❑ ❑ ❑ ❑

Nevus Seborrheic keratosis Basal cell carcinoma Vascular Dermatofibroma Squamous cell carcinoma Melanoma Other

DISPOSITION

3-50b

❑ No intervention ❑ Follow-up ❑ Histopathologic diagnosis

CASE 50 HISTORY A 70-year-old Haitian man with Fitzpatrick skin type VI came in to treat some warts. After a total body skin examination, this pigmented skin lesion was found hidden on his right buttock under the waistband of his underwear. 1. Scaliness, pigmented pseudofollicular openings, and pseudostreaks are typical of pigmented seborrheic keratosis in darker-skinned races. 2. Globules identify a melanocytic lesion. 3. The global pattern could be multicomponent or “Spitzoid.” 4. Asymmetry of color and structure, irregular pigment network, irregular dots and globules, irregular streaks, bluish-white color, and regression put melanoma in the differential diagnosis. 5. An incisional biopsy was consistent with tumoral melanosis (TM) with no suggestion of melanoma. This is a good dermoscopic–pathologic correlation.

197

198

DERMOSCOPY: AN ILLUSTRATED SELF-ASSESSMENT GUIDE

RISK

1

❑ Low ❑ Intermediate ✔ High ❑

DIAGNOSIS Nevus Seborrheic keratosis Basal cell carcinoma Vascular Dermatofibroma Squamous cell carcinoma Melanoma Other

4

3

3-50c

❑ ❑ ❑ ❑ ❑ ❑ ✔ ❑ ❑

2

ANSWERS Answers: 2,3,4

DISPOSITION ❑ No intervention ❑ Follow-up ✔ Histopathologic diagnosis ❑

DERMOSCOPIC CRITERIA ■







■ ■ ■ ■

Asymmetry of color and structure Multicomponent global pattern (1,2,3,4) Irregular pigment network (red boxes) Irregular dots and globules (white boxes) Irregular streaks (black arrows) Bluish-white color (red stars) Regression (black stars) Regular pigment network on normal skin (circle)

Discussion: ■







■ ■



This in situ melanoma was hidden under the waistband of this dark-skinned patient’s underwear. ■ Melanoma is most typically found at acral sites in darker-skinned individuals. Initial clinical and dermoscopic impression was seborrheic keratosis. ■ A significant number of melanomas have seborrheic keratosis as the primary diagnosis or is in the differential diagnosis on pathology request forms. ■ Irregular dots and globules thought to be pigmented pseudofollicular openings. ■ Streaks considered pseudostreaks of seborrheic keratosis. An Incisional biopsy consistent with tumoral melanosis with no sign of melanoma. ■ This was not a good dermoscopic–pathologic correlation. Tumoral melanosis (TM) is a rare histopathological finding: ■ A nodular or plaque-like accumulation of melanin-laden macrophages/melanophages in the dermis. ■ Immunohistochemical stains are important to confirm no melanocytes. ■ TM pigmented cells are negative for S-100 and HBM-45. ■ TM pigmented cells are positive for macrophage markers such as CD68. ■ Most biopsies are to rule out melanoma. ■ There can be a history of melanoma and TM is found in a satellite nodule or lymph node mass → metastatic melanoma. ■ TM may be the residual finding after partial or complete melanoma regression. ■ TM with complete regression should be considered a high risk melanoma. ■ A metastatic work-up is indicated. ■ Sentinel node biopsy and metastatic work-up negative in this case. Dramatic dermoscopy with well-developed melanoma-specific criteria. An atypical starburst “Spitzoid” global pattern is in the differential diagnosis since there is bluish-white color and irregular streaks. This was not a “Spitzoid” melanoma histopathologically.

PEARLS ■

■ ■

People of all skin types deserve total body skin examinations. ■ The incidence of melanoma is increasing in Latinos and African-Americans. ■ White Hispanics and African-Americans tend to have deeper melanomas and/ or metastatic melanoma at the time of diagnosis compared to Caucasians. Dermoscopy is beneficial in all skin types. Dermoscopic findings have been consistent in all skin types.

Chapter 3

Trunk and Extremities

RISK ❑ Low ❑ Intermediate ❑ High

DIAGNOSIS

3-51a

❑ ❑ ❑ ❑ ❑ ❑ ❑ ❑

Nevus Seborrheic keratosis Basal cell carcinoma Vascular Dermatofibroma Squamous cell carcinoma Melanoma Other

DISPOSITION

3-51b

❑ No intervention ❑ Follow-up ❑ Histopathologic diagnosis

CASE 51 HISTORY Eight years ago, a 48-year-old Latin female had a pigmented lesion on her left anterior thigh excised. Unfortunately, the lab report with the diagnosis was lost. Three years ago the lesion recurred in the same spot and has been getting progressively darker. The patient did not remember the name of the doctor who did the excision or the lab where the specimen was sent. 1. The lesion was excised with a histopathologic diagnosis of tumoral melanosis (TM). The margins were free and there was no evidence of melanoma. No further surgery or work-up is indicated. 2. Clinically and dermoscopically this should be considered a regressed melanoma even without histopathologic proof. 3. Widespread regression is diagnostic of melanoma. 4. The irregular black blotch with foci of bluish-white color is diagnostic of melanoma. 5. Reticular depigmentation is seen within and outside of the lesion and has no diagnostic significance.

199

200

DERMOSCOPY: AN ILLUSTRATED SELF-ASSESSMENT GUIDE

RISK ❑ Low ❑ Intermediate ✔ High ❑

1

1

DIAGNOSIS Nevus Seborrheic keratosis Basal cell carcinoma Vascular Dermatofibroma Squamous cell carcinoma Melanoma Other

2

3 3 3-51c

❑ ❑ ❑ ❑ ❑ ❑ ✔ ❑ ❑

DISPOSITION

ANSWERS

❑ No intervention ❑ Follow-up ✔ Histopathologic diagnosis ❑

Answers: 2,5

Discussion: ■



DERMOSCOPIC CRITERIA ■





■ ■

■ ■ ■

Asymmetry of color and structure (+) Multicomponent global pattern (1, 2, 3) Irregular dots and globules (white boxes) Irregular black blotch (arrows) Bluish-white color (over the irregular black blotch) Regression (stars) Peppering (circles) Reticular depigmentation (red boxes)







Two cases of tumoral melanosis (TM), a rare diagnosis seen within a 2-week period is remarkable. ■ Perhaps TM is under diagnosed. With clinical and dermoscopic diagnosis of melanoma and histopathologic diagnosis of TM with regression, contact dermatopathology posthaste! ■ There was not a good dermoscopic-pathologic correlation. ■ The block was sent to a pigmented lesion expert for another histopathologic opinion which was the same diagnosis → TM, regression without evidence of melanoma consider melanoma to a depth of 1.8mm. ■ The original pathology report could not be obtained. ■ Sentinel node biopsy and a metastatic work-up were negative. There is a differential diagnosis for the irregular black blotch. ■ Atypical melanocytes → melanoma. ■ TM → melanophages and free melanin in the dermis-> melanoma? The black globules have a differential diagnosis. ■ Nests of melanocytes → melanoma. ■ Nests of melanophages → TM. The reticular depigmentation seen within and outside of the lesion represents normal skin markings and has no diagnostic significance. ■ High risk criteria (eg, arborizing vessels and reticular depigmentation) are significant only when seen within the lesion.

PEARLS ■



Consider excision at the time of the initial visit with any patient that might have a melanoma to avoid loss to follow-up. ■ Patients can be unreliable and wait long periods of time before having suspicious lesions, that turn out to be melanoma, removed! TM is not an easy diagnosis to make and a consensus of educated, experienced clinicians may be needed to decide the proper disposition.

Chapter 3

Trunk and Extremities

RISK ❑ Low ❑ Intermediate ❑ High

DIAGNOSIS

3-52a

❑ ❑ ❑ ❑ ❑ ❑ ❑ ❑

Nevus Seborrheic keratosis Basal cell carcinoma Vascular Dermatofibroma Squamous cell carcinoma Melanoma Other

DISPOSITION

3-52b

❑ No intervention ❑ Follow-up ❑ Histopathologic diagnosis

CASE 52 HISTORY This lesion was found on the calf of a 72-year-old man who plays tennis three times a week in Florida. 1. Globules identify a melanocytic lesion. 2. There is asymmetry of color and structure and a multicomponent global pattern. 3. There are regular dots, globules, streaks, blotches, and bluish- white color diagnosing a dysplastic nevus. 4. This could be a collision tumor. 5. The melanoma-specific criteria are more than enough to diagnose a melanoma.

201

202

DERMOSCOPY: AN ILLUSTRATED SELF-ASSESSMENT GUIDE

RISK ❑ Low ❑ Intermediate ✔ High ❑

1

2

1

DIAGNOSIS Nevus Seborrheic keratosis Basal cell carcinoma Vascular Dermatofibroma Squamous cell carcinoma Melanoma Other

3

3

3-52c

❑ ❑ ❑ ❑ ❑ ❑ ✔ ❑ ❑

DISPOSITION ❑ No intervention ❑ Follow-up ✔ Histopathologic diagnosis ❑

ANSWERS Answers: 1,2,4,5

DERMOSCOPIC CRITERIA ■





■ ■



■ ■

Asymmetry of color and structure Multicomponent global pattern (1,2,3) Irregular dots and globules (circles) Irregular streaks (arrows) Irregular black blotch (white stars) Bluish-white color (over the irregular black blotch) Regression (black stars) Peppering (red box)

Discussion: ■ ■



The clinical suspicion of a melanoma is confirmed with dermoscopy. There is a pigmented and amelanotic component to this melanoma. ■ A collision tumor is in the differential diagnosis. The melanoma-specific criteria are well developed: ■ Asymmetry of color and structure → no mirror images of criteria. ■ Multicomponent global pattern → each section has several criteria. ■ Irregular dots and globules → different sizes and shapes, asymmetrical location. ■ Irregular streaks → only in one spot. ■ Six colors. ■ Irregular black blotch with overlying bluish-white color → no “veil.” ■ Regression with peppering. ■ There is a differential diagnosis of the peppering. ■ “Pinpoint” vessels if they look red to you. ■ “Peppering” if they look gray. ■ Milky-red color in the lower half.

PEARLS ■ ■

This is a straightforward case. One can practice their memorized checklist of points that should be covered in every case for completeness.

Chapter 3

Trunk and Extremities

RISK ❑ Low ❑ Intermediate ❑ High

DIAGNOSIS

3-53a

❑ ❑ ❑ ❑ ❑ ❑ ❑ ❑

Nevus Seborrheic keratosis Basal cell carcinoma Vascular Dermatofibroma Squamous cell carcinoma Melanoma Other

DISPOSITION

3-53b

❑ No intervention ❑ Follow-up ❑ Histopathologic diagnosis

CASE 53 HISTORY This 40-year-old woman had a nevus adjacent to the areola on her left breast for years and was not aware of any changes, but admits that she does not usually check her skin. 1. 2. 3. 4.

This is a melanocytic lesion by default. The presence of a “black lamella” rules out a melanoma. The black blotch is very irregular and it is the most obvious high risk criterion present in the lesion. Remnants of a pigment network and homogenous light brown color are clues that this melanoma arose in a pre-existing nevus. 5. The clinical lesion and dermoscopic criteria suggest that this is an in situ or thin invasive melanoma.

203

204

DERMOSCOPY: AN ILLUSTRATED SELF-ASSESSMENT GUIDE

1

RISK ❑ Low ❑ Intermediate ✔ High ❑

2

2

DIAGNOSIS Nevus Seborrheic keratosis Basal cell carcinoma Vascular Dermatofibroma Squamous cell carcinoma Melanoma Other

3

3 3

3-53c

❑ ❑ ❑ ❑ ❑ ❑ ✔ ❑ ❑

2

3

ANSWERS Answers: 3,4,5

DISPOSITION ❑ No intervention ❑ Follow-up ✔ Histopathologic diagnosis ❑

Discussion: ■



DERMOSCOPIC CRITERIA ■







■ ■





Asymmetry of color and structure Multicomponent global pattern (1,2,3) Irregular pigment network (black boxes) Irregular dots and globules (white boxes) Irregular streaks (black arrows) Irregular black blotch (white stars) Bluish-white color (yellow arrows) Regression (black stars)









Pigment network and globules are the criteria that diagnose this as a melanocytic lesion. → It is not melanocytic by default. ■ Consider a lesion to be melanocytic “by default” if there are no criteria to diagnose a melanocytic lesion, seborrheic keratosis, basal cell carcinoma, dermatofibroma, or vascular lesion. The irregular black blotch is not a “black lamella.” ■ A “black lamella” is a homogeneous black blotch with a shiny appearance. ■ The irregular black blotch is inhomogeneous → broken up. Some of the criteria are hard to see: ■ Pigment network is questionably present. ■ Focus of irregular streaks is questionably present. ■ Bluish-white color is very subtle and could be missed if one is in a hurry. ■ Regression is questionably present and could represent normal background skin color. ■ Dots and globules are easier to find scattered throughout the lesion. The irregular black blotch and asymmetry of color and structure are easy to see and these high risk criteria suggest this is a melanoma. Section 2 represents the melanoma arising in a dysplastic nevus in Section 3. ■ There are no criteria to suggest the nevus is dysplastic yet it turned out to be dysplastic histopathologically. ■ It was a feature-poor dysplastic nevus without high risk criteria. One can tell that this is not a deep melanoma by following indications: ■ Flat lesion clinically, not raised or nodular. ■ Black and brown color indicating the pathology/pigment is in the epidermis and/or the upper dermis. ■ An absence of criteria and colors of a deeper melanoma (eg, polymorphous vessels, prominent blue color). ■ The foci of bluish-white color are of no diagnostic significance in this lesion.

PEARLS ■



This case is easy to diagnose even for the novice provided they actively study and practice the technique. Dermoscopy cannot be learned by osmosis.

Chapter 3

Trunk and Extremities

RISK ❑ Low ❑ Intermediate ❑ High

DIAGNOSIS

3-54a

❑ ❑ ❑ ❑ ❑ ❑ ❑ ❑

Nevus Seborrheic keratosis Basal cell carcinoma Vascular Dermatofibroma Squamous cell carcinoma Melanoma Other

DISPOSITION

3-54b

❑ No intervention ❑ Follow-up ❑ Histopathologic diagnosis

CASE 54 HISTORY A 35-year-old woman reported that this small lesion on her abdomen got darker over a 3-month period. 1. While not obvious, the suggestion that there are globules identifies this as a melanocytic lesion. 2. Symmetry of color and structure and regular blotches diagnose a banal acquired nevus. 3. The blotches are not as dramatic as the ones in Case 53, yet they are still irregular and a red flag for concern. 4. The clinical and dermoscopic differential diagnosis includes a dysplastic nevus or in situ melanoma. 5. Subtle bluish-white color and erythema diagnose an in situ melanoma.

205

206

DERMOSCOPY: AN ILLUSTRATED SELF-ASSESSMENT GUIDE

RISK ❑ Low ❑ Intermediate ✔ High ❑

✔ ❑ ❑ ❑ ❑ ❑ ❑ ❑ ❑

Nevus Seborrheic keratosis Basal cell carcinoma Vascular Dermatofibroma Squamous cell carcinoma Melanoma Other

3-54c

DIAGNOSIS

ANSWERS Answers: 1,3,4

DISPOSITION ❑ No intervention ❑ Follow-up ✔ Histopathologic diagnosis ❑

Discussion: ■





DERMOSCOPIC CRITERIA ■

■ ■







Asymmetry of color and structure Homogeneous global pattern Irregular dots and globules (boxes) Irregular black blotches (black arrows) Bluish-white color (overlying the irregular dark blotches) Hypopigmentation (stars)









This is a gray zone lesion in which the picture is not so clear. ■ The differential diagnosis includes a dysplastic nevus or in situ melanoma. Asymmetry of color and structure and irregular black blotches are the melanoma-specific criteria that are easy to see. ■ Melanoma-specific criteria are more sensitive and specific for melanoma, but could be found in benign lesions (eg, banal or dysplastic nevi). The global pattern is homogeneous: ■ Most of the lesion is filled with homogeneous color. ■ There are a few areas with local criteria (eg, dots and globules). ■ Could be a multicomponent global pattern with three distinct areas of criteria. In general, whatever global pattern one finds, it is still essential to determine if it is regular or irregular, good or bad, low or high risk. The irregular globules seen in one of the blotches might be created by inhomogeneity of the blotch and not true globules. The hypopigmented area has an erythematous component: ■ Pressure with instrumentation blanched away the erythema leaving residual hypopigmentation. ■ Erythema is commonly found in dysplastic nevi. The high risk criteria that raise a red flag for concern include: ■ Even though it is small, a clinical lesion with the ABCD features. ■ E for evolving/changing is very worrisome and should be added → ABCDE. ■ Asymmetry of color and structure. ■ Irregular black blotches. ■ Diffuse erythema ■ Bluish-white color.

PEARLS ■ ■ ■ ■

Well developed or not, melanoma-specific criteria should not be ignored. Subtle foci of melanoma-specific criteria may be only clue of high risk. Before making a benign diagnosis look for poorly developed high risk criteria. Small melanomas (less than 6 mm) can be diagnosed with attention to history, clinical appearance, and subtle melanoma-specific criteria.

Chapter 3

Trunk and Extremities

RISK ❑ Low ❑ Intermediate ❑ High

DIAGNOSIS

3-55a

❑ ❑ ❑ ❑ ❑ ❑ ❑ ❑

Nevus Seborrheic keratosis Basal cell carcinoma Vascular Dermatofibroma Squamous cell carcinoma Melanoma Other

DISPOSITION

3-55b

❑ No intervention ❑ Follow-up ❑ Histopathologic diagnosis

CASE 55 HISTORY The husband of this 57-year-old woman found this lesion on her back. 1. A central white patch and peripheral network diagnose a dermatofibroma. 2. A central bluish-white blotch and regular streaks diagnose a “Spitz” nevus. 3. A large crypt, milia-like cysts, and pigmented pseudofollicular openings diagnose a pigmented seborrheic keratosis. 4. There are no well-developed melanoma-specific criteria in this irritated congenital nevus. 5. Multifocal hypopigmentation, regular peripheral globules, and a regular blotch diagnose an actively changing nevus.

207

208

DERMOSCOPY: AN ILLUSTRATED SELF-ASSESSMENT GUIDE

RISK ❑ Low ❑ Intermediate ✔ High ❑

1

2 3

❑ ❑ ❑ ❑ ❑ ❑ ✔ ❑ ❑

Nevus Seborrheic keratosis Basal cell carcinoma Vascular Dermatofibroma Squamous cell carcinoma Melanoma Other

3-55c

DIAGNOSIS

ANSWERS Answers: None are correct

Discussion:

DISPOSITION ❑ No intervention ❑ Follow-up ✔ Histopathologic diagnosis ❑







■ ■

DERMOSCOPIC CRITERIA ■





■ ■



■ ■ ■

Asymmetry of color and structure Multicomponent global pattern (1,2,3) Irregular dots and globules (circles) Irregular streaks (black arrows) Irregular dark blotch (yellow arrows) Bluish-white color (overlying the irregular dark blotch) Regression (stars) Milia-like cysts (yellow boxes) Five colors





■ ■



Critics of dermoscopy might say that one does not need the technique to realize this could be a melanoma and they are correct in this case. There is no pigment network, and one might argue that the bluish-white color is a variation of the morphology of the white color seen in dermatofibromas. There are no crypts → large crater-like invaginations filled with yellowish keratin found in seborrheic keratosis and dermal nevi. There is bluish-white color superimposed on areas of the irregular dark blotch. The streaks are questionably present and irregular ■ Asymmetrically located at points along the periphery. With bluish-white color and irregular streaks this could be considered to be an irregular starburst “Spitzoid” global pattern. Differential of larger brown peripheral globules may be irregular streaks. ■ Inter-observer agreement among expert dermoscopists is not always ideal. ■ Some might consider the globules to be streaks. There is widespread regression The milia-like cysts have a differential diagnosis: ■ They are created by inhomogeneity of the whitish-colored regression. ■ True milia-like cysts → milia-like cysts can be found in melanomas. The main dermoscopic features that diagnose this melanoma include: ■ Asymmetry of color and structure ■ Multicomponent or irregular starburst “Spitzoid” global pattern ■ Irregular dark blotch ■ Irregular streaks ■ Bluish-white color → there is no “veil” ■ Multiple colors

PEARLS ■ ■



Dermoscopy is not needed to diagnose all high risk lesions. Meta-analysis has shown that dermoscopy improves the diagnosis of melanomas over clinical examination alone by as much as 16%. Though not required in this case, not using dermoscopy risks: ■ Missing melanomas ■ Unnecessary surgery (malignant to benign ratio for dermoscopists is much better than for nondermoscopists).

Chapter 3

Trunk and Extremities

RISK ❑ Low ❑ Intermediate ❑ High

DIAGNOSIS

3-56a

❑ ❑ ❑ ❑ ❑ ❑ ❑ ❑

Nevus Seborrheic keratosis Basal cell carcinoma Vascular Dermatofibroma Squamous cell carcinoma Melanoma Other

DISPOSITION

3-56b

❑ No intervention ❑ Follow-up ❑ Histopathologic diagnosis

CASE 56 HISTORY This contentious 70-year-old man came in for some scaly spots on his face. He almost refused a total body skin examination that we routinely offer new patients. It was a surprise to find this lesion on his lower back. 1. 2. 3. 4.

Clinically and dermoscopically, this is highly suspicious for a melanoma. Pigment network and globules identify a melanocytic lesion. There are foci of irregular pigment network. It does not matter if one considers the global pattern to be homogeneous or multicomponent; it is still regular and low risk. 5. Asymmetry of color and structure, an irregular dark blotch and regression are major features of this lesion.

209

210

DERMOSCOPY: AN ILLUSTRATED SELF-ASSESSMENT GUIDE

RISK ❑ Low ❑ Intermediate ✔ High ❑

1

2 2

DIAGNOSIS Nevus Seborrheic keratosis Basal cell carcinoma Vascular Dermatofibroma Squamous cell carcinoma Melanoma Other

DISPOSITION ❑ No intervention ❑ Follow-up ✔ Histopathologic diagnosis ❑

2 2

3

ANSWERS Answers: 1,2,3,5

Discussion: ■

DERMOSCOPIC CRITERIA ■







■ ■ ■



Asymmetry of color and structure (+) Multicomponent global pattern (1,2,3) Irregular pigment network (boxes) Irregular dots and globules (circles) Irregular dark blotch (arrows) Regression (stars) Bluish-white and gray colors (within the regression) Multiple colors

3-56c

❑✔ ❑ ❑ ❑ ❑ ❑ ❑ ❑









The histopathologic diagnosis was a dysplastic nevus: ■ This was not a good clinico–dermoscopic–pathologic correlation. ■ There was a very strong feeling that this was an in situ melanoma. ■ A second histopathologic opinion by a pigmented lesion expert concurred with the diagnosis of a dysplastic nevus. ■ In situ melanoma arising in a nevus was in the differential diagnosis. There are well-developed melanoma-specific criteria: ■ Asymmetry of color and structure ■ Irregular pigment network ■ Regression ■ Composed of white, gray, and blue color. Regression was not seen clinically or histopathologically. ■ Hyperkeratosis and melanosis can cause bluish-white color → dermoscopic differential diagnosis in play. The irregular dark blotch is not well developed compared to the ones in Cases 51, 52, and 53. ■ It might not be considered a true blotch but hyperpigmentation. This is a false positive dermoscopic melanoma.

PEARLS ■ ■ ■

Dermoscopy helps one decide on a surgical approach. Many colleagues routinely shave off atypically pigmented skin lesions. When the suspicion for melanoma is high, we do not recommend a shave excision in which the chances of getting free margins are not as great as with a complete excision.

Chapter 3

Trunk and Extremities

RISK ❑ Low ❑ Intermediate ❑ High

DIAGNOSIS

3-57a

❑ ❑ ❑ ❑ ❑ ❑ ❑ ❑

Nevus Seborrheic keratosis Basal cell carcinoma Vascular Dermatofibroma Squamous cell carcinoma Melanoma Other

DISPOSITION

3-57b

❑ No intervention ❑ Follow-up ❑ Histopathologic diagnosis

CASE 57 HISTORY A 19-year-old woman noticed this pigmented lesion on her upper arm a month ago. 1. 2. 3. 4. 5.

Clinically and with a nonhomogeneous blue global pattern this could be a blue nevus. Clinically and with bluish-white color this could be a nodular melanoma. A few lacunae-like structures put a vascular lesion in the differential diagnosis. A paucity of local criteria suggests this is a heavily pigmented nodular melanoma. Clinically and dermoscopically this is suspicious for a nodular melanoma, however, one does not see nodular melanoma in this young age group.

211

212

DERMOSCOPY: AN ILLUSTRATED SELF-ASSESSMENT GUIDE

RISK ❑ Low ❑ Intermediate ✔ High ❑

DIAGNOSIS Nevus Seborrheic keratosis Basal cell carcinoma Vascular Dermatofibroma Squamous cell carcinoma Melanoma Other

DISPOSITION ❑ No intervention ❑ Follow-up ✔ Histopathologic diagnosis ❑

3-57c

❑ ❑ ❑ ❑ ❑ ❑ ❑✔ ❑

ANSWERS Answers: 1,2,3,4

Discussion: ■

DERMOSCOPIC CRITERIA ■ ■ ■ ■



Homogeneous global pattern Bluish-white color (stars) Reddish color (arrows) Irregular dots and globules (yellow boxes) Lacunae-like structures (white boxes)











Nodular melanoma can be the easiest diagnosis to make or miss. ■ Always maintain a high index of suspicion with a papule or nodule no matter what the history, clinical, and/or dermoscopic appearance is. Asymmetry of the shades of bluish-white color is a red flag for concern. ■ This is an inhomogeneous homogeneous global pattern. An absence or paucity of local criteria is often found in nodular melanoma. ■ Check the periphery for foci of local criteria. The histopathologic correlate of the reddish color is not known: ■ It could be erythema. ■ It is not the milky-red color that can be seen in melanoma. The suggestion of a few lacunae-like structures and blue color could be seen a vascular lesion. Nodular melanoma ■ Can be found in any age group, young and old alike. ■ Can develop overnight. ■ Grow rapidly or be present for a long time. ■ Be amelanotic, hypomelanotic, or heavily pigmented. ■ Patients may or may not be aware of changes in a long-standing lesion.

PEARLS ■

■ ■

Include nodular melanoma in your systematic checklist of points to think about in any nodular lesion. It could be one of the great masqueraders! Foci of local criteria are a red flag for concern in equivocal nodular lesions.

Chapter 3

Trunk and Extremities

RISK ❑ Low ❑ Intermediate ❑ High

DIAGNOSIS

3-58a

❑ ❑ ❑ ❑ ❑ ❑ ❑ ❑

Nevus Seborrheic keratosis Basal cell carcinoma Vascular Dermatofibroma Squamous cell carcinoma Melanoma Other

DISPOSITION

3-58b

❑ No intervention ❑ Follow-up ❑ Histopathologic diagnosis

CASE 58 HISTORY This black nodule was clearly an “ugly duckling” lesion found on the back of an 81-year-old man whose back was covered with typical brown seborrheic keratosis. 1. A sharply demarcated black nodule with very irregular black globules characterizes this nodular melanoma. 2. Sharp border demarcation, multiple milia-like cysts, and pigmented pseudofollicular openings characterize this classic seborrheic keratosis. 3. A black nodule with a paucity of local criteria help diagnose this nodular melanoma. 4. Diffuse bluish-white color favors the diagnosis of a melanoma. 5. There are no criteria to suggest that this is a melanoma.

213

214

DERMOSCOPY: AN ILLUSTRATED SELF-ASSESSMENT GUIDE

RISK ✔ Low ❑ ❑ Intermediate ❑ High

DIAGNOSIS Nevus Seborrheic keratosis Basal cell carcinoma Vascular Dermatofibroma Squamous cell carcinoma Melanoma Other

DISPOSITION ✔ No intervention ❑ ❑ Follow-up ❑ Histopathologic diagnosis

3-58c

❑ ✔ ❑ ❑ ❑ ❑ ❑ ❑ ❑

ANSWERS Answers: 2,5

Discussion: ■



DERMOSCOPIC CRITERIA ■

■ ■



Sharp border demarcation (black arrows) Milia-like cysts (boxes) Pigmented pseudofollicular openings (white arrows) Bluish-white color (yellow arrows)

■ ■







There are no criteria to diagnose a melanocytic lesion, basal cell carcinoma, dermatofibroma, vascular lesion, squamous cell carcinoma, or anything other than a classic seborrheic keratosis. Clinically this looks very similar to the nodular melanoma in Case 57, which points out the benefits of dermoscopic examination vs clinical examination alone. ■ This was an “ugly duckling” lesion clinically → a red flag for concern. ■ Nodular melanoma is clearly in the clinical differential diagnosis. ■ Nodular melanoma is clearly not in the dermoscopic differential diagnosis. Sharp border demarcation can be seen in seborrheic keratosis and melanoma. Multiple milia-like cysts and multiple-pigmented pseudofollicular openings are not typically found in melanoma. ■ A few milia-like cysts and pseudofollicular openings can be found in benign and malignant melanocytic lesions. One does not need to find the full constellation of criteria to diagnose seborrheic keratosis. The novice dermoscopist might have a difficult time deciding if there are irregular dots and globules or pigmented pseudofollicular openings. The subtle bluish-white and black color are commonly found in seborrheic keratosis and are not melanoma-specific criteria. ● Seborrheic keratosis can be heavily pigmented.

PEARLS ■



Clinical and/or dermoscopic “ugly duckling” lesions are not always high risk. ■ Just a red flag for concern that deserves your undivided attention. Beware! There are seborrehic keratosis-like melanomas → melanoma incognito. ■ Collision tumors consisting of seborrehic keratosis and melanoma are a common combination.

Chapter 3

Trunk and Extremities

RISK ❑ Low ❑ Intermediate ❑ High

DIAGNOSIS

3-59a

❑ ❑ ❑ ❑ ❑ ❑ ❑ ❑

Nevus Seborrheic keratosis Basal cell carcinoma Vascular Dermatofibroma Squamous cell carcinoma Melanoma Other

DISPOSITION

3-59b

❑ No intervention ❑ Follow-up ❑ Histopathologic diagnosis

CASE 59 HISTORY A 60-year-old man has this violaceous nodule on his back for many years. 1. 2. 3. 4. 5.

Milky-red areas and regression characterize this amelanotic nodular melanoma. The white stellate areas represent reticular depigmentation-(negative/white network). Homogeneous purple color, lacunae, and fibrous septa characterize this hemangioma. One cannot diagnose a vascular lesion because the color is too dark. At least 50% of a lesion should be filled with well-developed lacunae before one can diagnose a vascular lesion.

215

216

DERMOSCOPY: AN ILLUSTRATED SELF-ASSESSMENT GUIDE

RISK ✔ Low ❑ ❑ Intermediate ❑ High

DIAGNOSIS Nevus Seborrheic keratosis Basal cell carcinoma Vascular Dermatofibroma Squamous cell carcinoma Melanoma Other

DISPOSITION ✔ No intervention ❑ ❑ Follow-up ❑ Histopathologic diagnosis

3-59c

❑ ❑ ❑ ✔ ❑ ❑ ❑ ❑ ❑

ANSWERS Answers: 3

Discussion: ■



DERMOSCOPIC CRITERIA ■

■ ■



Homogeneous purple colors (stars) Lacunae (boxes) Fibrous septa (arrows) ■





■ ■

Clinically but not dermoscopically melanoma is in the differential diagnosis. ■ This lesion looks clinically similar to Cases 57 and 58. The history favors of a benign lesion. ■ Patient’s histories are not always accurate but should always be listened to. This is not a melanocytic lesion by default because there are criteria for a vascular lesion: ■ Lacunae ■ Homogeneous purple color ■ Fibrous septa Lacunae are: ■ Not always well developed. ■ May be few in number or diffusely fill a lesion. ■ There is no set number required to diagnose a vascular lesion. Milky-red areas found in melanomas are pinkish-white not purplish in color, with or without out-of-focus reddish globule-like structures. Reticular depigmentation (white pigment network, negative pigment network) found in melanoma typically is thinner, whiter, and in sharper focus. Fibrous septa (white color) are typically found in vascular lesions. Any shade of red or purple color can be found in vascular lesions. ■ The deeper in the dermis that the vascular spaces are the darker the color will be.

PEARLS ■



In equivocal cases search for foci of melanoma specific-criteria before diagnosing a benign vascular lesion. If in doubt, cut it out because there are hemangioma-like melanomas and hemangioma-like cutaneous metastatic melanoma → melanoma incognito.

Chapter 3

Trunk and Extremities

RISK ❑ Low ❑ Intermediate ❑ High

DIAGNOSIS 3-60a

❑ ❑ ❑ ❑ ❑ ❑ ❑ ❑

Nevus Seborrheic keratosis Basal cell carcinoma Vascular Dermatofibroma Squamous cell carcinoma Melanoma Other

DISPOSITION

3-60b

❑ No intervention ❑ Follow-up ❑ Histopathologic diagnosis

CASE 60 HISTORY A 47-year-old physician was very upset when he discovered this new lesion on his right lower abdomen. 1. 2. 3. 4. 5.

Globules identify a melanocytic lesion. The global pattern is “Spitzoid.” There are irregular dots and globules, irregular dark blotches, and reticular depigmentation. An atypical Spitz nevus and “Spitzoid” melanoma are in the differential diagnosis. It is not clear if there are globules or streaks at the periphery and does not matter because both are high risk.

217

218

DERMOSCOPY: AN ILLUSTRATED SELF-ASSESSMENT GUIDE

RISK ❑ Low ❑ Intermediate ✔ High ❑

❑✔ ❑ ❑ ❑ ❑ ❑ ❑ ❑

Nevus Seborrheic keratosis Basal cell carcinoma Vascular Dermatofibroma Squamous cell carcinoma Melanoma Other

3-60c

DIAGNOSIS

ANSWERS Answers: 1,2,3,4,5

DISPOSITION ❑ No intervention ❑ Follow-up ✔ Histopathologic diagnosis ❑

Discussion: ■





DERMOSCOPIC CRITERIA ■

■ ■





Asymmetry of color and structure “Spitzoid “ global pattern Irregular dots and globules (circles) Irregular dark blotches (white arrows) Reticular depigmentation/ white network (yellow arrows)









Even though the lesion does not demonstrate the ABCD criteria and even before it is examined with dermoscopy one should be thinking this could be high risk. ■ A new black lesion in an adult. The global pattern is atypical starburst: ■ Asymmetry of color and structure. ■ Foci of dots, globules, and/or streaks at the periphery. ■ Centrally located reticular depigmentation (eg, negative pigment network). The globules are irregular by definition: ■ Different sizes and shapes ■ Asymmetrical location The dark blotches are irregular by definition: ■ Irregular outline ■ Asymmetrical location Morphology is a variation of classic reticular depigmentation: ■ It should be with thinner white lines and with more defined reticulation ■ At times one has to use their imagination to identify criteria that are not classic. Reticular depigmentation can be found in dermatofibromas, dysplastic nevi, Spitz nevi and melanomas and is not a differentiating criterion. There is not a good dermoscopic–pathologic correlation: ■ The dermoscopic pattern in this Spitz nevus is atypical and “Spitzoid” yet there was no mention of atypia histopathologically. ■ One should communicate with their dermatopathologist and make sure atypical histopathologic features were not missed. ■ Consider another histopathologic opinion with a “Spitzod” lesion expert → they are out there.

PEARLS ■ ■

■ ■

Some believe Spitz nevi are atypical histopathologically and not high risk. Histopathologic criteria that subcategorize Spitz nevi as atypical, potentially high risk→ melanoma. Ensure pathologist is aware of the criteria in an atypical “Spitzoid” lesion. Sending clinical and dermoscopic images to the pathologist improves communication and can motivate them to search for high risk features that could otherwise be missed.

Chapter 3

Trunk and Extremities

RISK ❑ Low ❑ Intermediate ❑ High

DIAGNOSIS

3-61a

❑ ❑ ❑ ❑ ❑ ❑ ❑ ❑

Nevus Seborrheic keratosis Basal cell carcinoma Vascular Dermatofibroma Squamous cell carcinoma Melanoma Other

DISPOSITION

3-61b

❑ No intervention ❑ Follow-up ❑ Histopathologic diagnosis

CASE 61 HISTORY This solitary lesion was found on the anterior thigh of a 53-year-old woman. 1. The clinical appearance and presence of multiple milia-like cysts diagnose a pigmented seborrheic keratosis. 2. Peripheral globules identify a melanocytic lesion. 3. Bluish-white color and an irregular black blotch filling most of the lesion are red flags for concern. 4. Milia-like cysts are usually white or yellow and not bluish-white. 5. It is hard to decide if this is a heavily pigmented seborrheic keratosis or a melanoma which is an indication to make a histopathologic diagnosis.

219

220

DERMOSCOPY: AN ILLUSTRATED SELF-ASSESSMENT GUIDE

RISK ❑ Low ❑ Intermediate ✔ High ❑

1

DIAGNOSIS

2

Nevus Seborrheic keratosis Basal cell carcinoma Vascular Dermatofibroma Squamous cell carcinoma Melanoma Other

DISPOSITION ❑ No intervention ❑ Follow-up ✔ Histopathologic diagnosis ❑

4 3

5 3-61c

❑ ❑ ❑ ❑ ❑ ❑ ✔ ❑ ❑

ANSWERS Answers: 2,3,4,5

Discussion: ■

DERMOSCOPIC CRITERIA ■







■ ■

Asymmetry of color and structure Multicomponent global pattern (1,2,3,4,5) Irregular black and brown dots and globules (circles) Irregular bluish-white dots (boxes) Irregular black blotch (arrows) Bluish-white homogeneous color (stars)













The irregular black blotch filling most of the lesion is an immediate cause for concern. ■ Proceed with focused attention to completely analyze the lesion. With a potentially difficult and/or high risk lesion go directly into your organized checklist of features to look for. Step one → melanocytic vs nonmelanocytic ■ This is melanocytic because there are brown globules. Step two → Global features ■ Asymmetry of color and structure. ■ Multicomponent global pattern with four different areas of criteria. Step three → Local criteria ■ Poorly defined irregular black and brown dots and globules. ■ Irregular bluish-white dots and globules. ■ These are not milia-like cysts and probably represent hyperkeratosis (scale). ■ Irregular black blotch is filling most of the lesion. ■ Subtle foci of homogeneous bluish-white color → not a “veil.” Step four → Diagnosis and/or differential diagnosis ■ One cannot be sure if this is a heavily pigmented seborrheic keratosis or melanoma. Step five → Lesion disposition ■ Make a histopathologic diagnosis since melanoma is in the differential diagnosis.

PEARL ■

If one gets to the point of confusion, stop and think “If in doubt, cut it out.” Problem solved!

Chapter 3

Trunk and Extremities

RISK ❑ Low ❑ Intermediate ❑ High

DIAGNOSIS

3-62a

❑ ❑ ❑ ❑ ❑ ❑ ❑ ❑

Nevus Seborrheic keratosis Basal cell carcinoma Vascular Dermatofibroma Squamous cell carcinoma Melanoma Other

DISPOSITION

3-62b

❑ No intervention ❑ Follow-up ❑ Histopathologic diagnosis

CASE 62 HISTORY A 74-year-old woman has this nodule on her sternum for two years. 1. This is a beautiful heart-shaped benign seborrheic keratosis with multiple milia-like cysts and large pigmented crypts. 2. A central white patch and hyperpigmentation characterize this dermatofibroma. 3. This is a melanocytic lesion by default. 4. Irregular black blotches and an absence of local criteria other than a few irregular vessels at the periphery help to diagnose a nodular melanoma. 5. The whitish color could be created by hyperkeratosis (scale) or regression.

221

222

DERMOSCOPY: AN ILLUSTRATED SELF-ASSESSMENT GUIDE

1

RISK ❑ Low ❑ Intermediate ✔ High ❑

2

DIAGNOSIS Nevus Seborrheic keratosis Basal cell carcinoma Vascular Dermatofibroma Squamous cell carcinoma Melanoma Other

DISPOSITION ❑ No intervention ❑ Follow-up ✔ Histopathologic diagnosis ❑

3

4

3-62c

❑ ❑ ❑ ❑ ❑ ❑ ✔ ❑ ❑

ANSWERS Answers: 3,4,5

Discussion: ■



DERMOSCOPIC CRITERIA ■



■ ■ ■ ■

Asymmetry of color and structure Multicomponent global pattern (1,2,3,4) Irregular dark blotches (arrows) Hyperkeratosis (scale) (stars) Whitish dots (white boxes) Polymorphous vessels (yellow boxes)







An adorable heart-shaped lesion seen clinically is of no significance other than the fact that it is a black nodule. There is an absence of criteria to diagnose a melanocytic lesion, seborrheic keratosis, basal cell carcinoma, dermatofibroma, or hemangioma → melanocytic by default. Scaliness creates the whitish color. ■ Dry skin significantly diminishes what one can see with dermoscopy. ■ Mineral oil was used to eliminate the dryness → a glob of mineral oil covers 90% of the lesion. ■ The whitish dots are not milia-like cysts and are created by the dry scaly skin. Once a melanocytic lesion is diagnosed, the irregular dark blotches, absence of local criteria, and polymorphous vessels put nodular melanoma at the top of the diagnostic list. One has to look hard to identify the atypical vessels and for some they would be questionably present.

PEARLS ■ ■

It is essential to eliminate dry scaly skin to get a better dermoscopic view. With a vivid imagination interesting shapes can be see dermoscopically! ■ Hearts, Yorkies, and even invasive melanoma that looks like Mickey Mouse with two big ears. ■ An interesting observation that adds an aspect of frivolity to a very serious subject.

Chapter 3

Trunk and Extremities

RISK ❑ Low ❑ Intermediate ❑ High

DIAGNOSIS

3-63a

❑ ❑ ❑ ❑ ❑ ❑ ❑ ❑

Nevus Seborrheic keratosis Basal cell carcinoma Vascular Dermatofibroma Squamous cell carcinoma Melanoma Other

DISPOSITION

3-63b

❑ No intervention ❑ Follow-up ❑ Histopathologic diagnosis

CASE 63 HISTORY This solitary lesion was found on the back of a 40-year-old woman. 1. The irregular round to oval black structures could represent globules of a melanocytic lesion or pigmented pseudofollicular openings of a seborrheic keratosis. 2. The irregular black blotch could represent invasive melanoma or hyperpigmentation in a seborrheic keratosis. 3. The diffuse bluish-white color does not help to differentiate a melanoma from a seborrheic keratosis. 4. The sharp border demarcation does not help differentiate a melanoma from a seborrheic keratosis. 5. Multiple hairpin-shaped vessels suggest that this is a seborrheic keratosis.

223

224

DERMOSCOPY: AN ILLUSTRATED SELF-ASSESSMENT GUIDE

RISK ❑ Low ✔ Intermediate ❑ ❑ High

1

2

DIAGNOSIS Nevus Seborrheic keratosis Basal cell carcinoma Vascular Dermatofibroma Squamous cell carcinoma Melanoma Other

3 4

3-63c

❑ ✔ ❑ ❑ ❑ ❑ ❑ ❑ ❑

ANSWERS DISPOSITION ❑ No intervention ❑ Follow-up ✔ Histopathologic diagnosis ❑

Answers: 1,2,3,4,5

Discussion: ■



DERMOSCOPIC CRITERIA ■













■ ■



Asymmetry of color and structure Multicomponent global pattern (1,2,3,4) Sharp border demarcation (black arrows) Pigmented pseudofollicular openings (yellow arrows) Nonpigmented pseudofollicular opening (white box) Irregular brown dots (yellow box) Irregular black blotch (white arrows) Bluish-white color (stars) Hairpin-shaped vessels (black boxes) Polymorphous vessels (red boxes)







Asymmetry of color and structure and the multicomponent global pattern are not exclusively found in high risk melanocytic lesions. This case points out the need to think in terms of dermoscopic differential diagnosis. ■ Irregular dark globules of a melanocytic lesion vs pigmented pseudofollicular openings of a seborrheic keratosis. ■ The focus of small dots has no diagnostic significance. ■ Irregular black blotch of a melanoma vs hyperpigmentation of a seborrheic keratosis. ■ Bluish-white color is nonspecific and can be seen in seborrheic keratosis and in melanoma. ■ The absence of milia-like cysts does not rule out a seborrheic keratosis. ■ Polymorphous vessels with some hairpin shapes can be seen in melanoma and seborrheic keratosis. ■ One often has to use their imagination to see hairpin shapes. The hairpin-shaped vessels and nonpigmented pseudofollicular opening favors the diagnosis of a seborrheic keratosis. Pigmented pseudofollicular openings can have a three-dimensional appearance when seen in vivo that does not show up in digital dermoscopic images. For the experienced clinician, the diagnosis of an atypical seborrheic keratosis should be easier to make than by the novice dermoscopist. ■ A stuck on clinical appearance seen here is commonly found in seborrheic keratosis.

PEARLS ■



Beware! Melanomas can have a stuck on clinical appearance similar to seborrheic keratosis. A collision tumor, melanoma and seborrheic keratosis is in the differential diagnosis.

Chapter 3

Trunk and Extremities

RISK ❑ Low ❑ Intermediate ❑ High

DIAGNOSIS

3-64a

❑ ❑ ❑ ❑ ❑ ❑ ❑ ❑

Nevus Seborrheic keratosis Basal cell carcinoma Vascular Dermatofibroma Squamous cell carcinoma Melanoma Other

DISPOSITION

3-64b

❑ No intervention ❑ Follow-up ❑ Histopathologic diagnosis

CASE 64 HISTORY You spot this innocuous looking pink macule on the upper back of a 69-year-old man. 1. 2. 3. 4.

Globules identify a melanocytic lesion. Regression, polymorphous vessels, and irregular globules diagnose a regressive melanoma. A pink scaly macule with glomerular vessels diagnoses Bowen disease. An absence of pigment network, arborizing vessels, and pigmentation diagnose a pigmented basal cell carcinoma. 5. A pink lesion seen clinically is nonspecific and the vessels seen with dermoscopy can in many instances make a specific diagnosis.

225

226

DERMOSCOPY: AN ILLUSTRATED SELF-ASSESSMENT GUIDE

RISK ❑ Low ❑ Intermediate ✔ High ❑

1

DIAGNOSIS

2

Nevus Seborrheic keratosis Basal cell carcinoma Vascular Dermatofibroma Squamous cell carcinoma Melanoma Other

3

3-64c

❑ ❑ ✔ ❑ ❑ ❑ ❑ ❑ ❑

ANSWERS DISPOSITION

Answers: 4,5

❑ No intervention ❑ Follow-up ✔ Histopathologic diagnosis ❑

Discussion: ■



DERMOSCOPIC CRITERIA ■



■ ■

■ ■

Asymmetry of color and structure Multicomponent global pattern (1,2,3) Arborizing vessels (arrows) Brown dots and globules (black circles) Blue globule (red circle) Depigmentation (stars)





■ ■



This is a nonmelanocytic lesion with asymmetry of color and structure and a multicomponent global pattern. Pink macules and papules are nonspecific and can be melanocytic, nonmelanocytic, benign, malignant, or inflammatory. ■ Shiny semi-translucent clinical appearance suggests basal cell carcinoma. ■ Bowen disease (in situ squamous cell carcinoma) typically has: ■ Pinkish scaly macules, patches or plaques ■ Not shiny or semi-translucent ■ Contains diffuse or localized glomerular vessels → pinpoint or circular grouped telectangietatic vessels One sees classic arborizing vessels, both small and large bore, branching: ■ Sharply in focus due to surface location ■ Out-of-focus vessels are not on the surface of a lesion. ■ Out-of-focus arborizing vessels are a “red flag” and could be melanoma. Arborizing vessels are not exclusive to basal cell carcinoma. They can be in: ■ Melanoma ■ Sun-damaged skin ■ Scar tissue ■ Nevi ■ Sebaceous gland hyperplasia Nonarborizing vessels can also be seen in basal cell carcinomas. Pigmentation is commonly found in basal cell carcinomas. ■ The size, shape, and color are highly variable. ■ No specific pigmentation type is diagnostic of basal cell carcinoma. White color is commonly found in basal cell carcinomas and could be confused with the bony-white color of regression seen in melanomas.

PEARLS ■



Always keep in mind that the “classic” clinical and/or dermoscopic basal cell carcinoma could be a melanoma. Be vigilant and always maintain a high-melanoma index of suspicion.

Chapter 3

Trunk and Extremities

RISK ❑ Low ❑ Intermediate ❑ High

DIAGNOSIS

3-65a

❑ ❑ ❑ ❑ ❑ ❑ ❑ ❑

Nevus Seborrheic keratosis Basal cell carcinoma Vascular Dermatofibroma Squamous cell carcinoma Melanoma Other

DISPOSITION

3-65b

❑ No intervention ❑ Follow-up ❑ Histopathologic diagnosis

CASE 65 HISTORY This suspicious lesion was found by an internist on the chest of an 80-year-old man. 1. 2. 3. 4. 5.

Milky-red areas, pinpoint and arborizing vessels characterize this amelanotic melanoma. The white area has arborizing vessels and ulceration diagnosing a basal cell carcinoma. The brown area has globules and pigment network diagnosing a nevus. This could be a collision tumor or a melanoma arising in a nevus. A basal cell carcinoma and nevus are never seen in combination in collision tumors.

227

228

DERMOSCOPY: AN ILLUSTRATED SELF-ASSESSMENT GUIDE

RISK ❑ Low ❑ Intermediate ✔ High ❑

DIAGNOSIS Nevus Seborrheic keratosis Basal cell carcinoma Vascular Dermatofibroma Squamous cell carcinoma Melanoma Other

DISPOSITION ❑ No intervention ❑ Follow-up ✔ Histopathologic diagnosis ❑

3-65c

❑ ❑ ✔ ❑ ❑ ❑ ❑ ❑ ❑

ANSWERS Answers: 2,3,4

Discussion: ■



DERMOSCOPIC CRITERIA ■ ■

■ ■



Arborizing vessels (arrows) Polymorphous vessels (red boxes) Ulceration (black box) Irregular brown globules (circle) Regular fine pigment network (yellow box)











Strict dermoscopic evaluation diagnoses a collision tumor or melanoma arising in a nevus. There are classic arborizing vessels and a small ulceration that diagnoses a basal cell carcinoma. ■ Melanoma could also have similar appearing arborizing vessels and ulceration. The diffuse bony-white color could be seen in a basal cell or melanoma and is not a differentiating criterion. The irregular brown globules and pigment network are not well developed and could be easily overlooked. Any combination of benign and/or malignant tumors is theoretically possible in collision tumors. ■ A basal cell carcinoma and nevus would not be unusual. There are foci of nonarborizing vessels (eg, polymorphous vessels). ■ Not uncommonly, small telectangietatic vessels with various shapes (eg, linear, comma-shaped, pinpoint) are also found in basal cell carcinomas along with arborizing vessels. There are no milky-red globules. ■ The globules are brown and not milky-red in color. ■ There is not a pinkish background surrounding the globules.

PEARL ■

Be aware of the concept of collision tumors so that they are in the differential diagnosis when the dermoscopic criteria present themselves.

Chapter 3

Trunk and Extremities

RISK ❑ Low ❑ Intermediate ❑ High

DIAGNOSIS

3-66a

❑ ❑ ❑ ❑ ❑ ❑ ❑ ❑

Nevus Seborrheic keratosis Basal cell carcinoma Vascular Dermatofibroma Squamous cell carcinoma Melanoma Other

DISPOSITION

3-66b

❑ No intervention ❑ Follow-up ❑ Histopathologic diagnosis

CASE 66 HISTORY This very small lesion was found on the posterior shoulder of a 45-year-old man. 1. There are classic arborizing vessels of a basal cell carcinoma. 2. The combination of arborizing vessels, white color, and pigment diagnose a pigmented basal cell carcinoma. 3. The large brown irregular blotch with a focus of bluish-white color is a clue that this might be a melanoma. 4. Pigment network seen outside of the lesion has no diagnostic significance. 5. In focus arborizing vessels are never seen in basal cell carcinomas.

229

230

DERMOSCOPY: AN ILLUSTRATED SELF-ASSESSMENT GUIDE

RISK ❑ Low ❑ Intermediate ✔ High ❑

1 2

DIAGNOSIS Benign Nevus Seborrheic keratosis Basal cell carcinoma Vascular Dermatofibroma Squamous cell carcinoma Melanoma Other

DISPOSITION ❑ No intervention ❑ Follow-up ✔ Histopathologic diagnosis ❑

2

3

3-66c

❑ ❑ ❑ ❑ ❑ ❑ ✔ ❑ ❑

ANSWERS Answers: 1,3,4

Discussion: ■

DERMOSCOPIC CRITERIA ■





■ ■ ■ ■

Asymmetry of color and structure Multicomponent global pattern (1,2,3) Irregular brown blotch (black arrows) Bluish-white color (circle) Arborizing vessels (red arrows) Regression (stars) Pigment network (boxes)









Here it is, a melanoma that at first blush looks like a typical basal cell carcinoma that has: ■ Classic arborizing vessels ■ White color ■ Pigmentation One sees fine pigment network surrounding the lesion: ■ A normal skin finding when found outside of the lesion. ■ More commonly found in darker-skinned races. There are clues that this might be a melanoma: ■ Clinically it does not look like a basal cell carcinoma. ■ The large brown blotch with a focus of bluish-white color is not typically seen in basal cell carcinomas. The white color has a differential diagnosis: ■ Regression in a melanoma. ■ White color typically seen in basal cell carcinomas. In-focus classic arborizing vessels can also be found in melanomas. ■ Statistically they are much more commonly found in basal cell carcinomas.

PEARLS ■ ■ ■

Congratulate yourself if you diagnose a barely perceptable lesion like this. You have taken dermatology and dermoscopy to a higher level. Focus one’s attention on every patient at all times because patients’ lives are on the line any time of the day.

Chapter 3

Trunk and Extremities

RISK ❑ Low ❑ Intermediate ❑ High

DIAGNOSIS

3-67a

❑ ❑ ❑ ❑ ❑ ❑ ❑ ❑

Nevus Seborrheic keratosis Basal cell carcinoma Vascular Dermatofibroma Squamous cell carcinoma Melanoma Other

DISPOSITION

3-67b

❑ No intervention ❑ Follow-up ❑ Histopathologic diagnosis

(Reproduced, with permission, from Journal of Drugs in Dermatology. New Methods and Technologies. Nov 2008-Vol 7- Issue 11. Fig 1.)

CASE 67 HISTORY This nodule on the back of a 63-year-old man was found by his wife while in the swimming pool. 1. Clinically and dermoscopically, this is a classic basal cell carcinoma. 2. Arborizing vessels, blue ovoid nests of pigment, and brown globules diagnose a basal cell carcinoma. 3. Brown globules identify a melanocytic lesion. 4. Well-developed melanoma-specific criteria suggest that this could be a melanoma. 5. There is a large area of milky-red color with a few milky-red globules.

231

232

DERMOSCOPY: AN ILLUSTRATED SELF-ASSESSMENT GUIDE

RISK

1

❑ Low ❑ Intermediate ✔ High ❑

2

2

3

DIAGNOSIS Nevus Seborrheic keratosis Basal cell carcinoma Vascular Dermatofibroma Squamous cell carcinoma Melanoma Other

3

1 1

3-67c

❑ ❑ ❑ ❑ ❑ ❑ ✔ ❑ ❑

ANSWERS Answers: 3,4,5

Discussion:

DISPOSITION ❑ No intervention ❑ Follow-up ✔ Histopathologic diagnosis ❑







DERMOSCOPIC CRITERIA ■





■ ■ ■ ■

■ ■

Asymmetry of color and structure (+) Multicomponent global pattern (1,2,3) Irregular brown dots and globules (black boxes) Bluish-white color (blue stars) Regression (black stars) Peppering (yellow boxes) Polymorphous vessels (black arrows) Milky-red area (red arrows) Six colors





A dark nodule with a multicolor macular component is not what one would see clinically with a basal cell carcinoma. Brown globules identify a melanocytic lesion. ■ They are difficult to see. The lesion is filled with well-developed melanoma-specific criteria. ■ Asymmetry of color and structure ■ Polymorphous vessels ■ Multicomponent global pattern ■ Milky-red area ■ Regression with peppering ■ Six colors ■ Bluish-white color Some of the criteria have a differential diagnosis. ■ Bluish-white color of a melanoma vs blue ovoid nests of a basal cell carcinoma. ■ The blue color is not ovoid in shape, which goes against the diagnosis of a basal cell carcinoma. ■ Arborizing vessels of a basal cell carcinoma vs polymorphous vessels of a melanoma. ■ The vessels have very little branching, which goes against the diagnosis of a basal cell carcinoma. ■ The presence of other shapes (eg, pinpoint, linear) favors the diagnosis of a melanoma. There is a large milky-red area with a few out-of-focus reddish globules. ■ This favors the diagnosis of a melanoma. ■ Not typically seen in basal cell carcinomas. ■ This could easily be overlooked by the novice dermoscopist.

PEARLS ■





At first blush, one might focus on the basal cell-like linear vessels and blue color, and diagnose a basal cell carcinoma. It is essential to examine all of the criteria in a lesion before making a dermoscopic diagnosis. When it is not possible to differentiate a basal cell carcinoma from a melanoma, consider the lesion to be a melanoma and arrange to make a histopathologic diagnosis posthaste!

Chapter 3

Trunk and Extremities

RISK ❑ Low ❑ Intermediate ❑ High

DIAGNOSIS

3-68a

❑ ❑ ❑ ❑ ❑ ❑ ❑ ❑

Nevus Seborrheic keratosis Basal cell carcinoma Vascular Dermatofibroma Squamous cell carcinoma Melanoma Other

DISPOSITION

3-68b

❑ No intervention ❑ Follow-up ❑ Histopathologic diagnosis

(Reproduced, with permission, from Journal of Drugs in Dermatology. New Methods and Technologies. Nov 2008-Vol 7- Issue 11. Fig 2.)

CASE 68 HISTORY This case was e-mailed to you for your opinion. Is it a pigmented basal cell carcinoma or melanoma? 1. Clinically it looks like a melanoma. 2. Pigment network and globules identify a melanocytic lesion. 3. Asymmetry of color and structure and a multicomponent global pattern could be seen in both pathologies. 4. Lacks the arborizing vessels suggestive of a basal cell carcinoma. 5. The large irregular black blotch, white color with polymorphous vessels favors the diagnosis of a melanoma.

233

234

DERMOSCOPY: AN ILLUSTRATED SELF-ASSESSMENT GUIDE

RISK ❑ Low ❑ Intermediate ✔ High ❑

3

1

DIAGNOSIS ❑ ❑ ❑ ❑ ❑ ❑ ✔ ❑ ❑

2

Nevus Seborrheic keratosis Basal cell carcinoma Vascular Dermatofibroma Squamous cell carcinoma Melanoma Other

3 2

3 3-68c

3

DISPOSITION ❑ No intervention ❑ Follow-up ✔ Histopathologic diagnosis ❑

ANSWERS Answers: 1,2,3,4,5

Discussion: ■

DERMOSCOPIC CRITERIA ■









■ ■

■ ■

Asymmetry of color and structure Multicomponent global pattern (1,2,3) Irregular pigment network (black boxes) Irregular black globules (yellow boxes) Irregular black blotches (yellow arrows) Regression (stars) Polymorphous vessels ■ Pinpoint (circles) ■ Linear (black arrows) Six colors Hyperkeratosis (white box)









The lesion looks like a melanoma clinically: ■ Central dark blotch ■ ABCD criteria ■ Absence of the semi-translucency typically seen in basal cell carcinoma There are well-developed melanoma-specific criteria: ■ Asymmetry of color and structure ■ Multicomponent global pattern ■ Both could be seen in basal cell carcinoma or melanoma. ■ Irregular black globules ■ Polymorphous vessels ■ Irregular black blotches ■ Six colors ■ Regression The diffuse white color has a differential diagnosis: ■ Bony-white color of regression seen in melanoma ■ White color of a basal cell carcinoma Polymorphous vessels seen in the bony-white color favors the diagnosis of a melanoma. The absence of arborizing vessels goes against the diagnosis of a basal cell carcinoma.

PEARLS ■





Even experienced dermoscopists might confuse this with a pigmented basal cell carcinoma if one does not focus their attention and is in a hurry. Minimal pressure with instrumentation and use of a gel (ultrasound or hand sanitizer) applied to the skin will allow one to better visualize small telangiectatic vessels. Small vessels are better visualized with polarizing noncontact dermoscopy (dermlite 3GEN) when not using gel.

Chapter 3

Trunk and Extremities

RISK ❑ Low ❑ Intermediate ❑ High

DIAGNOSIS

3-69a

❑ ❑ ❑ ❑ ❑ ❑ ❑ ❑

Nevus Seborrheic keratosis Basal cell carcinoma Vascular Dermatofibroma Squamous cell carcinoma Melanoma Other

DISPOSITION

3-69b

❑ No intervention ❑ Follow-up ❑ Histopathologic diagnosis

CASE 69 HISTORY A 76-year-old man was seen by his dermatologist for a skin infection when this lesion was found on his posterior left upper arm. 1. Globules identify a melanocytic lesion. 2. Milky-red color and reticular depigmentation are clues that this is a melanoma. 3. A central white patch, peripheral pigment network, and arborizing vessels characterize this dermatofibroma. 4. Regression is seen clinically and dermoscopically. 5. The large area of regression is characterized by milky-red color, homogeneous gray color, and polymorphous vessels.

235

236

DERMOSCOPY: AN ILLUSTRATED SELF-ASSESSMENT GUIDE

RISK ❑ Low ❑ Intermediate ✔ High ❑

1

DIAGNOSIS

2

Nevus Seborrheic keratosis Basal cell carcinoma Vascular Dermatofibroma Squamous cell carcinoma Melanoma Other

DISPOSITION ❑ No intervention ❑ Follow-up ✔ Histopathologic diagnosis ❑

3 3-69c

❑ ❑ ❑ ❑ ❑ ❑ ✔ ❑ ❑

ANSWERS Answers: 1,2,4,5

Discussion: ■



DERMOSCOPIC CRITERIA ■





■ ■

■ ■





Asymmetry of color and structure Multicomponent global pattern (1,2,3) Irregular brown dots and globules (black circles) Irregular streaks (black arrows) Reticular depigmentation (white boxes) Regression (stars) Polymorphous vessels (yellow boxes) Homogenous gray color and peppering (yellow circles) Arborizing vessels (red boxes)













The brown globules are hard to see, nevertheless identify a melanocytic lesion. ■ This could also be considered a melanocytic lesion by default if one does not agree that there are brown globules. Reticular depigmentation (negative pigment network/white pigment network/white network) seems to blend in with stellate bony-white scar tissue in some areas. Milky-red/pink color fills most of the lesion. ■ Pink color in any form is always a red flag for concern. ■ However, pink color is not always associated with high risk pathology. Asymmetry of color and structure and the multicomponent global pattern are melanoma-specific criteria that can be seen in basal cell carcinoma and melanoma. One focus of irregular streaks is a melanoma-specific criterion but of little importance in the context of the other well-developed high risk criteria. The area of regression is made up of: ■ Bony-white color ■ Milky-red/pink color ■ Polymorphous vessels ■ Pinpoint ■ Linear ■ Homogenous gray color and peppering An atypical dermatofibroma is in the dermoscopic but not the clinical differential diagnosis. The arborizing vessels surrounding the lesion are a sign of chronic sun damage and have nothing to do with the lesion itself.

PEARLS ■ ■

It cannot be overemphasized: If there’s pink, stop and think! While not specific, pink color is an independent potentially high risk criterion.

Chapter 3

Trunk and Extremities

RISK ❑ Low ❑ Intermediate ❑ High

DIAGNOSIS

3-70a

❑ ❑ ❑ ❑ ❑ ❑ ❑ ❑

Nevus Seborrheic keratosis Basal cell carcinoma Vascular Dermatofibroma Squamous cell carcinoma Melanoma Other

DISPOSITION

3-70b

❑ No intervention ❑ Follow-up ❑ Histopathologic diagnosis

CASE 70 HISTORY A 64-year-old man was seen for his melanoma follow-up when this lesion was found on his back. 1. 2. 3. 4.

A central white patch and pigment network put a dermatofibroma in the differential diagnosis. Pigment network identifies a melanocytic lesion. An amelanotic nodule with a few pinpoint vessels puts melanoma in the differential diagnosis. Slightly irregular pigment network, a hypomelanotic central area, and diffuse erythema put an irritated nevus in the differential diagnosis. 5. Irregular pigment network and multifocal hypopigmentation put a dysplastic nevus in the differential diagnosis.

237

238

DERMOSCOPY: AN ILLUSTRATED SELF-ASSESSMENT GUIDE

RISK ❑ Low ✔ Intermediate ❑ ❑ High

✔ ❑ ❑ ❑ ❑ ❑ ❑ ❑ ❑

Nevus Seborrheic keratosis Basal cell carcinoma Vascular Dermatofibroma Squamous cell carcinoma Melanoma Other

3-70c

DIAGNOSIS

ANSWERS Answers: 1,2,3,4,5

Discussion: ■

DISPOSITION ❑ No intervention ❑ Follow-up ✔ Histopathologic diagnosis ❑ ■

DERMOSCOPIC CRITERIA ■





■ ■

■ ■

Reticular-homogeneous global pattern Asymmetry of color and structure Irregular pigment network (boxes) Irregular dots (black circles) Hypopigmentation (black stars) Regression (white stars) Pinpoint vessels (red circles)









Pigment network identifies a melanocytic lesion. ■ It could be considered regular or slightly irregular. ■ The brown dots are too small to be considered globules, which would be another criteria identifying a melanocytic lesion. ■ Dermatofibromas are nonmelanocytic lesions that can have a pigment network. The global pattern is reticular-homogeneous. ■ Reticular created by pigment network. ■ Homogeneous created by an amelanotic centrally located dermal nevus. Even though this lesion was found in an adult, this pattern is more commonly found on the scalp in children and is also referred to as: ■ Targetoid nevus ■ Reverse fried egg nevus ■ Fried egg nevus has a central dark area. ■ “ Crown on thorns” nevus (our preferred term). The differential diagnosis for the hypopigmented areas includes: ■ Regression ■ Hypopigmentation seen in a dermal nevus ■ Not uncommonly, dermal nevi have very little if any pigment. ■ Multifocal hypopigmentation commonly found in dysplastic nevi ■ A nodule of amelanotic melanoma The pinpoint vessels: ■ Are nonspecific and could be seen in regression or in a dermal nevus. ■ Might easily be missed since they are not well developed. Diffuse erythema is a nonspecific sign of inflammation that easily blanches away with pressure from instrumentation.

PEARLS ■



■ ■

The value of being able to create a dermoscopic differential diagnosis for global patterns and local criteria is highlighted in this case. “Crown of thorns” nevi are commonly found in the scalp of children and are usually benign. A “Crown of thorns” nevus in an adult is a red flag for concern. Whatever is seen with dermoscopy, the main goal is to determine if it is regular or irregular, low or high risk, good or bad.

Chapter 3

Trunk and Extremities

RISK ❑ Low ❑ Intermediate ❑ High

DIAGNOSIS 3-71a

❑ ❑ ❑ ❑ ❑ ❑ ❑ ❑

Nevus Seborrheic keratosis Basal cell carcinoma Vascular Dermatofibroma Squamous cell carcinoma Melanoma Other

DISPOSITION

3-71b

❑ No intervention ❑ Follow-up ❑ Histopathologic diagnosis

CASE 71 HISTORY A pink spot was found on the back of an 85-year-old man 1. 2. 3. 4. 5.

There are no criteria to diagnose a melanocytic lesion. There are no criteria to diagnose a seborrheic keratosis. There are no criteria to diagnose a dermatofibroma. There are no criteria to diagnose a vascular lesion. An absence of pigment network within the lesion and the presence of arborizing vessels, pigmentation, and ulceration diagnose a basal cell carcinoma.

239

240

DERMOSCOPY: AN ILLUSTRATED SELF-ASSESSMENT GUIDE

RISK ❑ Low ❑ Intermediate ✔ High ❑

DIAGNOSIS Nevus Seborrheic keratosis Basal cell carcinoma Vascular Dermatofibroma Squamous cell carcinoma Melanoma Other

DISPOSITION ❑ No intervention ❑ Follow-up ✔ Histopathologic diagnosis ❑

3-71c

❑ ❑ ✔ ❑ ❑ ❑ ❑ ❑ ❑

ANSWERS Answers: 1,2,3,4,5

Discussion: ■

DERMOSCOPIC CRITERIA ■

■ ■ ■

Arborizing vessels (yellow arrows) Pigmentation (white arrows) Ulceration (black arrows) Pigment network (boxes)

This is a classic basal cell carcinoma. ■ Absence of pigment network within the lesion: ■ There is pigment network at the periphery and outside of the lesion on the surrounding skin. ■ It is not really part of the lesion but could be considered a criterion to diagnose a melanocytic lesion → dermoscopic differential diagnosis. ■ Arborizing vessels: ■ They are all small bore. ■ Some branch while most do not. ■ Basal cell-like vessels do not always branch, which is a variation of the morphology that is commonly encountered. ■ Pigmentation: ■ Brown ■ Gray ■ Blue → no leaf-like structures are seen ■ Ulceration with a differential diagnosis: ■ Brown globules of a melanocytic lesion ■ Pigmented pseudofollicular openings of a seborrheic keratosis

PEARLS ■



There are innumerable variations of global patterns and local criteria that are routinely encountered in daily practice. Being aware of the classic presentations of global patterns and local criteria plus practice, practice and more practice allows one to identify variations of morphology that routinely occur.

Chapter 3

Trunk and Extremities

RISK ❑ Low ❑ Intermediate ❑ High

DIAGNOSIS

3-72a

❑ ❑ ❑ ❑ ❑ ❑ ❑ ❑

Nevus Seborrheic keratosis Basal cell carcinoma Vascular Dermatofibroma Squamous cell carcinoma Melanoma Other

DISPOSITION

3-72b

❑ No intervention ❑ Follow-up ❑ Histopathologic diagnosis

CASE 72 HISTORY This lesion was found on the thigh of a 30-year-old woman. 1. This could not be a melanocytic lesion because there is no pigment network. 2. Milia-like cysts, pigmented pseudofollicular openings, and hairpin vessels diagnose a seborrheic keratosis. 3. Brown globules identify a melanocytic lesion. 4. Pink, gray, white colors and pinpoint vessels characterize a large area of regression. 5. Asymmetry of color and structure, a multicomponent global pattern, regular globules, regular blotches, and regression characterize this Spitzoid melanoma.

241

242

DERMOSCOPY: AN ILLUSTRATED SELF-ASSESSMENT GUIDE

RISK ❑ Low ❑ Intermediate ✔ High ❑

1

2

DIAGNOSIS 3

Nevus Seborrheic keratosis Basal cell carcinoma Vascular Dermatofibroma Squamous cell carcinoma Melanoma Other

4

3-72c

❑ ❑ ❑ ❑ ❑ ❑ ✔ ❑ ❑

DISPOSITION ❑ No intervention ❑ Follow-up ✔ Histopathologic diagnosis ❑

ANSWERS Answers: 3,4

Discussion: ■

DERMOSCOPIC CRITERIA ■







■ ■

■ ■ ■

Asymmetry of color and structure Multicomponent global pattern (1,2,3,4) Irregular dots and globules (circles) Irregular dark blotches (yellow arrows) Regression (stars) Gray color in the area of regression (black arrows) Pinpoint vessels (box) Milia-like cyst (white arrow) Six colors

















With a quick glance, this lesion looks clinically and dermoscopically similar to Case 71. Brown globules identify a melanocytic lesion. ■ Pigment network is not needed to make that determination. There is one milia-like cyst. ■ Mucoid degeneration found in melanomas could look like a milia-like cyst. Pigmented pseudofollicular openings of a seborrheic keratosis are in the differential diagnosis of the focus of large brown irregular globules and irregular brown blotches at 3 o’clock. The irregular brown blotches are poorly defined with different shades of brown color. The large area of regression is made up of: ■ Milky-red/pink color ■ Gray homogeneous color without peppering ■ Bony-white color ■ Pinpoint vessels ■ It is hard to tell if the dots are brown or red. The absence of arborizing vessels and/or spoke-wheel structures goes against the diagnoses of a basal cell carcinoma. Of the potential six Spitzoid patterns, only the atypical pattern would be in the differential diagnosis of this case. Histopathologically, there was a nodular component that was not seen clinically or with dermoscopy.

PEARLS ■



Once this lesion was determined to be melanocytic, it looked more malignant than benign. The most atypical dermoscopic features could be associated with a benign lesion.

Chapter 3

Trunk and Extremities

RISK ❑ Low ❑ Intermediate ❑ High

DIAGNOSIS

3-73a

❑ ❑ ❑ ❑ ❑ ❑ ❑ ❑

Nevus Seborrheic keratosis Basal cell carcinoma Vascular Dermatofibroma Squamous cell carcinoma Melanoma Other

DISPOSITION

3-73b

❑ No intervention ❑ Follow-up ❑ Histopathologic diagnosis

CASE 73 HISTORY An 84-year-old man with a history of multiple skin cancers had this lesion on his back. 1. A milky-red area, irregular brown globules, polymorphous vessels, and milia-like cysts characterize this melanoma. 2. Multiple milia-like cysts, pigmented pseudofollicular openings, and hairpin vessels diagnose a seborrheic keratosis. 3. This could be a collision tumor: a seborrheic keratosis and amelanotic melanoma. 4. The bluish-white color is not diagnostic of a melanoma and could be seen in a seborrheic keratosis. 5. Multiple milia-like cysts and multiple pigmented pseudofollicular openings help differentiate a seborrheic keratosis from a melanocytic lesion that can have a few of these criteria.

243

244

DERMOSCOPY: AN ILLUSTRATED SELF-ASSESSMENT GUIDE

RISK ❑ Low ✔ Intermediate ❑ ❑ High

DIAGNOSIS Nevus Seborrheic keratosis Basal cell carcinoma Vascular Dermatofibroma Squamous cell carcinoma Melanoma Other

DISPOSITION ❑ No intervention ❑ Follow-up ✔ Histopathologic diagnosis ❑

1





■ ■

■ ■ ■

Asymmetry of color and structure Multicomponent global pattern (1,2,3) Milia-like cysts (circles) Pigmented pseudofollicular openings (arrows) Hairpin vessels (black box) Milky-red area (yellow box) Bluish-white color (stars)

3

ANSWERS Answers: 2,3,4,5

Discussion: ■

DERMOSCOPIC CRITERIA

2

3-73c

❑ ✔ ❑ ❑ ❑ ❑ ❑ ❑ ❑



■ ■



Asymmetry of color and structure and a multicomponent global pattern are commonly seen in seborrheic keratosis and are a nonspecific finding of little significance if there are criteria that definitively diagnose a seborrheic keratosis. Multiple milia-like cysts and pigmented pseudofollicular openings make the diagnosis. ■ The milia-like cysts look like “stars in the sky.” This seborrheic keratosis is heavily pigmented, which is the norm. The milky-red/pink area has a differential diagnosis. ■ Collision tumor: ■ Seborrheic keratosis and amelanotic melanoma ■ Seborrheic keratosis and basal cell carcinoma ■ Seborrheic keratosis and eccrine porocarcinoma ■ Focus of irritation One has to use their imagination to diagnose the hairpin vessels. ■ Polymorphous vessels (eg, linear, corkscrew) are in the differential diagnosis.

PEARLS ■

■ ■

Classic seborrheic keratosis without areas of potentially high risk pathology are routinely seen. Collision tumors commonly have seborrheic keratosis as one component. Beware! Melanomas are commonly mistaken for seborrheic keratosis.

Chapter 3

Trunk and Extremities

RISK ❑ Low ❑ Intermediate ❑ High

DIAGNOSIS

3-74a

❑ ❑ ❑ ❑ ❑ ❑ ❑ ❑

Nevus Seborrheic keratosis Basal cell carcinoma Vascular Dermatofibroma Squamous cell carcinoma Melanoma Other

DISPOSITION

3-74b

❑ No intervention ❑ Follow-up ❑ Histopathologic diagnosis

CASE 74 HISTORY A 34-year-old woman noticed a change in a nevus on her arm, which she had for many years. 1. Pigment network and brown globules identify a melanocytic lesion. 2. There is symmetry of color and structure, a globular global pattern, regular pigment network, and comma-shaped vessels. 3. Milky-red/pink color and pinpoint vessels are a red flag for concern. 4. Foci of the globular global pattern suggest that this melanoma arose in a pre-existing nevus. 5. Grayish homogeneous color without peppering suggests that there is regression in this melanoma.

245

246

DERMOSCOPY: AN ILLUSTRATED SELF-ASSESSMENT GUIDE

RISK ❑ Low ❑ Intermediate ✔ High ❑

1

2

3

DIAGNOSIS ❑ ❑ ❑ ❑ ❑ ❑ ✔ ❑ ❑

Nevus Seborrheic keratosis Basal cell carcinoma Vascular Dermatofibroma Squamous cell carcinoma Melanoma Other

2

4

2 3

3 3-74c

DISPOSITION ❑ No intervention ❑ Follow-up ✔ Histopathologic diagnosis ❑

ANSWERS Answers: 1,3,4,5

DERMOSCOPIC CRITERIA ■









■ ■ ■



Asymmetry of color and structure Multicomponent global pattern (1,2,3,4) Irregular pigment network (black boxes) Irregular purplish blotch (white arrows) Cobblestone-like globules (red boxes) Milky-red/pink color (stars) Pinpoint vessels (circles) Bluish-white color (yellow arrows) Gray homogeneous color and peppering (black arrows)

Discussion: ■











Clinically and dermoscopically this is clearly a high risk lesion. ■ There would not be a good clinico–dermoscopic–pathologic correlation if the pathologic report was not melanoma. Remnants of the cobblestone global pattern indicate that the melanoma arose in a pre-existing nevus. ■ The differential diagnosis includes reticular depigmentation (white network/negative pigment network). The milky-red/pink color with pinpoint vessels are dramatic and the first clue one might notice that this is a high risk lesion. ■ If there’s pink stop and think! ■ There is a component of bluish-white color that can be a component of a milky-red area. The milky-red/pink area has a differential diagnosis. ■ Neovascularization in a melanoma. ■ Amelanotic component of the pigmented melanoma. The irregular purplish blotch is another melanoma-specific criterion that has no diagnostic significance. The homogeneous grayish color with peppering represents foci of regression.

PEARLS ■



No pearls here, this is a clear-cut melanoma both clinically and with dermoscopy. At this point the diagnosis should be easy.

Chapter 3

Trunk and Extremities

RISK ❑ Low ❑ Intermediate ❑ High

DIAGNOSIS

3-75a

❑ ❑ ❑ ❑ ❑ ❑ ❑ ❑

Nevus Seborrheic keratosis Basal cell carcinoma Vascular Dermatofibroma Squamous cell carcinoma Melanoma Other

DISPOSITION

3-75b

❑ No intervention ❑ Follow-up ❑ Histopathologic diagnosis

CASE 75 HISTORY A 50-year-old woman found this lesion on her thigh one day, when she was in the shower. 1. Comma-shaped vessels, regular blotches, and regular globules diagnose a compound nevus. 2. This is a melanocytic lesion by default. 3. Diffuse bluish-white color, irregular dark blotches, milky-red color, and milky-red globules diagnose an invasive melanoma. 4. Hairpin vessels, pigmented pseudofollicular openings with sharp border demarcation diagnose a seborrheic keratosis. 5. Glomerular vessels and pigmentation diagnose pigmented Bowen disease.

247

248

DERMOSCOPY: AN ILLUSTRATED SELF-ASSESSMENT GUIDE

RISK ❑ Low ❑ Intermediate ✔ High ❑ 1

DIAGNOSIS Nevus Seborrheic keratosis Basal cell carcinoma Vascular Dermatofibroma Squamous cell carcinoma Melanoma Other

DISPOSITION ❑ No intervention ❑ Follow-up ✔ Histopathologic diagnosis ❑

ANSWERS Answers: 2,3

Discussion: ■

DERMOSCOPIC CRITERIA ■





■ ■ ■



2

3-75c

❑ ❑ ❑ ❑ ❑ ❑ ✔ ❑ ❑

3

Asymmetry of color and structure Multicomponent global pattern (1,2,3) Irregular dark blotches (white arrows) Diffuse bluish-white color Irregular dots (circle) Milky-red globules (black arrows) Milia-like cyst (red arrow)













This is a melanocytic lesion by default. ■ There is an absence of criteria to diagnose a melanocytic lesion, seborrheic keratosis, basal cell carcinoma, dermatofibroma, or hemangioma. ■ Two small dots are not enough to diagnose a melanocytic lesion. The irregular dark blotches are clues that this could be a melanocytic lesion. ■ Blotches are bigger than dot and globules. The asymmetry of color and structure and multicomponent global pattern are minor points in the context of the other high risk criteria. Diffuse bluish-white color, milky-red/pink color, and multiple milky-red globules are clues that this is an invasive melanoma. Milky-red globules should not be confused with lacunae of a benign hemangioma: ■ Lacunae are sharply demarcated vascular spaces, milky-red globules are not. Glomerular vessels of Bowen disease should be grouped, sharply demarcated coiled red spots. There is one milia-like cyst that has no diagnostic significance. ■ It could represent mucoid degeneration.

PEARLS ■ ■ ■

■ ■ ■

If there’s pink stop and think! When there’s white control your fright! If there’s blue they might sue! ■ Words to live by! Nodular melanoma can come out of nowhere and grow rapidly It is an easy diagnosis to make and miss! It competes with amelanotic melanoma for the prize of best masquerader!

Chapter 3

Trunk and Extremities

RISK ❑ Low ❑ Intermediate ❑ High

DIAGNOSIS

3-76a

❑ ❑ ❑ ❑ ❑ ❑ ❑ ❑

Nevus Seborrheic keratosis Basal cell carcinoma Vascular Dermatofibroma Squamous cell carcinoma Melanoma Other

DISPOSITION

3-76b

❑ No intervention ❑ Follow-up ❑ Histopathologic diagnosis

CASE 76 HISTORY This pink scaly slightly raised area was found on the forearm of a 50-year-old-golfer. 1. Pink scaly macules, papules, patches, plaques, or nodules could be melanocytic, nonmelanocytic, benign, malignant, or inflammatory. 2. Dermoscopy is not helpful to diagnose pink scaly lesions. 3. Pinpoint and glomerular vessels plus pigmentation diagnose a pigmented Bowen disease. 4. Glomerular vessels are pathognomonic for Bowen disease. 5. Irregular linear, glomerular, and pinpoint vessels, brown homogeneous pigmentation and white color put melanoma in the differential diagnosis.

249

250

DERMOSCOPY: AN ILLUSTRATED SELF-ASSESSMENT GUIDE

RISK ❑ Low ❑ Intermediate ✔ High ❑ 1

DIAGNOSIS 2

Nevus Seborrheic keratosis Basal cell carcinoma Vascular Dermatofibroma Squamous cell carcinoma Melanoma Other

DISPOSITION ❑ No intervention ❑ Follow-up ✔ Histopathologic diagnosis ❑

3

3-76c

❑ ❑ ❑ ❑ ❑ ✔ ❑ ❑ ❑

ANSWERS Answers: 1,3,5

Discussion: ■ ■

DERMOSCOPIC CRITERIA ■





■ ■



Asymmetry of color and structure Multicomponent global pattern (1,2,3) Homogeneous light brown color (black arrows) Pinpoint vessels (yellow circles) Glomerular vessels (black circles) Irregular linear vessels (black box)





■ ■





Pink scaly lesions are very common in the elderly sun-damaged population. Lesions are not always melanocytic by default when there are criteria to suggest another diagnosis not in that algorithm. Pink scaly lesions could be melanocytic, nonmelanocytic benign, malignant, or inflammatory, and dermoscopy is helpful to make a diagnosis. Pink lesions that lack melanoma-specific criteria usually have telangiectatic vessels that help make the diagnosis. ■ Rarely pink lesions are completely featureless. ■ One might have to look hard to identify and categorize the vessels. By definition, glomerular vessels are grouped and coiled. Bowen disease can be: ■ Pigmented or nonpigmented ■ Contain glomerular and/or pinpoint vessels Statistically, a pink scaly macule, papule, patch, plaque in an adult containing only pinpoint, and/or glomerular vessels usually is Bowen disease. ■ Amelanotic melanoma always is in the differential diagnosis. The focus of irregular linear vessels is not typically seen in Bowen disease and suggests a melanoma.

PEARLS ■

Vessels seen with dermoscopy are nonspecific, yet have the possibility of suggesting a specific diagnosis. ■ Arborizing—Basal cell carcinoma ■ Comma—Nevus ■ Glomerular—Bowen disease ■ Linear pinpoint-Clear cell ■ Polymorphous—Melanoma acanthoma ■ Hairpin-Seborrheic keratosis

Chapter 3

Trunk and Extremities

RISK ❑ Low ❑ Intermediate ❑ High

DIAGNOSIS

3-77a

❑ ❑ ❑ ❑ ❑ ❑ ❑ ❑

Nevus Seborrheic keratosis Basal cell carcinoma Vascular Dermatofibroma Squamous cell carcinoma Melanoma Other

DISPOSITION ❑ No intervention ❑ Follow-up ❑ Histopathologic diagnosis

CASE 77 HISTORY This lesion was found in an adult on the thigh and clinically looks exactly like the lesion in Case 76. 1. 2. 3. 4.

This lesion is filled with pinpoint vessels. A solitary pink lesion with diffuse pinpoint vessels could represent psoriasis. A solitary pink lesion filled with pinpoint vessels could represent Bowen disease. This lesion also demonstrates asymmetry of color and structure and bluish-white color that puts amelanotic melanoma in the differential diagnoses. 5. The fat serpiginous purple lines represent underlying blood vessels in a thin-skinned sun-damaged person.

251

252

DERMOSCOPY: AN ILLUSTRATED SELF-ASSESSMENT GUIDE

RISK ❑ Low ❑ Intermediate ✔ High ❑

DIAGNOSIS Nevus Seborrheic keratosis Basal cell carcinoma Vascular Dermatofibroma Squamous cell carcinoma Melanoma Other

DISPOSITION ❑ No intervention ❑ Follow-up ✔ Histopathologic diagnosis ❑

3-77b

❑ ❑ ❑ ❑ ❑ ✔ ❑ ❑ ❑

ANSWERS Answers: 1,2,3,4,5

Discussion: ■

DERMOSCOPIC CRITERIA ■

■ ■ ■ ■

Asymmetry of color and structure Pinpoint vessels (circles) Hypopigmentation (black stars) Bluish-white color (blue stars) Underlying cutaneous blood vessels (arrows) ■



Once again, a solitary pink scaly lesion filled with pinpoint vessels has a differential diagnosis. ■ In an adult, a lesion on sun-exposed skin with diffuse pinpoint vessels indicates that the number one possibility is Bowen disease. ■ This is a classic example, the most common presentation of nonpigmented Bowen disease that one will encounter. ■ A plaque of psoriasis could be indistinguishable. ■ One should perform a total body skin examination to discover other stigmata of psoriasis that might be present and easily clinch the correct diagnosis. ■ Amelanotic melanoma is lower on the differential diagnosis list. ■ One would like to see a less homogeneous distribution of polymorphous vessels. ■ A solitary lesion of nonspecific dermatitis can have this clinical and dermoscopic picture. Pigmented and nonpigmented Bowen disease are said to be diagnosed by finding glomerular vessels. ■ Glomerular and/or pinpoint vessels can be seen. ■ It might be more common to see pinpoint than true glomerular vessels. ■ Nonpigmented Bowen disease is much more common than the pigmented variety. ■ There can be a single or multiple lesions. The bluish-white color has no diagnostic significance in this case.

PEARLS ■ ■ ■

Bowen disease is ubiquitous in the elderly and not so elderly population. One of the major benefits of dermoscopy is patient reassurance. When one encounters a solitary pink lesion that could be amelanotic melanoma but classic vessels of Bowen disease are identified, it would be a nice gesture to pass that positive information to your patient.

Chapter 3

Trunk and Extremities

RISK ❑ Low ❑ Intermediate ❑ High

DIAGNOSIS

3-78a

❑ ❑ ❑ ❑ ❑ ❑ ❑ ❑

Nevus Seborrheic keratosis Basal cell carcinoma Vascular Dermatofibroma Squamous cell carcinoma Melanoma Other

DISPOSITION

3-78b

❑ No intervention ❑ Follow-up ❑ Histopathologic diagnosis

CASE 78 HISTORY This barely perceptible, asymptomatic pink macule was found on the upper back of a 50-year-old man while performing a total body skin examination. 1. 2. 3. 4.

Clinically and dermoscopically this is a classic example of Bowen disease. Comma and hairpin-shaped vessels help diagnose a seborrheic keratosis. Linear glomerular vessels suggest that this could be a clear cell acanthoma. The string of pearls circular arrangement of glomerular vessels diagnoses a clear cell acanthoma. 5. Different shades of pink color and milky-red globules put amelanotic melanoma in the differential diagnosis.

253

254

DERMOSCOPY: AN ILLUSTRATED SELF-ASSESSMENT GUIDE

RISK ✔ Low ❑ ❑ Intermediate ❑ High

DIAGNOSIS Nevus Seborrheic keratosis Basal cell carcinoma Vascular Dermatofibroma Squamous cell carcinoma Melanoma Other

3-78c

❑ ❑ ❑ ❑ ❑ ❑ ❑ ✔ ❑

ANSWERS DISPOSITION

Answers: 3,4,5

❑ No intervention ❑ Follow-up ✔ Histopathologic diagnosis ❑

Discussion: ■



DERMOSCOPIC CRITERIA ■ ■



■ ■ ■

Different shades of pink color String of pearls glomerular vessels (black arrows) Linear glomerular vessels (boxes) Pinpoint vessels (circles) Linear vessels (yellow arrows) Comma vessel (yellow box)







Clinically, this is not a typical clinical clear cell acanthoma which usually are: ■ Well circumscribed ■ Peripheral scale ■ 1-2 cm reddish moist nodules ■ Lower extremities The linear and circular arrangement of glomerular-like vessels immediately puts clear cell acanthoma at the top of the differential diagnosis list. ■ To date, this pattern of vessels is very sensitive and specific if not diagnostic of a clear cell acanthoma. ■ A large series of cases is needed to disprove this observation. The vessels are said to be arranged like a string of pearls and in this case they appear to be forming a circular necklace. Pinpoint, linear, and comma-shaped vessels are also seen. ■ Some of the vessels form circles or look like hairpins. The differential diagnosis includes: ■ Psoriasis → Usually has a homogeneous distribution of pinpoint vessels throughout the entire lesion. ■ Amelanotic melanoma → different shades of pink color, pinpoint and linear vessels, and a few milky-red globules should raise a red flag for concern. ■ Flat seborrheic keratosis → hairpin vessels. ■ This is very low on the differential diagnostic list. ■ One can see flat pink seborrheic keratosis without milia-like cysts, pseudofollicular openings, or other important criteria.

PEARLS ■







Once again, this case points out how useful dermoscopy can be to help diagnose nonspecific pink lesions. Even though there are several different-shaped telangiectatic vessels, the predominant pattern with linear and a circular arrangement of glomerular vessels makes the diagnoses. Don’t expect tiny vessels to always fit into specific categories of shapes. There can be shapes that have never been described and are hard to describe. Don’t get too hung up on the shapes!

Chapter 3

Trunk and Extremities

RISK ❑ Low ❑ Intermediate ❑ High

DIAGNOSIS 3-79a

❑ ❑ ❑ ❑ ❑ ❑ ❑ ❑

Nevus Seborrheic keratosis Basal cell carcinoma Vascular Dermatofibroma Squamous cell carcinoma Melanoma Other

DISPOSITION

3-79b

❑ No intervention ❑ Follow-up ❑ Histopathologic diagnosis

CASE 79 HISTORY The daughter of an 80-year-old man found this reddish lesion on the back of her father one day at the beach. 1. 2. 3. 4. 5.

This is a featureless pink lesion. This is a feature poor pink lesion. There are no melanoma-specific criteria. There are pinpoint, linear, and hairpin vessels. Different shades of pink color and polymorphous vessels could be seen in amelanotic melanoma and acute lichen planus-like keratosis (LPLK).

255

256

DERMOSCOPY: AN ILLUSTRATED SELF-ASSESSMENT GUIDE

1

RISK ❑ Low ❑ Intermediate ✔ High ❑

3 2

DIAGNOSIS Nevus Seborrheic keratosis Basal cell carcinoma Vascular Dermatofibroma Squamous cell carcinoma Melanoma Other

DISPOSITION ❑ No intervention ❑ Follow-up ✔ Histopathologic diagnosis ❑

4

3-79c

❑ ❑ ❑ ❑ ❑ ❑ ✔ ❑ ❑

ANSWERS Answers: 2,4,5

Discussion: ■ ■ ■

DERMOSCOPIC CRITERIA ■



■ ■ ■ ■ ■

Asymmetry of color and structure Multicomponent global pattern (1,2,3,4) Different shades of pink color Pinpoint vessels (black boxes) Linear vessels (yellow boxes) Hairpin vessels (white boxes) Clothing fibers (arrows)











This could be considered a melanocytic lesion by default. It is not featureless because there are polymorphous vessels. It is feature-poor because there are no other well-developed melanomaspecific criteria. The clinical differential diagnosis includes Bowen disease, basal cell carcinoma, a pink lichen planus-like keratosis, nonspecific dermatitis and amelanotic melanoma. Different shades of pink color are nonspecific and could be seen in amelanotic melanoma and pink lichen planus-like keratosis. Pinpoint and linear vessels are also nonspecific. ■ It might be difficult to differentiate the pinpoint and linear vessels. The presence of hairpin vessels favors the diagnosis of amelanotic melanoma. ■ Hairpin vessels are not seen in pink lichen planus-like keratosis. Completely amelanotic 100% featureless melanomas are not commonly found. ■ Statistically, a lesion that looks like this clinically and dermoscopically will not be a melanoma.

PEARLS ■



■ ■

Don’t be surprised that even the most innocuous looking pink lesion with minimal dermoscopic features could be an amelanotic melanoma. A patient can have multiple similar appearing pink lesions and only one might turn out to be an amelanotic melanoma. Don’t ignore pink lesions. Don’t fear pink lesions, respect them!

Chapter 3

Trunk and Extremities

RISK ❑ Low ❑ Intermediate ❑ High

DIAGNOSIS

3-80a

❑ ❑ ❑ ❑ ❑ ❑ ❑ ❑

Nevus Seborrheic keratosis Basal cell carcinoma Vascular Dermatofibroma Squamous cell carcinoma Melanoma Other

DISPOSITION

3-80b

❑ No intervention ❑ Follow-up ❑ Histopathologic diagnosis

CASE 80 HISTORY A 65-year-old woman presented with a slightly pruritic, pink scaly papule on her right thigh, which started a month ago and was slowly getting bigger. 1. Diffuse pinpoint vessels and scale characterize this isolated plaque of psoriasis. 2. Different shades of pink and brown color, pinpoint vessels, and regression diagnose this amelanotic melanoma. 3. This could be a melanocytic lesion by default. 4. This could be an acute pink lichen planus-like keratosis. 5. Clinically and dermoscopically it is not possible to make a specific diagnosis.

257

258

DERMOSCOPY: AN ILLUSTRATED SELF-ASSESSMENT GUIDE

RISK ❑ Low ❑ Intermediate ✔ High ❑ 2

1

DIAGNOSIS Nevus Seborrheic keratosis Basal cell carcinoma Vascular Dermatofibroma Squamous cell carcinoma Melanoma Other

5 3 4

3-80c

❑ ❑ ❑ ❑ ❑ ❑ ❑ ✔ ❑

ANSWERS Answers: 3,4,5

DISPOSITION ❑ No intervention ❑ Follow-up ✔ Histopathologic diagnosis ❑

Discussion: ■



DERMOSCOPIC CRITERIA ■



■ ■

■ ■ ■

Asymmetry of color and structure Multicomponent global pattern (1,2,3,4,5) Different shades of pink color Different shades of brown color (black arrows) Pinpoint vessels (circles) Scale (yellow arrows) Normal skin (stars)





There is no way to differentiate this nonspecific clinical and dermoscopic picture from amelanotic melanoma. ■ The history of sudden appearance, slight enlargement, and pruritus favors an acute lichen planus-like keratosis over a melanoma. Pink lichen-planus like keratosis develop de novo and are not associated with pre-existing lesions (eg, flat seborrheic keratosis, solar lentigo). ■ Lichen planus-like keratosis can be: ■ Acute → less than 3 months ■ Sub-acute → 3 months to a year ■ Chronic → more than a year ■ This is an acute short-lived lichen planus-like keratosis. ■ Acute lichen planus-like keratoses are usually pink. ■ Solitary lesion. ■ Patients can have two or three lesions at the same time. ■ Patients can develop new lesions at any time in the future. Different shades of pink color with or without polymorphous vessels and scale characterize typical pink lichen planus-like keratosis. ■ It is not possible to differentiate pink lichen planus-like keratosis from other feature-poor scaly pink lesions. The white color does not represent regression. ■ Typically some or all of the pink color blanches away with pressure from instrumentation.

PEARLS ■ ■



■ ■

Eliminate scale or dryness on a lesion to get a better dermoscopic view. Acute pink lichen planus-like keratosis can be seasonal and one can expect to see several cases in short order in the same or different patients. A patient’s luck is very important with melanoma survival, especially when a pink lesion that could be a deep amelanotic melanoma turns out to be an innocuous pink lichen planus-like keratosis. What a nice and welcomed surprise! Let your patient know how lucky they are.

Chapter 3

Trunk and Extremities

RISK ❑ Low ❑ Intermediate ❑ High

DIAGNOSIS

3-81a

❑ ❑ ❑ ❑ ❑ ❑ ❑ ❑

Nevus Seborrheic keratosis Basal cell carcinoma Vascular Dermatofibroma Squamous cell carcinoma Melanoma Other

DISPOSITION

3-81b

❑ No intervention ❑ Follow-up ❑ Histopathologic diagnosis

CASE 81 HISTORY A 43-year-old woman had this nonchanging, asymptomatic lesion on her right lower leg for 9 months. 1. 2. 3. 4. 5.

Brown globules identify a melanocytic lesion. Regression characterized by grayish-white color and peppering fill the lesion. Pinpoint vessels and brown color diagnose pigmented Bowen disease. The red dots represent pinpoint vessels. The clinical and dermoscopic differential diagnosis includes regressive melanoma and a subacute lichen planus-like keratosis.

259

260

DERMOSCOPY: AN ILLUSTRATED SELF-ASSESSMENT GUIDE

RISK

1

❑ Low ❑ Intermediate ✔ High ❑

2

DIAGNOSIS Nevus Seborrheic keratosis Basal cell carcinoma Vascular Dermatofibroma Squamous cell carcinoma Melanoma Other

DISPOSITION ❑ No intervention ❑ Follow-up ✔ Histopathologic diagnosis ❑

3

3-81c

❑ ❑ ❑ ❑ ❑ ❑ ❑ ✔ ❑

ANSWERS Answers: 2,4,5

Discussion: ■

DERMOSCOPIC CRITERIA ■



■ ■



Asymmetry of color and structure Multicomponent global pattern (1,2,3) Pinpoint vessels (circles) Homogeneous gray color and peppering (boxes) Bluish-white color of regression (stars)







The clinical and dermoscopic pictures are worrisome: ■ This is melanoma until proven otherwise with a histopathologic diagnosis. ■ The differential diagnosis includes sub-acute lichen planus-like keratosis. ■ A relatively rare lesion. There are well-developed melanoma-specific criteria: ■ Asymmetry of color and structure ■ Multicomponent global pattern ■ Pinpoint vessels ■ Widespread regression Sub-acute lichen planus-like keratosis are characterized by: ■ Dusky-red or violaceous color. ■ Irregularly pigmented with different shades of brown, gray, and/or white color. ■ Have a variable amount of regression with peppering. ■ The older the lesion, one sees more regression with peppering. A small sub-set of lichen planus-like keratosis is thought to represent immunologically-mediated regression of an existing solar lentigo or flat seborrheic keratosis. ■ One might see criteria associated with these two pathologies associated with lichen planus-like keratosis.

PEARL ■

Put lichen planus-like keratosis in your differential diagnosis of regressive lesions with peppering and you won’t be too surprised when a lesion you are concerned is a bad melanoma turns out not to be.

Chapter 3

Trunk and Extremities

RISK ❑ Low ❑ Intermediate ❑ High

DIAGNOSIS

3-82a

❑ ❑ ❑ ❑ ❑ ❑ ❑ ❑

Nevus Seborrheic keratosis Basal cell carcinoma Vascular Dermatofibroma Squamous cell carcinoma Melanoma Other

DISPOSITION

3-82b

❑ No intervention ❑ Follow-up ❑ Histopathologic diagnosis

CASE 82 HISTORY You discover this gray macule on the chest of a 75-year-old man with evidence of severe sun damage. 1. The patient should bathe and wash the dirt off. 2. The gray dots represent a classic example of peppering that represents melanophages and free melanin in the dermis. 3. This is a classic radiation tattoo. 4. Remnants of the fingerprint pattern and peppering suggest that this could be a chronic lichen planus-like keratosis. 5. The focus of brown globules could represent remnants of the fingerprint pattern.

261

262

DERMOSCOPY: AN ILLUSTRATED SELF-ASSESSMENT GUIDE

RISK ✔ Low ❑ ❑ Intermediate ❑ High

DIAGNOSIS Nevus Seborrheic keratosis Basal cell carcinoma Vascular Dermatofibroma Squamous cell carcinoma Melanoma Other

3-82c

❑ ❑ ❑ ❑ ❑ ❑ ❑ ✔ ❑

ANSWERS Answers: 2,4,5

DISPOSITION ❑ No intervention ✔ Follow-up ❑ ❑ Histopathologic diagnosis

Discussion: ■





DERMOSCOPIC CRITERIA ■ ■

■ ■

Peppering (black boxes) Solar lentigo (black arrows) ■ Irregular tan color ■ Fingerprint pattern (yellow boxes) Brown globules (yellow arrows) Scarred sun-damaged skin (stars)









If one performs a focused total body skin examination, subtle gray macules like this are not uncommonly found in people with extensive sun damage. The white color and peppering puts a lesion with regression at the top of the differential diagnosis list. With the identification of criteria to diagnose a solar lentigo, a chronic lichen planus-like keratosis now tops the differential diagnostic list. The solar lentigo consists of: ■ Irregular tan color. ■ Remnants of a fingerprint pattern with fine parallel line segments. ■ The brown globules look like the globules of a melanocytic lesion but represent broken up remnants of the fingerprint pattern. The white color is seen clinically on the skin surrounding the lesion and does not represent regression. ■ Diffuse white scar-like discoloration is one of the many findings that result from chronic sun exposure. For the novice dermoscopist, a histopathologic diagnosis is indicated. With experience, after evaluating all of the criteria one will not feel compelled to make a histopathologic diagnosis. A radiation tattoo is characterized by: ■ A small perfectly round blue macule. ■ Homogeneous blue color similar to a blue nevus. ■ Several similar lesions can be seen in the same area. ■ The patient will tell you that they had radiation in the area.

PEARLS ■



This is an example of a chronic lichen planus-like keratosis in which there appears to be regression of a pre-existing lesion. ■ The coexistence of a pre-existing lesion is an important clue that the lesion might not be a regressive melanoma. In our experience, one does not routinely find evidence of a pre-existing solar lentigo or flat seborrheic keratosis associated with acute, sub-acute, or chronic lichen planus-like keratosis.

Chapter 3

Trunk and Extremities

RISK ❑ Low ❑ Intermediate ❑ High

DIAGNOSIS

3-83a

❑ ❑ ❑ ❑ ❑ ❑ ❑ ❑

Nevus Seborrheic keratosis Basal cell carcinoma Vascular Dermatofibroma Squamous cell carcinoma Melanoma Other

DISPOSITION

3-83b

❑ No intervention ❑ Follow-up ❑ Histopathologic diagnosis

CASE 83 HISTORY A 63-year-old female discovered this dark spot on her thigh while drying off in the shower. 1. Clinically and dermoscopically, this is similar to Case 82 and represents a chronic lichen planus-like keratosis. 2. Globules identify a melanocytic lesion. 3. There are several melanoma-specific criteria without evidence of a solar lentigo or flat seborrheic keratosis. 4. The focus of bluish-white color in the brown area is a red flag for concern that this could be a melanoma. 5. Asymmetry of color and structure, a multicomponent global pattern, different shades of brown color, irregular dots and globules, plus regression characterize this melanoma.

263

264

DERMOSCOPY: AN ILLUSTRATED SELF-ASSESSMENT GUIDE

RISK ❑ Low ❑ Intermediate ✔ High ❑ 1 2

3

DIAGNOSIS Nevus Seborrheic keratosis Basal cell carcinoma Vascular Dermatofibroma Squamous cell carcinoma Melanoma Other

DISPOSITION ❑ No intervention ❑ Follow-up ✔ Histopathologic diagnosis ❑

1 1

3-83c

❑ ❑ ❑ ❑ ❑ ❑ ✔ ❑ ❑

ANSWERS Answers: 2,3,4,5

Discussion: ■

DERMOSCOPIC CRITERIA ■









Asymmetry of color and structure Multicomponent global pattern (1,2,3) Irregular dots and globules (circles) Different shades of brown color Regression (black stars) ■ Gray color and peppering (boxes) ■ Bluish-white color (arrows)









Clinically and dermoscopically, this is similar to Case 82. ■ Both are subtle gray macules clinically with regression dermoscopically but this is where the similarities end. ■ The suggestion of brown color seen clinically is a clue that this might be a high risk lesion. There are no criteria associated with a solar lentigo or flat seborrheic keratosis. The large irregular pigmented area is another clue that this could be a melanoma and contains: ■ Different shades of brown color. ■ Poorly-developed irregular dots and globules. ■ A focus of bluish-white color. Regression with gray color and foci of peppering is a nonspecific finding. ■ The bluish-white color suggests this could be a melanoma. Histopathologically, this was diagnosed as an in situ melanoma arising in a compound nevus without any mention of regression. ■ This is not a good dermoscopic-pathologic correlation. ■ One should communicate with the pathologist that there was regression seen with dermoscopy but it was not mentioned in the pathology report. ■ Further investigation is indicated to rule out a more invasive melanoma with regression.

PEARLS ■ ■ ■ ■

Take the time to check all gray macules with dermoscopy. Perform focused skin examinations so that they are not overlooked! Once discovered, passing them over could be a deadly mistake! Don’t be lazy!

Chapter 3

Trunk and Extremities

RISK ❑ Low ❑ Intermediate ❑ High

DIAGNOSIS

3-84a

❑ ❑ ❑ ❑ ❑ ❑ ❑ ❑

Nevus Seborrheic keratosis Basal cell carcinoma Vascular Dermatofibroma Squamous cell carcinoma Melanoma Other

DISPOSITION

3-84b

❑ No intervention ❑ Follow-up ❑ Histopathologic diagnosis

CASE 84 HISTORY This lesion was found on the back of a 66-year-old man. 1. Remnants of the fingerprint pattern, milia-like cysts, and regression diagnose a chronic lichen planus-like keratosis. 2. Pigment network identifies a melanocytic lesion. 3. There is asymmetry of color and structure plus a multicomponent global pattern. 4. Irregular pigment network and widespread regression are the other melanoma-specific criteria. 5. Even though there is widespread regression, the presence of regular pigment network and the absence of pinpoint vessels rules out a melanoma.

265

266

DERMOSCOPY: AN ILLUSTRATED SELF-ASSESSMENT GUIDE

RISK

1

❑ Low ❑ Intermediate ✔ High ❑

DIAGNOSIS

DISPOSITION ❑ No intervention ❑ Follow-up ✔ Histopathologic diagnosis ❑

DERMOSCOPIC CRITERIA ■









2

Nevus Seborrheic keratosis Basal cell carcinoma Vascular Dermatofibroma Squamous cell carcinoma Melanoma Other

Asymmetry of color and structure Multicomponent global pattern (1,2,3) Irregular pigment network (boxes) Regression ■ Bony-white color (stars) ■ Homogeneous gray color and peppering (black arrows) Milia-like cysts (yellow arrows)

3

2 3-84c

❑ ❑ ❑ ❑ ❑ ❑ ✔ ❑ ❑

ANSWERS Answers: 2,3,4

Discussion: ■











Clinically the lesion is difficult to see because it is almost completely regressed. ■ Melanomas can completely regress leaving a residual depigmented area or nothing at all to see. The widespread regression should not be confused with hypopigmentation. ■ Regression has a bony-white color. ■ Hypopigmentation can have different shades of light brown color. ■ Gray color and peppering are not found in hypopigmentation. The regression is comprised of: ■ Bony-white color ■ Homogeneous gray color ■ Peppering (very difficult to see) There are only remnants of mildly atypical pigment network. ■ Another dermoscopist might consider the pigment network to be regular. The criterion that looks like milia-like cysts could represent mucoid degeneration. ■ Milia-like cysts can be found in melanomas. The presence or absence of pinpoint vessels does not make or break the diagnosis of a melanoma. None are found in this regressive melanoma.

PEARLS ■

■ ■

Wood’s light examination in a dark room will help accentuate the bonywhite color of clinical regression and superficially located pigmentation. The Wood’s light can make invisible lesions become visible. If a primary cutaneous melanoma cannot be found, a Wood’s light examination should be performed in patients that present with metastatic melanoma to lymph nodes or other organs to try to identify the residual depigmented area of a completely regressed melanoma.

Chapter 3

Trunk and Extremities

RISK ❑ Low ❑ Intermediate ❑ High

DIAGNOSIS

3-85a

❑ ❑ ❑ ❑ ❑ ❑ ❑ ❑

Nevus Seborrheic keratosis Basal cell carcinoma Vascular Dermatofibroma Squamous cell carcinoma Melanoma Other

DISPOSITION

3-85b

❑ No intervention ❑ Follow-up ❑ Histopathologic diagnosis

CASE 85 HISTORY A 17-year-old male came in for his routine 6-month skin examination. He was a surfer with a history of extensive sun exposure and a few mildly dysplastic nevi. Digital monitoring found a change in this long-time-stable lesion on his left upper back. The patient was not aware of the change. He commented before this lesion was discovered that he did not think he needed skin examinations anymore! 1. 2. 3. 4. 5.

Blue ovoid nests, white and brown colors diagnose a pigmented basal cell carcinoma. Brown globules identify a melanocytic lesion. Regular blotches, hypopigmentation, and regular globules diagnose a dysplastic nevus. The bony-white color with blue globules is never seen in true regression. The well-developed melanoma-specific criteria diagnose melanoma with certainty.

267

268

DERMOSCOPY: AN ILLUSTRATED SELF-ASSESSMENT GUIDE

RISK ❑ Low ❑ Intermediate ✔ High ❑

1

3

1

DIAGNOSIS Nevus Seborrheic keratosis Basal cell carcinoma Vascular Dermatofibroma Squamous cell carcinoma Melanoma Other

DISPOSITION ❑ No intervention ❑ Follow-up ✔ Histopathologic diagnosis ❑

3

ANSWERS Answers: 2

Discussion: ■ ■

DERMOSCOPIC CRITERIA ■









Asymmetry of color and structure Multicomponent global pattern (1,2,3) Irregular dots and globules (circles) Irregular dark brown blotches (white arrows) Regression ■ Bony-white color (stars) ■ Bluish-gray color (red arrows)

1

3

3-85c

✔ ❑ ❑ ❑ ❑ ❑ ❑ ❑ ❑

2







Brown globules identify a melanocytic lesion. There are well-developed melanoma-specific criteria: ■ Asymmetry of color and structure ■ Irregular brown blotches ■ Multicomponent global pattern ■ Regression The irregular brown dots and globules are few and far between, and a minor criterion when compared to the other well-developed high risk criteria. The regression is composed of: ■ Bony-white color ■ Bluish-gray blotches ■ Peppering is absent. ■ The bluish tinge means that the melanophages and free melanin are deeper in the dermis → the Tyndall effect. ■ Gray and/or blue color with or without peppering can be seen in regression. This moderately dysplastic nevus looks more malignant than benign. ■ There is not a good dermoscopic-pathologic correlation since it is very suspicious for a melanoma. ■ A second dermatopathologists opinion was indicated. ■ Another experienced dermatopathologist could not find criteria to diagnose a melanoma with regression. ■ An experienced dermatopatholgist can differentiate regression associated with a benign or malignant lesion.

PEARLS ■



The patient commented at the beginning of the consultation that he felt there was no longer a need for follow-up skin examinations. Patients with a history of dysplastic nevi are at a higher risk to develop melanoma at any time in the future and should continue with periodic selfskin examinations and 6-month total body skin examinations, which of course should include dermoscopy.

Chapter 3

Trunk and Extremities

RISK ❑ Low ❑ Intermediate ❑ High

DIAGNOSIS

3-86a

❑ ❑ ❑ ❑ ❑ ❑ ❑ ❑

Nevus Seborrheic keratosis Basal cell carcinoma Vascular Dermatofibroma Squamous cell carcinoma Melanoma Other

DISPOSITION

3-86b

❑ No intervention ❑ Follow-up ❑ Histopathologic diagnosis

CASE 86 HISTORY A primary care physician found this solitary lesion on the back of a 52-year-old man and referred him to you for your evaluation. 1. 2. 3. 4. 5.

The darks dots and globules could be seen in a melanocytic lesion. The dark dots and globules could be seen in a pigmented basal cell carcinoma. The foci of pinpoint and linear vessels are hard to see and rule out a basal cell carcinoma. This could be a basal cell carcinoma without arborizing vessels. Pigmented pseudofollicular openings and milia-like cysts diagnose a seborrheic keratosis.

269

270

DERMOSCOPY: AN ILLUSTRATED SELF-ASSESSMENT GUIDE

RISK ❑ Low ❑ Intermediate ✔ High ❑ 1

DIAGNOSIS Nevus Seborrheic keratosis Basal cell carcinoma Vascular Dermatofibroma Squamous cell carcinoma Melanoma Other

3

3-86c

❑ ❑ ✔ ❑ ❑ ❑ ❑ ❑ ❑

2

ANSWERS DISPOSITION ❑ No intervention ❑ Follow-up ✔ Histopathologic diagnosis ❑

Answers: 1,2,4

Discussion: ■



DERMOSCOPIC CRITERIA ■







■ ■

Asymmetry of color and structure Multicomponent global pattern (1,2,3) Irregular grayish-black dots and globules (circles) Pinpoint and linear vessels (boxes) Ulcerations (arrows) Bluish-white color (stars)











At first blush this is not a classic basal cell carcinoma. ■ There is an absence of well-developed criteria to make the diagnosis. ■ Arborizing vessels ■ Spoke-wheel structures ■ Ulceration The conspicuous irregular grayish-black dots and globules have a differential diagnosis. ■ A variation of typical peppering with larger dots and globules. ■ Dots and globules of a melanocytic lesion (eg, melanoma). ■ Pigmentation found in basal cell carcinoma. There are innumerable variations of shapes and colors of pigmentation that can be found in basal cell carcinomas. ■ There is no need to try to categorize pigmentation into special shapes and/or descriptions. ■ Leaf-like structures ■ Ovoid nests There are no arborizing vessels. ■ The diagnosis of basal cell carcinoma can be made without finding arborizing vessels. There are pinpoint and linear vessels, which can be found in basal cell carcinomas but are more suggestive of a melanocytic lesion. There are poorly defined ulcerations that have a differential diagnosis. ■ Crust formation There are no criteria to diagnose a seborrheic keratosis.

PEARLS ■





In vivo, the pigmentation typically found in basal cell carcinomas appears to be in the context of a lesion with a semi-translucent clear look, which does not always come through in digital clinical images. Semi-translucency is an important clue that a lesion could be a basal cell carcinoma. If pigmented, it could appear like a dirty ice cube.

Chapter 3

Trunk and Extremities

RISK ❑ Low ❑ Intermediate ❑ High

DIAGNOSIS

3-87a

❑ ❑ ❑ ❑ ❑ ❑ ❑ ❑

Nevus Seborrheic keratosis Basal cell carcinoma Vascular Dermatofibroma Squamous cell carcinoma Melanoma Other

DISPOSITION

3-87b

❑ No intervention ❑ Follow-up ❑ Histopathologic diagnosis

CASE 87 HISTORY A 31-year-old man complained that this lesion on his right lower leg that has been present for years has lately become sensitive. 1. Clinically, the lesion looks like a basal cell carcinoma with a semi-translucent appearance. However, the polymorphous vessels rule this out. 2. Gray blotches diagnose a melanocytic lesion. 3. Irregular gray blotches and well-developed polymorphous vessels put melanoma in the differential diagnosis. 4. The differential diagnosis of the two large irregular dark blotches includes ulceration and pigmented blotches of a melanocytic lesion. 5. Polymorphous and arborizing vessels can be found in basal cell carcinomas.

271

272

DERMOSCOPY: AN ILLUSTRATED SELF-ASSESSMENT GUIDE

RISK ❑ Low ❑ Intermediate ✔ High ❑

2

DIAGNOSIS

1

Nevus Seborrheic keratosis Basal cell carcinoma Vascular Dermatofibroma Squamous cell carcinoma Melanoma Other

DISPOSITION ❑ No intervention ❑ Follow-up ✔ Histopathologic diagnosis ❑

3 4

3-87c

❑ ❑ ✔ ❑ ❑ ❑ ❑ ❑ ❑

ANSWERS Answers: 3,4,5

Discussion: ■



DERMOSCOPIC CRITERIA ■





■ ■



Asymmetry of color and structure Multicomponent global pattern (1,2,3,4) Irregular gray blotches (black boxes) Ulceration (black arrows) Dried blood in ulcerations (stars) Polymorphous vessels ■ Pinpoint (black circles) ■ Linear (yellow arrows) ■ Hairpin (white box) ■ Corkscrew (yellow box) ■ Arborizing (yellow circles) ■ Comma (red circle)











The semi-translucent clinical appearance is a clue that the lesion could be a basal cell carcinoma. This case points out that basal cell carcinomas can have polymorphous vessels in addition to typical arborizing vessels. The irregular gray blotches (homogeneous gray color) are the pigmented component of the basal cell and not a criterion that can diagnose a melanocytic lesion. There are two components to the ulceration: ■ Ulceration ■ Dried dark blood There appears to be vessels in the ulceration, which is unexplained and has a differential diagnosis: ■ An irregular dark blotch of a melanocytic lesion with polymorphous vessels. Ulcerations can be found in both basal cell carcinomas and melanomas, and are not a differentiating point. ■ Statistically, ulcerations are more commonly found in basal cell carcinomas. The final thought after evaluating all of the clinical and dermoscopic criteria is that this could be a melanoma with ulceration and polymorphous vessels, or an unusual basal cell carcinoma with polymorphous vessels and lacking classic arborizing vessels. Either way, prompt histopathologic diagnosis is indicated.

PEARLS ■ ■

This is a great case to study the different shapes of telangiectatic vessels. Be aware that a sub-set of basal cell carcinomas lack well-developed arborizing vessels and can contain polymorphous vessels or no vessels at all.

Chapter 3

Trunk and Extremities

RISK ❑ Low ❑ Intermediate ❑ High

DIAGNOSIS

3-88a

❑ ❑ ❑ ❑ ❑ ❑ ❑ ❑

Nevus Seborrheic keratosis Basal cell carcinoma Vascular Dermatofibroma Squamous cell carcinoma Melanoma Other

DISPOSITION

3-88b

❑ No intervention ❑ Follow-up ❑ Histopathologic diagnosis

CASE 88 HISTORY A 64-year-old man noticed this lesion on his back 4 weeks before going to his dermatologist. 1. 2. 3. 4. 5.

Clinically and dermoscopically this looks like a compound nevus. A focus of brown globules identifies a melanocytic lesion. The bony- and bluish-white colors do not represent regression because there is no peppering. A central white patch and a fine peripheral pigment network diagnose a dermatofibroma. At least 50% of the lesion is replaced with regression, which is suspicious for a melanoma.

273

274

DERMOSCOPY: AN ILLUSTRATED SELF-ASSESSMENT GUIDE

RISK ❑ Low ❑ Intermediate ✔ High ❑

2 1

Nevus Seborrheic keratosis Basal cell carcinoma Vascular Dermatofibroma Squamous cell carcinoma Melanoma Other

DISPOSITION ❑ No intervention ❑ Follow-up ✔ Histopathologic diagnosis ❑

1 2 3 3-88c

❑ ❑ ❑ ❑ ❑ ❑ ✔ ❑ ❑

ANSWERS Answers: 2,5

Discussion: ■



DERMOSCOPIC CRITERIA ■







Asymmetry of color and structure Multicomponent global pattern (1,2,3) Irregular brown globules (circle) Regression ■ Bony-white color (black stars) ■ Bluish-white color (blue stars)

1

2

DIAGNOSIS









Clinically, the regression is hard to see and dermoscopy makes it glaringly obvious. The focus of globules is not well developed and one could consider this to be a melanocytic lesion by default. The regression is extensive and consists of bony- and bluish-white colors without peppering. ■ Peppering is not always found in regression. Asymmetry of color and structure and the multicomponent global pattern are the only other well-developed melanoma-specific criteria. The focus of brown globules are asymmetrically located, and irregular in size and shape. ■ They are a minor component of the entire atypical picture. Dermatofibromas do have significant variation of morphology that can be seen along with the central white patch. ■ The regression does not fall within the possibility of being considered a central white patch. ■ The lesion does not have pigment network. ■ Dermatofibromas do not all have pigment network.

PEARLS ■

■ ■



Once again, this case points out how much more can be seen with dermoscopy over naked eye examination or with the other nondermoscopic magnification clinicians use. Regression by itself is an independent, potentially high risk criterion. The more the regression filling a lesion, the greater the chance that it is a melanoma. Regression can be found in benign lesions.

Chapter 3

Trunk and Extremities

RISK ❑ Low ❑ Intermediate ❑ High

DIAGNOSIS

3-89a

❑ ❑ ❑ ❑ ❑ ❑ ❑ ❑

Nevus Seborrheic keratosis Basal cell carcinoma Vascular Dermatofibroma Squamous cell carcinoma Melanoma Other

DISPOSITION

3-89b

❑ No intervention ❑ Follow-up ❑ Histopathologic diagnosis

CASE 89 HISTORY This firm papule was found on the right shin of a 48-year-old woman. 1. The pigment network could be seen in a melanocytic lesion or dermatofibroma. 2. The irregular bony-white color could represent regression in a melanoma or an atypical central white patch of a dermatofibroma. 3. Giant milia-like cysts and pseudofollicular openings diagnose a seborrheic keratosis. 4. The differential diagnosis includes melanoma and an atypical dermatofibroma. 5. Atypical dermatofibromas warrant a histopathologic diagnosis.

275

276

DERMOSCOPY: AN ILLUSTRATED SELF-ASSESSMENT GUIDE

1

RISK 1

❑ Low ❑ Intermediate ✔ High ❑

2 1 2

2

DIAGNOSIS

1 2

Nevus Seborrheic keratosis Basal cell carcinoma Vascular Dermatofibroma Squamous cell carcinoma Melanoma Other

3

3-89c

❑ ❑ ❑ ❑ ✔ ❑ ❑ ❑ ❑

ANSWERS Answers: 1,2,4,5

DISPOSITION ❑ No intervention ❑ Follow-up ✔ Histopathologic diagnosis ❑

Discussion: ■



DERMOSCOPIC CRITERIA ■









■ ■

Asymmetry of color and structure Multicomponent global pattern (1,2,3) Regular pigment network (boxes) Purplish-white color (black arrows) Fingerprint pattern (yellow arrows) Central white patch (stars) Ring-like structures (circles)











A strict interpretation of the global pattern and local criteria favors the diagnosis of a melanoma over dermatofibroma. ■ The history of a long-standing, nonchanging lesion that is firm to palpation favors the diagnosis of a dermatofibroma. A classic nonatypical dermatofibroma is made up of: ■ Centrally located bony-white homogeneous color. ■ Fine peripheral pigment network. ■ This pattern is commonly encountered. When atypical, the white color of a dermatofibroma is indistinguishable from regression seen in melanomas and can take many forms (eg, reticular depigmentation, multifocal white areas). Giant milia-like cysts exist. However, they have sharp well-defined borders that differ from the white color seen here where the borders are not sharp at all. One must be aware that there are innumerable variations of morphology and color that can be found in dermatofibromas. ■ The diffuse erythema seen clinically blanches away with pressure and is a nonspecific finding. ■ The irregular purplish-white blotches could represent deep pigmentation of a melanocytic lesion or vascularization that could be seen in a melanocytic lesion or in a dermatofibroma. ■ It is not the milky-red color typically seen in melanomas. The pseudofollicular opening-like circles could represent ring-like fragments of pigment network. ■ Dermatofibromas can have thick ring-like circles making up the pigment network. The parallel line segments suggesting a fingerprint pattern represent fragments of the pigment network.

PEARLS ■ ■

Be aware that atypical dermatofibromas exist. In order not to miss dermatofibroma-like melanomas, all atypical dermatofibromas warrant a histopathologic diagnosis.

Chapter 3

Trunk and Extremities

RISK ❑ Low ❑ Intermediate ❑ High

DIAGNOSIS

3-90a

❑ ❑ ❑ ❑ ❑ ❑ ❑ ❑

Nevus Seborrheic keratosis Basal cell carcinoma Vascular Dermatofibroma Squamous cell carcinoma Melanoma Other

DISPOSITION

3-90b

❑ No intervention ❑ Follow-up ❑ Histopathologic diagnosis

CASE 90 HISTORY A 19-year-old developed white color around a pre-existing nevus. 1. Globules identify a melanocytic lesion. 2. Irregular globules, irregular dark blotches, and regression characterize this regressive melanoma. 3. A peripheral white patch and irregular pigment network characterize this reverse dermatofibroma. 4. A symmetrical white halo around the central pigmentation characterizes this halo nevus. 5. Asymmetrical pigmentation that extends outside of a white halo could represent a recurrent melanoma.

277

278

DERMOSCOPY: AN ILLUSTRATED SELF-ASSESSMENT GUIDE

RISK ❑ Low ✔ Intermediate ❑ ❑ High

DIAGNOSIS Nevus Seborrheic keratosis Basal cell carcinoma Vascular Dermatofibroma Squamous cell carcinoma Melanoma Other

3-90c

✔ ❑ ❑ ❑ ❑ ❑ ❑ ❑ ❑

ANSWERS DISPOSITION

Answers: 1,4,5

❑ No intervention ❑ Follow-up ✔ Histopathologic diagnosis ❑

Discussion: ■ ■ ■ ■ ■

DERMOSCOPIC CRITERIA ■



■ ■





Irregular dots and globules (boxes) Irregular dark brown blotches (black arrows) Hypopigmentation (blue stars) Depigmented Halo (black stars) Follicular openings (yellow arrows) Perifollicular hypopigmentation (circles)







■ ■





The clinical diagnosis of a halo nevus is usually straightforward. Identify if pigmented component is low, intermediate, or high risk. In most but not all cases melanocytic lesion are low risk (eg, nevus). Halo phenomenon can be found around melanomas. In this case, centrally located criteria are atypical and raise a red flag. ■ Irregular dots and globules with an asymmetric location and irregular size and shape ■ Irregular dark brown blotches with an irregular size and shape Low risk criteria in a halo nevus might include a featureless pink color, homogeneous brown color with/or without regular dots and/or globules. The follicular openings and perifollicular hypopigmentation have no diagnostic significance. ■ Perifollicular hypopigmentation can be seen in congenital melanocytic nevi. Recurrent nevus or recurrent melanoma is in the differential diagnosis and the history is important. ■ A halo forming around a pre-existing pigmented skin lesion favors the diagnosis of a halo nevus. ■ The recurrence of pigmentation in a previously excised melanocytic lesion scar could be seen in a recurrent nevus or recurrent melanoma. ■ It is essential to review the original pathology of a recurrent pigmented skin lesion. Centrally-located pigmentation favors a benign process. Halo phenomenon with asymmetrical pigmentation especially if it extends outside of the halo, is a red flag that could represent recurrent melanoma. In this case a histopathologic diagnosis is indicated because of atypical centrally located criteria. To date a reverse dermatofibroma has not been reported.

PEARLS ■



New halo nevi or the development of vitiligo in a patient with a history of melanoma could be a sign of metastatic melanoma. A metastatic work-up is indicated even if the patient is asymptomatic.

Chapter 3

Trunk and Extremities

RISK ❑ Low ❑ Intermediate ❑ High

DIAGNOSIS

3-91a

❑ ❑ ❑ ❑ ❑ ❑ ❑ ❑

Nevus Seborrheic keratosis Basal cell carcinoma Vascular Dermatofibroma Squamous cell carcinoma Melanoma Other

DISPOSITION

3-91b

❑ No intervention ❑ Follow-up ❑ Histopathologic diagnosis

CASE 91 HISTORY A 24-year-old man had 10 melanocytic nevi on his back and this was one of them. It was clinically and dermoscopically an “ugly duckling” lesion. 1. This case is similar to Case 89 with regression and an irregular dark blotch. 2. With a multicomponent global pattern, an irregular dark blotch, and irregular globules, melanoma is in the differential diagnosis. 3. This lesion demonstrates symmetry of color and structure that favors low risk pathology. 4. This is a benign “fried egg” appearing nevus with a centrally located regular dark blotch and hypopigmentation. 5. This appears to be an “ugly duckling” lesion that warrants an immediate excision to rule out melanoma.

279

280

DERMOSCOPY: AN ILLUSTRATED SELF-ASSESSMENT GUIDE

RISK ✔ Low ❑ ❑ Intermediate ❑ High

1

2

DIAGNOSIS

3

Nevus Seborrheic keratosis Basal cell carcinoma Vascular Dermatofibroma Squamous cell carcinoma Melanoma Other

DISPOSITION ✔ No intervention ❑ ❑ Follow-up ❑ Histopathologic diagnosis

3-91c

✔ ❑ ❑ ❑ ❑ ❑ ❑ ❑ ❑

ANSWERS Answers: 3,4

Discussion: ■

DERMOSCOPIC CRITERIA ■



■ ■

■ ■

Symmetry of color and structure Multicomponent global pattern (1,2,3) Hypopigmentation (stars) Regular dark brown blotch (arrows) Regular brown dots (circle) White dots (boxes)







■ ■

Clinically and dermoscopically the lesion looks low risk: ■ Symmetry of color and structure ■ Absence of melanoma-specific criteria ■ One might consider the central dark blotch to be irregular. Most “fried egg” appearing nevi are not high risk: ■ Beware! Melanoma can have a “fried egg” clinical and/or dermoscopic appearance. ■ There should be melanoma-specific criteria. Hypopigmentation should not be confused with regression. ■ It is not bony-white. ■ It is light brown color. The focus of barely visible brown dots has no diagnostic significance and could be easily overlooked. ■ If one thinks they are globules that would be a criterion to diagnose a melanocytic lesion. ■ They are too small to be considered globules. ■ This is melanocytic by default. There are no melanoma-specific criteria. The white specks represent reflection artifact from the mineral oil used to take the image, and are not milia-like cysts.

PEARLS ■



“Ugly duckling” and “fried egg” appearing melanocytic nevi are not always high risk. Every case should be evaluated based on its own merits by a full evaluation of all of the clinical and dermoscopic criteria.

Chapter 3

Trunk and Extremities

RISK ❑ Low ❑ Intermediate ❑ High

DIAGNOSIS

3-92a

❑ ❑ ❑ ❑ ❑ ❑ ❑ ❑

Nevus Seborrheic keratosis Basal cell carcinoma Vascular Dermatofibroma Squamous cell carcinoma Melanoma Other

DISPOSITION

3-92b

❑ No intervention ❑ Follow-up ❑ Histopathologic diagnosis

CASE 92 HISTORY A 34-year-old woman noticed an enlarging skin lesion on her left shoulder. She waited 5 months before going to her dermatologist. 1. 2. 3. 4.

Globules identify a melanocytic lesion. There is asymmetry of color and structure, and a homogeneous global pattern. One cannot be sure whether there is true regression or hypopigmentation. The differential diagnosis includes nodular melanoma, atypical dermatofibroma, and hypopigmented seborrheic keratosis. 5. This could not be a nodular melanoma because the lesion is dark clinically and light dermoscopically.

281

282

DERMOSCOPY: AN ILLUSTRATED SELF-ASSESSMENT GUIDE

RISK ❑ Low ❑ Intermediate ✔ High ❑

❑ ❑ ❑ ❑ ❑ ❑ ✔ ❑ ❑

Nevus Seborrheic keratosis Basal cell carcinoma Vascular Dermatofibroma Squamous cell carcinoma Melanoma Other

3-92c

DIAGNOSIS

ANSWERS Answers: 1,2,3

DISPOSITION ❑ No intervention ❑ Follow-up ✔ Histopathologic diagnosis ❑

Discussion: ■



■ ■

DERMOSCOPIC CRITERIA ■

■ ■





Asymmetry of color and structure Homogeneous global pattern Irregular dots and globules (boxes) Irregular streaks (yellow arrows) Regression: ■ White color (yellow stars) ■ Grayish color (black stars)



■ ■





One does not need dermoscopy to realize that this is a high risk lesion that needs to be removed as soon as possible. There is not a good clinico–dermoscopic correlation, which should be a red flag for concern. ■ The lesion is dark clinically and hypopigmented dermoscopically. Irregular globules identify a melanocytic lesion. The exact description of the global pattern is debatable and is considered to be homogeneous because it is practically devoid of local criteria (eg, dots, globules, pigment network) and composed mainly of different homogeneous colors. ■ It could also be considered a multicomponent global pattern. The diffuse light color is a clue that the lesion could be high risk and has a differential diagnosis. ■ Hypopigmentation vs regression: ■ The gray color favors regression over hypopigmentation. However, it is not bony-white that favors hypopigmentation over regression. ■ Thus, one sees that local criteria cannot always be categorized with certainty. A focus of streaks is debatably present but is of no diagnostic significance. Even with a stretch of one’s imagination, there are no criteria to diagnose a dermatofibroma or seborrheic keratosis. Hypopigmented melanomas are more difficult to diagnose because there is usually a paucity of well-developed high risk criteria. Putting the history, and clinical and dermoscopic features together, one can only come up with one conclusion: This is a melanoma until proven otherwise and should be removed posthaste.

PEARLS ■

■ ■

Sequential digital monitoring of a potential nodular melanoma is contraindicated. Valuable time will be lost that could adversely affect the patients prognosis. This entire clinical scenario would be a perfect example of when one should consider removing a lesion at first consultation, so that valuable time is not lost and an unreliable patient is not lost to follow-up.

Chapter 3

Trunk and Extremities

RISK ❑ Low ❑ Intermediate ❑ High

DIAGNOSIS

3-93a

❑ ❑ ❑ ❑ ❑ ❑ ❑ ❑

Nevus Seborrheic keratosis Basal cell carcinoma Vascular Dermatofibroma Squamous cell carcinoma Melanoma Other

DISPOSITION

3-93b

❑ No intervention ❑ Follow-up ❑ Histopathologic diagnosis

CASE 93 HISTORY A 71-year-old man was concerned about this skin lesion on his left upper thigh. 1. Milia-like cysts can be seen clinically suggesting that this irregularly pigmented nodule is a seborrheic keratosis. 2. Asymmetry of color and structure, irregular dark globules, regression, and bluish-white color diagnose a nodular melanoma. 3. Multiple milia-like cysts and pseudofollicular openings are the main criteria to diagnose this seborrheic keratosis. 4. One large pigmented crypt also favors the diagnosis of a seborrheic keratosis. 5. The diffuse bluish-white color is pathognomonic for a melanoma.

283

284

DERMOSCOPY: AN ILLUSTRATED SELF-ASSESSMENT GUIDE

RISK ✔ Low ❑ ❑ Intermediate ❑ High

❑ ✔ ❑ ❑ ❑ ❑ ❑ ❑ ❑

Nevus Seborrheic keratosis Basal cell carcinoma Vascular Dermatofibroma Squamous cell carcinoma Melanoma Other

3-93c

DIAGNOSIS

ANSWERS Answers: 1,3,4

Discussion:

DISPOSITION ✔ No intervention ❑ ❑ Follow-up ❑ Histopathologic diagnosis

DERMOSCOPIC CRITERIA ■



■ ■





Asymmetry of color and structure Sharp border demarcation (black arrows) Milia-like cysts (white arrows) Pigmented pseudofollicular openings (yellow arrows) Nonpigmented pseudofollicular openings (blue arrows) Pigmented crypt (red arrows)

















Clinically, but not dermoscopically, this dark nodule is somewhat suspicious for a nodular melanoma. At times, dermoscopic criteria can be seen clinically and that clue helps point one in the direction of the correct diagnosis. ■ Some dermoscopic criteria that can be seen clinically include: ■ The white milia-like cysts of a seborrheic keratosis. ■ The dark streaks of a Spitz nevus. ■ Peripheral globules in a melanocytic nevus. This is a classic seborrheic keratosis with some, but not all, of the criteria used to make the diagnosis: ■ Sharp border demarcation ■ Multiple pigmented and nonpigmented ■ Multiple milia-like cysts pseudofollicular openings Crypts are large pseudofollicular openings which: ■ Vary in size and shape ■ Are not exclusively seen in seborrheic keratoses ■ Are also found in papillomatious melanocytic nevi Absent criteria used to make the diagnosis include: ■ Fissures and ridges, which can, but may not always, form a brain-like pattern ■ Fat-fingers ■ Hairpin vessels Milia-like cysts can be of an opaque white color or can be brilliant like shinning “stars in the sky.” ■ Different sizes of these roundish structures are the norm. Pseudofollicular openings are more irregular in size and shape and can be pigmented or nonpigmented. ■ Oxidation of sebaceous material in the epidermal invaginations results in a dark color, creating the pigmentation of the openings. The diffuse bluish-white color is a nonspecific finding that is not diagnostic of a high risk lesion (eg, melanoma).

PEARL ■

It is fun to be able to recognize dermoscopic criteria clinically. If you can do this, you can feel proud that you have taken your dermoscopic skills to a higher level. Congratulations!

Chapter 3

Trunk and Extremities

RISK ❑ Low ❑ Intermediate ❑ High

DIAGNOSIS 3-94a

❑ ❑ ❑ ❑ ❑ ❑ ❑ ❑

Nevus Seborrheic keratosis Basal cell carcinoma Vascular Dermatofibroma Squamous cell carcinoma Melanoma Other

DISPOSITION

3-94b

❑ No intervention ❑ Follow-up ❑ Histopathologic diagnosis

CASE 94 HISTORY A 23-year-old woman developed this new nodule that seemed to be growing rapidly. 1. 2. 3. 4.

Globules identify a melanocytic lesion. There is significant asymmetry of color and structure, a red flag for concern. Diffuse bluish- and bony-white colors along with peppering diagnoses regression. Irregular dots and globules, an irregular dark blotch, and polymorphous vessels are more criteria to help diagnose a melanoma. 5. If this was felt to be a benign lesion histopathologically, there would not be a good dermoscopic-pathologic correlation.

285

286

DERMOSCOPY: AN ILLUSTRATED SELF-ASSESSMENT GUIDE

RISK 1

❑ Low ❑ Intermediate ✔ High ❑

DIAGNOSIS

2

Nevus Seborrheic keratosis Basal cell carcinoma Vascular Dermatofibroma Squamous cell carcinoma Melanoma Other

DISPOSITION ❑ No intervention ❑ Follow-up ✔ Histopathologic diagnosis ❑

3

3

3-94c

❑ ❑ ❑ ❑ ❑ ❑ ✔ ❑ ❑

ANSWERS Answers: 1,2,3,4,5

Discussion: ■



DERMOSCOPIC CRITERIA ■











Asymmetry of color and structure Multicomponent global pattern (1,2,3) Irregular dots and globules (black boxes) Irregular dark brown blotch (yellow arrows) Polymorphous vessels (red boxes) Regression ■ Bluish-white color (stars) ■ Peppering (yellow boxes)







Clinically and dermoscopically, this is highly suspicious for a nodular melanoma: a rapidly growing nodule with dramatic melanoma-specific criteria. It would be very difficult to misdiagnose this as a benign lesion. Melanoma-specific criteria are dramatically present: ■ Asymmetry of color and structure ■ Multicomponent global pattern ■ There is so much going on that it is hard to divide the lesion into three or more separate components, which is a minor point in the context of the entire atypical picture: ■ Irregular dots and globules ■ Irregular dark brown blotch ■ Regression The polymorphous vessels are hard to see, but at any rate represent only a minor criterion in the context of the overall atypical picture. There are no differential diagnostic possibilities here other than a bad melanoma!

PEARLS ■





Even experienced clinicians could miss this case if their eyes and minds are closed to the benefits of dermoscopy. ■ It is remarkable how many of the patients we have seen with a history of melanoma have been told by one or several dermatologists that their melanomas were nothing to worry about! Invariably, they were never examined with dermoscopy. Dermoscopy should be like the seat belts in a car. One should not leave home without using them! Once you have begun to incorporate dermoscopy into your daily practice, you will fully understand this point: It is impossible to feel comfortable practicing without it.

Chapter 3

Trunk and Extremities

RISK ❑ Low ❑ Intermediate ❑ High

3-95a

DIAGNOSIS ❑ ❑ ❑ ❑ ❑ ❑ ❑ ❑

Nevus Seborrheic keratosis Basal cell carcinoma Vascular Dermatofibroma Squamous cell carcinoma Melanoma Other

DISPOSITION

3-95b

❑ No intervention ❑ Follow-up ❑ Histopathologic diagnosis

CASE 95 HISTORY This lesion was found on the back of a 19-year-old woman who frequented tanning parlors. 1. 2. 3. 4. 5.

Pigment network identifies a melanocytic lesion. The global pattern is reticular. There are foci of irregular pigment network, and irregular dots and globules. Hypopigmentation not regression fills the lesion. There are enough melanoma-specific criteria to diagnose a melanoma.

287

288

DERMOSCOPY: AN ILLUSTRATED SELF-ASSESSMENT GUIDE

RISK ❑ Low ✔ Intermediate ❑ ❑ High

DIAGNOSIS Nevus Seborrheic keratosis Basal cell carcinoma Vascular Dermatofibroma Squamous cell carcinoma Melanoma Other

3-95c

✔ ❑ ❑ ❑ ❑ ❑ ❑ ❑ ❑

ANSWERS Answers: 1,2,3,4

DISPOSITION

Discussion:

❑ No intervention ❑ Follow-up ✔ Histopathologic diagnosis ❑







DERMOSCOPIC CRITERIA ■ ■







Reticular global pattern Asymmetry of color and structure Irregular pigment network (boxes) Irregular dots and globules (circles) Multifocal hypopigmentation (stars)







This is a mildly dysplastic nevus without enough criteria to diagnose a melanoma. This is a classic example of a dysplastic nevus: ■ Asymmetry of color and structure ■ Foci of irregular pigment network and irregular dots and globules ■ No greater than two colors ■ Multifocal hypopigmentation The global pattern is reticular because most of the lesion is filled with pigment network. ■ Whatever the global pattern is, criteria must be determined as regular or irregular. ■ In this case the pigment network is irregular without a uniform honeycomb pattern. It is thickened, branched, and broken up. ■ The dots and globules are of different sizes and shapes and asymmetrically located, making them irregular. Hypopigmentation that fills the lesion should not be confused with regression. ■ It is light tan color not bony-white. An experienced dermoscopist might consider this a low risk lesion that does not need a histopathologic diagnosis. ■ Sequential digital monitoring would be an alternative to excision. ■ With sequential monitoring, most nevi that look like this do not change. ■ Of those that do change, most, but not all, remain low risk. ■ All nevi that change do not need to be removed if there are no high risk criteria. If this were felt to be a melanoma histopathologically, there would not be a good clinico–dermoscopic–pathologic correlation and another dermatopathologist’s opinion should be obtained.

PEARLS ■



One of the major benefits of dermoscopy is avoiding unnecessary surgery as would be the case here with an experienced dermoscopist. A novice might decide a lesion like this needs a histopathologic diagnosis. ■ With experience, this clinical scenario will look less and less worrisome to you.

Chapter 3

Trunk and Extremities

RISK ❑ Low ❑ Intermediate ❑ High

DIAGNOSIS

3-96a

❑ ❑ ❑ ❑ ❑ ❑ ❑ ❑

Nevus Seborrheic keratosis Basal cell carcinoma Vascular Dermatofibroma Squamous cell carcinoma Melanoma Other

DISPOSITION

3-96b

❑ No intervention ❑ Follow-up ❑ Histopathologic diagnosis

CASE 96 HISTORY A 61-year-old man noticed this lesion on his left shoulder. 1. 2. 3. 4. 5.

Clinically and dermoscopically, this is more worrisome than the last case. This looks more malignant than benign but it is not a clear cut melanoma. There is a large regular dark blotch with regular dots and globules. There is an irregular dark blotch and irregular dots and globules. The differential diagnosis includes a dysplastic nevus or in situ melanoma.

289

290

DERMOSCOPY: AN ILLUSTRATED SELF-ASSESSMENT GUIDE

RISK

1

❑ Low ❑ Intermediate ✔ High ❑

2 2

3

DIAGNOSIS Nevus Seborrheic keratosis Basal cell carcinoma Vascular Dermatofibroma Squamous cell carcinoma Melanoma Other

3 2

3

3-96c

✔ ❑ ❑ ❑ ❑ ❑ ❑ ❑ ❑

ANSWERS Answers: 1,2,4,5

Discussion:

DISPOSITION ❑ No intervention ❑ Follow-up ✔ Histopathologic diagnosis ❑

DERMOSCOPIC CRITERIA ■









■ ■ ■

Asymmetry of color and structure Multicomponent global pattern (1, 2, 3) Regular pigment network (boxes) Irregular brown dots and globules (circles) Irregular dark brown blotch (white arrows) Bluish-white color (yellow star) Hypopigmentation (black stars) Normal skin (blue stars)

■ ■









Pigment network and globules identify a melanocytic lesion. Even though the pigment network in Case 95 is more widespread and irregular, the global picture here is more atypical. ■ The irregular dark brown blotch is much larger, darker with a suggestion of bluish color in the center. ■ The dots and globules are larger, darker, and numerous. ■ Both lesions have hypopigmentation with regression in the differential diagnosis. In this case, there is clear cut hypopigmentation with lighter shades of brown but also a much whiter area. It is questionable if the white area is true regression because it is not whiter than the surrounding skin. ■ Not uncommon, one can see normal skin within a melanocytic lesion, which appears to be the case here. A dark blotch by definition should be devoid of local criteria. That does not always hold true, as seen in this case because there are foci or irregular globules. This dysplastic nevus appears more malignant than benign, with a dysplastic nevus and in situ melanoma in the differential diagnosis. There are criteria in favor of a superficial vs invasive melanoma: ■ Flat lesion clinically ■ Color associated with a superficial location (eg, shades of brown) ■ Absence of criteria associated with invasive melanoma (eg, raised or nodular lesion, 5 or 6 colors including pink and blue, polymorphous vessels).

PEARLS ■ ■



One can see a progression of atypical features in Cases 95 and 96. It is essential to be able to create a differential diagnosis of global patterns and local criteria. With experience, one will have mental images of past cases to draw on when dealing with real life clinical situations. ■ You will remember specific global dermoscopic patterns or specific patterns of local criteria with their histopathologic diagnoses, and realize that the lesion you are looking at looks similar to the past lesions. ■ The more one uses dermoscopy, the larger a mental data base will be created → Experience!

Chapter 3

Trunk and Extremities

RISK ❑ Low ❑ Intermediate ❑ High

DIAGNOSIS

3-97a

❑ ❑ ❑ ❑ ❑ ❑ ❑ ❑

Nevus Seborrheic keratosis Basal cell carcinoma Vascular Dermatofibroma Squamous cell carcinoma Melanoma Other

DISPOSITION

3-97b

❑ No intervention ❑ Follow-up ❑ Histopathologic diagnosis

CASE 97 HISTORY This solitary lesion was found on the back of a 40-year-old woman. 1. 2. 3. 4. 5.

Regular pigment network and regular globules identify a melanocytic lesion. There is symmetry of color and structure and a reticular global pattern. Multifocal hypopigmentation and regular pigment network diagnose a dysplastic nevus. The pigment network, dots and globules, and dark blotches are all irregular. Regression with milky-red globules and the other melanoma-specific criteria diagnose an invasive melanoma.

291

292

DERMOSCOPY: AN ILLUSTRATED SELF-ASSESSMENT GUIDE

RISK

1

❑ Low ❑ Intermediate ✔ High ❑

2 3

DIAGNOSIS

4

Nevus Seborrheic keratosis Basal cell carcinoma Vascular Dermatofibroma Squamous cell carcinoma Melanoma Other

DISPOSITION ❑ No intervention ❑ Follow-up ✔ Histopathologic diagnosis ❑

5 5

3-97c

❑ ❑ ❑ ❑ ❑ ❑ ✔ ❑ ❑

ANSWERS Answers: 4,5

Discussion: ■

DERMOSCOPIC CRITERIA ■









■ ■



Asymmetry of color and structure Multicomponent global pattern (1,2,3,4,5) Irregular pigment network (boxes) Irregular dots and globules (black circles) Irregular dark brown blotches (arrows) Regression (stars) Milky-red globules (yellow circles) Milia-like cysts (yellow box)







■ ■





Irregular, not regular, pigment network and irregular globules diagnose a melanocytic lesion. ■ The pigment network is thickened, branched, and in some areas broken up. ■ The dots and globules are of different sizes and shapes, dark and asymmetrically located in the lesion. ■ They are larger than in Cases 95 and 96. Compared to Cases 95 and 96, with similar criteria, the global picture is more irregular. There are, however, features that are less irregular: ■ The irregular brown blotches are much smaller compared to those in Case 96. Bluish-white color with foci of milky-red globules fills the lesion and represents the invasive component of this melanoma. ■ The milky-red globules could be considered pinpoint vessels. A pinkish hue can also be appreciated. The two milia-like cysts have no diagnostic significance. ■ Milia-like cysts are not exclusively found in seborrheic keratoses and can be found in benign and malignant melanocytic lesions. One could say with near certainty that this is a melanoma. ■ Never tell a patient that they have melanoma 100% because a very atypical historical, clinical, and dermoscopic picture could turn out to be benign. A dysplastic nevus is not in the differential diagnosis.

PEARLS ■ ■

This is an easy melanoma to diagnose. If you had trouble, are unsure of yourself, or missed it completely, go directly to jail. Do not pass. Go and do not collect $100!

Chapter 3

Trunk and Extremities

RISK ❑ Low ❑ Intermediate ❑ High

DIAGNOSIS

3-98a

❑ ❑ ❑ ❑ ❑ ❑ ❑ ❑

Nevus Seborrheic keratosis Basal cell carcinoma Vascular Dermatofibroma Squamous cell carcinoma Melanoma Other

DISPOSITION

3-98b

❑ No intervention ❑ Follow-up ❑ Histopathologic diagnosis

CASE 98 HISTORY A 3-year-old was referred for evaluation of a darkening lesion on his right shin. 1. 2. 3. 4. 5.

Irregular pigment network and irregular globules identify a melanocytic lesion. The global pattern is “Spitzoid”. There are foci of irregular streaks at the periphery. Regular streaks surround the lesion at the periphery. Asymmetry of color and structure, irregular pigment network, dots, and streaks diagnose a nodular melanoma.

293

294

DERMOSCOPY: AN ILLUSTRATED SELF-ASSESSMENT GUIDE

RISK ❑ Low ✔ Intermediate ❑ ❑ High

DIAGNOSIS Nevus Seborrheic keratosis Basal cell carcinoma Vascular Dermatofibroma Squamous cell carcinoma Melanoma Other

3-98c

✔ ❑ ❑ ❑ ❑ ❑ ❑ ❑ ❑

ANSWERS DISPOSITION

Answers: 1,2,4

❑ No intervention ❑ Follow-up ✔ Histopathologic diagnosis ❑

Discussion: ■





DERMOSCOPIC CRITERIA ■

■ ■





Symmetry of color and structure “Spitzoid “ global pattern Irregular pigment network (white boxes) Irregular dots and globules (red circles) Regular streaks (arrows)







A dark lesion developing in a child is more likely to be a Spitz nevus than melanoma. Clinically, one can appreciate streaks at the periphery before using dermoscopy. Irregular pigment network and irregular globules identify a melanocytic lesion. ■ They are, however, symmetrically located within the lesion. ■ They intermingle with each other, which has no diagnostic significance. The global pattern is “Spitzoid”. ■ This is the black pigment network/reticular variant. ■ Any of the six global patterns suggestive of a Spitz nevus are referred to as having a “Spitzoid” global pattern: ■ Starburst ■ Black pigment network/reticular ■ Globular ■ Homogeneous ■ The pink and nonspecific patterns often cannot be diagnosed as being “Spitzoid”. The streaks are considered to be regular because they symmetrically surround the lesion at the periphery. ■ Shape does not determine if streaks are regular or irregular. ■ Foci of streaks at the periphery define them as being irregular. Dermoscopically, the diagnosis is clear with the caveat that a small percentage of symmetrical “Spitzoid” patterns turns out to be melanoma. ■ A new and changing black lesion in a patient of any age, even with a symmetrical “Spitzoid” pattern, is a red flag for concern and should be removed.

PEARLS ■





Be confident and reassure your patient and/or their parents that this will turn out in most cases to be a benign Spitz nevus. Spend time with the patient for a good explanation of what a Spitz nevus is, to allay anxiety. Sequential digital follow-up of a “Spitzoid” lesion has been reported, is fool hearted, puts the patient at risk, and is contraindicated.

Chapter 3

Trunk and Extremities

RISK ❑ Low ❑ Intermediate ❑ High

DIAGNOSIS

3-99a

❑ ❑ ❑ ❑ ❑ ❑ ❑ ❑

Nevus Seborrheic keratosis Basal cell carcinoma Vascular Dermatofibroma Squamous cell carcinoma Melanoma Other

DISPOSITION

3-99b

❑ No intervention ❑ Follow-up ❑ Histopathologic diagnosis

CASE 99 HISTORY A 71-year-old woman was not sure how long this lesion has been present on her back. 1. 2. 3. 4. 5.

Regular pigment network identifies a melanocytic lesion. Moth-eaten borders and the fingerprint pattern diagnose a solar lentigo. An irregular black blotch plus irregular pigment network diagnose a small nodular melanoma. The black blotch is regular and created by a “black lamella.” The “black lamella” is created by pigmented parakeratosis and is not associated with melanoma.

295

296

DERMOSCOPY: AN ILLUSTRATED SELF-ASSESSMENT GUIDE

RISK ✔ Low ❑ ❑ Intermediate ❑ High

DIAGNOSIS Nevus Seborrheic keratosis Basal cell carcinoma Vascular Dermatofibroma Squamous cell carcinoma Melanoma Other

DISPOSITION ✔ No intervention ❑ ❑ Follow-up ❑ Histopathologic diagnosis

3-99c

✔ ❑ ❑ ❑ ❑ ❑ ❑ ❑ ❑

ANSWERS Answers: 1,4,5

Discussion: ■

DERMOSCOPIC CRITERIA ■







Symmetry of color and structure Homogeneous-reticular global pattern Regular pigment network (boxes) Regular black blotch (star)

■ ■









The global pattern is reticular-homogeneous: ■ Reticular created by the pigment network. ■ Homogeneous created by the regular black blotch. This is a nice example of global symmetry of color and structure. The pigment network symmetrically surrounds the lesion and is not considered to be irregular. ■ It looks the same throughout the lesion. ■ The fingerprint pattern has parallel, not honeycomb-like line segments, which make up the pigment network. The black blotch is considered to be regular: ■ It has a uniform oval shape and symmetrically fills most of the lesion. The black blotch is also shinny, and is referred to as a “black lamella” created by pigmented parakeratosis. Tape stripping usually, but not always, results in specks of the “black lamella” sticking to the tape. Melanoma is not in the differential diagnosis of this symmetrical black lesion. ■ A black lesion or black color, seen with dermoscopy, is not always high risk. ■ Trans-epidermal elimination of melanin dermoscopically looks black; however, it is not high risk histopathologically.

PEARLS ■



Patients are impressed with tape stripping especially if specks of the “black lamella” stick to the tape. Play it up, your patients will think you are a genius and the best dermatologist in town! You might get a referral or two.

Chapter 3

Trunk and Extremities

RISK ❑ Low ❑ Intermediate ❑ High

DIAGNOSIS

3-100a

❑ ❑ ❑ ❑ ❑ ❑ ❑ ❑

Nevus Seborrheic keratosis Basal cell carcinoma Vascular Dermatofibroma Squamous cell carcinoma Melanoma Other

DISPOSITION

3-100b

❑ No intervention ❑ Follow-up ❑ Histopathologic diagnosis

CASE 100 HISTORY This “ugly duckling” black spot was found on the back of a 79-year-old woman. 1. Globules identify a melanocytic lesion. 2. Multiple milia-like cysts and pigmented pseudofollicular openings diagnose a seborrheic keratosis. 3. Irregular dark blotches and regression diagnose a melanoma. 4. The irregular dark blotches represent the hyperpigmentation typically found in seborrheic keratosis. 5. Hyperpigmentation is rarely found in seborrheic keratosis.

297

298

DERMOSCOPY: AN ILLUSTRATED SELF-ASSESSMENT GUIDE

RISK ✔ Low ❑ ❑ Intermediate ❑ High

DIAGNOSIS Nevus Seborrheic keratosis Basal cell carcinoma Vascular Dermatofibroma Squamous cell carcinoma Melanoma Other 3-100c

❑ ✔ ❑ ❑ ❑ ❑ ❑ ❑ ❑

DISPOSITION ✔ No intervention ❑ ❑ Follow-up ❑ Histopathologic diagnosis

ANSWERS Answers: 2,4

Discussion: ■

DERMOSCOPIC CRITERIA ■ ■ ■





Sharp border demarcation Milia-like cysts (white arrows) Pigmented pseudofollicular openings (yellow arrows) Dots and globular-like structures (boxes) Hyperpigmentation (stars)









There is an absence of criteria to diagnose a melanocytic lesion. Multiple well-developed milia-like cysts and poorly developed pigmented pseudofollicular openings diagnose a seborrheic keratosis. The dots and globular-like structures seen at the periphery of the lesion and in the surrounding skin are not part of the lesion. Some of the pigmented pseudofollicular openings look like the brown globules of a melanocytic lesion. ■ It is not always possible to perfectly categorize local criteria, hence the need for dermoscopic differential diagnosis. The irregular dark blotches represents hyperpigmentation commonly found in seborrheic keratosis.

PEARLS ■ ■

The multiple well-developed milia-like cysts clinch the diagnosis. For the less-experienced dermoscopist, always remember, “If in doubt, cut it out!”

Chapter 3

Trunk and Extremities

RISK ❑ Low ❑ Intermediate ❑ High

DIAGNOSIS

3-101a

❑ ❑ ❑ ❑ ❑ ❑ ❑ ❑

Nevus Seborrheic keratosis Basal cell carcinoma Vascular Dermatofibroma Squamous cell carcinoma Melanoma Other

DISPOSITION

3-101b

❑ No intervention ❑ Follow-up ❑ Histopathologic diagnosis

CASE 101 HISTORY A 66-year-old man had this long-standing asymptomatic lesion on his chest. 1. Clinically and dermoscopically, this is similar to the last case especially with the regular blotch that fills most of the lesion. 2. There is symmetry of color and structure, a homogeneous global pattern with regular dots and globules, and a regular blotch. 3. Multifocal hypopigmentation plus an irregular pigment network diagnose a dysplastic nevus. 4. There is a good clinico–dermoscopic correlation because this dark nodule looks like a melanoma and there are well-developed melanoma-specific criteria. 5. Irregular dots and globules, an irregular dark blotch, and bluish-white color suggest that this could be a melanoma.

299

300

DERMOSCOPY: AN ILLUSTRATED SELF-ASSESSMENT GUIDE

RISK ❑ Low ❑ Intermediate ✔ High ❑

2

DIAGNOSIS Nevus Seborrheic keratosis Basal cell carcinoma Vascular Dermatofibroma Squamous cell carcinoma Melanoma Other

DISPOSITION ❑ No intervention ❑ Follow-up ✔ Histopathologic diagnosis ❑

2 3

1

4

ANSWERS Answers: 4,5

Discussion: ■



DERMOSCOPIC CRITERIA ■











Asymmetry of color and structure Multicomponent global pattern (1,2,3,4) Irregular brown dots and globules (circles) Irregular dark brown blotch (white arrows) Multifocal hypopigmentation (stars) Bluish-white color (white box)

3-101c

❑ ❑ ❑ ❑ ❑ ❑ ✔ ❑ ❑

1









Irregular globules diagnose a melanocytic lesion. ■ There is no pigment network. Global melanoma-specific criteria include: ■ Asymmetry of color and structure ■ A multicomponent global pattern Local melanoma-specific criteria include: ■ Irregular brown dots and globules ■ Irregular dark brown blotch ■ Bluish-white color There is multifocal hypopigmentation, which is more commonly associated with dysplastic nevi. The dermoscopic picture in this case is diametrically opposed to Case 99. ■ Symmetry of color and structure vs asymmetry of color and structure. ■ A flat vs nodular lesion. ■ An absence of melanoma-specific criteria vs well-developed melanomaspecific criteria. There is a good clinico–dermoscopic–pathologic correlation because clinically, dermoscopically, and histopathologically this is a melanoma.

PEARLS ■ ■

There is no pearl in this straightforward case. It is a nice study of contrasts between good and evil!

Chapter 3

Trunk and Extremities

RISK ❑ Low ❑ Intermediate ❑ High

DIAGNOSIS

3-102a

❑ ❑ ❑ ❑ ❑ ❑ ❑ ❑

Nevus Seborrheic keratosis Basal cell carcinoma Vascular Dermatofibroma Squamous cell carcinoma Melanoma Other

DISPOSITION

3-102b

❑ No intervention ❑ Follow-up ❑ Histopathologic diagnosis

CASE 102 HISTORY The patient’s dermatologist followed this lesion for 3 years and said it was a benign seborrheic keratosis. It was never examined with dermoscopy. The patient came for a second opinion because his wife was worried, thinking it could be a skin cancer. 1. Multiple milia-like cysts and pigmented pseudofollicular openings diagnose a seborrheic keratosis. 2. Hairpin vessels and fat-fingers clinch the diagnosis of a seborrheic keratosis. 3. Irregular globules and a large irregular dark blotch suggest this is a melanoma. 4. This could be a collision tumor consisting of a melanoma and a seborrheic keratosis. 5. Melanoma as part of a collision tumor has never been reported.

301

302

DERMOSCOPY: AN ILLUSTRATED SELF-ASSESSMENT GUIDE

RISK ❑ Low ❑ Intermediate ✔ High ❑

1

2

DIAGNOSIS 2

Nevus Seborrheic keratosis Basal cell carcinoma Vascular Dermatofibroma Squamous cell carcinoma Melanoma Other

DISPOSITION ❑ No intervention ❑ Follow-up ✔ Histopathologic diagnosis ❑

3

3-102c

❑ ✔ ❑ ❑ ❑ ❑ ❑ ✔ ❑ ❑

ANSWERS Answers: 3,4

Discussion: ■



DERMOSCOPIC CRITERIA ■











Asymmetry of color and structure Multicomponent global pattern (1,2,3) Irregular brown globules (yellow boxes) Irregular dark black blotch (white stars) Regression ■ White color (black stars) ■ Peppering (black box) Seborrheic keratosis (black arrows) ■ Milia-like cysts (yellow arrows) ■ Pigmented pseudofollicular openings (white arrows)

■ ■





Clinically, this dark nodule with an eccentric bony-white halo is very suspicious for a melanoma. The verrucous component suggests this could be a collision tumor. ■ The lesion looks like a seborrheic keratosis with milia-like cysts and pigmented pseudofollicular openings or hyperpigmentation. ■ This is by far not a classic seborrheic keratosis. Poorly defined irregular brown globules identify a melanocytic lesion. The global melanoma-specific criteria include: ■ Asymmetry of color and structure. ■ A multicomponent global pattern. ■ Compared to Case 94 in which the zones of the multicomponent global pattern overlap, here they are very sharply demarcated. The local melanoma-specific criteria include: ■ Poorly developed irregular brown globules. ■ A well-developed irregular black blotch. ■ Regression composed of bony-white color and peppering. ■ The regression is more obvious clinically than dermoscopically. Melanoma as part of a collision tumor is a known phenomenon. ■ Melanoma and seborrheic keratosis is one of the most common associations.

PEARLS ■



When there is a possibility that you are dealing with a collision tumor, alert your pathologist so that they will know to search for both pathologies. In a collision tumor that does not contain a potential melanoma, if it is possible, submit two specimens in separate formalin containers to help make the diagnosis.

Chapter 3

Trunk and Extremities

RISK ❑ Low ❑ Intermediate ❑ High

DIAGNOSIS

3-103a

❑ ❑ ❑ ❑ ❑ ❑ ❑ ❑

Nevus Seborrheic keratosis Basal cell carcinoma Vascular Dermatofibroma Squamous cell carcinoma Melanoma Other

DISPOSITION

3-103b

❑ No intervention ❑ Follow-up ❑ Histopathologic diagnosis

CASE 103 HISTORY This lesion was found on the chest of a 65-year-old man. 1. This is a melanocytic lesion by default. 2. The bluish-white color is a clue that this dark nodule is a melanoma. 3. There is a paucity of superficially located local criteria that puts a nodular melanoma at the top of the differential diagnosis list. 4. There is an irregular dark blotch which is not sharply demarcated. 5. Ovoid nests of bluish-white color and a focus of milky-red color diagnose a pigmented basal cell carcinoma.

303

304

DERMOSCOPY: AN ILLUSTRATED SELF-ASSESSMENT GUIDE

RISK ❑ Low ❑ Intermediate ✔ High ❑

1

DIAGNOSIS ❑ ❑ ❑ ❑ ❑ ❑ ✔ ❑ ❑

1

Nevus Seborrheic keratosis Basal cell carcinoma Vascular Dermatofibroma Squamous cell carcinoma Melanoma Other

2

3 4

1

DISPOSITION 3-103c

❑ No intervention ❑ Follow-up ✔ Histopathologic diagnosis ❑

ANSWERS Answers: 1,2,3,4

DERMOSCOPIC CRITERIA ■





■ ■ ■ ■

Asymmetry of color and structure Multicomponent global pattern (1,2,3,4) Irregular dark brown blotch (white arrows) Bluish-white color (white stars) Regression (black stars) Milky-red area (yellow box) Five colors

Discussion: ■ ■









This is melanocytic by default. Clinically and dermoscopically this is a good example of a nodular melanoma. The melanoma-specific criteria are well developed: ■ Asymmetry of color and structure ■ Regression ■ Multicomponent global pattern ■ Milky-red area ■ Irregular dark brown blotch ■ Five colors ■ Bluish-white color The irregular dark blotch correlates clinically with what appears to be ulceration. ■ Ulceration was reported histopathologically. Typically, nodular melanomas have a paucity of superficially located local criteria (eg, pigment network, brown dots, and globules). Compared with the last two cases, this is the end of the line of how bad a primary melanoma can be!

PEARLS ■





The goal of performing periodic total body skin examinations utilizing dermoscopy is to prevent melanoma from getting this bad. It is not the luck of the Irish if one presents with an ulcerated deep nodular melanoma. It is one of life’s disasters that have terrible consequences for the patient and their loved ones. Every clinician’s motivation should be to be as good as they can be every minute of the working day, to prevent this from ever happening.

Chapter 3

Trunk and Extremities

RISK ❑ Low ❑ Intermediate ❑ High

DIAGNOSIS

3-104a

❑ ❑ ❑ ❑ ❑ ❑ ❑ ❑

Nevus Seborrheic keratosis Basal cell carcinoma Vascular Dermatofibroma Squamous cell carcinoma Melanoma Other

DISPOSITION

3-104b

❑ No intervention ❑ Follow-up ❑ Histopathologic diagnosis

CASE 104 HISTORY This was found on the upper back of a 67-year-old woman. 1. 2. 3. 4. 5.

Clinically, there is a semi-translucent quality suggesting this could be a basal cell carcinoma. This cannot be a basal cell carcinoma because there are no arborizing vessels or ulceration. The presence of spoke-wheel structures suggests this is a basal cell carcinoma. Irregular streaks and regression diagnose a hypomelanotic melanoma. Spoke-wheel structures always have a perfect set of lines radiating from the central hub.

305

306

DERMOSCOPY: AN ILLUSTRATED SELF-ASSESSMENT GUIDE

RISK ❑ Low ❑ Intermediate ✔ High ❑

1

2

DIAGNOSIS Nevus Seborrheic keratosis Basal cell carcinoma Vascular Dermatofibroma Squamous cell carcinoma Melanoma Other

DISPOSITION ❑ No intervention ❑ Follow-up ✔ Histopathologic diagnosis ❑

3

3-104c

❑ ❑ ✔ ❑ ❑ ❑ ❑ ❑ ❑

ANSWERS Answers: 1,3

Discussion: ■



DERMOSCOPIC CRITERIA ■



■ ■



Asymmetry of color and structure Multicomponent global pattern (1,2,3) Brown pigmentation (circles) Blue ovoid pigmentation (red arrow) Spoke-wheel structures ■ Central hub (yellow arrows) ■ Spokes (black arrows)









A focused skin examination with a high index of suspicion and dermoscopy are needed to find and diagnose a small basal cell carcinoma like this one. The suggestion of the presence of atypical spoke-wheel structures was the first clue that this could be a basal cell carcinoma. Classic spoke-wheel structures are made up of: ■ A central pigmented hub that looks like a globule. ■ Fine pigmented lines that radiate from all points of the hub (like the spokes of a bicycle wheel). In this case, the central hubs are irregular, the spokes look like thickened streaks, and they do not radiate from all points of the central hub. ■ Often one has to use their imagination to identify atypical spoke-wheel structures. ■ It is not uncommon to find multiple well-developed spoke-wheel structures. One blue ovoid area of pigmentation associated with pigmented basal cell carcinomas appears to have three spoke-like structures radiating from it. Arborizing vessels and ulceration are not needed to make the diagnosis.

PEARLS ■





Inter-observer agreement is not always good even among experienced dermoscopists. Some might not think there are spoke-wheel structures in this lesion. We identified the spoke-wheel structures immediately, and based on that made the correct dermoscopic diagnosis before the lesion was removed. With knowledge and imagination, there is no telling where dermoscopy will take you!

Chapter 3

Trunk and Extremities

RISK ❑ Low ❑ Intermediate ❑ High

3-105a

DIAGNOSIS ❑ ❑ ❑ ❑ ❑ ❑ ❑ ❑

Nevus Seborrheic keratosis Basal cell carcinoma Vascular Dermatofibroma Squamous cell carcinoma Melanoma Other

DISPOSITION

3-105b

❑ No intervention ❑ Follow-up ❑ Histopathologic diagnosis

CASE 105 HISTORY White color developed around several nevi on a 19-year-old woman. 1. Globules identify a melanocytic lesion. 2. This could represent the fissure and ridge pattern of a halo seborrheic keratosis. 3. Asymmetry of color and structure, irregular dots and globules, and regression diagnose a melanoma. 4. This could be a halo nevus with a benign cobblestone global pattern. 5. A wide diameter halo around a melanocytic lesion is diagnostic of melanoma.

307

308

DERMOSCOPY: AN ILLUSTRATED SELF-ASSESSMENT GUIDE

RISK ✔ Low ❑ ❑ Intermediate ❑ High

DIAGNOSIS Nevus Seborrheic keratosis Basal cell carcinoma Vascular Dermatofibroma Squamous cell carcinoma Melanoma Other

DISPOSITION ✔ No intervention ❑ ❑ Follow-up ❑ Histopathologic diagnosis

3-105c

✔ ❑ ❑ ❑ ❑ ❑ ❑ ❑ ❑

ANSWERS Answers: 1,2,4

Discussion: ■ ■

DERMOSCOPIC CRITERIA ■ ■



Cobblestone global pattern Regular dots and globules (boxes) Regression (stars)



Brown globules identify a melanocytic lesion. Large angulated globules with a few dots make up the cobblestone global pattern. ■ The fissure and ridge pattern of a seborrheic keratosis is in the differential diagnosis of a cobblestone global pattern. This is a halo nevus with an unusually wide halo. ■ The size of a white halo has no diagnostic significance and does not diagnose melanoma. ■ Rarely, you may see halo seborrheic keratosis.

PEARLS ■



Sequential digital clinical and dermoscopic monitoring is a cutting-edge way to follow this patient. ■ Halo nevi usually disappear over a variable period of time. The “wobble sign” created by pressure from instrumentation directly on a lesion and side-to-side movement can be used to differentiate a seborrheic keratosis from a melanocytic nevus. ■ A seborrheic keratosis is usually fixed to the skin and is immobile → a negative “wobble sign.” ■ An elevated nevus usually moves easily from side-to-side → a positive “wobble sign.”

Chapter 3

Trunk and Extremities

RISK ❑ Low ❑ Intermediate ❑ High

3-106a

DIAGNOSIS ❑ ❑ ❑ ❑ ❑ ❑ ❑ ❑

Nevus Seborrheic keratosis Basal cell carcinoma Vascular Dermatofibroma Squamous cell carcinoma Melanoma Other

DISPOSITION

3-106

❑ No intervention ❑ Follow-up ❑ Histopathologic diagnosis

CASE 106 HISTORY This lesion was found on the knee of an 82-year-old man. There could be a history of trauma while working in his yard. However, the patient was not sure. 1. Clinically and dermoscopically this is consistent with an abrasion and topical antibiotics should be recommended. 2. If you miss the focus of brown globules, the homogeneous brown color suggests this red nodule could be a melanocytic lesion. 3. Different shades of pink color are a red flag for concern. 4. Pinpoint vessels and milky-red globules suggest this could be an amelanotic melanoma. 5. This feature-poor lesion does not have well-developed melanoma-specific criteria.

309

310

DERMOSCOPY: AN ILLUSTRATED SELF-ASSESSMENT GUIDE

RISK

1

❑ Low ❑ Intermediate ✔ High ❑ 2

3

DIAGNOSIS Nevus Seborrheic keratosis Basal cell carcinoma Vascular Dermatofibroma Squamous cell carcinoma Melanoma Other

4 3-106c

❑ ❑ ❑ ❑ ❑ ❑ ✔ ❑ ❑

DISPOSITION

ANSWERS

❑ No intervention ❑ Follow-up ✔ Histopathologic diagnosis ❑

Answers: 2,3,4,5

Discussion: ■



DERMOSCOPIC CRITERIA ■







■ ■

■ ■ ■

Asymmetry of color and structure Multicomponent global pattern (1,2,3,4) Irregular brown dots and globules (black box) Homogeneous brown color (yellow arrows) Different shades of pink color Milky-red globules (black arrows) Pinpoint vessels (yellow boxes) Ulceration (white box) Hypopigmentation (stars)











Clinically, but not dermoscopically, an infectious process is in the differential diagnosis. ■ Dermoscopic criteria for infectious lesions have not been well studied. A focus of irregular brown globules identifies a melanocytic lesion. ■ If the globules are not identified, this could be considered to be melanocytic by default. The focus of homogeneous brown color suggests the lesion could be melanocytic. ■ Amelanotic melanoma, devoid of the typical well-developed, melanomaspecific criteria seen in pigmented melanomas, often have foci of homogeneous brown pigmentation at the periphery. Different shades of pink color are an important yet nonspecific clue that this lesion is an amelanotic melanoma. ■ These can be seen in melanocytic, nonmelanocytic, benign, malignant, or inflammatory pink lesions. Foci of milky-red globules in a background of pink color are another important clue that this could be an amelanotic melanoma. ■ The pinpoint vessels could be considered milky-red globules. The differentiation is a minor point because both types of vessels indicate the lesion might be high risk. Hypopigmentation is created by the pressure of the camera. ■ In most pink lesions pink blanches away with adequate pressure. The small ulceration has no diagnostic significance.

PEARLS ■





Finding as many clues as possible might be the only way to diagnose amelanotic melanoma. It would be a rare lesion that is 100% featureless. Most have vessels that can be identified with minimal pressure or the use of ultrasound or hand sanitizer gel. Polarizing light is better than nonpolarizing for visualizing blood vessels.

Chapter 3

Trunk and Extremities

RISK ❑ Low ❑ Intermediate ❑ High

DIAGNOSIS

3-107a

❑ ❑ ❑ ❑ ❑ ❑ ❑ ❑

Nevus Seborrheic keratosis Basal cell carcinoma Vascular Dermatofibroma Squamous cell carcinoma Melanoma Other

DISPOSITION ❑ No intervention ❑ Follow-up ❑ Histopathologic diagnosis

CASE 107 HISTORY You were e-mailed this case for your dermoscopic evaluation. No clinical image was sent. It was a firm papule on the right shin of a 22-year-old woman. She was not aware of its presence. 1. Brown globules identify a melanocytic lesion. 2. Fine peripheral pigment network and a central white patch diagnose a dermatofibroma. 3. The central white patch is composed of homogeneous bony-white color and reticular depigmentation. 4. The globular-like structures are created by the reticular depigmentation over a background of dark brown color. 5. Reticular depigmentation is diagnostic of a dermatofibroma.

311

312

DERMOSCOPY: AN ILLUSTRATED SELF-ASSESSMENT GUIDE

RISK ✔ Low ❑ ❑ Intermediate ❑ High

DIAGNOSIS Nevus Seborrheic keratosis Basal cell carcinoma Vascular Dermatofibroma Squamous cell carcinoma Melanoma Other

DISPOSITION ✔ No intervention ❑ ❑ Follow-up ❑ Histopathologic diagnosis

3-107b

❑ ❑ ❑ ❑ ✔ ❑ ❑ ❑ ❑

ANSWERS Answers: 2,3,4

Discussion: ■



DERMOSCOPIC CRITERIA ■





Regular pigment network (black boxes) Central white patch ■ Homogeneous bony-white color (stars) ■ Reticular depigmentation (white boxes) Globular-like structures (yellow arrows)









A clinical image is not needed to help make the diagnosis. The history and dermoscopic criteria are sufficient. This is not the classic central white patch that is made up of centrally-located, homogeneous, bony-white color, characteristic of a dermatofibroma. Reticular depigmentation (negative or white network) is made up of sharply in-focus honeycomb-like or reticular bony-white line segments. ■ The thickness, extent of the line segments, and intensity of the color varies from lesion to lesion. Not uncommonly, reticular depigmentation makes up the central white patch of a dermatofibroma. The fine, thin peripheral pigment network is one of the major criteria to diagnose dermatofibromas. ■ This is the exception to the rule that pigment network diagnoses a melanoctyic lesion. ■ Dermatofibromas are not melanocytic, and pigment network helps make their diagnosis. The supposedly well-developed brown globules are not what they appear to be. They are created by reticular depigmentation over a background of homogeneous dark brown color. ■ The novice dermoscopist could be easily led astray and consider them to be the brown globules of a melanocytic lesion.

PEARL ■

Beware! While not the case here, reticular depigmentation/white network might be the only clue that a pigmented lesion is a melanoma!

Chapter 3

Trunk and Extremities

RISK ❑ Low ❑ Intermediate ❑ High

DIAGNOSIS

3-108a

❑ ❑ ❑ ❑ ❑ ❑ ❑ ❑

Nevus Seborrheic keratosis Basal cell carcinoma Vascular Dermatofibroma Squamous cell carcinoma Melanoma Other

DISPOSITION

3-108b

❑ No intervention ❑ Follow-up ❑ Histopathologic diagnosis

CASE 108 HISTORY At a routine follow-up skin examination, this small pigmented lesion was found on the shin of a 30-year-old woman. 1. There is not a good clinico–dermoscopic correlation because the lesion looks low risk based on ABCD clinical criteria, yet with dermoscopy it looks suspicious. 2. Globules identify a melanocytic lesion. 3. A large irregular dark blotch and reticular depigmentation diagnose a dermatofibroma. 4. A large irregular dark blotch, irregular dots and globules, and reticular depigmentation suggest this is a melanoma. 5. Sequential digital monitoring is indicated for this gray zone slightly suspicious pigmented skin lesion.

313

314

DERMOSCOPY: AN ILLUSTRATED SELF-ASSESSMENT GUIDE

RISK ❑ Low ❑ Intermediate ✔ High ❑

DIAGNOSIS Nevus Seborrheic keratosis Basal cell carcinoma Vascular Dermatofibroma Squamous cell carcinoma Melanoma Other

1 2 3

4

3-108c

❑ ❑ ❑ ❑ ❑ ❑ ✔ ❑ ❑

ANSWERS DISPOSITION ❑ No intervention ❑ Follow-up ✔ Histopathologic diagnosis ❑

Answers: 1,2,4

Discussion: ■

■ ■

DERMOSCOPIC CRITERIA ■











Asymmetry of color and structure Multicomponent global pattern (1,2,3,4) Irregular brown dots and globules (circles) Irregular dark brown blotch (yellow arrows) Reticular depigmentation (yellow boxes) Hypopigmentation (stars)







This is a melanoma- incognito aka false negative clinical melanoma that would be missed if it were not examined with dermoscopy. ■ It lacks the ABCD clinical criteria and looks low risk. Irregular brown globules identify a melanocytic lesion. Some of the melanoma-specific criteria are well-developed and include: ■ Asymmetry of color and structure ■ Multicomponent global pattern with four different areas ■ A large irregular dark blotch ■ Reticular depigmentation There are poorly defined irregular dots and globules and hypopigmentation both of which are of minor diagnostic significance. ■ Section 4 could be an area of regression. Reticular depigmentation (negative/white network) is an important clue that a lesion might be melanoma. Reticular depigmentation can be found in: ■ Banal nevi ■ Dermatofibromas ■ Dysplastic nevi ■ Melanoma ■ Spitz nevi

PEARLS ■







To improve your dermoscopic skills, it is best to have your instrumentation handy at all times. ■ What a shame for you and more importantly your patient, to miss being able to examine an interesting lesion because for one reason or another you do not have dermoscopic instrumentation! Always keep in mind that the most banal appearing lesion could be melanoma-incognito. Dermoscopy should not only be used to examine clinically atypical lesions but also to gather a reference base from typical lesions, which is important. Refrain from outbursts of surprise when you find a strikingly bad dermoscopic picture. It will scare your patient; it is unprofessional yet instinctive, often uncontrollable!

Chapter 3

Trunk and Extremities

RISK ❑ Low ❑ Intermediate ❑ High

DIAGNOSIS 3-109a

❑ ❑ ❑ ❑ ❑ ❑ ❑ ❑

Nevus Seborrheic keratosis Basal cell carcinoma Vascular Dermatofibroma Squamous cell carcinoma Melanoma Other

DISPOSITION

3-109b

❑ No intervention ❑ Follow-up ❑ Histopathologic diagnosis

CASE 109 HISTORY This new lesion was found on the anterior thigh of a 19-year-old tanning parlor enthusiast. 1. Pigment network and globules identify a melanocytic lesion. 2. There is a significant asymmetry of color and structure and the multicomponent global pattern. 3. Some of the high risk criteria are easy to see while others are not. 4. The irregular dots and globules, irregular black blotches, and bluish-white color are easy to see. 5. Irregular pigment network, irregular streaks, and regression are not that easy to see.

315

316

DERMOSCOPY: AN ILLUSTRATED SELF-ASSESSMENT GUIDE

RISK ❑ Low ❑ Intermediate ✔ High ❑

1

DIAGNOSIS Nevus Seborrheic keratosis Basal cell carcinoma Vascular Dermatofibroma Squamous cell carcinoma Melanoma Other

DISPOSITION ❑ No intervention ❑ Follow-up ✔ Histopathologic diagnosis ❑

2 3

ANSWERS Answers: 1,2,3,4,5

Discussion: ■ ■

DERMOSCOPIC CRITERIA ■





■ ■



■ ■ ■

Asymmetry of color and structure Multicomponent global pattern (1,2,3) Irregular pigment network (yellow box) Irregular streaks (arrows) Irregular brown dots and globules (white boxes) Irregular black blotches (yellow stars) Bluish-white color (white stars) Regression (gray stars) Five colors

1

3-109c

❑ ❑ ❑ ❑ ❑ ❑ ✔ ❑ ❑

1





Clinically and dermoscopically, this lesion is high risk and easy to diagnose. Most of the high risk criteria are easy to identify: ■ Asymmetry of color and structure ■ Multicomponent global pattern ■ Each zone has several local criteria within it. ■ Irregular brown dots and globules ■ Irregular black blotches ■ Bluish-white color There is one focus of irregular pigment network with some streaks coming off it at the periphery. ■ There are irregular streaks scattered around at the periphery. Regression is seen clinically and with dermoscopy but it is not clear cut and could be missed.

PEARLS ■

If you were insecure about the analysis of this lesion at this point in the book: ■ Review Chapter 1. ■ Find a colleague who has experience to help you. ■ Find a dermoscopy meeting to attend ASAP. ■ Don’t give up, the learning curve is steep yet not insurmountable.

Chapter 3

Trunk and Extremities

RISK ❑ Low ❑ Intermediate ❑ High

DIAGNOSIS 6 month follow-up

12 month follow-up

18 month follow-up

❑ ❑ ❑ ❑ ❑ ❑ ❑ ❑ 3-110a

Baseline

Nevus Seborrheic keratosis Basal cell carcinoma Vascular Dermatofibroma Squamous cell carcinoma Melanoma Other

DISPOSITION ❑ No intervention ❑ Follow-up ❑ Histopathologic diagnosis

CASE 110 HISTORY This is one of several nevi that were being followed with digital dermoscopy to look for important changes over time. 1. Asymmetry of color and structure, the multicomponent global pattern, a very irregular pigment network, and regression in the baseline image diagnose an in situ melanoma. The lesion should have been removed at that time. 2. Asymmetry of color and structure, a reticular-homogeneous global pattern, a mildly irregular pigment network, different shades of brown color, and multifocal hypopigmentation diagnose a mildly dysplastic nevus. 3. A side-by-side comparison of the baseline image with the same lesion at 6, 12, and 18 months shows that the lesion is basically without change. 4. The lesion has been changing slowly over time with high risk criteria in the 18-month followup image. 5. Significant dermoscopic changes over time include: asymmetrical enlargement, the presence of new melanoma-specific criteria, or new colors.

317

318

DERMOSCOPY: AN ILLUSTRATED SELF-ASSESSMENT GUIDE

RISK

DIAGNOSIS ✔ ❑ ❑ ❑ ❑ ❑ ❑ ❑ ❑

❑ No intervention ✔ Follow-up ❑ ❑ Histopathologic diagnosis





12 month follow-up

18 month follow-up

ANSWERS Answers: 2,3,5

Discussion: ■

DERMOSCOPIC CRITERIA



6 month follow-up

Nevus Seborrheic keratosis Basal cell carcinoma Vascular Dermatofibroma Squamous cell carcinoma Melanoma Other

DISPOSITION



Baseline

Asymmetry of color and structure Reticular-homogeneous global pattern Irregular pigment network (all colored boxes) Multifocal hypopigmentation (all colored stars)

3-110b

✔ Low ❑ ❑ Intermediate ❑ High







The criteria in the baseline image diagnose a mildly dysplastic nevus and include: ■ Asymmetry of color and structure ■ Reticular-homogeneous global pattern: ■ Mildly irregular pigment network creating the reticular component ■ Multifocal hypopigmentation not regression creating the homogeneous component ■ Light and dark brown color There are no significant melanoma-specific criteria. ■ In this case, the asymmetry of color and structure and slightly irregular pigment network are not high risk. A side-by-side comparison of the baseline image and the lesion at 6, 12, and 18 months show that they are essentially without significant changes. ■ If you compare the different-colored boxes and stars, the changes are minimal. For long-term digital dermoscopic monitoring every 6 to 12 months, potentially significant changes would include: ■ Asymmetrical enlargement ■ Asymmetrical changes within the lesion without enlargement ■ The presence of new melanoma-specific criteria (eg, irregular pigment network, dots, globules, streaks, blotches, regression) ■ Disappearance of well-developed local criteria ■ New colors

PEARLS ■ ■ ■



Most nevi that change over time are not high risk. All nevi that change do not need to be removed. Beware! There are melanomas that do not appear to be high risk clinically or with dermoscopy. They are only diagnosed after monitoring and finding dermoscopic changes over time when comparing baseline with subsequent digital images. Consider adding digital dermoscopy to your practice.

Chapter 3

Baseline

Trunk and Extremities

6 month fol

RISK

3-111a

❑ Low ❑ Intermediate ❑ High

DIAGNOSIS ❑ ❑ ❑ ❑ ❑ ❑ ❑ ❑

Nevus Seborrheic keratosis Basal cell carcinoma Vascular Dermatofibroma Squamous cell carcinoma Melanoma Other

3-111b

DISPOSITION ❑ No intervention ❑ Follow-up ❑ Histopathologic diagnosis

CASE 111 HISTORY A 43-year-old man returned for a routine 6-month follow-up digital dermoscopic examination. He had a baseline database of 15 nevi that were being followed. He performed monthly self-skin examinations and did not report any new, changing, or symptomatic lesions. 1. The baseline image does not have enough criteria to diagnose a melanoma. There is mild asymmetry of color and structure, foci of minimally irregular pigment network, different shades of brown color, and a focus of reticular depigmentation. 2. A side-by-side comparison of the baseline and follow-up image demonstrates that the lesion has not changed at all. 3. A side-by-side comparison of the baseline and follow-up image demonstrates symmetrical enlargement without any new high risk criteria. 4. There has been a dramatic asymmetrical enlargement that is a red flag for concern. Without well-developed melanoma-specific criteria continued monitoring is indicated. 5. Even though the lesion does not have well-developed, melanoma-specific criteria, the change itself is significant and an indication for an excision.

319

320

DERMOSCOPY: AN ILLUSTRATED SELF-ASSESSMENT GUIDE

Baseline

6 month follow-up

RISK ❑ Low ❑ Intermediate ✔ High ❑

❑ ❑ ❑ ❑ ❑ ❑ ✔ ❑ ❑

Nevus Seborrheic keratosis Basal cell carcinoma Vascular Dermatofibroma Squamous cell carcinoma Melanoma Other

3-111c

DIAGNOSIS

ANSWERS Answers: 1,5

Discussion:

DISPOSITION ❑ No intervention ❑ Follow-up ✔ Histopathologic diagnosis ❑





DERMOSCOPIC CRITERIA: BASELINE IMAGE ■







Asymmetry of color and structure Homogeneous-reticular global pattern Irregular pigment network (red box) Reticular depigmentation (white arrows)







The baseline image does not have enough criteria to diagnose a high risk dysplastic nevus or melanoma. There is only mild asymmetry of color and structure, a minimally irregular pigment network, and a small focus of reticular depigmentation. A side-by-side comparison of the baseline image and the same nevus, 6 months later, demonstrates significant and dramatic changes. ■ The shape has asymmetrically expanded and it looks like a completely different lesion. ■ When there is such a dramatic change, make sure that you are comparing the same lesions. ■ The focus of irregular pigment network has expanded. ■ The hyperpigmentation within the lesion has expanded and is darker. ■ The hyperpigmentation is not well-demarcated and cannot be considered an irregular blotch. Even though the lesion has changed significantly, if it were examined by itself, it would not have enough criteria to diagnose a high risk dysplastic nevus or melanoma. The focus of reticular depigmentation in the baseline image is no longer seen in the follow-up image and is of no diagnostic significance. ■ Reticular depigmentation should always be a red flag for concern but that does not mean that it is always found in high risk lesions. The overall changes are significant and warrant a histopathologic diagnosis to rule out melanoma-incognito.

PEARLS ■

This case points out the benefits of digital dermoscopic follow-up examination. ■ The patient was not aware of the change. ■ The change was only found by digital-dermoscopic follow-up. ■ The dermoscopic changes were significant even though the lesion by itself could have been overlooked as not being high risk with an absence of well-developed melanoma-specific criteria. ■ There are cases when melanoma can only be diagnosed by finding dermoscpic changes over time.

Chapter 3

Trunk and Extremities

RISK

Baseline

6 month follow-up

3-112a

❑ Low ❑ Intermediate ❑ High

DIAGNOSIS ❑ ❑ ❑ ❑ ❑ ❑ ❑ ❑

Nevus Seborrheic keratosis Basal cell carcinoma Vascular Dermatofibroma Squamous cell carcinoma Melanoma Other

DISPOSITION

3-112b

❑ No intervention ❑ Follow-up ❑ Histopathologic diagnosis

CASE 112 HISTORY This is a follow-up of another nevus on the back of the patient from the last case. 1. The baseline image looks low risk with mildly irregular features. 2. A side-by-side comparison of the baseline and follow-up image demonstrates symmetrical darkening and that the lesion has expanded in size. 3. Even though the lesion is the same size at follow-up, there is asymmetrical darkening within the lesion. 4. At a higher magnification, one can see in the follow-up lesion asymmetry of color and structure, foci of irregular pigment network, a few irregular globules, bluish-white color, and multifocal hypopigmentation. 5. The changes are enough to warrant an excision yet the lesion might still not be a melanoma.

321

322

DERMOSCOPY: AN ILLUSTRATED SELF-ASSESSMENT GUIDE

RISK ❑ Low ✔ Intermediate ❑ ❑ High

DIAGNOSIS ✔ ❑ ❑ ❑ ❑ ❑ ❑ ❑ ❑

Nevus Seborrheic keratosis Basal cell carcinoma Vascular Dermatofibroma Squamous cell carcinoma Melanoma Other

DISPOSITION 3-112c

❑ No intervention ❑ Follow-up ✔ Histopathologic diagnosis ❑

ANSWERS DERMOSCOPIC CRITERIA: FOLLOW-UP IMAGE ■

■ ■







Asymmetry of color and structure Reticular global pattern Irregular pigment network (black boxes) Irregular globules (yellow boxes) Multifocal hypopigmentation (stars) Bluish-white color

Answers: 1,3,4,5

Discussion: ■





● ●

You can clearly see that even though the size is the same at follow-up, the nevus has changed: ■ Asymmetrical darkening At a higher magnification you can see the criteria to diagnose a mildly dysplastic nevus: ■ Asymmetry of color and structure ■ A few irregular globules ■ Foci of irregular pigment network ■ Multifocal hypopigmentation The bluish-white color has no diagnostic significance. The changes are significant and warrant a histopathologic diagnosis. There is a good dermoscopic–pathologic correlation with the diagnosis of a mildly dysplastic nevus. ■ The follow-up lesion looks more benign than malignant. ■ There are not enough criteria to diagnose a high grade dysplastic nevus or melanoma.

PEARLS ■



Experienced dermoscopists might still feel comfortable monitoring this mildly dysplastic nevus. Beware! There is not always a good correlation with the dermoscopic criteria to differentiate mild, moderate, or severely dysplastic nevi. ■ Nevi with very irregular criteria might be diagnosed as being mildly dysplastic nevi and feature-poor lesions might actually be severely dysplastic nevi.

Chapter 3

Trunk and Extremities

RISK ❑ Low ❑ Intermediate ❑ High

Baseline

6 month follow-up

3-113a

DIAGNOSIS ❑ ❑ ❑ ❑ ❑ ❑ ❑ ❑

Nevus Seborrheic keratosis Basal cell carcinoma Vascular Dermatofibroma Squamous cell carcinoma Melanoma Other

DISPOSITION ❑ No intervention ❑ Follow-up ❑ Histopathologic diagnosis

CASE 113 HISTORY This is the 6-month follow-up of a nevus on the abdomen of a 30-year-old man. 1. The criteria in the baseline image diagnose a mildly dysplastic nevus. 2. A side-by-side comparison of the baseline and follow-up image demonstrates symmetrical enlargement. 3. A side-by-side comparison of the baseline and follow-up image demonstrates an asymmetrical enlargement, asymmetrical darkening, and a diffuse pinkish hue. 4. The differential diagnosis of the lesion at the 6-month follow-up includes: a moderate to severely dysplastic nevus or melanoma arising in a dysplastic nevus. 5. The changes are significant and warrant a histopathologic diagnosis.

323

324

DERMOSCOPY: AN ILLUSTRATED SELF-ASSESSMENT GUIDE

RISK ❑ Low ❑ Intermediate ✔ High ❑

❑ ❑ ❑ ❑ ❑ ❑ ✔ ❑ ❑

Nevus Seborrheic keratosis Basal cell carcinoma Vascular Dermatofibroma Squamous cell carcinoma Melanoma Other

Baseline

6 month follow-up

3-113b

DIAGNOSIS

ANSWERS Answers: 1,3,4,5

Discussion: ■

DISPOSITION ❑ No intervention ❑ Follow-up ✔ Histopathologic diagnosis ❑





DERMOSCOPIC CRITERIA: BASELINE IMAGE ■



■ ■

Asymmetry of color and structure Homogeneous-reticular global pattern Irregular pigment network Multifocal hypopigmentation

Once again, the baseline image is not atypical enough to warrant an excision, yet not perfectly banal appearing. This is an indication for sequential monitoring. ■ It looks more benign than malignant, a “gray zone” lesion. The lesion has changed significantly over time. ■ Expansion of the right side of the lesion (black arrows). ■ Asymmetrical expansion of the centrally located hyperpigmentation (white arrows). Histopathologically, this was felt to be an in situ melanoma arising in a dysplastic nevus, which is a good dermoscopic–pathologic correlation.

PEARLS ■







Critics of sequential digital dermoscopic monitoring might argue that with this technique, you could miss a melanoma at the initial examination. Studies have shown that most melanomas that are diagnosed with sequential digital dermoscopic monitoring are either in situ or early invasive lesions. ■ In one’s career, everyone misses or misdiagnoses melanoma with or without special diagnostic techniques. Sequential digital dermoscopic monitoring allows you to follow patients with multiple nevi over time and helps to avoid unnecessary surgery. Yet, it facilitates the early diagnosis of obvious melanomas or melanoma-incognito. Digital dermoscopy takes your skills to the highest level and might be the only way to diagnose melanoma that is 100% curable!

Chapter 4

Palms, Soles, Nails

General Instructions: You will find a list of True/False statements following each case history. Select any statements, which you believe to be true. There may be one, more than one or no true statements for any given case. Choose the correct risk, diagnosis and disposition for each case. Then, turn the page to find a detailed discussion and pearls for each case.

Chapter 4

Palms, Soles, Nails

RISK ❑ Low ❑ Intermediate ❑ High

4-1a

DIAGNOSIS ❑ ❑ ❑ ❑ ❑ ❑ ❑ ❑

Nevus Seborrheic keratosis Basal cell carcinoma Vascular Dermatofibroma Squamous cell carcinoma Melanoma Other

DISPOSITION

4-1b

❑ No intervention ❑ Follow-up ❑ Histopathologic diagnosis

CASE 1 HISTORY The parents of this 5-year-old girl noticed a dark spot on her palm. 1. There is asymmetry of color and structure and the malignant parallel ridge pattern (PRP). 2. The parallel brown lines are in the furrows of the skin diagnosing the malignant parallel furrow pattern. 3. The parallel brown lines are in the furrows of the skin diagnosing the benign parallel furrow pattern. 4. The parallel furrow pattern is the most common benign pattern found on glabrous skin. 5. The parallel furrow pattern is the most common benign pattern found on nonglabrous skin.

327

328

DERMOSCOPY: AN ILLUSTRATED SELF-ASSESSMENT GUIDE

RISK ✔ Low ❑ ❑ Intermediate ❑ High

✔ ❑ ❑ ❑ ❑ ❑ ❑ ❑ ❑

Nevus Seborrheic keratosis Basal cell carcinoma Vascular Dermatofibroma Squamous cell carcinoma Melanoma Other

4-1c

DIAGNOSIS

ANSWERS Answers: 3, 4

Discussion:

DISPOSITION ❑ No intervention ✔ Follow-up ❑ ❑ Histopathologic diagnosis

■ ■





DERMOSCOPIC CRITERIA ■



Symmetry of color and structure Parallel furrow pattern ■ Furrows (white arrows) ■ Ridges (yellow arrows) ■ Acrosyringia (black arrows)





There is perfect symmetry of color and structure. This nevus is on the volar surface of the hand (ie, glabrous skin). ■ Glabrous skin is nonhair-bearing skin. The unique anatomy of the skin on the palms and soles with its distinctive dermatoglyphic pattern is made up of ridges (gyri) and furrows (sulci or fissures) that create parallel site-specific patterns. ■ Parallel patterns can also be found on mucosal surfaces. The parallel brown lines are in the furrows of the skin creating the benign parallel furrow pattern. ■ This is the most common benign pattern seen on glabrous skin. ■ There is no such entity as the malignant parallel furrow pattern. ■ The ridges and furrows can be seen clinically, which is a clue to the dermoscopic pattern. The lighter, hypopigmented parallel lines represent the ridges of the skin: ■ When pigmentation is found in the ridges that creates the parallel ridge pattern (PRP), which is very sensitive and specific yet not completely diagnostic for acral melanoma. Acrosyringia are: ■ The intra-epidermal section of eccrine sweat gland ducts (“eccrine pores”). ■ Always in the ridges and never in the furrows of the skin. ■ Appear as centrally located uniform white dots with a linear arrangement. ■ They are said to look like “a string of pearls.”

PEARLS ■







Digital sequential clinical and dermoscopic monitoring is a cutting-edge way to follow such a lesion for changes over time that might warrant a histopathologic diagnosis in the future. Patients and/or their parents are always very impressed when digital dermoscopic images are taken for follow-up purposes. It is cutting-edge: Why shouldn’t they be? When trying to remember whether pigment in ridges or furrows represents a malignant or benign pattern, think only that “furrows are fine!” Dermoscopy helps to prevent surgical intervention in most, but not all, acral pigmented skin lesions especially in children!

Chapter 4

Palms, Soles, Nails

RISK ❑ Low ❑ Intermediate ❑ High

DIAGNOSIS

4-2a

❑ ❑ ❑ ❑ ❑ ❑ ❑ ❑

Nevus Seborrheic keratosis Basal cell carcinoma Vascular Dermatofibroma Squamous cell carcinoma Melanoma Other

DISPOSITION

4-2b

❑ No intervention ❑ Follow-up ❑ Histopathologic diagnosis

CASE 2 HISTORY This lesion was found on the foot of a 43-year-old woman. 1. This is the benign parallel furrow pattern, double-line variant. 2. Irregular dots and globules, irregular thickened lines, and wide ridges are a red flag for concern. 3. There are variations of the parallel furrow pattern, which can contain a single line of pigment in the furrows or two parallel lines on each side of the furrows with or without dots and globules. 4. The parallel furrow pattern could be made up completely of a linear arrangement of dots and globules. 5. The parallel furrow pattern always contains a single row of pigmentation in the furrows.

329

330

DERMOSCOPY: AN ILLUSTRATED SELF-ASSESSMENT GUIDE

RISK ✔ Low ❑ ❑ Intermediate ❑ High

DIAGNOSIS Nevus Seborrheic keratosis Basal cell carcinoma Vascular Dermatofibroma Squamous cell carcinoma Melanoma Other

4-2c

✔ ❑ ❑ ❑ ❑ ❑ ❑ ❑ ❑

DISPOSITION

ANSWERS

❑ No intervention ✔ Follow-up ❑ ❑ Histopathologic diagnosis

Answers: 1,3,4

Discussion: ■



DERMOSCOPIC CRITERIA ■



■ ■

Symmetry of color and structure Parallel furrow pattern ■ Double lines in the furrows (black lines) ■ Singe lines in furrows (yellow arrows) Ridges (black arrows) Globules (circles)







Clinically, but not dermoscopically, this lesion looks similar to that in Case 1 except that it has the double line parallel furrow pattern. The classic parallel furrow pattern is made up of a single brown line in the skin furrows (“singles line variant”). However, there are several commonly encountered variations of this benign pattern. This is the double-line variant with parallel brown lines on both sides of the hypopigmented furrows. ■ There are also a few globules. One can see dots and globules in combination with linear pigmentation or dots and globules by themselves. ■ A single row of dots and globules in the furrows without linear pigmentation (“the single-dotted line variant”). ■ Two rows of dots and globules on both sides of hypopigmented furrows without linear pigmentation (“the double-dotted line variant”). The width of the ridges has no diagnostic significance in this case. ■ Irregularly pigmented and thickened ridges can be seen in acral melanoma.

PEARLS ■



It is essential to be aware of all of the variations that can be seen in the benign parallel furrow pattern, so that they are not mistaken for high risk lesions resulting in unnecessary surgical procedures. It is not necessary to remember the specific names for the variants, just remember that the pigmentation has to be in the furrows (“furrows are fine”) and there can be one or two lines with or without dots and globules, or just dots and globules alone.

Chapter 4

Palms, Soles, Nails

RISK ❑ Low ❑ Intermediate ❑ High

4-3a

DIAGNOSIS ❑ ❑ ❑ ❑ ❑ ❑ ❑ ❑

Nevus Seborrheic keratosis Basal cell carcinoma Vascular Dermatofibroma Squamous cell carcinoma Melanoma Other

DISPOSITION ❑ No intervention ❑ Follow-up ❑ Histopathologic diagnosis

CASE 3 HISTORY This lesion was found on the foot of a 6-year-old boy and looked clinically similar to the last two cases. 1. Asymmetry of color and structure, irregular globules, and irregular hyperpigmentation diagnose a melanoma. 2. This is a variant of the benign parallel furrow pattern with brown globules and linear hyperpigmentation in the skin furrows. 3. Irregularities of the parallel furrow pattern are routinely encountered and are not high risk. 4. Irregularities of the parallel furrow pattern warrant immediate excision. 5. If in doubt, cut it out!

331

332

DERMOSCOPY: AN ILLUSTRATED SELF-ASSESSMENT GUIDE

RISK ✔ Low ❑ ❑ Intermediate ❑ High

DIAGNOSIS Nevus Seborrheic keratosis Basal cell carcinoma Vascular Dermatofibroma Squamous cell carcinoma Melanoma Other

DISPOSITION ❑ No intervention ✔ Follow-up ❑ ❑ Histopathologic diagnosis

4-3b

✔ ❑ ❑ ❑ ❑ ❑ ❑ ❑ ❑

ANSWERS Answers: 2,3,5

Discussion: ■

DERMOSCOPIC CRITERIA ■

Parallel furrow pattern Globules (white boxes) ■ Skin furrows (black arrows) ■ Hyperpigmentation (stars) Skin ridges (yellow arrows) Lattice-like pattern (white lines)





■ ■







The global picture in this case is more irregular than in the last two cases. However, it is a variant of the benign parallel furrow pattern with foci of the lattice-like pattern. ■ The young age of the patient is also important and favors a benign acral nevus. ■ Acral lentiginous melanoma in a child is extremely improbable. The parallel furrow pattern in this case is made up of: ■ Linear pigmentation in the furrows. ■ Brown globules in the furrows impinging into the hypopigmented ridges. ■ Irregular hyperpigmentation in the furrows impinging into the ridges. ■ Criteria impinging into the ridges are commonly seen. They do not diagnose the parallel ridge pattern and are not associated with melanoma. Lattice-like pattern: ■ Is made up of brown lines in the furrows and brown lines running perpendicular to the furrows creating a ladder-like appearance. ■ Refer to Figure 1-5 in Chapter 1 to see a classic lattice like pattern. ■ There are three complete foci of this pattern with some of the globules trying to form the rungs of a ladder in other areas. It is not uncommon to have a combination of benign parallel patterns in one lesion—as long as there are no criteria associated with melanoma, the combinations in most cases still represent benign nevi. The wide hypopigmented ridges are easily identified. In this case there are no acrosyringia. ■ Ridges can be with or without acrosyringia.

PEARLS ■ ■

There are innumerable variations of the benign parallel furrow pattern. At times, the global picture in acral nevi is not clear cut. If this happens, and if you have to think too long about what is going on, fall back on the very important principle: If in doubt, cut it out.

Chapter 4

Palms, Soles, Nails

RISK ❑ Low ❑ Intermediate ❑ High

4-4a

DIAGNOSIS ❑ ❑ ❑ ❑ ❑ ❑ ❑ ❑

Nevus Seborrheic keratosis Basal cell carcinoma Vascular Dermatofibroma Squamous cell carcinoma Melanoma Other

DISPOSITION

4-4b

❑ No intervention ❑ Follow-up ❑ Histopathologic diagnosis

CASE 4 HISTORY A 24-year-old man was concerned about a pigmented lesion on his left foot. 1. This is the classic parallel furrow pattern with pigmentation in the skin furrows. 2. This is a variation of the morphology that can be seen with the lattice-like pattern with pigmentation in the furrows and running perpendicular to the furrows. 3. Acrosyringia are clearly seen in the furrows. 4. Acrosyringia are clearly seen in the ridges and on normal skin. 5. One does not see acrosyringia on normal skin.

333

334

DERMOSCOPY: AN ILLUSTRATED SELF-ASSESSMENT GUIDE

RISK ✔ Low ❑ ❑ Intermediate ❑ High

DIAGNOSIS Nevus Seborrheic keratosis Basal cell carcinoma Vascular Dermatofibroma Squamous cell carcinoma Melanoma Other

DISPOSITION ❑ No intervention ✔ Follow-up ❑ ❑ Histopathologic diagnosis

4-4c

✔ ❑ ❑ ❑ ❑ ❑ ❑ ❑ ❑

ANSWERS Answers: 2,4

Discussion: ■

DERMOSCOPIC CRITERIA ■



■ ■

Asymmetry of color and structure Lattice-like pattern ■ Furrows (black arrows) ■ Perpendicular lines (yellow arrows) Ridges (red arrows) Acrosyringia ■ Inside lesion (black boxes) ■ Outside lesion (red boxes)





The benign lattice-like pattern is more developed in this nevus compared to the last case, with multiple pigmented linear lines in the furrows and pigmented lines running between furrows. ■ Refer to Figure 1-5 in Chapter 1 to see a classic lattice-like pattern. This is a variation of the morphology than can be seen in the lattice-like pattern and falls within the range of a benign acral nevus. ■ There are only foci of pigmentation within the furrows and pigmented lines coming off them. ■ The angles of the rungs of the ladder in this case are not all 90°. They are not all perpendicular and do not have to be perpendicular to be within the range of the criteria to diagnose the lattice-like pattern. Multiple white dots representing eccrine pores (acrosyringia) are seen in a linear and irregular distribution in the lesion and in the surrounding skin. ■ Acrosyringia are commonly seen on the palms and soles of normal skin, the normal anatomy of the palms and soles.

PEARLS ■ ■

Variations of benign patterns can be found in acral lentiginous melanoma. Make sure that you do not miss criteria associated with melanoma before diagnosing a benign acral nevus.

Chapter 4

Palms, Soles, Nails

RISK ❑ Low ❑ Intermediate ❑ High

4-5a

DIAGNOSIS ❑ ❑ ❑ ❑ ❑ ❑ ❑ ❑

Nevus Seborrheic keratosis Basal cell carcinoma Vascular Dermatofibroma Squamous cell carcinoma Melanoma Other

DISPOSITION

4-5b

❑ No intervention ❑ Follow-up ❑ Histopathologic diagnosis

CASE 5 HISTORY A nevus on the left fifth finger of a 15-year-old girl increased in size over the past year and itches. 1. Asymmetry of color and structure plus bluish-white color diagnose a nodular melanoma. 2. Foci of the parallel furrow pattern suggest that this could be a benign lesion. 3. Both a benign parallel furrow pattern and criteria to diagnose acral lentiginous melanoma can be seen in the same lesion. 4. This is the dotted parallel ridge pattern with bluish-white color and irregular acrosyringia, diagnosing a nodular melanoma. 5. Criteria associated with melanomas seen on nonglabrous skin can also be found on glabrous skin.

335

336

DERMOSCOPY: AN ILLUSTRATED SELF-ASSESSMENT GUIDE

RISK ❑ Low ❑ Intermediate ✔ High ❑

DIAGNOSIS Nevus Seborrheic keratosis Basal cell carcinoma Vascular Dermatofibroma Squamous cell carcinoma Melanoma Other

DISPOSITION ❑ No intervention ❑ Follow-up ✔ Histopathologic diagnosis ❑

4-5c

✔ ❑ ❑ ❑ ❑ ❑ ❑ ❑ ❑

ANSWERS Answers: 2,3,5

Discussion: ■



DERMOSCOPIC CRITERIA ■

■ ■ ■ ■ ■

Asymmetry of color and structure Furrows (black arrows) Ridges (yellow arrows) Acrosyringia (black circles) Brown globules (yellow circles) Bluish-white color (stars)









The history of a changing, itchy lesion is a red flag for concern. ■ Focus your attention and be on the look-out for high risk criteria. The bluish-white color is another red flag for concern. ■ Criteria associated with melanoma (eg, irregular streaks, irregular blotches, bluish-white color) on nonglabrous skin can also be found on the palms and soles (glabrous or nonhair-bearing skin). Foci of the benign parallel furrow pattern can be seen at the periphery: ■ Pigmentation in the furrows on each side of hypopigmented ridges. ■ Acrosyringia are seen in some of the ridges. ■ Beware! Benign acral patterns can be found in melanoma. The blend of a linear and nonlinear arrangement of acrosyringia within the bluish-white area is an irregular feature, indeed the most irregular feature in this lesion. The malignant parallel ridge pattern, which would be seen as linear pigmentation filling the ridges and is associated with melanoma, is not identified anywhere in this lesion. Rather there are a few brown globules in the ridges, which have no diagnostic significance. The differential diagnosis of what turned out to be histopathologically an irritated melanoctyic nevus, includes a dysplastic nevus or melanoma.

PEARLS ■



If there are any clues that an acral lesion might be high risk, be cautious and make a histopathologic diagnosis as soon as possible. Acral lentiginous melanomas are among the most difficult melanomas to diagnose. A working knowledge of the important dermoscopic patterns can help enormously. If you do not put in time with book learning, classic image examination, and using the technique as often as possible, your dermoscopic skills will not get to the level where they should be. Laziness could be a fatal error that should be avoided at all costs!

Chapter 4

Palms, Soles, Nails

RISK

4-6b

4-6a

❑ Low ❑ Intermediate ❑ High

DIAGNOSIS ❑ ❑ ❑ ❑ ❑ ❑ ❑ ❑

Nevus Seborrheic keratosis Basal cell carcinoma Vascular Dermatofibroma Squamous cell carcinoma Melanoma Other

DISPOSITION

4-6c

❑ No intervention ❑ Follow-up ❑ Histopathologic diagnosis

CASE 6 HISTORY This lesion developed de novo on the foot of a 57-year-old man. 1. Even though this looks like a melanoma clinically and dermoscopically, without the malignant parallel ridge pattern the diagnosis cannot be made. 2. One expects to find parallel patterns on the dorsal and ventral (volar) surfaces of the feet. 3. One expects to find parallel patterns only on the dorsal surfaces of the feet. 4. The criteria to diagnose melanoma are similar on the dorsal surface of the feet, legs, and trunk. 5. There are well-developed melanoma-specific criteria typically found on the trunk.

337

338

DERMOSCOPY: AN ILLUSTRATED SELF-ASSESSMENT GUIDE

RISK ❑ Low ❑ Intermediate ✔ High ❑

1

2

DIAGNOSIS Nevus Seborrheic keratosis Basal cell carcinoma Vascular Dermatofibroma Squamous cell carcinoma Melanoma Other

DISPOSITION ❑ No intervention ❑ Follow-up ✔ Histopathologic diagnosis ❑

DERMOSCOPIC CRITERIA ■







■ ■





Asymmetry of color and structure Multicomponent global pattern (1,2,3,4) Irregular pigment network (black arrows) Irregular black dots/globules (circles) Irregular streaks (red arrows) Irregular black blotches (white stars) Reticular depigmentation (white boxes) Bluish-white color (black stars)

3

4

4-6d

❑ ❑ ❑ ❑ ❑ ❑ ✔ ❑ ❑

ANSWERS Answers: 4,5

Discussion: ■







This case points out that parallel patterns are only found on the palms, soles, and mucosa (glabrous skin) and not on the dorsal surfaces of the hands and feet (hair-bearing nonglabrous skin). ■ Glabrous vs nonglabrous skin The dorsal surfaces of the hands and feet are evaluated the same way as are other areas of hair-bearing skin, such as the trunk, arms, legs, and scalp. Well-developed, melanoma-specific criteria are present: ■ Asymmetry of color and structure ■ Multicomponent global pattern (four different areas of criteria within the lesion) ■ Irregular black dots/globules ■ Irregular black blotches ■ Irregular streaks ■ Bluish-white color ■ Five colors ■ Reticular depigmentation The irregular pigment network is a minor features in this lesion.

PEARLS ■





Parallel patterns are only found on the glabrous skin of the palms, soles, and mucosae. De novo dark lesions developing in any location in any patient young, old, or in between, are not always high risk but rather are a red flag for concern that should not be ignored. Dermoscopy is a great tool to get an on the spot second opinion if an equivocal new dark lesion is low, intermediate, or high risk!

Chapter 4

Palms, Soles, Nails

RISK ❑ Low ❑ Intermediate ❑ High

4-7a

DIAGNOSIS ❑ ❑ ❑ ❑ ❑ ❑ ❑ ❑

Nevus Seborrheic keratosis Basal cell carcinoma Vascular Dermatofibroma Squamous cell carcinoma Melanoma Other

DISPOSITION

4-7b

❑ No intervention ❑ Follow-up ❑ Histopathologic diagnosis

CASE 7 HISTORY This lesion was found on the sole of a 46-year-old man while performing a routine total body skin examination. The patient said the lesion was there since he was a child but might be changing. 1. 2. 3. 4. 5.

Globules identify a melanocytic lesion. Lacunae-like structures put a hemangioma in the differential diagnosis. A bluish-white haze and globules put a Spitz nevus in the differential diagnosis. Globular patterns are seen on the trunk but never on the palms or soles. Irregular globules, irregular blotches, and bluish-white color diagnose a nodular melanoma.

339

340

DERMOSCOPY: AN ILLUSTRATED SELF-ASSESSMENT GUIDE

RISK ❑ Low ✔ Intermediate ❑ ❑ High

✔ ❑ ❑ ❑ ❑ ❑ ❑ ❑ ❑

Nevus Seborrheic keratosis Basal cell carcinoma Vascular Dermatofibroma Squamous cell carcinoma Melanoma Other

4-7c

DIAGNOSIS

ANSWERS Answers: 1,2,3

Discussion:

DISPOSITION ❑ No intervention ❑ Follow-up ✔ Histopathologic diagnosis ❑





DERMOSCOPIC CRITERIA ■ ■





Globular global pattern Irregular dots and globules (black arrows) Irregular blotches (yellow arrows) Bluish-white haze



Clinically, this compound nevus is a low risk lesion. Dermoscopically, it is a gray-zone lesion that looks more benign than malignant. It is not a perfectly banal lesion. Dermoscopic differential diagnosis comes into play with this lesion. ■ There are minimally irregular dots and globules with a reddish-brown color. ■ Melanocytic lesion (brown dots and globules) vs vascular lesion (violaceous lacunae) ■ Irregular blotches ■ Confluent globules vs irregular blotches ■ The bluish-white haze puts the globular variant of a Spitz nevus into the differential diagnosis. ■ When one sees bluish-white color, a Spitz nevus usually is in the differential diagnosis. ■ Centrally located bluish/bluish-white color is a clue that the lesion might be a Sptiz nevus. This is a gray-zone lesion that might be handled differently depending on the experience of the clinician. ■ No intervention because this is a benign nevus with a globular pattern. ■ Sequential digital clinical and dermoscopic monitoring to look for important changes over time. ■ Histopathologic diagnosis for the novice dermoscopist because of some gray zone, nondiagnostic irregular features. ■ If in doubt, cut it out! ■ It is better to be safe than sorry!

PEARLS ■ ■ ■



■ ■

One does not always find parallel patterns on glabrous skin. Benign and malignant nonparallel patterns can be found. One can find melanocytic lesions on the palms/soles that contain a pigment network, dots/globules, or homogeneous color without parallel patterns. Any shade, distribution or intensity of blue/bluish-white color could be associated with a high risk lesion. Remember: If there’s blue, they might sue! As a general rule, whatever global pattern or local criteria are seen, you still have to determine if they are regular or irregular, low or high risk, good or bad. ■ Don’t take things for granted! ■ Try to always keep on your thinking cap!

Chapter 4

Palms, Soles, Nails

RISK ❑ Low ❑ Intermediate ❑ High

DIAGNOSIS

4-8a

❑ ❑ ❑ ❑ ❑ ❑ ❑ ❑

Nevus Seborrheic keratosis Basal cell carcinoma Vascular Dermatofibroma Squamous cell carcinoma Melanoma Other

DISPOSITION

4-8b

❑ No intervention ❑ Follow-up ❑ Histopathologic diagnosis

CASE 8 HISTORY This asymptomatic lesion was found on the foot of a 67-year-old man at a routine follow-up skin examination. 1. 2. 3. 4.

Clinically and dermoscopically, this is a high risk lesion. Clinically and dermoscopically, this is a gray-zone lesion. Clinically and dermoscopically, this is a banal acral nevus. Asymmetry of color and structure, irregular pigment network, and regression diagnose a regressive melanoma. 5. Regular pigment network, indistinct borders, and island of normal skin diagnose a benign acral nevus.

341

342

DERMOSCOPY: AN ILLUSTRATED SELF-ASSESSMENT GUIDE

RISK ✔ Low ❑ ❑ Intermediate ❑ High

DIAGNOSIS Nevus Seborrheic keratosis Basal cell carcinoma Vascular Dermatofibroma Squamous cell carcinoma Melanoma Other

DISPOSITION ❑ No intervention ✔ Follow-up ❑ ❑ Histopathologic diagnosis

4-8c

✔ ❑ ❑ ❑ ❑ ❑ ❑ ❑ ❑

ANSWERS Answers: 3,5

Discussion: ■



DERMOSCOPIC CRITERIA ■

■ ■

■ ■

Asymmetry of color and structure Reticular global pattern Regular pigment network (boxes) Islands of normal skin (stars) Indistinct margins (arrows)













Clinically and dermoscopically, there are no red flags for concern. This is a banal clinical and dermoscopic picture. The pigment network is regular, not irregular, and diagnoses a melanocytic lesion: ■ The line segments are thin, light, and look the same throughout the lesion. This case points out that you can see pigment network without parallel patterns on acral surfaces. The global pattern is reticular because most of the lesion is filled with pigment network. There is mild asymmetry of color and structure, which in this case is not high risk. Indistinct margins without an abrupt cut-off is considered a sign of low risk pathology. Two shades of brown color and an absence of melanoma-specific criteria point in the direction of banal melanocytic nevus. Normal skin seen within a lesion, on any part of the body, is a common occurrence and should not be confused with regression. ■ Regression can be seen on glabrous skin.

PEARLS ■





By definition, regression should be lighter than the surrounding skin or a milky/bony-white color. The distinction is not always possible to make especially for the novice dermoscopist. Regression is an independently high risk criterion and might be the one clue to help diagnose melanoma. The more the regression that fills a lesion, the greater the chance it is a melanoma.

Chapter 4

Palms, Soles, Nails

RISK ❑ Low ❑ Intermediate ❑ High

4-9a

DIAGNOSIS ❑ ❑ ❑ ❑ ❑ ❑ ❑ ❑

Nevus Seborrheic keratosis Basal cell carcinoma Vascular Dermatofibroma Squamous cell carcinoma Melanoma Other

DISPOSITION

4-9b

❑ No intervention ❑ Follow-up ❑ Histopathologic diagnosis

CASE 9 HISTORY A 35-year-old woman discovered this asymptomatic and nonchanging lesion on the lateral surface of her right foot. 1. Clinically and dermoscopically, this is a high risk lesion. 2. Dermoscopically, but not clinically, this is a high risk lesion. 3. Regular pigment network, islands of normal skin, and sharp border demarcation diagnose a banal acral nevus. 4. Asymmetry of color and structure and a very irregular pigment network are red flags for concern. 5. A dysplastic nevus and in situ melanoma are in the dermoscopic but not clinical differential diagnosis.

343

344

DERMOSCOPY: AN ILLUSTRATED SELF-ASSESSMENT GUIDE

RISK ❑ Low ❑ Intermediate ✔ High ❑

DIAGNOSIS Nevus Seborrheic keratosis Basal cell carcinoma Vascular Dermatofibroma Squamous cell carcinoma Melanoma Other

DISPOSITION ❑ No intervention ❑ Follow-up ✔ Histopathologic diagnosis ❑

4-9c

✔ ❑ ❑ ❑ ❑ ❑ ❑ ❑ ❑

ANSWERS Answers: 2,4,5

Discussion: ■



DERMOSCOPIC CRITERIA ■

■ ■

■ ■



Asymmetry of color and structure Reticular global pattern Irregular pigment network (boxes) Irregular streaks (red arrows) Multifocal hypopigmentation (stars) Sharp border demarcation (black arrows)











The dermoscopic picture is a surprise because it is very irregular while clinically the lesion does not look high risk: ■ This case points out how much more important information you can obtain when using dermoscopy vs clinical examination alone. The global pattern is reticular because the majority of the lesion is made up of pigment network. ■ Compared to the last case, the pigment network is very irregular because it is thickened, branched, and broken-up. There is multifocal hypopigmentation typically found in dysplastic nevi, which should not be confused with regression, a common feature of melanoma. The foci of irregular streaks are a minor point compared to the irregular pigment network that fills the lesion. The shades of brown color are much more intense compared to the last case: ■ Intensities of color have diagnostic significance. In general, the darker they are, the more high risk the lesion might be. A sharp lesional border is considered a high risk criterion. An exception to this rule is found in seborrheic keratosis that typically have sharp borders. ■ There can always be an exception to every rule! ■ Compare this dysplastic nevus with the last case to see the difference between nonsharp vs sharp borders. The dermoscopic differential diagnosis includes dysplastic nevus vs in situ melanoma. ■ In situ melanoma can be diagnosed simply by having a very irregular pigment network as the major melanoma-specific criterion.

PEARLS ■



There is not a good clinico-dermoscopic correlation, which should be a red flag for concern. Outbursts of shock and surprise with unexpected dramatic dermoscopic pictures do not sit well with patients. Control yourself!

Chapter 4

Palms, Soles, Nails

RISK ❑ Low ❑ Intermediate ❑ High

4-10a

DIAGNOSIS ❑ ❑ ❑ ❑ ❑ ❑ ❑ ❑

Nevus Seborrheic keratosis Basal cell carcinoma Vascular Dermatofibroma Squamous cell carcinoma Melanoma Other

DISPOSITION

4-10b

❑ No intervention ❑ Follow-up ❑ Histopathologic diagnosis

CASE 10 HISTORY The podiatrist of this 57-year-old woman found this lesion and referred the patient to you because he did not want to do a skin biopsy. 1. 2. 3. 4.

Clinically and dermoscopically, this is a low risk lesion. Clinically and dermoscopically, this is a gray-zone lesion. The fingerprint pattern and moth-eaten borders diagnose a solar lentigo. A slightly irregular pigment network, asymmetry of color and structure, and multifocal hypopigmentation are red flags for concern. 5. The high risk clinical picture outweighs the banal appearing dermoscopic picture and a histopathologic diagnosis is indicated.

345

346

DERMOSCOPY: AN ILLUSTRATED SELF-ASSESSMENT GUIDE

RISK ❑ Low ❑ Intermediate ✔ High ❑

DIAGNOSIS Nevus Seborrheic keratosis Basal cell carcinoma Vascular Dermatofibroma Squamous cell carcinoma Melanoma Other

4-10c

❑ ❑ ❑ ❑ ❑ ❑ ✔ ❑ ❑

ANSWERS Answers: 2,4,5

DISPOSITION ❑ No intervention ❑ Follow-up ✔ Histopathologic diagnosis ❑

Discussion: ■



DERMOSCOPIC CRITERIA ■

■ ■







■ ■

Asymmetry of color and structure Reticular global pattern Irregular pigment network (black boxes) Multifocal hypopigmentation (white stars) Fingerprint pattern (yellow box) Subungual hematoma (yellow arrow) Nail plate (black stars) Reflection artifact (black arrows)







Pigment network diagnoses a melanocytic lesion. There is no acral parallel pattern because the lesion is located on the dorsal surface of the toe (ie, nonglabrous skin). ■ By definition, this is the reticular global pattern because the lesion is filled with a pigment network. Criteria to diagnose a solar lentigo other than a focus of the fingerprint pattern are not present (eg, moth-eaten borders) and the lesion does not look like a solar lentigo clinically. ■ Beware! A solar lentigo can be associated with high risk melanocytic pathology such as atypical melanocytic hyperplasia, dysplastic nevi, and melanoma. The clinical picture looks more high risk than the banal appearing dermoscopic picture of: ■ Mild asymmetry of color and structure ■ Minimally irregular pigment network ■ Multifocal hypopigmentation The two foci of bony-white color represent reflection artifact or scale formation. They are not areas of regression. This is a gray-zone lesion with red flags for concern but no definitive diagnosis can be made with dermoscopy.

PEARLS ■



■ ■



In most cases, an atypical clinical picture is overridden by a banal dermoscopic picture. At times, an atypical clinical picture trumps a banal dermoscopic picture and it is better to err on the side of conservatism and make a histopathologic diagnosis. Think in terms of a good clinico–dermoscopic–pathologic correlation. Multiple skin biopsies are often needed to diagnose melanoma when the clinical lesion is large. Use the most atypical dermoscopic area for an incisional biopsy.

Chapter 4

Palms, Soles, Nails

RISK ❑ Low ❑ Intermediate ❑ High

DIAGNOSIS

4-11a

❑ ❑ ❑ ❑ ❑ ❑ ❑ ❑

Nevus Seborrheic keratosis Basal cell carcinoma Vascular Dermatofibroma Squamous cell carcinoma Melanoma Other

DISPOSITION

4-11b

❑ No intervention ❑ Follow-up ❑ Histopathologic diagnosis

(Reprinted, with permission, from Stolz W, Braun-Falco O, Bilek P et al. (2002) Color Atlas of Dermatoscopy. Second Edition. Blackwell Publishing, Oxford.)

CASE 11 HISTORY The patient says that this spot has been on his foot since he was a child and has not changed in any way. 1. 2. 3. 4. 5.

Clinically and dermoscopically, this is a high risk lesion. Clinically but not dermoscopically, this is a high risk lesion. This is a banal acral nevus with the lattice-like pattern. This is a banal nevus with the parallel furrow pattern. Fine oblique lines diagnose the benign fibrillar pattern.

347

348

DERMOSCOPY: AN ILLUSTRATED SELF-ASSESSMENT GUIDE

RISK ✔ Low ❑ ❑ Intermediate ❑ High

DIAGNOSIS Nevus Seborrheic keratosis Basal cell carcinoma Vascular Dermatofibroma Squamous cell carcinoma Melanoma Other

DISPOSITION ❑ No intervention ✔ Follow-up ❑ ❑ Histopathologic diagnosis

4-11c

✔ ❑ ❑ ❑ ❑ ❑ ❑ ❑ ❑

ANSWERS Answers: 5

Discussion: ■



DERMOSCOPIC CRITERIA ■

■ ■

Fibrillar global pattern ■ Oblique parallel line segments (\\\) Ridges (stars) Furrows (arrows)

■ ■ ■

This is a classic fine filamentous fibrillar acral pattern, characterized by thin oblique parallel line segments. Oblique parallel line segments that are thickened and irregular can be seen in melanoma. The history and clinical and dermoscopic pictures are all low risk. There are no high risk criteria. With pressure, the lattice-like pattern can transform into the fibrillar pattern.

PEARLS ■







Acral nevi should be followed like any other nevus on other areas of the body. It is a misconception held by some that acral nevi are more likely to transform into melanoma and they should be removed as a preventative measure. Contrary to popular belief, they do not need to be removed simply because of their location unless there are high risk criteria. Sequential digital clinical and dermoscopic monitoring is the best way to follow low risk acral nevi.

Chapter 4

Palms, Soles, Nails

RISK ❑ Low ❑ Intermediate ❑ High

4-12a

DIAGNOSIS ❑ ❑ ❑ ❑ ❑ ❑ ❑ ❑

Nevus Seborrheic keratosis Basal cell carcinoma Vascular Dermatofibroma Squamous cell carcinoma Melanoma Other

DISPOSITION

4-12b

❑ No intervention ❑ Follow-up ❑ Histopathologic diagnosis

(Reprinted, with permission, from Stolz W, Braun-Falco O, Bilek P et al. (2002) Color Atlas of Dermatoscopy. Second Edition. Blackwell Publishing, Oxford.)

CASE 12 HISTORY A 64-year-old woman had a biopsy on this acral lesion 2 years ago and a lentiginous nevus was diagnosed. The lesion has been slowly getting bigger and darker. 1. 2. 3. 4. 5.

This is a classic parallel furrow pattern with hypopigmented ridges and pigmented furrows. This is a classic malignant parallel ridge pattern with pigmentation in the ridges. Acrosyringia are clearly seen in the skin furrows. Acrocringia are in the pigmented ridges. The biopsy was benign and this is a gray-zone lesion that warrants sequential digital dermoscopic follow-up.

349

350

DERMOSCOPY: AN ILLUSTRATED SELF-ASSESSMENT GUIDE

RISK ❑ Low ❑ Intermediate ✔ High ❑

❑ ❑ ❑ ❑ ❑ ❑ ✔ ❑ ❑

Nevus Seborrheic keratosis Basal cell carcinoma Vascular Dermatofibroma Squamous cell carcinoma Melanoma Other

4-12c

DIAGNOSIS

ANSWERS Answers: 2,4

Discussion: ■

DISPOSITION ❑ No intervention ❑ Follow-up ✔ Histopathologic diagnosis ❑



■ ■

DERMOSCOPIC CRITERIA ■

Parallel ridge pattern ■ Pigmented ridges with linear acrosyringia (boxes) ■ Pigmented ridges without acrosyringia (black arrows) ■ Furrows (black lines) ■ Acrosyringia (yellow arrows)











One does not need dermoscopy to realize that this is a high risk lesion that needs a histopathologic diagnosis. This is a classic malignant parallel ridge pattern that confirms the clinical diagnosis of a melanoma. ■ A review of the pathology of an older biopsy in a malignant lesion is indicated in some situations. Possibly, the original biopsy was not read by an experienced dermatopathologist and the correct diagnosis was missed. Pigmentation is in the skin ridges. There are ridges with and without acrosyringia. ■ One does not always see acrosyringia in the ridges. Acrosyringia are important landmarks to help determine where pigmentation is. In a straightforward case, with furrows and ridges, they are always in the ridges. In this case, the furrows are hypopigmented. The opposite of the benign parallel furrow pattern in which they are pigmented. It is not known why pigmentation favors skin ridges in malignant lesions vs furrows in benign acral lesions. The sensitivity and specificity for this is, however, is very high. When a lesion is made up exclusively of the parallel ridge pattern, it is either an in situ or early invasive acral lentiginous melanoma. Foci of the parallel ridge pattern can be found in more invasive acral lentiginous melanoma usually with other melanoma-specific criteria (eg, irregular streaks, dark irregular blotches, bluish-white color, regression, pink color, atypical vessels).

PEARLS ■

■ ■

If you see the parallel ridge pattern with a nonmalignant pathology report, it would indicate a poor dermoscopic–pathologic correlation and should raise a red flag for concern. Consider performing several biopsies with large lesions at acral sites. Beware! Even experienced dermatopathologists can have trouble diagnosing atypical acral melanocytic lesions. If there is any question, do not hesitate to get another dermatopathologist’s opinion, preferably a national pigmented lesion expert. Even the super experts are not always correct in diagnosing acral melanomas. You can’t always win. Unfortunately, your patient might die!!

Chapter 4

Palms, Soles, Nails

RISK ❑ Low ❑ Intermediate ❑ High

4-13a

DIAGNOSIS ❑ ❑ ❑ ❑ ❑ ❑ ❑ ❑

Nevus Seborrheic keratosis Basal cell carcinoma Vascular Dermatofibroma Squamous cell carcinoma Melanoma Other

DISPOSITION

4-13b

❑ No intervention ❑ Follow-up ❑ Histopathologic diagnosis

CASE 13 HISTORY A 72-year-old tennis player with a history of extensive sun exposure and nonmelanoma skin cancer found this lesion on his right thumb. He came in right away because a friend had just died from melanoma. 1. 2. 3. 4. 5.

Clinically and dermoscopically, this is a worrisome lesion. Irregular dots and globules and the parallel furrow pattern diagnose a melanoma. Irregular dots and globules and the parallel ridge pattern diagnose melanoma. Blood pebbles and blood in the skin ridges diagnose skin hemorrhage. The parallel ridge pattern is diagnostic of melanoma. There is no differential diagnosis.

351

352

DERMOSCOPY: AN ILLUSTRATED SELF-ASSESSMENT GUIDE

RISK ✔ Low ❑ ❑ Intermediate ❑ High

DIAGNOSIS Nevus Seborrheic keratosis Basal cell carcinoma Vascular Dermatofibroma Squamous cell carcinoma Melanoma Other

DISPOSITION ❑ No intervention ✔ Follow-up ❑ ❑ Histopathologic diagnosis

4-13c

❑ ❑ ❑ ❑ ❑ ❑ ❑ ✔ ❑

ANSWERS Answers: 4

Discussion: ■



DERMOSCOPIC CRITERIA ■



Parallel ridge pattern Ridges (yellow arrows) ■ Furrows (black arrows) Blood pebbles (circles) ■





This is a purplish, not brown or black, parallel ridge pattern associated with skin hemorrhage. The parallel ridge pattern is very sensitive and specific to diagnose melanoma; however, it is not pathognomonic of melanoma. The parallel ridge pattern can also be seen in: ■ Skin hemorrhage ■ Banal acral melanocytic nevi in darker skinned races ■ Diffusely as a normal finding on the palms and soles of darker skin races ■ Postinflammatory changes in darker skinned races ■ Acral hemangiomas The peripheral dots/globules represent “blood pebbles.” ■ Dots/globules of a melanocytic lesion are in the differential diagnosis.

PEARLS ■





Sequential digital clinical and dermoscopic monitoring can be used to follow the expected natural color changes one sees with the breakdown and disappearance of blood. For a faster way to diagnose hemorrhage one can: ■ Pick out the blood with a 20-gauge needle. ■ Perform a heme guaiac test—blood in the skin will be guaiac positive. Beware! The presence of blood does not rule out melanoma. Acral melanomas can be associated with blood. Always search for melanomaspecific criteria before diagnosing pure skin hemorrhage.

Chapter 4

Palms, Soles, Nails

RISK ❑ Low ❑ Intermediate ❑ High

4-14a

DIAGNOSIS ❑ ❑ ❑ ❑ ❑ ❑ ❑ ❑

Nevus Seborrheic keratosis Basal cell carcinoma Vascular Dermatofibroma Squamous cell carcinoma Melanoma Other

DISPOSITION

4-14b

❑ No intervention ❑ Follow-up ❑ Histopathologic diagnosis

CASE 14 HISTORY A 76-year-old man has this lesion on his foot for several years. There was a history of trauma in the area that never healed. 1. With a history of trauma, gray color and peppering, one can diagnose post-inflammatory hyperpigmentation. 2. Asymmetry of color and structure, and gray color are red flags for concern. 3. There are remnants of the parallel ridge pattern, which puts melanoma in the differential diagnosis. 4. There are remnants of the benign parallel furrow pattern, which can be seen in invasive acral lentiginous melanoma. 5. Clinically and dermoscopically, this is a high risk lesion that warrants a histopathologic diagnosis posthaste.

353

354

DERMOSCOPY: AN ILLUSTRATED SELF-ASSESSMENT GUIDE

RISK ❑ Low ❑ Intermediate ✔ High ❑

DIAGNOSIS Nevus Seborrheic keratosis Basal cell carcinoma Vascular Dermatofibroma Squamous cell carcinoma Melanoma Other

DISPOSITION ❑ No intervention ❑ Follow-up ✔ Histopathologic diagnosis ❑

4-14c

❑ ❑ ❑ ❑ ❑ ❑ ✔ ❑ ❑

ANSWERS Answers: 2,3,4,5

Discussion: ■

DERMOSCOPIC CRITERIA ■







Asymmetry of color and structure Parallel ridge pattern (black arrows) Parallel furrow pattern (yellow arrows) Regression (stars) ■ Gray color and peppering (boxes)







The history of trauma is misleading and with gray color and peppering one might mistakenly diagnose post-traumatic dyschromia if one does not examine the entire lesion. Regression is seen clinically and dermoscopically in the form of white color and gray color with peppering. Compared to the last case, the parallel patterns are poorly developed: ■ The black arrows identify faint areas of the parallel ridge pattern without acrosyringia in the ridges. ■ One cannot be sure if the yellow arrows identify the parallel ridge or parallel furrow pattern. ■ Both patterns can be found in acral lentiginous melanoma. Combinations of parallel patterns can be found in invasive acral melanomas as opposed to in situ lesions that usually only contain the parallel ridge pattern.

PEARLS ■

■ ■

At times, it is difficult to differentiate the gray dots and globules of peppering from dots and globules representing malignant melanocytes in melanoma. Err on the side of caution and make the diagnosis as soon as possible. Sequential digital clinical and dermoscopic monitoring is contraindicated in this case. Valuable time would be lost in making the correct diagnosis.

Chapter 4

Palms, Soles, Nails

RISK ❑ Low ❑ Intermediate ❑ High

4-15a

DIAGNOSIS ❑ ❑ ❑ ❑ ❑ ❑ ❑ ❑

Nevus Seborrheic keratosis Basal cell carcinoma Vascular Dermatofibroma Squamous cell carcinoma Melanoma Other

DISPOSITION

4-15b

❑ No intervention ❑ Follow-up ❑ Histopathologic diagnosis

(Reprinted, with permission, from Stolz W, Braun-Falco O, Bilek P et al. (2002) Color Atlas of Dermatoscopy. Second Edition. Blackwell Publishing, Oxford.)

CASE 15 HISTORY A 68-year-old woman has a pigmented lesion on the plantar surface of her foot that has been getting thicker over the past few years. 1. Without the parallel ridge pattern, melanoma cannot be diagnosed. 2. The bluish-white color is a red flag for concern that this might be a melanoma. 3. The irregular brown globular-like structures could represent remnants of the parallel ridge pattern. 4. The nonlinear arrangement of acrosyringia favors a benign lesion. 5. The nonlinear arrangement of the acrosyringia is irregular and represents destruction of the normal architecture by invasive melanoma.

355

356

DERMOSCOPY: AN ILLUSTRATED SELF-ASSESSMENT GUIDE

RISK ❑ Low ❑ Intermediate ✔ High ❑

1

2 1

DIAGNOSIS 3

Nevus Seborrheic keratosis Basal cell carcinoma Vascular Dermatofibroma Squamous cell carcinoma Melanoma Other

DISPOSITION ❑ No intervention ❑ Follow-up ✔ Histopathologic diagnosis ❑

2

ANSWERS Answers: 2, 3, 5

Discussion: ■



DERMOSCOPIC CRITERIA ■





■ ■

Asymmetry of color and structure Multicomponent global pattern (1,2,3) Irregular brown globules (boxes) Bluish-white color (stars) Acrosyringia (arrows)

2

4-15c

❑ ❑ ❑ ❑ ❑ ❑ ✔ ❑ ❑









The history and clinical appearance are worrisome. The irregular dermoscopic picture confirms that this could be a high risk lesion→ melanoma. The bluish-white color is a red flag for concern and represents deep involvement of the pathologic process. ■ Different shades of blue color represent pigmentation in the deeper dermis. Without the expected parallel patterns, one should evaluate the lesions as if it were on nonglabrous skin. There are several melanoma-specific criteria: ■ Asymmetry of color and structure ■ Multicomponent global pattern ■ Bluish-white color The irregular brown globular-like structures have a differential diagnosis. Irregular globules of a melanocytic lesion, or remnants of the parallel ridge pattern that have been destroyed by the regressive process. ■ In some but not all areas they have a linear appearance, which favors remnants of a parallel pattern. The acrosyringia have foci of linear and nonlinear distribution, which is irregular and a red flag for concern.

PEARLS ■





Everything in this case, from the history to the clinical and dermoscopic pictures, points to the diagnosis of invasive acral lentiginous melanoma. In reality, this case can only be a melanoma. Inform your patient about the possibility of malignancy. Hold out hope that there is a chance that it might be benign. The most atypical clinical scenario can still turn out not to be malignant. It happens!

Chapter 4

Palms, Soles, Nails

RISK ❑ Low ❑ Intermediate ❑ High

DIAGNOSIS

4-16a

❑ ❑ ❑ ❑ ❑ ❑ ❑ ❑

Nevus Seborrheic keratosis Basal cell carcinoma Vascular Dermatofibroma Squamous cell carcinoma Melanoma Other

DISPOSITION

4-16b

❑ No intervention ❑ Follow-up ❑ Histopathologic diagnosis

CASE 16 HISTORY A 75-year-old woman developed persistent pain while walking after treatment for a callus on the sole of her right foot. 1. The parallel furrow pattern and peppering diagnose a benign nevus with post-inflammatory changes secondary to the treatment of the “callus”. 2. The malignant parallel ridge pattern is clearly seen indicating that this was not a callus but a melanoma mistakenly treated as a callus. 3. Bluish-white, gray, and pink color plus pinpoint vessels indicate this is not an in situ melanoma. 4. This represents the benign parallel ridge pattern created by hemorrhage secondary to the treatment of the callus. 5. Sequential digital dermoscopic monitoring is indicated to follow the natural disappearance of the hemorrhage.

357

358

DERMOSCOPY: AN ILLUSTRATED SELF-ASSESSMENT GUIDE

RISK ❑ Low ❑ Intermediate ✔ High ❑

DIAGNOSIS Nevus Seborrheic keratosis Basal cell carcinoma Vascular Dermatofibroma Squamous cell carcinoma Melanoma Other

DISPOSITION ❑ No intervention ❑ Follow-up ✔ Histopathologic diagnosis ❑

4-16c

❑ ❑ ❑ ❑ ❑ ❑ ✔ ❑ ❑

ANSWERS Answers: 2,3

Discussion: ■



DERMOSCOPIC CRITERIA ■



■ ■ ■ ■

Asymmetry of color and structure Parallel ridge pattern ■ Ridges (black lines) ■ Furrows (arrows) Bluish-white color (stars) Gray color (boxes) Diffuse pink color Pinpoint vessels (circles)







The malignant parallel ridge pattern is clearly seen with pigmentation in the thicker ridges flanked on both sides by the lighter colored furrows. The bluish-white and gray color has a differential diagnoses, invasive melanoma vs post-inflammatory changes secondary to the treatment of a “callus”. Asymmetry of color and structure, pink color, and pinpoint vessels are high risk features. The color is not good for hemorrhage because it is brown not purplish, ruling out the parallel ridge pattern created by blood. Sequential digital clinical and dermoscopic monitoring is contraindicated. There is too much evidence that this is not a simple callus but rather an invasive acral lentiginous melanoma.

PEARLS ■

■ ■ ■

This will not be the first nor the last time acral lentiginous melanoma is misdiagnosed. Always keep melanoma in mind when dealing with acral lesions. Equivocal lesions need close monitoring. Don’t lose your patient to follow-up!

Chapter 4

Palms, Soles, Nails

RISK ❑ Low ❑ Intermediate ❑ High

4-17a

DIAGNOSIS ❑ ❑ ❑ ❑ ❑ ❑ ❑ ❑

Nevus Seborrheic keratosis Basal cell carcinoma Vascular Dermatofibroma Squamous cell carcinoma Melanoma Other

DISPOSITION

4-17b

❑ No intervention ❑ Follow-up ❑ Histopathologic diagnosis

CASE 17 HISTORY A 44-year-old man was concerned about this spot on his foot that would not go away. 1. Clinically and dermoscopically this is a worrisome lesion. 2. There is asymmetry of color and structure but the malignant parallel ridge pattern is not identified, therefore melanoma cannot be diagnosed. 3. There is subtle pigmentation in the ridges which is enough to diagnose the malignant parallel ridge pattern. 4. Irregular dots and globules scattered throughout the lesion and irregular dark blotches are high risk criteria. 5. The banal dermoscopic features trump the high risk clinical appearance, and a histopathologic diagnosis is not indicated.

359

360

DERMOSCOPY: AN ILLUSTRATED SELF-ASSESSMENT GUIDE

RISK ❑ Low ❑ Intermediate ✔ High ❑

DIAGNOSIS Nevus Seborrheic keratosis Basal cell carcinoma Vascular Dermatofibroma Squamous cell carcinoma Melanoma Other

DISPOSITION ❑ No intervention ❑ Follow-up ✔ Histopathologic diagnosis ❑

4-17c

❑ ❑ ❑ ❑ ❑ ❑ ✔ ❑ ❑

ANSWERS Answers: 1,3,4

Discussion: ■ ■

DERMOSCOPIC CRITERIA ■

■ ■

■ ■ ■



Asymmetry of color and structure Parallel ridge pattern (stars) Parallel furrow pattern (yellow arrows) Ridges (yellow lines) Furrows (black arrows) Irregular brown dots and globules (circles) Irregular dark brown blotches (red arrows)







Clinically and dermoscopically, this is a high risk lesion. The parallel ridge pattern is present with tan pigmentation in some but not all of the ridges. ■ Compared to some of the other cases, one can see that there can be subtle presentations of this high risk criterion. ■ One has to focus their attention and specifically look for the pigmentation in the ridges so they are not missed. Foci of the parallel furrow pattern with linear pigmentation in some furrows are also seen. A haphazard distribution of irregular brown dots/globules and irregular pigmentation are red flags for concern. At times, a suspicious clinical appearance trumps a not so high risk looking dermoscopic picture and a histopathologic diagnosis should be considered.

PEARLS ■





Before you diagnose a low risk lesion, make sure that you do not miss high risk criteria that might be present yet are not well-developed. This is a busy image with lots of marks to identify. One has to focus this attention to study the image just as one should focus their attention and study the dermoscopic criteria in a difficult case. If one has to spend too much time thinking about the dermoscopic picture, it is better to err on the conservative side and make a histopathologic diagnosis.

Chapter 4

Palms, Soles, Nails

RISK ❑ Low ❑ Intermediate ❑ High

4-18a

DIAGNOSIS ❑ ❑ ❑ ❑ ❑ ❑ ❑ ❑

Nevus Seborrheic keratosis Basal cell carcinoma Vascular Dermatofibroma Squamous cell carcinoma Melanoma Other

DISPOSITION

4-18b

❑ No intervention ❑ Follow-up ❑ Histopathologic diagnosis

CASE 18 HISTORY A 32-year-old woman has had this lesion since childhood and thinks it has been getting slowly bigger over many years. 1. This is a perfectly banal appearing homogeneous global pattern that is of no concern. 2. The black color is not homogeneously seen throughout the lesion, which is a red flag for concern. 3. There are foci of pigmentation in some ridges, which diagnose a low risk parallel ridge pattern. 4. There are foci of pigmentation in some ridges, which diagnose the malignant parallel ridge pattern. 5. There is a linear and haphazard distribution of acrosyringia, which is a red flag for concern.

361

362

DERMOSCOPY: AN ILLUSTRATED SELF-ASSESSMENT GUIDE

RISK ❑ Low ❑ Intermediate ✔ High ❑

DIAGNOSIS Nevus Seborrheic keratosis Basal cell carcinoma Vascular Dermatofibroma Squamous cell carcinoma Melanoma Other

DISPOSITION ❑ No intervention ❑ Follow-up ✔ Histopathologic diagnosis ❑

4-18c

❑ ❑ ❑ ❑ ❑ ❑ ✔ ❑ ❑

ANSWERS Answers: 2,4,5

Discussion: ■



DERMOSCOPIC CRITERIA ■ ■







■ ■

Homogeneous global pattern Asymmetry of color and structure Bluish-white and black colors (stars) Ridges in normal skin (black lines) Furrows in normal skin (black arrows) Acrosyringia (yellow boxes) Parallel ridge pattern without acrosyringia (yellow arrows)





■ ■

The long-term banal history conflicts with the high risk clinical and dermoscopic picture. There can be a nonparallel homogeneous pattern seen on glabrous skin. A low risk homogeneous pattern is made up of uniform light color without any high risk criteria. This lesion has several high risk features suggesting that it could be a melanoma: ■ Asymmetry of color and structure ■ Bluish-white color ■ Irregular black color filling the lesion ■ A linear and haphazard distribution of acrosyringia There are foci of the malignant parallel ridge pattern without acrosyringia. Compared with the last case, this is a more advanced acral lentiginous melanoma that is easier to diagnose. ■ There is no doubt about the significance of the black pigmentation. ■ The black color is intense and fills the lesion. ■ The bluish-white and black colors are signs of invasion.

PEARLS ■





There are no clues clinically or dermoscopically that this is not a high risk lesion. The high risk clinical and dermoscopic pictures trumps the relatively low risk history, and points out how much useful information can be obtained with dermoscopy. Keep in mind that patient’s histories are not always accurate.

Chapter 4

Palms, Soles, Nails

RISK ❑ Low ❑ Intermediate ❑ High

4-19a

DIAGNOSIS ❑ ❑ ❑ ❑ ❑ ❑ ❑ ❑

Nevus Seborrheic keratosis Basal cell carcinoma Vascular Dermatofibroma Squamous cell carcinoma Melanoma Other

DISPOSITION

4-19b

❑ No intervention ❑ Follow-up ❑ Histopathologic diagnosis

CASE 19 HISTORY This lesion was present for 3 years on the heel of a 78-year-old woman. It started to bleed 2 weeks before she came in for a consultation. 1. Clinically and dermoscopically, this is a typical “black heel” with ulceration secondary to trauma. 2. Different shades of blue color are a red flag for concern. 3. Well-developed acrosyringia suggests that this is a melanocytic lesion. 4. The central area has a differential diagnosis that includes ulceration of a nodular melanoma. 5. Hemorrhagic crusts rule out melanoma and favor senile purpura, which is commonly seen in this age group.

363

364

DERMOSCOPY: AN ILLUSTRATED SELF-ASSESSMENT GUIDE

RISK ❑ Low ❑ Intermediate ✔ High ❑

DIAGNOSIS Nevus Seborrheic keratosis Basal cell carcinoma Vascular Dermatofibroma Squamous cell carcinoma Melanoma Other

DISPOSITION ❑ No intervention ❑ Follow-up ✔ Histopathologic diagnosis ❑

4-19c

❑ ❑ ❑ ❑ ❑ ❑ ✔ ❑ ❑

ANSWERS Answers: 2,3,4

Discussion: ■



DERMOSCOPIC CRITERIA ■ ■ ■

■ ■

Homogeneous global pattern Bluish-white color (white stars) Dark blue homogeneous color (yellow stars) Acrosyringia (boxes) Hemorrhagic crusts (white arrows)







There is nothing in the history, clinical, or dermoscopic picture to suggest that this is not a high risk lesion. The absence of the purplish color typical of blood and well-developed acrosyringia rules out “black heel” (talon noir) and senile purpura and favors a melanocytic lesion. The central darker area represents a deeper nodular component and the bluish-white color with linear acrosyringia represents the more superficial malignant parallel ridge pattern. Clinically and dermoscopically, there is ulceration with foci of hemorrhagic crusts. This is the ultimate expression of how bad an acral melanoma can be.

PEARLS ■



It is controversial if total body skin examinations should be the standard of care for dermatologists and other categories of physicians. Our colleagues that do not perform total body skin examinations would miss this lesion if it were asymptomatic and not the chief complain of the patient. One happier scenario would be that it was discovered at an in situ 100% curable stage at an earlier visit to a physician for an unrelated complaint and a complete skin examination was performed.

Chapter 4

Palms, Soles, Nails

RISK ❑ Low ❑ Intermediate ❑ High

4-20a

DIAGNOSIS ❑ ❑ ❑ ❑ ❑ ❑ ❑ ❑

Nevus Seborrheic keratosis Basal cell carcinoma Vascular Dermatofibroma Squamous cell carcinoma Melanoma Other

DISPOSITION

4-20b

❑ No intervention ❑ Follow-up ❑ Histopathologic diagnosis

CASE 20 HISTORY A 75-year-old woman had a painful hyperkeratosis on the sole of her foot for one year. 1. 2. 3. 4. 5.

The presence of dried blood rules out melanoma. The absence of a malignant parallel ridge pattern rules out melanoma. Bluish-white and milky-red colors are clues that this could be a melanoma. One focus of irregular globules is enough to identify a melanocytic lesion. Asymmetry of color and structure and a multicomponent global pattern are more clues that this could be a melanoma.

365

366

DERMOSCOPY: AN ILLUSTRATED SELF-ASSESSMENT GUIDE

RISK ❑ Low ❑ Intermediate ✔ High ❑

1

DIAGNOSIS 2

Nevus Seborrheic keratosis Basal cell carcinoma Vascular Dermatofibroma Squamous cell carcinoma Melanoma Other

DISPOSITION ❑ No intervention ❑ Follow-up ✔ Histopathologic diagnosis ❑

3

4

5

ANSWERS Answers: 3,4,5

Discussion: ■



DERMOSCOPIC CRITERIA ■



■ ■

■ ■ ■

Asymmetry of color and structure Multicomponent global pattern (1,2,3,4,5) Irregular brown globules (box) Irregular brown blotches (yellow arrows) Bluish-white color (blue star) Milky-red color (black arrows) Regression (black stars)

4-20c

❑ ❑ ❑ ❑ ❑ ❑ ✔ ❑ ❑









The last case represented how bad a pigmented acral melanoma can be. This invasive melanoma is just as invasive yet it is not heavily pigmented. The milky-red and bluish-white colors are clues that this could be a melanoma. ■ If there’s pink, stop and think! ■ If there’s blue, they might sue! ■ If there’s white, control your fright! Invasive acral melanomas that lack parallel patterns should be evaluated like melanomas found on the trunk or extremities. There are well-developed melanoma-specific criteria: ■ Asymmetry of color and structure ■ A multicomponent global pattern with five different areas ■ Bony-white color suggesting there is regression The irregular dark blotches and focus of irregular globules could represent dried blood. ■ Blood can be associated with acral melanomas. A callus or plantar wart are in the historical, clinical but not dermoscopic differential diagnosis.

PEARLS ■ ■



Always maintain a high “melanoma index of suspicion” for equivocal cases. Have your dermoscopic instrumentation handy and ready to be used at all times, so important clues that can only be gained by dermoscopy are not missed that might help diagnose a difficult case. This is a difficult case, with subtle dermoscopic features that could easily be missed even by an experienced dermoscopist.

Chapter 4

Palms, Soles, Nails

RISK ❑ Low ❑ Intermediate ❑ High

DIAGNOSIS

4-21a

❑ ❑ ❑ ❑ ❑ ❑ ❑ ❑

Nevus Seborrheic keratosis Basal cell carcinoma Vascular Dermatofibroma Squamous cell carcinoma Melanoma Other

DISPOSITION

4-21b

❑ No intervention ❑ Follow-up ❑ Histopathologic diagnosis

CASE 21 HISTORY This pink macule was found on the left foot of a 65-year-old woman while performing a total body skin examination. The patient was not aware of its presence. 1. 2. 3. 4.

Diffuse pinpoint vessels and pink color could represent Bowen disease. Nonspecific dermatitis and psoriasis are in the clinical and dermoscopic differential diagnosis. Amelanotic melanoma and acute lichen planus-like keratosis are in the differential diagnosis. A solitary pink macule could be melanocytic, nonmelanocytic, benign, malignant, or inflammatory. 5. Diagnostic sensitivity and specificity are higher when dermoscopy is used to diagnose pigmented vs pink, feature poor or featureless skin lesions.

367

368

DERMOSCOPY: AN ILLUSTRATED SELF-ASSESSMENT GUIDE

RISK ❑ Low ✔ Intermediate ❑ ❑ High

DIAGNOSIS Nevus Seborrheic keratosis Basal cell carcinoma Vascular Dermatofibroma Squamous cell carcinoma Melanoma Other

4-21d

❑ ❑ ❑ ❑ ❑ ❑ ❑ ✔ ❑

ANSWERS Answers: 1,2,3,4,5

DISPOSITION ❑ No intervention ❑ Follow-up ✔ Histopathologic diagnosis ❑

Discussion: ■



DERMOSCOPIC CRITERIA ■ ■ ■

Pinpoint vessels (boxes) Different shades of pink color White color (stars)



■ ■







A solitary pink lesion is nonspecific and could be melanocytic, nonmelanocytic, benign, malignant, or inflammatory. ■ To our surprise, the histopathologic report was that of a nonspecific dermatitis without evidence of a fungus. Dermoscopy is usually not as good a tool to diagnose pink, feature poor, or featureless vs pigmented lesions that have well-developed criteria (eg, pigment network, streaks, dots, globules, blotches). Telangiectatic vessels can give us clues to help diagnose pink lesions. However, they are also relatively nonspecific. This lesion is filled with pinpoint vessels. Pinpoint and glomerular vessels (diffuse or clustered coiled vessels) are a variation of the same pathology. Small vessels running perpendicular to the skin surface. The differential diagnosis of this lesion includes: ■ Bowen disease ■ Nonspecific dermatitis ■ Amelanotic melanoma ■ Psoriasis ■ Acute lichen planus-like keratosis Clues to narrow down the differential diagnosis can be obtained by knowing the history of the lesion, age of the patient, distribution of the lesion, and if it is symptomatic or not. ■ The absence of scale and the plantar location are points against an acute lichen planus-like keratosis. ■ Even though one can see a solitary lesion of psoriasis, it is not very common and is a point against this diagnosis. The white color has a differential diagnosis that includes regression or blanching of the blood supply with pressure when the image was taken.

PEARLS ■



Polarizing instrumentation, minimal pressure, and fluid is the best way to visualize telangietatic vessels (eg, ultrasound gel). It is not always necessary to make a histopathologic diagnosis of a solitary pink lesion at the first visit. Quite often they disappear after a few weeks and surgical interventions can be avoided.

Chapter 4

Palms, Soles, Nails

RISK ❑ Low ❑ Intermediate ❑ High

4-22a

DIAGNOSIS ❑ ❑ ❑ ❑ ❑ ❑ ❑ ❑

Nevus Seborrheic keratosis Basal cell carcinoma Vascular Dermatofibroma Squamous cell carcinoma Melanoma Other

DISPOSITION

4-22b

❑ No intervention ❑ Follow-up ❑ Histopathologic diagnosis

CASE 22 HISTORY An adult male presented with this nonhealing area on the plantar surface of his left foot. There was no history of trauma. 1. 2. 3. 4.

Milky-red color and regression diagnose an amelanotic melanoma. Homogeneous red color and a peripheral white collarette diagnose a pyogenic granuloma. There is an absence of color associated with melanin, which favors a pyogenic granuloma. Homogeneous red color and a peripheral white collarette are seen in 100% of pyogenic granulomas. 5. Homogeneous red color, peripheral white collarette, white lines within the lesion, and ulceration can all be found in pyogenic granulomas.

369

370

DERMOSCOPY: AN ILLUSTRATED SELF-ASSESSMENT GUIDE

RISK ✔ Low ❑ ❑ Intermediate ❑ High

DIAGNOSIS Nevus Seborrheic keratosis Basal cell carcinoma Vascular Dermatofibroma Squamous cell carcinoma Melanoma Other

4-22c

❑ ❑ ❑ ❑ ❑ ❑ ❑ ✔ ❑

DISPOSITION

ANSWERS

❑ No intervention ❑ Follow-up ✔ Histopathologic diagnosis ❑

Answers: 2,3,5

DERMOSCOPIC CRITERIA ■ ■

■ ■ ■ ■ ■

Homogeneous red color White peripheral collarette (stars) Acrosyringia (boxes) Furrows (arrows) Ridges (black lines) Pinpoint vessels (circles) Thread of material from a sock (yellow arrow)

Discussion: ■













Pyogenic granuloma is a misnomer. It is neither pyogenic nor is it a granuloma. ■ Histopathologically, it is a vascular tumor. ■ The etiology is unknown, and it is thought to be associated with trauma. ■ A reactive process “proud flesh.” In most cases, it is a clinical diagnosis based on: ■ Recent onset ■ Reddish color ■ Areas associated with trauma (eg, fingers, hands, mucosal surfaces) ■ Friability and easily bleeding The dermoscopic criteria used to diagnose pyogenic granulomas include: ■ Homogeneous reddish color ■ White collarette at the periphery ■ The thickness of the collarette varies from very thin to thick as in this case. ■ White lines within the lesion ■ Ulceration None of the criteria are seen all the time: ■ Homogeneous red color and peripheral white collarette are the most frequently found criteria. Amelanotic melanoma is the most important tumor that should always be in the differential diagnosis of a pyogenic granuloma. Criteria in favor of melanoma vs pyogenic granuloma include: ■ Criteria seen in melanocytic lesions (eg, pigment network, dots, globules, streaks, blotches) ■ Color associated with melanin (eg, black, brown, gray, blue) The differential diagnosis of the pinpoint vessel-like dots includes pinpoint vessels and dots created by blood.

PEARLS ■



Amelanotic melanoma is the great masquerader, and should always be included in the differential diagnosis of a “pyogenic granuloma.” No age group is excluded from this general principle.

Chapter 4

Palms, Soles, Nails

RISK ❑ Low ❑ Intermediate ❑ High

4-23a

DIAGNOSIS ❑ ❑ ❑ ❑ ❑ ❑ ❑ ❑

Nevus Seborrheic keratosis Basal cell carcinoma Vascular Dermatofibroma Squamous cell carcinoma Melanoma Other

DISPOSITION

4-23b

❑ No intervention ❑ Follow-up ❑ Histopathologic diagnosis

CASE 23 HISTORY This 7-year-old boy was referred for a biopsy of pigmentation of the nail apparatus to rule out melanoma. The lesion had been present for 2 years without change. 1. Pigmented nail bands are rare in childhood. The main differential diagnosis includes nail apparatus melanoma. 2. The longitudinal bands are irregular in thickness with a loss of parallelism, which confirms the clinical diagnosis of nail apparatus melanoma. 3. The periungual pigmentation seen here is known as Hutchinson sign and is diagnostic of nail apparatus melanoma. 4. The pigmented bands are homogeneous in color without loss of parallelism, which favors the diagnosis of a benign melanocytic nevus. 5. Nail apparatus melanoma is rare in children but should always be included in the differential diagnosis of melanonychia striata.

371

372

DERMOSCOPY: AN ILLUSTRATED SELF-ASSESSMENT GUIDE

RISK ❑ Low ❑ Intermediate ✔ High ❑

✔ ❑ ❑ ❑ ❑ ❑ ❑ ❑ ❑

Nevus Seborrheic keratosis Basal cell carcinoma Vascular Dermatofibroma Squamous cell carcinoma Melanoma Other

4-23c

DIAGNOSIS

ANSWERS Answers: 4,5

Discussion: ■

DISPOSITION ❑ No intervention ❑ Follow-up ✔ Histopathologic diagnosis ❑





DERMOSCOPIC CRITERIA ■







Homogeneous light and dark brown background Uniform thick dark bands (double red arrows) Uniform thin dark bands (white arrows) Hutchinson sign (yellow arrows)











■ ■

Melanonychia striata (longitudinal pigmented banding) is usually a worrisome clinical finding in any age group. This history and the clinical and dermoscopic findings help to determine if the pigmentation is low risk (eg, nevus, lentigo, ethnic variation, drug induced, post-inflammatory, post-traumatic) or high risk (eg, nail apparatus melanoma). A history that points in a benign direction includes: ■ Onset during childhood ■ Stable without change over time ■ A family history of similar nail pigmentation ■ Exposure to pigmentogenic medication (eg, acyclovir, minocyline, hydroxyurea) ■ History of trauma or inflammation Low risk dermoscopic features include: ■ A light or dark brown homogeneous background may or may not be present. ■ Regular longitudinal line segments with regard to the color, spacing, thickness, parallelism. Parallelism refers to uniform unbroken line segments: ■ Loss of parallelism (broken line segments) created by irregular melanin production can be associated with nail apparatus melanoma. Pigmentation that extends into the periungual tissue is called Hutchinson sign and can be associated with nail apparatus melanoma, but not always. Pseudo-Hutchinson sign refers to pigment in the cuticle, and is not associated with melanoma. The criteria in this case that favors a benign nevus include: ■ Onset during childhood ■ Stable over time Even though the color of the bands is uniform, the widths are not, which is a red flag for concern. The periungual pigmentation (Hutchinson sign) is another red flag for concern. There were enough red flags for concern which prompted a biopsy. The histopathologic diagnosis was that of a severely atypical melanocytic process and the lesion was excised.

PEARLS ■







Nail apparatus biopsy can be avoided in most children because the incidence of melanoma is extremely low. A red flag for concern in a child is a rapid change in any type of nail apparatus pigmentation. Sequential digital clinical and dermoscopic monitoring is the cutting-edge way to follow patients with nail apparatus pigmentation. Oil or gel must be used to see through the thick, dry nail plate.

Chapter 4

Palms, Soles, Nails

RISK ❑ Low ❑ Intermediate ❑ High

4-24a

DIAGNOSIS ❑ ❑ ❑ ❑ ❑ ❑ ❑ ❑

Nevus Seborrheic keratosis Basal cell carcinoma Vascular Dermatofibroma Squamous cell carcinoma Melanoma Other

DISPOSITION

4-24b

❑ No intervention ❑ Follow-up ❑ Histopathologic diagnosis

CASE 24 HISTORY A 17-year-old girl has had this pigmentation on her fifth finger for 2 years. There is no history of trauma and the color seems to be changing. 1. 2. 3. 4.

Uniform bands diagnose a benign nevus. There is no loss of parallelism. There is no Hutchinson sign. The lines are irregular in thickness and spacing with loss of parallelism which suggests this could be a melanoma. 5. Compared to the last case, the color of the bands is much lighter with minimal periungual pigmentation which rules out nail apparatus melanoma.

373

374

DERMOSCOPY: AN ILLUSTRATED SELF-ASSESSMENT GUIDE

RISK ❑ Low ❑ Intermediate ✔ High ❑

DIAGNOSIS ❑ ❑ ❑ ❑ ❑ ❑ ✔ ❑ ❑

Nevus Seborrheic keratosis Basal cell carcinoma Vascular Dermatofibroma Squamous cell carcinoma Melanoma Other

4-24c

DISPOSITION ❑ No intervention ❑ Follow-up ✔ Histopathologic diagnosis ❑

ANSWERS Answers: 4

DERMOSCOPIC CRITERIA ■

■ ■

Irregular pigmented bands (black arrows) Loss of parallelism (red arrow) Hutchinson sign (yellow arrows)

Discussion: ■









Even though the patient is young, the history and clinical and dermoscopic features are worrisome. ■ There is a history of change. ■ There is no history of a potentially benign cause for the pigmentation (trauma, inflammation). The bands are irregular: ■ Variable thickness ■ Irregular spacing ■ Variegate color (light/dark brown, gray) ■ The intensity of the band color is not a defining point to help differentiate benign from malignant pathology. Adding up the high risk criteria help diagnose Hutchinson sign vs pseudoHutchinson sign with periungual pigmentation. There is no feature of this case other than the patient’s age that favors a benign lesion. This is a melanoma until proven otherwise!

PEARLS ■

■ ■

Do not put a histopathologic diagnosis off if melanoma is in the differential diagnosis of melanonychia striata. Digital monitoring is contraindicated in this case. Be persuasive in your convictions that a patient might have a nail apparatus melanoma if you have to convince a colleague to perform the biopsy for you.

Chapter 4

Palms, Soles, Nails

RISK ❑ Low ❑ Intermediate ❑ High

4-25a

DIAGNOSIS ❑ ❑ ❑ ❑ ❑ ❑ ❑ ❑

Nevus Seborrheic keratosis Basal cell carcinoma Vascular Dermatofibroma Squamous cell carcinoma Melanoma Other

DISPOSITION

4-25b

❑ No intervention ❑ Follow-up ❑ Histopathologic diagnosis

CASE 25 HISTORY A 36-year-old woman noticed this discoloration on one big toe nail a few months ago. There was no history of trauma even though she jogs three times a week. 1. Gray color diagnoses post-inflammatory hyperpigmentation secondary to trauma. 2. The presence of blood spots rules out melanoma and confirms that this is secondary to the patients jogging. 3. There is no Hutchinson sign, which must be present to diagnose melanoma. 4. The bands have different widths with a loss of parallelism, which suggests this could be a nail apparatus melanoma. 5. The homogeneous light and dark brown bands diagnose a benign nail apparatus nevus.

375

376

DERMOSCOPY: AN ILLUSTRATED SELF-ASSESSMENT GUIDE

RISK ❑ Low ❑ Intermediate ✔ High ❑

DIAGNOSIS Nevus Seborrheic keratosis Basal cell carcinoma Vascular Dermatofibroma Squamous cell carcinoma Melanoma Other

4-25c

❑ ❑ ❑ ❑ ❑ ❑ ✔ ❑ ❑

DISPOSITION

ANSWERS

❑ No intervention ❑ Follow-up ✔ Histopathologic diagnosis ❑

Answers: 4

Discussion: ■

■ ■

DERMOSCOPIC CRITERIA ■







Irregular thick pigmented bands (yellow and white double arrows) Thin fine pigmented bands (red arrow) Loss of parallelism (black arrows) Irregular dark dots and globules (circles)





All aspects of this case, the history, clinical, and dermoscopic features are high risk and suggest this could be a nail apparatus melanoma. The clinical appearance is more atypical than what is seen with dermoscopy. There are subtle high risk criteria that could be missed by the novice dermoscopist or experienced dermoscopist who is in a hurry and/or does not make a focused examination: ■ Thick pigmented bands with different widths ■ Multiple fine thin bands ■ The loss of parallelism is questionably present The presence of irregular dark dots and globules could be confused with blood spots which can be associated with nail apparatus melanoma. Post-inflammatory melanonychia striata created by trauma is usually symmetrically seen on both feet and has homogeneous gray bands.

PEARLS ■



Do not diagnose low risk melanonychia striata before a focused search is made for high risk criteria that might not be well-developed. If only one nail is involved, raise a red flag for concern.

Chapter 4

Palms, Soles, Nails

RISK ❑ Low ❑ Intermediate ❑ High

DIAGNOSIS

4-26a

❑ ❑ ❑ ❑ ❑ ❑ ❑ ❑

Nevus Seborrheic keratosis Basal cell carcinoma Vascular Dermatofibroma Squamous cell carcinoma Melanoma Other

DISPOSITION

4-26b

❑ No intervention ❑ Follow-up ❑ Histopathologic diagnosis

CASE 26 HISTORY This solitary pigmented band has been present for 15 years on the index finger of a 47-year-old man. 1. The pressure of the micro-Hutchinson sign diagnoses a slow growing nail apparatus melanoma. 2. Irregular bands and loss of parallelism diagnose a melanoma. 3. Homogeneous light bands with definite areas of gray color put lentigo simplex in the differential diagnosis. 4. Gray bands in one or multiple nails can be found in lentigo simplex, ethnic pigmentation, after trauma, inflammation, or after the intake of pigmentogenic drugs. 5. Gray bands are an indication for a nail apparatus biopsy no matter what the history or clinical appearance of the nail.

377

378

DERMOSCOPY: AN ILLUSTRATED SELF-ASSESSMENT GUIDE

RISK ✔ Low ❑ ❑ Intermediate ❑ High

DIAGNOSIS Nevus Seborrheic keratosis Basal cell carcinoma Vascular Dermatofibroma Squamous cell carcinoma Melanoma Other

DISPOSITION ❑ No intervention ✔ Follow-up ❑ ❑ Histopathologic diagnosis

4-26c

❑ ❑ ❑ ❑ ❑ ❑ ❑ ✔ ❑

ANSWERS Answers: 3,4

Discussion: ■



DERMOSCOPIC CRITERIA ■







Uniform thick gray band (black double arrows) Uniform thin gray bands (yellow arrows) Darker homogenous gray color (box) Dry cuticle (red arrows)





The long-term stable history, absence of trauma or inflammation, and gray color put a lentigo simplex at the top of the differential diagnostic list. The thin lines are not well-developed and the loss of parallelism might be considered to be present. ■ Loss of parallelism is not diagnostic of melanoma. There is no history or other physical findings used to diagnose other etiologies of gray melanonychia striata. ■ Ethnic bands are usually polydactylic (multiple nails) with a positive family history of similar bands. ■ There is no history of ingestion of pigmentogenic drugs (eg, acyclovir, minocyline, hydroxyurea) and only one nail is involved. Drug-induced bands are polydactylic. ■ A negative history of trauma, or inflammation, rules out postinflammatory melanonychia striata. Micro-Hutchinson sign is defined as pigmentation in the cuticle that can only be seen with dermoscopy. It is a nonspecific finding that has to be evaluated in the context of the other dermoscopic findings. It is not diagnostic of nail apparatus melanoma.

PEARLS ■



There are always exceptions to every rule. Gray bands are usually associated with low risk pathology but theoretically they could be seen in nail apparatus melanoma. What looks like gray color to one person might be blackish to another. If in doubt, cut it out!

Chapter 4

Palms, Soles, Nails

RISK ❑ Low ❑ Intermediate ❑ High

DIAGNOSIS

4-27a

❑ ❑ ❑ ❑ ❑ ❑ ❑ ❑

Nevus Seborrheic keratosis Basal cell carcinoma Vascular Dermatofibroma Squamous cell carcinoma Melanoma Other

DISPOSITION

4-27b

❑ No intervention ❑ Follow-up ❑ Histopathologic diagnosis

CASE 27 HISTORY The patient hit his index finger with a hammer but the spot does not seem to be going away. 1. Irregular pigmented bands seen clinically proximal to the main area of discoloration, plus several melanoma-specific criteria, diagnose nail bed melanoma. 2. Nail bed melanoma exists but is rare compared to nail matrix melanoma. 3. Different shades of purple color plus blood spots diagnose subungual hemorrhage. 4. A round proximal edge and a filamentous distal edge can be seen in some, but not all, cases of subungual hemorrhage. 5. The reddish longitudinal bands seen clinically represent the normal, nonpathologic vasculature that can be seen in some but not all nails.

379

380

DERMOSCOPY: AN ILLUSTRATED SELF-ASSESSMENT GUIDE

RISK ✔ Low ❑ ❑ Intermediate ❑ High

DIAGNOSIS Nevus Seborrheic keratosis Basal cell carcinoma Vascular Dermatofibroma Squamous cell carcinoma Melanoma Other

DISPOSITION ❑ No intervention ✔ Follow-up ❑ ❑ Histopathologic diagnosis

4-27c

❑ ❑ ❑ ❑ ❑ ❑ ❑ ✔ ❑

ANSWERS Answers: 2,3,4,5

Discussion: ■ ■

DERMOSCOPIC CRITERIA ■ ■

■ ■

Dried blood (white arrows) Blood spots (blood pebbles) (boxes) Heme breakdown (stars) Longitudinal erythronychia (black arrows clinical image)

■ ■







No feature of this case points to the diagnosis of a nail apparatus melanoma. Criteria that diagnose subungual hemorrhage include: ■ History of trauma ■ Distal location ■ Purple color ■ Purple round blood spots (blood pebbles) ■ An absence of pigmented longitudinal lines The other colors, excluding white, represent heme breakdown. Trauma created the white color and it should not be confused with the bony-white color seen with regression. Lateral nail fold and nail bed melanoma exist but is rare compared to nail matrix melanoma. The faint reddish longitudinal bands seen clinically represent longitudinal erythronychia. ■ In a single nail, it could be associated with amelanotic nail apparatus melanoma. ■ In multiple nails it is a normal variant of the nail apparatus vasculature. ■ The reddish bands should partially or totally blanch easily with pressure and are usually seen on several but not all fingernails. ■ The incidence of longitudinal erythronychia in the general population is not known. Melanoma is the most common pathology included in the differential diagnosis of a subungual hemorrhage.

PEARLS ■



Longitudinal erythronychia is a common finding that can easily be over looked. Longitudinal erythronychia should not be confused with pigmented nail apparatus bands.

Chapter 4

Palms, Soles, Nails

RISK ❑ Low ❑ Intermediate ❑ High

4-28a1

DIAGNOSIS ❑ ❑ ❑ ❑ ❑ ❑ ❑ ❑

Nevus Seborrheic keratosis Basal cell carcinoma Vascular Dermatofibroma Squamous cell carcinoma Melanoma Other

DISPOSITION

4-28b

4-28a2

❑ No intervention ❑ Follow-up ❑ Histopathologic diagnosis

CASE 28 HISTORY There is a questionable history of trauma in this long-standing blue discoloration in a 35-year-old man. 1. Purplish color and a distal filamentous border diagnose subungual hemorrhage. 2. Nail dystrophy and blood spots are seen clinically, which confirm that this is subungual hemorrhage. 3. Homogeneous blue color suggests that this could be a blue nevus. 4. Bluish-white color, polymorphous vessels, and blood spots put a malignant blue nevus in the differential diagnosis. 5. A long-standing history eliminates the need to make a histopathologic diagnosis.

381

382

DERMOSCOPY: AN ILLUSTRATED SELF-ASSESSMENT GUIDE

RISK ❑ Low ✔ Intermediate ❑ ❑ High

DIAGNOSIS Nevus Seborrheic keratosis Basal cell carcinoma Vascular Dermatofibroma Squamous cell carcinoma Melanoma Other

4-28c

✔ ❑ ❑ ❑ ❑ ❑ ❑ ❑ ❑

DISPOSITION ❑ No intervention ❑ Follow-up ✔ Histopathologic diagnosis ❑

ANSWERS Answers: 3,4

Discussion: ■

DERMOSCOPIC CRITERIA ■ ■

■ ■

Bluish-white color (stars) Irregular hairpin vessels (dark arrows) Polymorphous vessels (box) Subungual hemorrhage (red arrows)













This is a difficult case with a rare, unstudied tumor that brings the concept of differential diagnosis into play. The bluish color seen clinically with a long-standing stable history rules out subungual hemorrhage. A banal dermoscopic presentation of a nail apparatus blue nevus would include homogeneous blue color similar to a blue nevus seen on another part of the body that fills the lunula. Atypical features include: ■ Nonhomogeneous bluish-white color filling the lunula ■ Irregular hairpin vessels ■ Polymorphous vessels (eg, pinpoint and irregular linear vessels) ■ Nail plate dystrophy (black arrows in the clinical images) ■ Foci of subungual hemorrhages (boxes in the clinical image) The irregular hairpin vessels could be confused with the distal filamentous component that can be seen in subungual hemorrhage, which would be a solid purple color. The differential diagnosis includes a blue nevus vs malignant blue nevus. ■ The stable history favors a benign blue nevus. ■ The atypical dermoscopic features favor a malignant blue nevus. Histopathology confirmed the benign nature of this lesion. Unusual vasculature, mild inflammation, and extravasated red blood cells help to explain what is seen clinically and dermoscopically.

PEARLS ■ ■

Sequential digital monitoring was used to confirm the stable nature of this lesion. One cannot see what one does not know. Now you know about this rare tumor that you might encounter someday.

Chapter 4

Palms, Soles, Nails

RISK

4-29a

❑ Low ❑ Intermediate ❑ High

DIAGNOSIS ❑ ❑ ❑ ❑ ❑ ❑ ❑ ❑

Nevus Seborrheic keratosis Basal cell carcinoma Vascular Dermatofibroma Squamous cell carcinoma Melanoma Other

4-29b

DISPOSITION ❑ No intervention ❑ Follow-up ❑ Histopathologic diagnosis

CASE 29 HISTORY The physician of a 62-year-old woman noticed this nail discoloration and referred her for dermoscopic evaluation to rule out melanoma. 1. 2. 3. 4.

Bluish-white color and irregular streaks diagnose nail apparatus nodular melanoma. Cuticular mega-capillary loops confirm the diagnosis of melanoma. Bluish-white color and irregular streaks diagnose a malignant blue nevus. Bluish-white homogeneous color, well-demarcated lateral and proximal borders, and a purplish filamentous distal border diagnose a subungual hemorrhage. 5. Bluish-white color rules out a subungual hemorrhage.

383

384

DERMOSCOPY: AN ILLUSTRATED SELF-ASSESSMENT GUIDE

RISK ✔ Low ❑ ❑ Intermediate ❑ High

DIAGNOSIS Nevus Seborrheic keratosis Basal cell carcinoma Vascular Dermatofibroma Squamous cell carcinoma Melanoma Other

DISPOSITION ❑ No intervention ✔ Follow-up ❑ ❑ Histopathologic diagnosis

4-29c

❑ ❑ ❑ ❑ ❑ ❑ ❑ ✔ ❑

ANSWERS Answers: 4

Discussion: ■

DERMOSCOPIC CRITERIA ■







Homogeneous bluish-white color (stars) Well-demarcated proximal and lateral borders (yellow arrows) Purple filamentous distal border (black arrows) Cuticular hairpin-shaped capillary loops (boxes)







This is a classic example of a subungual hemorrhage: ■ Homogeneous bluish-white and purple color ■ Well-demarcated proximal and lateral borders ■ Purple solid distal filamentous border The filamentous lines are solid and have no resemblance to large irregular hairpin vessels seen in the last case. There are normal small hairpin-shaped cuticular capillary loops, which should be compared to the irregular mega-capillary loops (Figure 1-34 in Chapter 1) that can be seen in collagen vascular diseases. ■ Cuticular capillary loops are not a criterion used to diagnose nail apparatus melanoma. Purple is not the only color that can be seen in subungual hemorrhage. Shades of black, blue, brown, and yellow can also be seen and depend on the state of heme breakdown

PEARLS ■



Even though the solid purplish filamentous distal border is not seen all the time, it is a major clue to diagnose subungual hemorrhage and should specifically be looked for with any nail pigmentation. Be cutting-edge and take digital clinical and dermoscopic images of subungual hemorrhage to ensure that you can document the expected distal migration of the discoloration.

Chapter 4

Palms, Soles, Nails

RISK ❑ Low ❑ Intermediate ❑ High

DIAGNOSIS

4-30a

❑ ❑ ❑ ❑ ❑ ❑ ❑ ❑

Nevus Seborrheic keratosis Basal cell carcinoma Vascular Dermatofibroma Squamous cell carcinoma Melanoma Other

DISPOSITION

4-30b

❑ No intervention ❑ Follow-up ❑ Histopathologic diagnosis

CASE 30 HISTORY A podiatrist noticed a progressive darkening of the first toe nail on the right foot of a 58-year-old woman and referred her to rule out melanoma. 1. A normal nail plate and homogeneous brown color diagnose a nail matrix nevus. 2. Purplish color and partial nail plate destruction diagnose a subungual hemorrhage secondary to trauma. 3. Homogeneous brown color and irregular longitudinal bands suggest this could be a nail apparatus melanoma. 4. Loss of parallelism is another point that favors a diagnosis of nail apparatus melanoma. 5. The absence of the Hutchinson sign does not rule out melanoma.

385

386

DERMOSCOPY: AN ILLUSTRATED SELF-ASSESSMENT GUIDE

RISK ❑ Low ❑ Intermediate ✔ High ❑

DIAGNOSIS Nevus Seborrheic keratosis Basal cell carcinoma Vascular Dermatofibroma Squamous cell carcinoma Melanoma Other

4-30c

❑ ❑ ❑ ❑ ❑ ❑ ✔ ❑ ❑

DISPOSITION

ANSWERS

❑ No intervention ❑ Follow-up ✔ Histopathologic diagnosis ❑

Answers: 3,4,5

Discussion: ■ ■

DERMOSCOPIC CRITERIA ■







Diffuse dark homogeneous color (stars) Irregular longitudinal bands (black arrows) Loss of parallelism (yellow arrows) Nail plate destruction (white arrows)

■ ■

There are no criteria to suggest that this could be a subungual hemorrhage. All of the features suggest that this is a melanoma: ■ No history of trauma or inflammation ■ Progressive darkening ■ Partial nail plate destruction ■ Homogeneous dark color ■ Bluish-white color ■ Irregular bands ■ Loss of parallelism The absence of the Hutchinson sign does not rule out melanoma. Amelanotic nail apparatus melanoma, the antithesis of this case, is not rare and could be devoid of any pigmentation to suggest the correct diagnosis. ■ Nail plate destruction is an important clue to suggest this diagnosis in a nonpigmented or pink nail apparatus tumor.

PEARLS ■



You will not have the opportunity to diagnose a case like this if you do not perform a total body skin examination which includes the feet. Have nail polish remover handy so that you can check your female patient’s nails that use nail polish.

Chapter 4

Palms, Soles, Nails

RISK ❑ Low ❑ Intermediate ❑ High

4-31a

DIAGNOSIS ❑ ❑ ❑ ❑ ❑ ❑ ❑ ❑

Nevus Seborrheic keratosis Basal cell carcinoma Vascular Dermatofibroma Squamous cell carcinoma Melanoma Other

DISPOSITION

4-31b

❑ No intervention ❑ Follow-up ❑ Histopathologic diagnosis

CASE 31 HISTORY A dermatologist was treating this finger nail as a fungus for 2 years without improvement. 1. Bluish-green color seen clinically and nail plate destruction diagnose a chronic pseudomonas infection. 2. Nail plate destruction and different colors seen with dermoscopy suggest that this could be a melanocytic lesion. 3. Gray globules identify a melanocytic lesion. 4. There is bony-white color that could represent scarring or regression. 5. The dermoscopic features are nonspecific and melanoma should be included in the differential diagnosis.

387

388

DERMOSCOPY: AN ILLUSTRATED SELF-ASSESSMENT GUIDE

RISK ❑ Low ❑ Intermediate ✔ High ❑

DIAGNOSIS Nevus Seborrheic keratosis Basal cell carcinoma Vascular Dermatofibroma Squamous cell carcinoma Melanoma Other

DISPOSITION ❑ No intervention ❑ Follow-up ✔ Histopathologic diagnosis ❑

4-31c

❑ ❑ ❑ ❑ ❑ ❑ ✔ ❑ ❑

ANSWERS Answers: 2,4,5

Discussion: ■

DERMOSCOPIC CRITERIA ■

■ ■ ■

Light and dark brown color (arrows) Bluish-gray color (black boxes) Peppering (yellow boxes) Scarring (stars)









For the astute clinician, the brown color seen clinically and dermoscopically is a clue that this might be a melanoma. ■ Brown not gray dots and/or globules (peppering) are one criterion used to diagnose a melanocytic lesion. Tinea is clearly in the differential diagnosis and the unresponsiveness to treatment does not rule tinea out. The dermoscopic criteria are nonspecific yet there are clues that point in the direction of the correct diagnosis: ■ Foci of brown color ■ Peppering (gray dots) ■ Foci of bluish color The bony-white color represents the scarring typically found in desmoplastic melanoma, which could be missed by the most experienced dermoscopist. ■ Blanching of the nail apparatus vasculature is in the differential diagnosis of the white color. This is a desmoplastic melanoma which is usually a histopathologic surprise diagnosis for most clinicians. This should not prevent it from being in the differential diagnosis of a scar-like lesion anywhere on the body.

PEARLS ■ ■

No one can win them all, especially when it comes to diagnosing melanoma. Dermoscopy, knowledge, experience, and a high index of suspicion increase the odds that you will not get burned!

Chapter 5

Genitalia

General Instructions: You will find a list of True/False statements following each case history. Select any statements, which you believe to be true. There may be one, more than one or no true statements for any given case. Choose the correct risk, diagnosis and disposition for each case. Then, turn the page to find a detailed discussion and pearls for each case.

Chapter 5

Genitalia

RISK ❑ Low ❑ Intermediate ❑ High

5-1a

DIAGNOSIS ❑ ❑ ❑ ❑ ❑ ❑ ❑ ❑

Nevus Seborrheic keratosis Basal cell carcinoma Vascular Dermatofibroma Squamous cell carcinoma Melanoma Other

DISPOSITION

5-1b

❑ No intervention ❑ Follow-up ❑ Histopathologic diagnosis

CASE 1 HISTORY A 53-year-old woman was referred by her gynecologist who thought she had a melanoma. 1. 2. 3. 4. 5.

Pigment network puts a melanocytic lesion in the differential diagnosis. Pigment network puts a lentigo in the differential diagnosis. The pigment network is prominent yet falls within the low risk category. The pigment network is highly irregular, which favors the diagnosis of a vulvar melanoma. Melanoma specific-criteria are absent, which favors the diagnosis of a benign vulvar lentigo.

391

392

DERMOSCOPY: AN ILLUSTRATED SELF-ASSESSMENT GUIDE

RISK ❑ Low ✔ Intermediate ❑ ❑ High

DIAGNOSIS Nevus Seborrheic keratosis Basal cell carcinoma Vascular Dermatofibroma Squamous cell carcinoma Melanoma Other

DISPOSITION ❑ No intervention ❑ Follow-up ✔ Histopathologic diagnosis ❑

5-1c

❑ ❑ ❑ ❑ ❑ ❑ ❑ ✔ ❑

ANSWERS Answers: 1,2,3,5

Discussion: ■ ■ ■

DERMOSCOPIC CRITERIA ■







Regular pigment network (white boxes) Homogeneous black color (stars) Irregular black globules (yellow box) Irregular black dots (circles)









Clinically, this genital lentigo is more worrisome than the dermoscopic picture. A melanocytic lesion and lentigo both can have a pigment network. The pigment network is thick with sharp border demarcation, reminiscent of an ink-spot lentigo. ■ An ink-spot lentigo is usually found on sun-exposed areas and does not have this clinical appearance. Typically, they are flat black macules as opposed to this lesion that appears raised. The jet black color is unusual and a red flag for concern. Genital lentigines are usually a light or dark brown color, but can be difficult to differentiate clinically and/or dermoscopically from melanoma. Genital melanoma is rare (4% of all melanomas) and presents with melanoma-specific criteria similar to those seen on the trunk and extremities (eg, asymmetry of color and structure, multicomponent global pattern, irregular pigment network, irregular dots, globules, irregular blotches, irregular streaks, multiple colors, regression, polymorphous vessels). The focus of irregular globules represents fragments of the pigment network. The irregular black dots have no diagnostic significance.

PEARLS ■





Dermoscopic evaluation can help to avoid potentially aggressive, often mutilating, surgery to rule out melanoma in this very sensitive area. In this case, sequential digital clinical and dermoscopic monitoring rather than histopathologic evaluation can be used by the more experienced and confident dermoscopist. The presence of a chaperone should be the standard of care when examining genital lesions.

Chapter 5

Genitalia

RISK ❑ Low ❑ Intermediate ❑ High

5-2a

DIAGNOSIS ❑ ❑ ❑ ❑ ❑ ❑ ❑ ❑

Nevus Seborrheic keratosis Basal cell carcinoma Vascular Dermatofibroma Squamous cell carcinoma Melanoma Other

DISPOSITION

5-2b

❑ No intervention ❑ Follow-up ❑ Histopathologic diagnosis

(Reprinted, with permission, from Stolz W, Braun-Falco O, Bilek P et al. (2002) Color Atlas of Dermatoscopy. Second Edition. Blackwell Publishing, Oxford.)

CASE 2 HISTORY A 34-year-old woman has this dark brown pigmentation on her vulva for many years. 1. A multicomponent global pattern, irregular dots, globules, streaks, and irregular black blotch diagnose a melanoma. 2. This is a gray-zone lesion with a benign globular and benign parallel pattern and an irregular black blotch. 3. If it were not for the irregular black blotch, this would be one of the benign patterns that can be seen in a genital lentigo. 4. The parallel line segments are similar to the parallel patterns that can be seen on the palms and soles. 5. Combinations of benign patterns can be seen in the genital area.

393

394

DERMOSCOPY: AN ILLUSTRATED SELF-ASSESSMENT GUIDE

RISK ❑ Low ✔ Intermediate ❑ ❑ High 1 2

DIAGNOSIS Nevus Seborrheic keratosis Basal cell carcinoma Vascular Dermatofibroma Squamous cell carcinoma Melanoma Other

4 3

5-2c

❑ ❑ ❑ ❑ ❑ ❑ ❑ ✔ ❑

DISPOSITION

ANSWERS

❑ No intervention ❑ Follow-up ✔ Histopathologic diagnosis ❑

Answers: 2,3,4,5

Discussion: ■



DERMOSCOPIC CRITERIA ■

■ ■ ■

■ ■

Multicomponent global pattern (1,2,3,4) Parallel pattern (black arrows) Globular pattern (black circles) Irregular black blotch (white arrows) Homogeneous gray color (box) White dots (white circles)



■ ■





The history is low risk, whereas clinically and dermoscopically there are worrisome features. There is a multicomponent global pattern (melanoma-specific criterion) with four different areas that put melanoma in the differential diagnosis: ■ Globular pattern made up of regular dots and globules (zone 1). ■ Parallel pattern made up of regular thick parallel line segments (zone 2). ■ Irregular black blotch (zone 3). ■ Homogeneous gray color (zone 4). Other than the prominent irregular black blotch, this lesion has a combination of benign patterns that can be seen in a lentigo. ■ A multicomponent global pattern is not diagnostic for a melanoma. More than one benign pattern can be seen in a genital lentigo. The focus of homogeneous gray color represents melanophages that can result from inflation or trauma. A specific name for the white dots is not known. They have no diagnostic significance. The differential diagnosis includes a lentigo, melanocytic nevus, combined nevus (blue nevus underlying junctional/compound nevus), and collision tumor (eg, lentigo and nevus, lentigo and melanoma).

PEARLS ■





If it is necessary to make a histopathologic diagnosis, one can use the most atypical dermoscopic area for an incisional biopsy and/or biopsies to avoid more aggressive surgical techniques. To rule out a collision tumor, more than one incisional biopsy should be performed. Alert your pathologist to be on the look-out for a possible collision tumor when one is in the differential diagnosis.

Chapter 5

Genitalia

RISK ❑ Low ❑ Intermediate ❑ High

5-3a

5-3a

DIAGNOSIS ❑ ❑ ❑ ❑ ❑ ❑ ❑ ❑

Nevus Seborrheic keratosis Basal cell carcinoma Vascular Dermatofibroma Squamous cell carcinoma Melanoma Other

DISPOSITION

5-3b

❑ No intervention ❑ Follow-up ❑ Histopathologic diagnosis

CASE 3 HISTORY The gynecologist of a 62-year-old woman discovered this lesion and referred the patient for a biopsy. 1. Irregular dark blotches, irregular dots and globules, plus bluish-white color diagnose a mucosal melanoma. 2. This is a melanocytic nevus with a globular pattern. 3. Ring-like structures put a lentigo in the differential diagnosis. 4. Irregular dark blotches and the clinical appearance of the lesion trump the low risk, ring-like pattern, and a histopathologic diagnosis is indicated. 5. The completely banal dermoscopic picture trumps the worrisome clinical appearance and a histopathologic diagnosis is not indicated.

395

396

DERMOSCOPY: AN ILLUSTRATED SELF-ASSESSMENT GUIDE

RISK ❑ Low ✔ Intermediate ❑ ❑ High

DIAGNOSIS Nevus Seborrheic keratosis Basal cell carcinoma Vascular Dermatofibroma Squamous cell carcinoma Melanoma Other

DISPOSITION ❑ No intervention ❑ Follow-up ✔ Histopathologic diagnosis ❑

5-3c

❑ ❑ ❑ ❑ ❑ ❑ ❑ ✔ ❑

ANSWERS Answers: 3, 4

Discussion: ■



DERMOSCOPIC CRITERIA ■ ■

Ring-like pattern (boxes) Irregular dark blotches (arrows)





This genital lentigo another gray-zone lesion that is not perfectly banal nor with sufficient high risk criteria to diagnose a melanoma. Ring-like structures and not dots and globules are identifiable: ■ Multiple round to oval white or tan structures with central clearing. ■ Well-demarcated hyperpigmented borders. ■ They can be diffuse, clustered, and appear grape-like. A ring-like pattern can be seen in genital lentigo, bowenoid papulosis, dermatofibromas, seborrheic keratosis, and lentigo maligna on the face (circle within a circle). The irregular dark blotches are a red flag for concern and together with the irregular clinical appearance is an indication to make a histopathologic diagnosis.

PEARLS ■



Glabrous skin (eg, lips, palms, soles, mucocutaneous junction, and mucosa) can have the same dermoscopic patterns that are considered site-specific. The dermoscopic features of the genitalia have not been well studied. If there are any clues (eg, history, clinical appearance, melanoma-specific criteria) that you might be dealing with a high risk lesion, it is better to err on the side of caution and make a histopathologic diagnosis.

Chapter 5

Genitalia

RISK ❑ Low ❑ Intermediate ❑ High

DIAGNOSIS

5-4a

❑ ❑ ❑ ❑ ❑ ❑ ❑ ❑

Nevus Seborrheic keratosis Basal cell carcinoma Vascular Dermatofibroma Squamous cell carcinoma Melanoma Other

DISPOSITION

5-4b

❑ No intervention ❑ Follow-up ❑ Histopathologic diagnosis

CASE 4 HISTORY A 21-year-old woman discovered this lesion and asked to have it checked during a visit for an unrelated skin complaint. 1. 2. 3. 4.

Ring-like, fish scale-like, and parallel patterns diagnose a lentigo. There is no pattern characterized as fish scale-like. There are no high risk dermoscopic criteria to suggest this is a melanoma. The fish scale-like pattern is a variant of the ring pattern and is thought to be created by pressure from instrumentation. 5. In this case, sequential digital clinical and dermoscopic monitoring can be used instead of making a histopathologic diagnosis.

397

398

DERMOSCOPY: AN ILLUSTRATED SELF-ASSESSMENT GUIDE

RISK ✔ Low ❑ ❑ Intermediate ❑ High

DIAGNOSIS Nevus Seborrheic keratosis Basal cell carcinoma Vascular Dermatofibroma Squamous cell carcinoma Melanoma Other

DISPOSITION ❑ No intervention ✔ Follow-up ❑ ❑ Histopathologic diagnosis

5-4c

❑ ❑ ❑ ❑ ❑ ❑ ❑ ✔ ❑

ANSWERS Answers: 1,3,4,5

Discussion: ■



DERMOSCOPIC CRITERIA ■





Ring-like pattern (white boxes) Fish scale-like pattern (black boxes) Parallel pattern (arrows)





Clinically, but not dermoscopically, this genital lentigo is worrisome characterized by asymmetry, irregular borders, and different shades of brown color. Dermoscopically, the picture is low risk with variants of the ring, fish scale-like, and parallel patterns. ■ Only a few of the ring structures have hyperpigmented borders. ■ When there are only two parallel line segments they are referred to as “train tracks.” The fish-scale pattern is thought to be a variant of the ring-like pattern created by pressure from instrumentation and is characterized by multiple curves or semicircular U- or V-shapes mimicking fish scales. ■ The hyphal pattern not seen here, is made up of short-branched line segments that look like fungal hyphae and is considered a variant of the ring-like and fish-scale pattern. There are no dermoscopic criteria to suggest that this is a melanoma.

PEARLS ■



Dermoscopy is an invaluable tool to help diagnose genital pigmentation that may or may not look high risk clinically. It is essential to learn all of the site-specific criteria and patterns to avoid misdiagnosis.

Chapter 5

Genitalia

RISK ❑ Low ❑ Intermediate ❑ High

5-5a

DIAGNOSIS ❑ ❑ ❑ ❑ ❑ ❑ ❑ ❑

Nevus Seborrheic keratosis Basal cell carcinoma Vascular Dermatofibroma Squamous cell carcinoma Melanoma Other

DISPOSITION

5-5b

❑ No intervention ❑ Follow-up ❑ Histopathologic diagnosis

CASE 5 HISTORY A 50-year-old woman was at the gynecologist who found this pigmented lesion. 1. A multicomponent global pattern and bluish-white and gray colors put melanoma in the differential diagnosis. 2. The bluish-white and gray colors could represent discoloration secondary to trauma or inflammation. 3. Foci of ring and fish scale-like structures and parallel line segments put a lentigo in the differential diagnosis. 4. This is a worrisome lesion clinically and dermoscopically. A wide excision without a preliminary incisional biopsy is indicated because it must be a melanoma. 5. To avoid aggressive and unnecessary surgery, because this could be a low risk lesion, multiple incisional biopsies should be considered in order to make an accurate diagnosis.

399

400

DERMOSCOPY: AN ILLUSTRATED SELF-ASSESSMENT GUIDE

RISK

1

❑ Low ❑ Intermediate ✔ High ❑ 3

DIAGNOSIS Nevus Seborrheic keratosis Basal cell carcinoma Vascular Dermatofibroma Squamous cell carcinoma Melanoma Other

2

5

5-5c

❑ ❑ ❑ ❑ ❑ ❑ ❑ ✔ ❑

4

DISPOSITION ❑ No intervention ❑ Follow-up ✔ Histopathologic diagnosis ❑

ANSWERS Answers: 1,2,3,5

Discussion: ■

DERMOSCOPIC CRITERIA ■







■ ■



Asymmetry of color and structure Multicomponent global pattern (1,2,3,4,5) Bluish-white and gray colors (arrows) Ring-like structures (black boxes) Parallel structures (yellow box) Fish scale-like pattern (white box) Homogeneous brownish-gray color (stars)











This is a worrisome dermoscopic picture because there are several melanoma-specific criteria: ■ Asymmetry of color and structure ■ Multicomponent global pattern ■ Bluish-white color ■ Gray color There are foci of poorly-developed low risk patterns: ■ Ring-like (individual rings) ■ Parallel The fish scale-like pattern is well developed with inverted v-shaped line segments. Bluish-white color should always be a red flag for concern that a lesion might be a melanoma. Gray color indicates the presence of melanophages created by inflammation or trauma both commonly seen in the genital area. Even with the identification of low risk criteria, the overall picture is high risk and a histopathologic diagnosis is indicated posthaste.

PEARLS ■





The diagnosis of a lentigo was not a good dermoscopic–pathologic correlation. Make sure that your pathologist has experience diagnosing pigmented genital lesions. One should not be politically correct when a patient’s well-being is on the line. Asking for second histopathologic opinion from a more experienced dermatopathologist should be your standard of care in certain situations. Multiple incisional biopsies are indicated at times with multicomponent lesions so that the correct diagnosis is not missed.

Chapter 5

Genitalia

RISK ❑ Low ❑ Intermediate ❑ High

DIAGNOSIS

5-6a

❑ ❑ ❑ ❑ ❑ ❑ ❑ ❑

Nevus Seborrheic keratosis Basal cell carcinoma Vascular Dermatofibroma Squamous cell carcinoma Melanoma Other

DISPOSITION

5-6b

❑ No intervention ❑ Follow-up ❑ Histopathologic diagnosis

CASE 6 HISTORY A 45-year-old woman with biopsy-proven genital lichen sclerosus et atrophicus was seen at her routine 6-month follow-up examination. Historically, the area was bony-white. She was not aware of the new dark discoloration. 1. Milky-red areas and bluish-white color diagnose a melanoma arising in chronic lichen sclerosus et atrophicus. 2. Polymorphous vessels confirm this to be a melanoma. 3. This represents a partially pigmented and amelanotic genital melanoma. 4. Purple color suggests that the new discoloration could simply represent hemorrhage. 5. A wide excision is indicated. There is no need for a preliminary incisional biopsy.

401

402

DERMOSCOPY: AN ILLUSTRATED SELF-ASSESSMENT GUIDE

RISK ✔ Low ❑ ❑ Intermediate ❑ High

DIAGNOSIS Nevus Seborrheic keratosis Basal cell carcinoma Vascular Dermatofibroma Squamous cell carcinoma Melanoma Other

DISPOSITION ❑ No intervention ❑ Follow-up ✔ Histopathologic diagnosis ❑

5-6c

❑ ❑ ❑ ❑ ❑ ❑ ❑ ✔ ❑

ANSWERS Answers: 4

Discussion: ■



DERMOSCOPIC CRITERIA ■ ■ ■

Hemorrhage (stars) Blood pebbles (box) Lichen sclerosus et atrophicus (arrows) ■





The purple color is a clue that this new discoloration might be secondary to hemorrhage, and it is. One should still think in terms of dermoscopic differential diagnosis. ■ The bluish-white color could represent invasive melanoma or different shades of hemorrhage. ■ The homogeneous purplish-red color could represent blood, the milkyred areas of a melanoma, or the amelanotic component of a partially pigmented melanoma. ■ The bony-white color could represent regression in a melanoma or lichen sclerosus et atrophicus. There are no polymorphous vessels (eg, pinpoint, irregular linear, corkscrew). Squamous cell carcinoma and not melanoma has been reported to develop in chronic lichen sclerosus et atrophicus. This is not the dermoscopic picture of squamous cell carcinoma: ■ Glomerular or irregular hairpin-shaped vessels Since the differential diagnosis includes melanoma, a histopathologic diagnosis is indicated.

PEARLS ■



At the time the discoloration was first discovered, since hemorrhage was at the top of the differential diagnosis, short-term (q 3 months) digital clinical and dermoscopic monitoring would be an alternative to a surgical intervention. One should see the expected changes in heme over time, which would confirm the diagnosis. Close observation is not always a bad idea in certain situations.

Chapter 5

Genitalia

RISK ❑ Low ❑ Intermediate ❑ High

DIAGNOSIS

5-7a

❑ ❑ ❑ ❑ ❑ ❑ ❑ ❑

Nevus Seborrheic keratosis Basal cell carcinoma Vascular Dermatofibroma Squamous cell carcinoma Melanoma Other

DISPOSITION

5-7b

❑ No intervention ❑ Follow-up ❑ Histopathologic diagnosis

CASE 7 HISTORY According to the patient, this lesion literally came out of nowhere within a 4-month period. There was no history of trauma or any pre-existing genital problem. 1. 2. 3. 4.

Clear cut polymorphous vessels suggest this could be a melanoma. Different shades of purple and bluish-white color diagnose hemorrhage. Bluish-white color is a red flag for concern that this could be a melanoma. There is sufficient clinical and dermoscopic evidence to justify a wide excision to rule out invasive melanoma. 5. One does not need dermoscopy to realize that this could be a bad melanoma, but it helps to confirm the clinical impression and guide one’s surgical approach.

403

404

DERMOSCOPY: AN ILLUSTRATED SELF-ASSESSMENT GUIDE

RISK ❑ Low ❑ Intermediate ✔ High ❑

DIAGNOSIS Nevus Seborrheic keratosis Basal cell carcinoma Vascular Dermatofibroma Squamous cell carcinoma Melanoma Other

DISPOSITION ❑ No intervention ❑ Follow-up ✔ Histopathologic diagnosis ❑

5-7c

❑ ❑ ❑ ❑ ❑ ❑ ✔ ❑ ❑

ANSWERS Answers: 1,3,4,5

Discussion: ■

DERMOSCOPIC CRITERIA ■

■ ■

Polymorphous vessels ■ Pinpoint (white boxes) ■ Irregular linear and corkscrew (red box) ■ Combinations of irregular shapes (yellow boxes) Bluish-white color (white stars) Amelanotic component (red stars)







The presence of polymorphous vessels (eg, pinpoint, irregular linear, cork screw) rules out hemorrhage. ■ Polymorphous vessels may or may not be sharply in focus and easy to identify. This lesion has a reddish amelanotic and bluish-white pigmented component. Both colors are associated with invasive melanoma. There are no differential diagnostic possibilities. A wide excision of the entire lesion is indicated without the need for preliminary incisional biopsy. This is the end of the line as far as how bad a genital/mucosal melanoma can be. The goal is to diagnose a melanoma before it gets this bad or better yet, a benign premalignant lesion.

PEARLS ■



One can see a progression of severity in the last three cases characterized by bluish-white color. Bluish-white color is always a red flag for concern, and a careful focused dermoscopic evaluation is indicated.

Chapter 5

Genitalia

RISK ❑ Low ❑ Intermediate ❑ High

DIAGNOSIS

5-8a

❑ ❑ ❑ ❑ ❑ ❑ ❑ ❑

Nevus Seborrheic keratosis Basal cell carcinoma Vascular Dermatofibroma Squamous cell carcinoma Melanoma Other

DISPOSITION

5-8b

❑ No intervention ❑ Follow-up ❑ Histopathologic diagnosis

CASE 8 HISTORY A 35-year-old man has this pigmentation for several years without change. 1. Brown globules identify a melanocytic lesion. 2. Asymmetry of color and structure, irregular brown globules, and irregular streaks diagnose a dysplastic nevus. 3. Multifocal pigmentation seen clinically plus globular and parallel patterns diagnose benign penile melanosis. 4. The globular pattern is only found in melanocytic nevi and never in the genital area. 5. The extensive nature of the pigmentation and irregular fingerprint pattern diagnose a melanoma.

405

406

DERMOSCOPY: AN ILLUSTRATED SELF-ASSESSMENT GUIDE

RISK ✔ Low ❑ ❑ Intermediate ❑ High

DIAGNOSIS Nevus Seborrheic keratosis Basal cell carcinoma Vascular Dermatofibroma Squamous cell carcinoma Melanoma Other

5-8c

❑ ❑ ❑ ❑ ❑ ❑ ❑ ✔ ❑

DISPOSITION

ANSWERS

❑ No intervention ✔ Follow-up ❑ ❑ Histopathologic diagnosis

Answers: 3

Discussion: ■



DERMOSCOPIC CRITERIA ■





Linear globular pattern (yellow boxes) Nonlinear globular pattern (black boxes) Parallel pattern (arrows)









There are no clinical or dermoscopic features to suggest this could be a melanoma. This is a classic clinical and dermoscopic example of penile melanosis (lentigines). Extensive genital melanosis can have a single or multiple dermoscopic patterns. In this case there is the globular and parallel pattern. The globular pattern is made up of aggregated round structures that have a linear and nonlinear distribution. There are only a few foci of parallel lines, which can be similar to the parallel patterns seen on the palms and soles. More extensive parallel lines can resemble the fingerprint pattern seen in lentigines on sun-exposed areas.

PEARLS ■



In order not to miss high risk dermoscopic criteria (eg, irregular dark blotches, bluish-white color, polymorphous vessels) one should examine the entire lesion. There is no association between melanosis and melanoma. The only problem for the novice dermoscopist is to not know the important patterns and criteria, and make a misdiagnosis that could result in inappropriate aggressive surgery or no surgery at all.

Chapter 5

Genitalia

RISK ❑ Low ❑ Intermediate ❑ High

5-9a

DIAGNOSIS ❑ ❑ ❑ ❑ ❑ ❑ ❑ ❑

Nevus Seborrheic keratosis Basal cell carcinoma Vascular Dermatofibroma Squamous cell carcinoma Melanoma Other

DISPOSITION

5-9b

❑ No intervention ❑ Follow-up ❑ Histopathologic diagnosis

CASE 9 HISTORY A 46-year-old man, with a history of lichen sclerosus et atrophicus and condyloma, has pigmentation on his penis for several years. 1. The pigmentation is post-inflammatory, secondary to the destructive treatment of his condyloma. 2. The scarring seen clinically, regression seen dermoscopically, and irregular globules diagnose a penile melanoma. 3. This is a classic example of melanoma arising in a condyloma. 4. The globular and fingerprint patterns diagnose a lentigo. 5. Fingerprint parallel patterns can have thick and/or thin line segments.

407

408

DERMOSCOPY: AN ILLUSTRATED SELF-ASSESSMENT GUIDE

RISK ✔ Low ❑ ❑ Intermediate ❑ High

DIAGNOSIS Nevus Seborrheic keratosis Basal cell carcinoma Vascular Dermatofibroma Squamous cell carcinoma Melanoma Other

5-9c

❑ ❑ ❑ ❑ ❑ ❑ ❑ ✔ ❑

DISPOSITION ❑ No intervention ✔ Follow-up ❑ ❑ Histopathologic diagnosis

ANSWERS Answers: 4,5

Discussion: ■



DERMOSCOPIC CRITERIA ■ ■



Globular pattern (black boxes) Fingerprint pattern (yellow boxes) Scarring (stars)



The patient has an irregularly pigmented lesion with scarring secondary to lichen sclerosus et atrophius. Even though the clinical appearance is worrisome the dermoscopic criteria are low risk: ■ Foci of a globular pattern ■ Foci of the parallel fingerprint pattern with thick line segments There are no high risk dermoscopic features. ■ The grayish color has no diagnostic significance. ■ The wart seen clinically has no relationship to the lentigo.

PEARL ■

Once again, dermoscopy gives us a cleaner picture of a clinically-worrisome pigmentation, eliminating the need for a surgical intervention.

Chapter 5

Genitalia

RISK ❑ Low ❑ Intermediate ❑ High

DIAGNOSIS

5-10a

❑ ❑ ❑ ❑ ❑ ❑ ❑ ❑

Nevus Seborrheic keratosis Basal cell carcinoma Vascular Dermatofibroma Squamous cell carcinoma Melanoma Other

DISPOSITION

5-10b

❑ No intervention ❑ Follow-up ❑ Histopathologic diagnosis

CASE 10 HISTORY A 26-year-old man noticed a fast-growing lesion on his penis, which has been stable for 3 months. 1. Brown globules identify a melanocytic lesion. 2. Grayish -black color is a red flag for concern. 3. There is asymmetry of color and structure and a multicomponent global pattern that can be seen in banal and high risk pathology. 4. This is a high risk lesion without a differential diagnosis that warrants a complete excision even though it will be very destructive in this sensitive area. 5. Picking the most atypical dermoscopic area for an incisional biopsy is the better way to go rather than a complete excision.

409

410

DERMOSCOPY: AN ILLUSTRATED SELF-ASSESSMENT GUIDE

RISK ❑ Low ✔ Intermediate ❑ ❑ High

1 2

DIAGNOSIS

3

Nevus Seborrheic keratosis Basal cell carcinoma Vascular Dermatofibroma Squamous cell carcinoma Melanoma Other

4

5-10c

✔ ❑ ❑ ❑ ❑ ❑ ❑ ❑ ❑

ANSWERS DISPOSITION ❑ No intervention ❑ Follow-up ✔ Histopathologic diagnosis ❑

Answers: 1,2,3,5

Discussion: ■

DERMOSCOPIC CRITERIA ■













Asymmetry of color and structure Multicomponent global pattern (1,2,3,4) Irregular brown dots/globules (white box) Irregular black dots/globules (black boxes) Irregular grayish-black blotches (yellow boxes) Multifocal hypopigmentation (stars) Diffuse erythema (pink color)







At first blush, the global dermoscopic picture of what turned out to be a dermal nevus with blue nevus component (combined nevus) is worrisome but one must keep in mind the concept of a dermoscopic differential diagnosis. ■ Even a very irregular clinical and/or dermoscopic picture might not represent high risk pathology. The lesion has well-developed melanoma-specific criteria: ■ Asymmetry of color and structure ■ Multicomponent global pattern ■ Irregular black dots/globules ■ Irregular grayish-black blotches/color ■ Melanoma-specific criteria are very sensitive and specific for high risk pathology (eg, dysplastic nevi, melanoma) but not diagnostic and can be seen in melanocytic, nonmelanocytic benign, and malignant pathology. Diffuse erythema is always a red flag for concern that can be seen in banal and in high risk pathology. The differential diagnosis includes: nevus, dysplastic nevus, combined nevus (blue nevus with overlying junctional/compound/dermal nevus), melanoma.

PEARLS ■







Aggressive surgical intervention should be avoided if the differential diagnosis includes low risk pathology. If diffuse erythema blanches away leaving low risk criteria, in most cases the lesion will be low risk. If erythema blanches away leaving high risk or nonspecific criteria then you could be dealing with a high risk lesion. If you refer a patient with a lesion in this location to a colleague in or out of our field to make the histopathologic diagnosis for you, make sure that you convey the concept of a conservative surgical approach to them. If in doubt, cut it out “does not always mean with a heavy hand!”

Chapter 5

Genitalia

RISK ❑ Low ❑ Intermediate ❑ High

5-11a

DIAGNOSIS ❑ ❑ ❑ ❑ ❑ ❑ ❑ ❑

Nevus Seborrheic keratosis Basal cell carcinoma Vascular Dermatofibroma Squamous cell carcinoma Melanoma Other

DISPOSITION

5-11b

❑ No intervention ❑ Follow-up ❑ Histopathologic diagnosis

CASE 11 HISTORY A 20-year-old man had this lesion for 6 months. 1. 2. 3. 4. 5.

This could be diagnosed a melanocytic lesion because there are globules or by default criteria. There is a pigmented and amelanotic component, which are potentially high risk criteria. Well-demarcated lacunae and hemorrhage diagnose a thrombosed hemangioma. This is the banal homogeneous pattern that can be seen in a lentigo. A dysplastic nevus and melanoma are in the differential diagnosis; therefore, a histopathologic diagnosis should be made posthaste.

411

412

DERMOSCOPY: AN ILLUSTRATED SELF-ASSESSMENT GUIDE

RISK ❑ Low ❑ Intermediate ✔ High ❑

DIAGNOSIS Nevus Seborrheic keratosis Basal cell carcinoma Vascular Dermatofibroma Squamous cell carcinoma Melanoma Other

2 3

5-11c

✔ ❑ ❑ ❑ ❑ ❑ ❑ ❑ ❑

1

DISPOSITION ❑ No intervention ❑ Follow-up ✔ Histopathologic diagnosis ❑

ANSWERS Answers: 1,2,5

Discussion: ■

DERMOSCOPIC CRITERIA ■



■ ■





Asymmetry of color and structure Multicomponent global pattern (1,2,3) Irregular brown globules (box) Irregular grayish-black blotch (black arrows) Polymorphous vessels (yellow arrows) Amelanotic component (stars)









Clinically and dermoscopically, this is a high risk lesion (eg, dysplastic nevus, melanoma) until proven otherwise by making a histopathologic diagnosis. ■ The histopathologic diagnosis was a dysplastic nevus with “Spitzoid” features. There are well-developed melanoma-specific criteria with a differential diagnosis: ■ Multicomponent global pattern ■ Irregular grayish-black blotches that could represent atypical melanocytes or melanophages. ■ Amelanotic component that represents one component of a benign nevus or the amelanotic component of a partially pigmented melanoma. The focus of brown globules and polymorphous vessels are not easy to identify. The vessels should be compared to the well-developed vessels in Case 7. One can see a homogeneous global pattern in a lentigo or melanoma characterized by homogeneous color without local criteria. ■ A banal homogeneous global pattern has uniform light and/or dark color. ■ A high risk homogeneous global pattern has different shades of black, brown, gray, and/or blue color. There are no criteria to diagnose a hemangioma (eg, lacunae) or a collision tumor.

PEARLS ■



Based on the history, clinical, and dermoscopic findings, this could be a high risk lesion. It is small and an incisional biopsy is not practical and could miss the important pathology. Therefore, in this case a complete, yet conservative excision is indicated. You can always go back and make a wider excision with high risk pathology. However, an aggressive wide excision cannot be undone if the pathology turns out to be banal!

Chapter 5

Genitalia

RISK ❑ Low ❑ Intermediate ❑ High

5-12a

DIAGNOSIS ❑ ❑ ❑ ❑ ❑ ❑ ❑ ❑

Nevus Seborrheic keratosis Basal cell carcinoma Vascular Dermatofibroma Squamous cell carcinoma Melanoma Other

DISPOSITION

5-12b

❑ No intervention ❑ Follow-up ❑ Histopathologic diagnosis

CASE 12 HISTORY A 54-year-old man had this lesion on his penis for several years without symptoms or any changes. 1. 2. 3. 4.

Fat-fingers diagnose a pigmented seborrheic keratosis. The homogeneous pattern diagnoses a lentigo. Irregular brown globules put a melanocytic lesion in the differential diagnosis. Nonhomogeneous hyperpigmentation with a bluish-white tinge, irregular brown globules, and irregular streaks put melanoma in the differential diagnosis. 5. Without banal criteria a histopathologic diagnosis is indicated.

413

414

DERMOSCOPY: AN ILLUSTRATED SELF-ASSESSMENT GUIDE

RISK ❑ Low ❑ Intermediate ✔ High ❑

DIAGNOSIS Nevus Seborrheic keratosis Basal cell carcinoma Vascular Dermatofibroma Squamous cell carcinoma Melanoma Other

DISPOSITION ❑ No intervention ❑ Follow-up ✔ Histopathologic diagnosis ❑

5-12c

❑ ❑ ❑ ❑ ❑ ❑ ❑ ✔ ❑

ANSWERS Answers: 3,4,5

Discussion: ■



DERMOSCOPIC CRITERIA ■





■ ■

Asymmetry of color and structure Irregular brown globules (yellow boxes) Nonhomogeneous hyperpigmentation with bluish-white tinge (stars) Irregular streaks (arrows ) Parallel line segments (red boxes)





Clinically and dermoscopically, this is a worrisome lesion that warrants a histopathologic diagnosis. There are no criteria associated with a lentigo (eg, homogeneous, globular, ring-like patterns) or seborrheic keratosis. ■ Fat-fingers with digit-like line segments are not identifiable. There are melanoma-specific criteria that put a dysplastic nevus or melanoma in the differential diagnoses: ■ Asymmetry of color and structure ■ Irregular brown globules ■ Irregular streaks ■ Nonhomogeneous/irregular hyperpigmentation with a bluish-white tinge This turned out to be bowenoid papulosis which is a clinical and pathologic diagnosis. The dermoscopic features have not been studied. ■ Bowenoid papulosis may be considered to be a transition state between a genital wart and Bowen disase.

PEARLS ■





Dermoscopy has done its job by ruling out low risk pathology that does not need a surgical intervention. Getting up close is not fun but a must in the genital area with a suspicious lesion that warrants dermoscopic examination. The presence of a chaperone should be the standard of care in these situations.

Chapter 5

Genitalia

RISK ❑ Low ❑ Intermediate ❑ High

DIAGNOSIS

5-13a

❑ ❑ ❑ ❑ ❑ ❑ ❑ ❑

Nevus Seborrheic keratosis Basal cell carcinoma Vascular Dermatofibroma Squamous cell carcinoma Melanoma Other

DISPOSITION

5-13b

❑ No intervention ❑ Follow-up ❑ Histopathologic diagnosis

CASE 13 HISTORY A colleague referred this 70-year-old man for a routine skin examination. There was a history of squamous cell carcinoma on his penis. The patient brought this spot to our attention. 1. Clinically, but not dermoscopically, this is a worrisome lesion. 2. Well-developed, ring-like structures put a lentigo at the top of the differential diagnosis. 3. Regression, polymorphous vessels, and irregular globules diagnose an in situ acral lentiginous melanoma. 4. Bowenoid papulosis is in the clinical but not dermoscopic differential diagnosis. 5. Bowenoid papulosis should be in the clinical and dermoscopic differential diagnosis.

415

416

DERMOSCOPY: AN ILLUSTRATED SELF-ASSESSMENT GUIDE

RISK ❑ Low ✔ Intermediate ❑ ❑ High

DIAGNOSIS Nevus Seborrheic keratosis Basal cell carcinoma Vascular Dermatofibroma Squamous cell carcinoma Melanoma Other

5-13c

❑ ❑ ❑ ❑ ❑ ❑ ❑ ✔ ❑

DISPOSITION ❑ No intervention ❑ Follow-up ✔ Histopathologic diagnosis ❑

ANSWERS Answers: 1,2,4,5

Discussion: ■

DERMOSCOPIC CRITERIA ■ ■

Ring-like structures (boxes) Normal skin color (stars)



■ ■ ■

We now have a series of bowenoid papulosis cases that look completely different clinically and dermoscopically. One with melanoma-specific criteria and one with the ring-like pattern associated with a genital lentigo. Ring-like structures are well-developed and should not be confused with the globules associated with a melanocytic lesion. The normal skin color should not be confused with regression. There are no high risk dermoscopic criteria. The irregular clinical appearance trumped the low risk dermoscopic pattern and led to an incisional biopsy.

PEARLS ■



Do not ignore gut feelings that a lesion might be high risk, especially in the genital area where much work needs to be done to clarify the global patterns and local criteria associated with low, intermediate, and high risk pathology. There was not a good clinico–dermoscopic–pathologic correlation since a ring-like pattern has only been associated with lentigines in the genital area. A review of the pathology by another experienced dermatopathologist confirmed the diagnosis of bowenoid papulosis.

Chapter 5

Genitalia

RISK

DIAGNOSIS ❑ ❑ ❑ ❑ ❑ ❑ ❑ ❑

Nevus Seborrheic keratosis Basal cell carcinoma Vascular Dermatofibroma Squamous cell carcinoma Melanoma Other

DISPOSITION ❑ No intervention ❑ Follow-up ❑ Histopathologic diagnosis

5-14b2

5-14b1

5-14a

❑ Low ❑ Intermediate ❑ High

CASE 14 HISTORY The brown discoloration and pink nodule were found in a 72-year-old woman when her primary care physician was about to perform a routine rectal examination. 1. The bluish-white and brown discoloration seen in both dermoscopic images are a red flag for concern that this could be a rectal melanoma. 2. Rectal melanoma has never been reported and this represents a thrombosed hemorrhoid. 3. Multiple melanoma-specific criteria are seen in dermoscopy Image 2 (eg, irregular brown dots and globules, irregular hyperpigmentation, polymorphous vessels, and bluish-white color), and put melanoma at the top of the differential diagnostic list. 4. Purplish-brown color puts senile purpura in the differential diagnosis. 5. Since this is a difficult area to biopsy and the clinical and dermoscopic features are equivocal, sequential digital clinical and dermoscopic monitoring is indicated instead of making a histopathologic diagnosis. The patient should return in 6 months for a follow-up examination.

417

418

DERMOSCOPY: AN ILLUSTRATED SELF-ASSESSMENT GUIDE

RISK ❑ Low ❑ Intermediate ✔ High ❑

DIAGNOSIS ❑ ❑ ❑ ❑ ❑ ❑ ✔ ❑ ❑

5-14c1

Nevus Seborrheic keratosis Basal cell carcinoma Vascular Dermatofibroma Squamous cell carcinoma Melanoma Other

DISPOSITION 5-14c2

❑ No intervention ❑ Follow-up ✔ Histopathologic diagnosis ❑

ANSWERS DERMOSCOPIC CRITERIA ■

Image 1 Bluish-white color (stars) ■ Homogeneous brown color (arrows) ■ Irregular brown dots and globules (boxes) Image 2 ■ Irregular brown dots and globules (black boxes) ■ Irregular hyperpigmentation (yellow stars) ■ Bluish-white color (black stars) ■ Polymorphous vessels (red boxes) ■



Answers: 1,3

Discussion: ■



■ ■ ■

Any mucosal surface can develop melanoma. Theoretically, the genital area should routinely be included in a total body skin examination, but for many reasons that is not possible. There are several high risk criteria that put melanoma at the top of the differential diagnostic list: ■ Bluish-white color ■ Irregular brown dots and globules ■ Diffuse irregular hyperpigmentation ■ Polymorphous vessels There are no criteria to suggest that this is hemorrhage. The criteria seen in the second image come from the pink nodule. Several years after the initial diagnosis, the patient was found to have metastatic melanoma to the brain and recently died!

PEARLS ■



It is the luck of the draw who will develop a melanoma in an unusual place and who will find a dermatologist/physician that will try to be as good as they can be and routinely perform total body skin examinations and become proficient in dermoscopy – a tissue sparing, potentially lifesaving technique. Dermoscopy can improve the diagnosis of melanocytic, nonmelanocytic, benign, malignant, and inflammatory lesions. It can help avoid unnecessary surgical procedures and help one decide which would be the best way to make a histopathologic diagnosis. The learning curve is relatively steep but no comparison to subjects every physician has had to learn (eg, human anatomy, biochemistry, genetics) to get his/her medical degree.

Index Page numbers followed by “f ” denote figures; those followed by “t” denote tables. A ABCD rule, 3, 4t Abdomen melanoma of, 165–166, 323–324 nevus on, 161–162, 205–206, 217–218 Acquired nevus clinical features of, 17 illustration of, 5f, 7f, 13f Acral hemorrhage, 7f Acral lentiginous melanoma, 7f, 356, 362, 366 Acral nevus, 6f, 348 Acral patterns, 6 Acrosyringia, 7, 328, 334, 336, 350, 356, 362 Actinic keratosis characteristics of, 21 pigmented, 80 Actinic lentigo, 16, 21 African-Americans, 198 Agminated nevi, 109 Alopecia areata, 23 Amelanotic melanoma, 5f, 11, 11f, 15, 17, 19–21, 20f, 25, 55–56, 113–115, 122, 178, 202, 212, 215, 227, 238, 243–244, 246, 248, 250–258, 309–310, 367–370, 380, 386, 401–402, 412 Amelanotic nail-apparatus melanoma, 386 Amelanotic nodule, 114 Androgenic alopecia, 23 Angiokeratoma, 120, 122 Annular-granular structures, 12t, 16f, 19, 21, 42, 64 description of, 15 of face, 79–80, 85–86, 88, 90 peppering vs., 90 Appendageal openings, 70, 74 Arborizing telangiectases, 73 Arborizing vessels, 5t, 18, 23–25, 35–37, 47–48, 53–54, 57–60, 73–74, 166, 175–176, 182, 200, 225–233, 235–236, 239–240, 242, 269–272, 305–306 of basal cell carcinoma, 9, 9f, 60, 226, 228, 240, 250 description of, 9, 9f of melanoma, 176, 230, 236 of nevus, 175–176 Areola melanoma, 203–204 Arm melanoma of, 155–156, 171–172, 185–186, 211–212, 235–236, 245–246 nevus of, 179–180, 187–188, 191–192

Asymmetrical follicular pigmentation, 15, 40, 62, 66, 78, 82, 84, 86, 88, 90, 92, 94 Atypical dermatofibroma, 10f, 170 Atypical melanonychia striata, 16 Atypical Spitzoid pattern, 18–19, 25, 45–46, 56, 148, 191–192, 194–196, 198, 217–218 Axillary seborrheic keratosis, 127–128 B Back basal cell carcinoma of, 225–226, 239–240, 269–270, 305–306 melanoma of, 101–102, 105–108, 151–152, 167–168, 207–208, 231–232, 255–256, 265–266, 273–274, 291–292, 301–302, 319–320 nevus of, 129–130, 143–144, 175–176, 193–194, 209–210, 237–238, 267–268, 287–288, 295–296, 307–308, 321–322 seborrheic keratosis of, 131–132, 213–214, 223–224, 243–244, 297–298, 301–302 vascular lesions of, 215–216 Basal cell carcinoma arborizing vessels of, 9, 9f, 60, 226, 228, 240, 250 of back, 225–226, 239–240, 269–270, 305–306 bony-white color of, 36, 228 of chest, 227–228 criteria for, 5t, 9f, 9–10, 23 of face, 57–58 irregular dots and globules of, 48, 182 of leg, 271–272 nonarborizing vessels of, 74, 228 of nose, 35–35, 73–74 pigmented, 9, 9f, 142, 181–182, 225–226 polymorphous vessels of, 228, 272 of scalp, 47–48, 59–60 spoke-wheel structures in, 9–10, 181–182, 305–306 ulceration in, 9, 36, 48, 58, 60, 181–182, 228, 240, 270, 272 Basal cell-like vessels, 38 Benign pigmented nail bands, 16 “Black” blue nevi, 32 Black dots, 4t, 5, 22–23, 270, 338, 392, 410 “Black heel,” 363–364 “Black lamella,” 32, 77–78, 189, 193–194, 204, 296 “Blood pebbles,” 352, 380, 402 Blotches black, 44, 46, 62, 64, 76, 78, 160, 164, 172 blue, 118, 162, 168, 172 brown, 13, 17f, 22, 34, 175–176, 176, 230, 242, 268, 278, 280, 286, 290, 292, 300, 304, 314, 360, 366

420

Index

Blotches (Cont.): description of, 13 grayish-black, 40, 48 irregular, 44 of nevus, 50 Blue blotches, 118, 162, 168, 172 Blue globules, 74, 166 Blue nevus of arm, 180 case studies of, 31–32, 180 in child, 31–32 clinical features of, 6, 6f, 17 polychromatic, 32 Blue ovoid nest of pigmentation of basal cell carcinoma, 182, 184, 303, 306 differential diagnosis of, 182 Blue-white veil, 13, 104 Bluish-white color asymmetry of, 212 of basal cell carcinoma, 60 description of, 14 of genitalia, 400, 402 of melanoma, 44, 58, 92, 104, 126, 166, 172, 186, 212, 230, 300, 316, 356, 366, 404, 418 of nevus, 124, 164, 174, 290, 336 of seborrheic keratosis, 128, 214, 224 Bony-white color of basal cell carcinoma, 36, 228 of desmoplastic melanoma, 388 of melanoma, 274 of regression, 156 Bowen disease characteristics of, 15, 15f, 21, 226, 250 glomerular vessels of, 250, 252 Bowenoid papulosis, 413–414, 415–416 Breast melanoma, 203–204 Brown blotches, 13, 17f, 22, 34, 175–176, 176, 230, 242, 268, 278, 280, 286, 290, 292, 300, 304, 314, 360, 366 Brown dots, 4t, 5, 15, 17, 24, 74, 80, 105, 106, 114, 166, 220, 224, 226, 232, 236, 238, 268, 280, 290, 300, 304, 310, 314, 316, 340, 360, 410, 417–418 Brown globules of basal cell carcinoma, 74, 240 of melanoma, 118, 232, 236, 242, 274, 308 of nevus, 336 in parallel furrows, 332 Brown pigmentation, irregular, 110 C Cherry hemangiomas, 120 Chest basal cell carcinoma of, 227–228 melanoma of, 108–109, 125–126, 133–136, 159–160, 195–196, 221–222, 299–300, 303–304 nevus of, 173–174

Children nail-apparatus melanoma in, 371 nevus in of back, 107–108, 193–194 blue, 31–32 congenital melanocytic nevi, 149–150 of ear, 55–56 of face, 69–70 of fingers, 335–336 of foot, 331–332 of scalp, 49–52 Spitzoid melanoma in, 194 Chronic lichen planus-like keratosis, 261–262 Cicatricial alopecia, 23 Clear cell acanthoma, 250, 253–254 Cobblestone globules, 138, 164 Cobblestone pattern, 12, 92, 108, 138–140, 246, 308 Collision tumors, 21, 21f, 62, 116, 244, 302 Combined nevus, 17, 177–178, 410 Comma-shaped vessels, 250, 254 Compound nevus, 264, 340 Congenital melanocytic nevi, 136, 149–150 Congenital nevi, 17, 17f, 163–164 “Crown on thorns” nevus, 238 “Crown vessels,” 38, 142 Crypts, 142, 284 Cutaneous lupus, 22f Cutaneous metastatic melanoma description of, 20, 118 features of, 11f, 178, 187–188 D Dark homogeneous areas, 12t, 15, 188, 364, 372, 386 Depigmentation of basal cell carcinoma, 226 reticular in dermatofibroma, 312 in melanoma, 154, 156, 216, 236, 314, 320, 338 in nevus, 218 in tumoral melanosis, 200 Dermatofibroma atypical, 10f, 276 characteristics of, 208, 274 criteria for, 5t, 10, 10f, 24–25, 152 of extremities, 153–154 of leg, 153–154, 275–276, 311–312 nonatypical, 276 pigment network of, 10 polymorphous vascular structures, 10, 10f reticular depigmentation in, 312 ring-like structures of, 10 senile, 159 Dermatomyositis, 22

Index

Dermoscopy benefits of, 2 definition of, 2 digital monitoring uses of, 2 two-step algorithm for, 3–7 Desmoplastic melanoma, 388 Diffuse erythema, 108, 154, 238, 410 Dots and globules. See also Globules black, 4t, 5, 13, 22–23, 270, 338, 392, 410 brown, 4t, 5, 13, 15, 17, 24, 74, 80, 105, 106, 114, 166, 220, 224, 226, 232, 236, 238, 268, 280, 290, 300, 304, 310, 314, 316, 340, 360, 410, 417–418 cobblestone, 138 color of, 5 description of, 5 gray, 5, 13 irregular. See Irregular dots and globules melanocytic, 13 nevus and, 34 red, 5, 13 regular, 13, 70, 138 Dysplastic nevus clinical features of, 108, 168, 268, 288, 290, 317–318 criteria for, 18, 18f, 24–25, 322 E Ear melanoma of, 12t, 45–46, 65–66, 77–78 nevus of, 55–56 Eccrine pores. See Acrosyringia Erythronychia, 17, 20, 380 Eumelanin, 14 Extremities. See also Arm; Leg dermatofibroma of, 153–154 melanoma of, 12t, 111–114, 117–118, 145–146, 155–158, 169–170 nevi of, 97–98, 147–148 in situ melanoma of, 19 F Face annular-granular structures of, 79–80 basal cell carcinoma of, 57–58 lentigo maligna of, 57–58 melanoma of, 12t, 43–44, 61–64, 83–90, 93–94 nevus of, 31–34, 69–70 seborrheic keratosis of, 29–30 solar lentigo of, 67–68, 88 False negative melanoma, 20 “Fat fingers,” 8, 8f, 138, 140, 142 Fibrillar acral pattern, 6, 348 Fibrous border, 120 Fibrous septa, 120 Fingerprint pattern, 8, 15f, 68, 90, 132, 136, 408 Fingers, nevus of, 335–336

421

Fish scale-like pattern, 397–398, 400 Fissures, 8, 140, 142 Flat melanoma, 20 Follicular openings, 80 in basal cell carcinoma, 36, 62 in melanoma, 40, 44, 64, 66, 78, 82, 84, 86, 88 in nevus, 30, 32 Follicular pigmentation, 90 Foot melanoma of, 337–338, 345–346, 349–350, 353–354, 355–356, 357–358, 359–360, 361–362, 363–364 nevus of, 329–330, 331–332, 333–334, 341–342, 343–344, 347–348 Forearm, squamous cell carcinoma of, 249–250 Forehead melanoma, 89–92 Free melanin, 268 “Fried egg” appearance, 33–34, 238, 279–280 Furrows, 6, 6f. See also Parallel furrow patterns G Genital lentigines, 392 Genital lentigo, 391–392, 395–398 Genitalia Bowenoid papulosis of, 413–414, 415–416 melanoma of, 403–404, 417–418 nevus of, 409–410, 411–412 Glabrous skin description of, 327–328, 396 volar surfaces of, 6 Global patterns description of, 12 homogeneous, 12, 364, 412 multicomponent. See Multicomponent global pattern Globular pattern description of, 150 of nevus, 206 nonlinear, 406 of penile melanosis, 406 Globular-like structures, 154, 183–184 Globules. See also Dots and globules blue, 74, 166 brown of basal cell carcinoma, 74, 240 of melanoma, 118, 232, 236, 242, 274, 308 of nevus, 336 in parallel furrows, 332 cobblestone, 138, 164 description of, 5, 14f Glomerular vessels Bowen disease and, 250, 254 description of, 15, 15f linear, 254 Gray dots, 5 Gray macules, 262 Grayish-black blotches, 40, 48

422

Index

H Hair shafts abnormalities of, 22–23 illustration of, 90 Hairpin vessels, 8, 8f, 54, 56, 244, 256, 382 Halo nevus, 118, 277–278, 308 Halo phenomenon, 278 Heel, 363–364 Hemangioma, 10f, 11, 120, 194, 216 Hemorrhage acral, 7f skin, 351–352 Hemorrhagic crusts, 122, 364 Homogeneous blue pigmentation, 6 Homogeneous pattern, 6, 12, 130, 364, 412 Hutchinson sign, 16, 371–372, 374 Hyperkeratosis, 106, 210, 222, 234 Hyperpigmentation description of, 298 irregular, 130 Hyperpigmented globules, 150 Hypopigmentation description of, 14, 98, 133, 144, 196 differential diagnosis, 238, 280 of melanoma, 160, 282, 310, 314 multifocal, 84, 98, 100, 108, 110, 112, 148, 300, 318, 322, 344, 410 of nevus, 162, 164, 206, 238, 288, 290 perifollicular, 150, 278 regression vs., 280, 282 of seborrheic keratosis, 30 of squamous cell carcinoma, 252 Hypopigmented fissures, 138 I In situ melanoma, 19, 19f, 264, 344 Ink-spot lentigo case studies of, 41–42, 392 criteria for, 21 Invasive melanoma, 14–15, 47, 62, 72, 94, 102, 104, 110, 114, 118, 126, 133, 135, 146, 157, 203, 222–223, 247–248, 264, 290–291, 355, 358, 366, 402–404 Irregular black blotches, 44, 46, 62, 64, 76, 78, 160, 164, 172 Irregular dots and globules of basal cell carcinoma, 48, 182 black, 234 of blue nevus, 180 description of, 13 of melanoma, 44, 46, 64, 66, 76, 82, 84, 92, 94, 102, 104, 110, 126, 136, 146, 152, 156, 158, 168, 174, 186, 192, 202, 204, 220, 242, 310, 314, 360 of nevus, 50, 52, 108, 162, 278, 288 of Spitzoid pattern, 196 of tumoral melanosis, 200 Irregular linear globules, 72

K Keratoacanthoma, 21 Knee melanoma, 309 L Lacunae, 10–11, 116, 120, 123–124, 216, 248 Lagoons, 10–11 Lanugo hairs, 150 Latinos, 198 Lattice-like pattern, 6, 332, 334 “Leaf-like” structures, 142, 182 Leg basal cell carcinoma of, 271–272 dermatofibroma of, 153–154, 275–276, 311–312 melanoma of, 157–158, 169–170, 201–202, 313–314 nevus of, 293–294 Lentigo actinic, 21 genital, 391–392, 395–396, 397–398 solar case studies of, 67–68, 71–72 characteristics of, 21, 262, 346 of chest, 133 of face, 88 of lip, 71–72 Lentigo maligna characteristics of, 5, 11, 16–17, 19, 21, 29–30, 57, 62, 77–80, 82–86, 88–90, 132, 145, 396 of ear lobe, 16f of face, 57–58, 88 Lentigo maligna melanoma, 11, 15–16, 19, 30, 77–78 Lichen planus, 22 Lichen planus-like keratosis acute, 258 characteristics of, 15f, 80 chronic, 261–262 pink, 18 subacute, 259–260 Lichen sclerosus et atrophicus of penis, 407–408 of vulva, 401–402 Linear globules, irregular, 72 Linear glomerular vessels, 54, 56, 254, 256, 270 Linear hypopigmentation, 100 Linear pinpoint vessels, 250 Lip, solar lentigo of, 71–72 Lupus erythematosus, 22 Lymphangiomas, 120 Lymph-hemangiomas, 120 M Melanocytic lesions analysis of, 11–21 criteria for, 4–7 diagnosis of, 23

Index

Melanocytic lesions (Cont.): global patterns of, 12 local criteria for, 13–17, 23 pattern analysis of, 11–12 pigment network, 14f Melanoma of abdomen, 165–166, 323–324 acral lentiginous, 7f, 356, 362, 366 amelanotic, 5f, 11, 11f, 15, 17, 19–21, 20f, 25, 55–56, 113–115, 122, 178, 202, 212, 215, 227, 238, 243–244, 246, 248, 250–258, 309–310, 362, 367–370, 380, 386, 401–402, 412 arborizing vessels of, 176, 230, 236. See also Arborizing vessels of areola, 203–204 of arm, 155–156, 171–172, 185–186, 211–212, 235–236, 245–246 of back, 101–102, 105–108, 151–152, 167–168, 207–208, 231–232, 255–256, 265–266, 273–274, 291–292, 301–302, 319–320 characteristics of, 123 of chest, 108–109, 125–126, 133–136, 159–160, 195–196, 221–222, 299–300, 303–304 desmoplastic, 388 of ears, 45–46, 65–66, 77–78 of extremities, 12t, 111–114, 117–118, 145–146, 155–156, 157–158, 169–170 of face, 43–44, 61–64, 83–90, 93–94 featureless, 20 feature-poor, 20 of foot, 337–338, 345–346, 349–350, 353–354, 355–356, 357–358, 359–360, 361–362, 363–364 of forehead, 89–92 of genitalia, 403–404, 417 in situ, 19, 19f, 264, 344 invasive, 14–15, 47, 62, 72, 94, 102, 104, 110, 114, 118, 126, 133, 135, 146, 157, 203, 222–223, 247–248, 264, 290–291, 355, 358, 366, 402–404 of knee, 309 of leg, 157–158, 169–170, 201–202, 313–314 lentigo maligna, 11, 15–16, 19, 30, 77–78 nail-apparatus. See Nail-apparatus melanoma of nose, 39–40, 81–82 polymorphous vessels of, 222, 232, 234, 250 pyogenic granuloma vs., 370 recurrent, 278 regression of, 46, 64, 66, 76, 94, 102, 104, 106, 110, 118, 134, 136, 146, 152, 158, 172, 242, 266, 274, 304, 316, 366 of scalp, 75–76 of shoulder, 229–230, 281–282 of sole, 12t, 365–366 Spitzoid, 19, 191–192 superficial spreading, 19–20, 24–25 of thigh, 199–200, 219–220, 241–242, 247–248, 263–264, 315–316 of trunk, 103–104, 197–198, 233–234 types of, 11 ulceration in, 40, 94, 310

423

Melanoma incognito, 2, 20, 314 Melanoma-specific criteria, 12, 12t, 14f, 16–17, 19–21, 24–25, 64, 66, 70, 84, 91–92, 98, 112, 114, 132, 134, 136, 154, 159–160, 168, 171, 174, 176, 191–192, 198, 201–202, 206–207, 210, 214, 231–232, 234, 236, 250, 255, 260, 263, 265, 267–268, 274, 280, 286–287, 299–300, 302, 304, 309, 314, 317–320, 337–338, 342, 350, 356, 366, 379, 392, 396, 400, 410, 412, 414, 416–417 Melanonychia striata, 16, 372, 376 Melanophages, 14, 268, 394 Melanosis description of, 210 penile, 405–406 tumoral, 197–198, 199–200 Menzies method, 3, 4t Micro-Hutchinson sign, 16, 377–378 Milia-like cysts, 7–8, 30, 56, 128, 166, 180, 186, 208, 214, 244, 248, 266, 276, 284, 292 Milky-red areas description of, 15, 116 of melanoma, 114, 186, 216, 232, 236, 246, 292, 304, 310, 366 of nevus, 52, 174 of seborrheic keratosis, 244 of vascular lesions, 124 Molluscum contagiosum, 38 Monilethrix, 22 “Moth-eaten borders,” 9, 30, 68, 82, 84, 86, 88, 132 “Mountain and valley” pattern, 140 Multicomponent global pattern, 12, 64, 72, 76, 92, 104, 106, 108, 114, 118, 126, 136, 146, 152, 156, 162, 164, 168, 170, 172, 174, 176, 256, 300, 302, 306, 316 Multifocal hypopigmentation, 84, 98, 100, 108, 110, 112, 148, 300, 318, 322, 344, 410 N Nail(s) gray bands in, 377–378 nevus of, 371–372, 381–382 subungual hematoma, 16, 17f, 346, 379–380, 383–384 Nail bands, 16, 371, 377–378 Nail folds, 22, 22f Nail matrix melanoma, 380 Nail-apparatus melanoma amelanotic, 386 in children, 371 criteria for, 12t features of, 16f, 20–21, 371, 373–374, 375–376, 385–386 Netherton syndrome, 23 Nevus of abdomen, 161–162, 205–206, 217–218 acquired, 5f, 7f, 13f, 17 acral, 6f, 348 of arm, 179–180, 187–188 of back, 129–130, 143–144, 175–176, 209–210, 237–238, 267–268, 287–288, 295–296, 307–308, 321–322

424

Index

Nevus (Cont.): blue. See Blue nevus of chest, 173–174 in children. See Children, nevus in combined, 17, 410 compound, 264, 340 congenital, 17, 17f, 136, 163–164 “crown on thorns,” 238 dysplastic. See Dysplastic nevus of ears, 55–56 of extremities, 97–98, 147–148 of face, 31–34, 69–70 of fingers, 335–336 of foot, 329–330, 331–332, 333–334, 341–342, 343–344, 347–348 “fried egg” appearance of, 33–34 of genitalia, 409–410, 411–412 halo, 118, 277–278, 308 of leg, 293–294 of nails, 371–372, 381–382 of palm, 327–328 of penis, 409–410, 411–412 recurrent, 17–18, 18f, 278 of scalp, 49–52 of shoulder, 289–290 of sole, 339–340 Spitz, 18–19, 19f, 24–25, 143–144, 187–188, 193–194, 218, 294 of trunk, 97–100, 115–116, 137–140 types of, 11 Nevus spilus, 109 Nodular melanoma, 20, 121–122, 212, 284, 304 Nonarborizing vessels, of basal cell carcinoma, 74, 228 Nonlinear globular pattern, 406 Nonpigmented pseudofollicular opening, 132, 142, 224 Nose basal cell carcinoma of, 35–36, 73–74 melanoma of, 12t, 39–40, 81–82 O Ovoid nests of basal cell carcinoma, 182, 184, 303, 306 differential diagnosis of, 182 P Palm melanoma of, 12t nevus of, 327–328 Parallel furrow patterns brown globules in, 332 description of, 6 double-line variant, 329–330 of foot, 329–330, 332 irregularities of, 331–332 lattice-like pattern, 334

Parallel furrow patterns (Cont.): in melanoma, 354 of palm, 327–328 single-line variant, 330 Parallel line segments, 90, 276, 414 Parallel patterns, 6 Parallel ridge pattern acrosyringia in, 7 description of, 7, 328 differential diagnosis, 352 malignant, 336, 358, 362 in melanoma, 354, 360, 362 pigmented, 350 Parallelism description of, 372 loss of, 386 Pattern analysis, 3, 4t, 11–12. See also specific pattern Pediculosis capitis, 22 Pediculosis pubis, 22 Penis Bowenoid papulosis of, 413–414, 415–416 lichen sclerosus et atrophicus of, 407–408 melanosis of, 405–406 nevus of, 409–410, 411–412 Peppering description of, 5 differential diagnosis of, 202 general images of, 50, 71–72, 80, 106, 128, 262 of melanoma, 64, 90, 106, 152, 156, 172, 202, 232, 246 of seborrheic keratosis, 128 of tumoral melanosis, 200 Perifollicular hypopigmentation, 150, 278 Peripheral erythema, 122, 192 Periungual pigmentation, 371 Pigment networks of dermatofibroma, 10 description of, 13 irregular, 98, 100, 104, 108, 110, 112, 114, 126, 130, 134, 136, 146, 152, 160, 176, 238, 266, 292, 294, 320, 338 of melanoma, 230, 292, 338 of nevus, 98, 290, 294, 342 regular, 98, 102, 114, 116, 134, 154 Pigmentation asymmetrical, 15 basal cell carcinoma, 9, 9f, 142, 181–182, 240 blue ovoid nest of of basal cell carcinoma, 182, 184, 303, 306 differential diagnosis of, 182 homogenous blue, 6 hyperpigmentation, 130, 298 hypopigmentation. See Hypopigmentation Pigmented actinic keratosis, 79–80, 85–86, 89 Pigmented pseudofollicular openings, 30, 128, 224, 242, 284, 298 Pili annulati, 23 Pink lichen planus-like keratosis, 17, 18, 18f

Index

Pinpoint vessels, 54, 56, 170, 238, 242, 252, 254, 256, 258, 270, 310, 368 Polychromatic blue nevus, 32 Polymorphous vascular pattern, 14–15 Polymorphous vessels of basal cell carcinoma, 228, 272 of melanoma, 40, 62, 106, 222, 232, 234, 404 of nevus, 412 Pseudofollicular openings, in melanoma, 36, 44 Pseudo-Hutchinson sign, 16, 372 Pseudomelanoma, 17–18 Pseudonetwork, 67–68, 80, 82, 86 Pseudopigment network, 70 Pseudopods, 13, 194 Psoriasis, 22, 252, 254 Purplish color of basal cell carcinoma, 58 of melanoma, 146, 246 of nevus, 162 of vascular lesions, 183–184 Pyogenic granuloma, 20, 119–120, 369–370 R Radial streaming, 194 Radiation tattoo, 262 Recurrent melanoma, 278 Recurrent nevus, 17–18, 18f, 278 Red dots, 5, 13 Reflection artifact, 346 Regression bony-white color of, 156 characteristics of, 13, 186, 238, 264, 302 hypopigmentation vs., 280, 282 of melanoma, 46, 64, 66, 76, 94, 102, 104, 106, 110, 118, 134, 136, 146, 152, 158, 172, 242, 266, 274, 304, 316, 366 of tumoral melanosis, 200 white color with, 13, 170 Reticular depigmentation in dermatofibroma, 312 in melanoma, 154, 156, 158, 216, 236, 314, 320, 338 in nevus, 218 in tumoral melanosis, 200 Reticular global pattern, 12, 238, 288, 296, 320, 346 Rhomboid structures, 15, 62, 78 Ridges, 8 Ring-like pattern, 10, 395–396, 397–398, 400, 416 S Saccules, 10–11 Scabies, 21 Scaliness, 222 Scalp basal cell carcinoma of, 47–48, 59–60 melanoma of, 75–76

425

Scalp (Cont.): nevus of, 49–52 seborrheic keratosis of, 53–54 Scleroderma nail-fold capillaries in, 22f pattern associated with, 22 Sebaceous gland hyperplasia case study of, 37–38 characteristics of, 21 Seborrheic keratosis atypical, 224 of axilla, 127–128 of back, 131–132, 213–214, 223–224, 243–244, 297–298, 301–302 criteria for, 5t, 7–9, 8f, 23 of face, 29–30 flat, 254 incipient, 142 milia-like cysts associated with, 7f, 7–8 papillomatous, 8 of scalp, 53–54 of thigh, 283–284 of trunk, 141–142 Senile dermatofibroma, 159 Senile hemangiomas, 120 Seven-point checklist, 3, 4t Sharp border demarcation, 142 Sharp demarcation, 9 Shoulder melanoma of, 229–230, 281–282 nevus of, 289–290 Skin hemorrhage, 351–352 Skin lesions criteria for, 5t melanocytic. See Melanocytic lesions Solar lentigines characteristics of, 10, 68 on lips, 72 Solar lentigo case studies of, 67–68, 71–72 characteristics of, 21, 262, 346 of chest, 133 of face, 88 of lip, 71–72 Sole melanoma of, 12t, 365–366 nevus of, 339–340 Spitz nevus, 18–19, 19f, 24–25, 143–144, 187–188, 193–194, 218, 294 Spitzoid melanoma in children, 194 description of, 19, 191–192 Spitzoid pattern atypical, 18–19, 25, 45–46, 56, 148, 191–192, 194–196, 195, 198, 217–218 description of, 45, 143–144, 148

426

Index

Spitzoid pattern (Cont.): irregular, 191–192, 208 of nevus, 218, 294 regular, 188 Spoke-wheel structures, of basal cell carcinoma, 9–10, 181–182, 305–306 Squamous cell carcinoma of forearm, 249–250 of thigh, 251–252 “Starburst” pattern, 12, 14f, 18, 19f, 46, 187–188, 191–194 Sternal melanoma, 221–222 Streaks description of, 13 irregular, 13, 46, 126, 236, 316, 414 of melanoma, 46, 126, 236, 316 of nevus, 187–188, 193–194 regular, 13, 194 Subacute lichen planus-like keratosis, 259–260 Subungual hematoma, 16, 17f, 346, 379–380, 383–384 Sun-damaged skin, 262–263 Superficial spreading melanoma characteristics of, 19–20 criteria for, 24–25 Synonyms, 2 T Tape stripping, 296 Thick melanoma, 20 Thigh melanoma of, 199–200, 219–220, 241–242, 247–248, 263–264, 315–316 seborrheic keratosis of, 283–284 squamous cell carcinoma of, 251–252 vascular lesions of, 183–184 Three-point checklist, 3, 4t Thrombosed hemangioma, 123 Thrombosed lacunae, 124 “Train tracks,” 72, 398 Trichoscopy, 22

Trunk basal cell carcinoma of, 181–182 melanoma of, 12t, 103–104, 197–198, 233–234 nevi of, 97–100, 115–116, 137–140 seborrheic keratosis of, 141–142 in situ melanoma of, 19 vascular lesions of, 119–124, 183–184 Tumoral melanosis, 197–198, 199–200 Tyndall effect, 14, 128, 180, 268 U “Ugly duckling” appearance, 98, 163–164, 213–214, 279–280, 297 Ulceration in basal cell carcinoma, 9, 36, 48, 58, 60, 181–182, 228, 240, 270, 272 in melanoma, 40, 94, 310 V Vascular lesions assessment of, 10–11 of back, 215–216 criteria for, 10–11, 24–25 milky-red areas of, 124 of thigh, 183–184 of trunk, 119–124, 183–184 Vulva lichen sclerosus et atrophicus of, 401–402 melanoma of, 417 W Warts, 22 White color differential diagnosis, 234, 262 with regression, 13 of vascular lesions, 183–184 white network/white pigment network, 10, 19, 22, 153–154, 192, 215, 216, 218, 236, 246, 312, 314 Whitish dots of melanoma, 222 “Wobble” sign, 138, 308 Wood’s light examination, 266